Sei sulla pagina 1di 250

FISICA 1

Autor: Hugo Medina Guzmn


Profesor de la Pontificia Universidad Catlica del Per
Agosto 2009
PRESENTACIN

Me agrad saber que Hugo Medina Guzmn estaba por publicar un texto sobre Fsica. Haba
dos razones suficientes para este sentimiento. Por un lado, tena curiosidad de saber lo que
podra aportar un texto ms de Fsica sobre los otros ya disponibles. Por otro lado, conozco de
la larga carrera de Hugo Medina como cultor de la enseanza de [a Fsica, y tena curiosidad
de ver cmo este compromiso como docente y experiencia se manifestaran en su texto. Tuve
la suerte de conocer al Ing. Jos Castro Mendvil en su taller, donde despleg una destacada
labor en el diseo y construccin de equipo de laboratorio para la enseanza de la Fsica.
Considero que Hugo es un digno discpulo del Ing. Castro Mendvil e igualmente ha dedicado
una fraccin considerable de su tiempo a la docencia, y al diseo y construccin de equipo de
laboratorio para resaltar los conceptos bsicos de la Fsica.
He revisado el contenido de este texto y veo con gran satisfaccin que su autor utiliza un
enfoque muy acertado. Toma como punto de partida una observacin experimental y a partir
de all desarrolla los conceptos fsicos que permiten interpretar esta observacin utilizando la
formulacin matemtica ms sencilla. Todo esto lo hace con el detalle suficiente de manera
que el lector pueda seguir el argumento lgico con facilidad. Considero que ste es un gran
aporte de este texto. Este enfoque contrasta con textos que enfatizan la formulacin
matemtica y dejan al alumno hurfano de una orientacin para aplicarla a una realidad fsica
concreta.
El contenido de temas de la Fsica General que son desarrollados en este texto se ajusta al
programa de estudios de la PUCP. El desarrollo de cada tema incluye ejemplos bien
seleccionados que son desarrollados con un detalle muy esmerado. Al final de cada captulo
se incluye un conjunto de preguntas y problemas propuestos; se incluye las respuestas.
Algunos problemas plantean configuraciones complejas pero que contienen ciertas
propiedades de simetra que permiten su reduccin a configuraciones sencillas. Al final del
texto encontramos un listado de referencias bibliogrficas a un buen nmero de textos de
Fsica General que han servido de consulta al autor.
En general, considero que este texto constituye una representacin grfica de la obra cotidiana
que Hugo ha venido desarrollando durante su carrera docente y, por lo tanto, es un aporte muy
valioso para la comunidad acadmica y pblico en general.

Lima, julio de 2007


PRLOGO

Los estudiantes a menudo se preguntan por qu llevan un curso de Fsica. La mejor razn por la que se
estudia Fsica es porque proporciona un mtodo coherente y lgico para comprender el mundo que nos
rodea; una persona que comprende lo que sucede a su alrededor, es capaz de convivir en su entorno de
manera racional y efectiva. Sin embargo, en ocasiones los estudiantes ignoran el potencial que tiene la
Fsica para explicar el entorno en trminos fciles de entender;
Este libro tiene por objeto brindar a los estudiante s de la Fsica General una ayuda para dom inar los
principios fsicos que son la base de la t ecnologa moderna. En ste libro se asume que los es tudiantes
tienen una base de lgebra, geom etra, y trigonom etra. Es m ucho m s co mpacto que los libros de
texto tradicionales, proporciona muchos ejemplos trabajados y pide resolver problemas
Este libro ser til tam bin como texto para una pe rsona que repasa o que consolida su conoci miento
de la Fsica.
La discusin y las explicaciones narrativas son sufici entemente claras y completas para poder utilizar
el libro o como texto, o como suplemento a un texto ms amplio.
La forma de aprender la fsica es trabaj ar real mente con problema s. Al usar este libro, el estudiante
debe ser activo. Debe intentar trabajar cada uno de los problemas y los ejem plos. Debe mirar las
soluciones solamente si no logra dar con el camino a su solucin.
Los ejem plos en este lib ro estn trabajados exhaustivamente, de modo que puedan servi r com o
modelos para el propio trabajo de los estudiantes. En este sentido se considera que los e studiantes se
benefician al observar los clculos reali zados en ms de una manera, por lo que se han incluido varios
mtodos para efectuar los clculos.
Adems, se tuvo especial cuidado en incluir proble mas y pregu ntas que co mbinan el material del
captulo en cuestin, con materi al de captulos anteriores. Tales problemas y preguntas destacan el
hecho importante de que d iversas reas de la Fsica se manifiestan de manera simultnea en el m undo
real. Adems, este mtodo de temas mltiples propor ciona una manera para que los estudiantes
repasen lo estudiado y ayuda a mejorar la habilidad para resolver problemas.
El diseo grfico es de gran im portancia, y para mejorar su funcin se ha intentado enfocar solamente
una idea principal en cada figura en lo posible. Por consiguiente, las figuras del libro a menudo se
dividen en dos o ms partes, para evitar la confusin de mezclar varias ideas en la misma figura.
Los profesores conocen la importancia de los diagramas de cuerpo libre cuando utilizan la segunda ley
de movimiento de Newton, y to dos lo s estudiantes aprenden de ellos a medida que estudian Fsica.
Tales diagramas se utilizan en to do e l libro, no solam ente en los primeros captulos en los que se
presenta y aplica la segunda ley de Newton. Por ejemplo, cuando se analiza la relaci n en las
oscilaciones, ta mbin entre la presi n y profundidad en un fluido, el anlisis se simplifica
considerablemente por medio de un diagrama de cuerpo libre. De manera semejante, cuando se deduce
la expresin para la rapid ez de una onda transversa l en una cuerda, un diagra ma de cuerpo libre es
muy til.
Cifras significativas. A lo largo de todo el libro se siguen los procedimientos normales para las cifras
significativas.
Se espera que el esfuerzo en la elaboracin de este libro sea de utilidad tanto para los estudiantes como
para los profesores. Toda opinin al respecto ser bienvenida.

Hugo Medina Guzmn


Lima Per
AGRADECIMIENTOS

El auto r ag radece prim eramente a los es tudiantes, quienes han contri buido bastante en la
elaboracin de este libro a travs d e su influe ncia en el es tablecimiento de las t cnicas y
principios de enseanza y a los profesores que con sus sugerencias y revisiones a las
separatas de los captulos hicieron notar puntos que necesitaban una mayor aclaracin.

Hugo Medina Guzmn


CONTENIDO
CAPTULO 1. Unidades, magnitudes fsicas y vectores
Introduccin al curso. Magnitudes fsicas: escalares y vectores. Unidades. Sistema
internacional de unidades.
Precisin y cifras significativas.
CAPTULO 2. Movimiento rectilneo
Definicin de partcula. Concepto de movimiento de traslacin y rotacin. Sistemas de
referencia. Posicin y desplazamiento. Movimiento en una dimensin. Velocidad.
Aceleracin. Movimiento con aceleracin constante. Movimiento vertical con aceleracin de
la gravedad. Grficos en cinemtica: obtencin de la velocidad y de la aceleracin por
derivacin de la funcin posicin versus tiempo, obtencin de la velocidad y de la posicin
por integracin de la funcin aceleracin versus tiempo.
CAPTULO 3. Movimiento en un plano y en el espacio
Sistemas de referencia y el sistema de coordenadas cartesianas en dos dimensiones.
Componentes de los vectores y vectores unitarios en coordenadas cartesianas. Adicin
vectorial. Movimiento en un plano. Vector posicin, desplazamiento y trayectoria. Velocidad.
Rapidez. Aceleracin. Movimiento parablico. Movimiento circular: descripcin horaria
(posicin, velocidad y aceleracin angular) y descripcin vectorial cartesiana.
Componentes normal y tangencial de la aceleracin. Velocidad y aceleracin relativas.
Generalizacin del movimiento a tres dimensiones en coordenadas cartesianas.
CAPTULO 4. Dinmica de una partcula
Leyes de Newton del movimiento. Sistemas de referencia inerciales. Masa y fuerza. Masa y
peso. Fuerzas de contacto y a distancia (Ley de gravitacin universal). Diagrama de cuerpo
libre. Aplicaciones de las leyes de Newton: partculas en equilibrio (Esttica) y en
movimiento acelerado (Dinmica), fuerzas de friccin.
Dinmica del movimiento circular. Dinmica en sistemas de referencia no inerciales.
CAPTULO 5. Trabajo y energa
Producto escalar de vectores. Trabajo de una fuerza. Energa cintica. Trabajo y energa
cintica. Fuerzas conservativas y no conservativas. Energa potencial gravitacional y elstica.
Energa mecnica.
Generalizacin de la ley de conservacin de la energa mecnica. Potencia.
CAPTULO 6. Sistema de partculas
Centro de masa. Posicin, velocidad y aceleracin del centro de masa. Cantidad de
movimiento lineal de una partcula y de un sistema de partculas. Impulso de una fuerza.
Segunda ley de Newton y la conservacin de la cantidad de movimiento lineal para un
sistema de partculas. Energa cintica de un sistema de partculas.
Colisin elstica e inelstica.
CAPTULO 7. Cuerpo rgido
Producto vectorial. Torque. Segunda condicin de equilibrio (Esttica del cuerpo rgido).
Cantidad de movimiento angular. Momento de inercia. Rotacin alrededor de un eje fijo.
Conservacin de la cantidad de movimiento angular. Energa en el movimiento de rotacin.
Energa cintica de rotacin. Rodadura.
CAPITULO 1

INTRODUCCIN AL
CURSO

QUE ES LA FISICA? 1
METODOLOGIA DE LA FISICA 1
PARTES DE LA FISICA 1
MAGNITUDES FSICAS: ESCALARES Y VECTORES. 1
UNIDADES. SISTEMA INTERNACIONAL DE UNIDADES. 2
MEDICIN. 2
UNIDADES. 2
Unidades fundamentales 2
Unidades derivadas 3
Prefijos comnmente encontrados. 3
CONVERSION DE UNIDADES 3
Factores de Conversin 3
ANALISIS DIMENSIONAL 4
a) Verificacin de una frmula especfica. 4
b) Desarrollo de ecuaciones. 4
c) Convertir un sistema de unidades a otro. 4
CIFRAS S1GNIFICATIVAS 5
Regla 1: Redondeo de un nmero 6
Regla 2: Suma y Resta 6
Regla 3: Multiplicacin y Divisin 6
ERRORES 6
Error absoluto 7
Error relativo 7
Porcentaje de error 7
Clasificacin de errores. 7
a) Error inherente 7
b) Error de truncado 7
c) Error de redondeo 7
d) Error de interpolacin 7
e) Error de aproximacin 7
PROPAGACION ERRORES 8
a) Suma de dos o ms variables. 9
b) Diferencia de dos variables. 9
c) Producto de dos o ms variables. 9
d) Potencias y races. 10
e) Cocientes. 10
PRECISIN Y EXACTITUD 11
RANGO DE ERROR O INCERTIDUMBRE 11
ESTIMADOS Y CLCULOS DEL ORDEN DE MAGNITUD 12
MODELOS IDEALIZADOS 13
COMO ESTUDIAR FISICA? 13
PREGUNTAS Y PROBLEMAS 14
CAPITULO 2

Movimiento rectilneo

DEFINICIN DE PARTCULA 1
CONCEPTO DE MOVIMIENTO DE TRASLACIN Y ROTACIN 1
CONCEPTO DE MOVIMIENTO 1
CLASIFICACIN DEL MOVIMIENTO 1
SISTEMAS DE REFERENCIA. POSICIN Y DESPLAZAMIENTO 1
Sistemas de referencia 1
Vector Posicin 2
Desplazamiento 2
Trayectoria y Ecuacin Horaria del Movimiento 2
VELOCIDAD Y RAPIDEZ 3
Rapidez 3
Derivadas de algunas funciones 4
Velocidad 4
Velocidad instantnea 5
ACELERACIN 6
Aceleracin Media 6
Aceleracin Instantnea o simplemente aceleracin 7
MOVIMIENTO RECTILNEO UNIFORME 8
MOVIMIENTO RECTILNEO UNIFORMEMENTE VARIADO 8
La Ecuacin de Torricelli 9
MOVIMIENTO VERTICAL CON ACELERACIN DE LA GRAVEDAD. 11
a) Cada libre 12
b) Lanzamiento hacia arriba 12
c) Lanzamiento hacia abajo 12
PROBLEMA INVERSO - CLCULO INTEGRAL 18
Pequea Tabla de Integrales 19
CINEMTICA DE PARTCULAS LIGADAS. MOVIMIENTOS DEPENDIENTES. 21
PREGUNTAS Y PROBLEMAS 23
CAPITULO 3

Movimiento en un plano y en el
espacio

MOVIMIENTO CIRCULAR 1
Posicin angular 1
Velocidad angular 1
Aceleracin angular 1
RELACIN ENTRE LAS MAGNITUDES ANGULARES Y LINEALES 1
Hallar el desplazamiento angular a partir de la velocidad angular. 2
Hallar el cambio de velocidad angular a partir de la aceleracin angular. 2
MOVIMIENTO CIRCULAR UNIFORME 2
MOVIMIENTO CIRCULAR UNIFORMEMENTE ACELERADO 2
COMPONENTES NORMAL Y TANGENCIAL DE LA ACELERACIN 2
Velocidad. 2
Aceleracin. 2
MOVIMIENTO CURVILNEO 7
El radio de curvatura 7
MOVIMIENTO PARABLICO 10
Ecuacin de la trayectoria 10
Tiempo de vuelo 11
El alcance horizontal 11
La altura mxima 11
VELOCIDAD Y ACELERACIN RELATIVAS 18
Movimiento Relativo de Traslacin Uniforme. La Relatividad de Galileo 18
PREGUNTAS Y PROBLEMAS 26
CAPTULO 4

Dinmica de una partcula

INTRODUCC1ON 1
EL ORIGEN DEL MOVIMIENTO 1
PRIMERA LEY DE NEWTON DEL MOVIMIENTO 1
QU ES FUERZA? 1
CAMBIO DE VELOCIDAD 2
SEGUNDA LEY DE NEWON DEL MOVIMIENTO 3
UNIDADES DE FUERZA Y MASA 3
PESO DE UN CUERPO 4
ACCION Y REACCIN 3
TERCERA LEY DE NEWTON DEL MOVIMIENTO 4
APLICACIONES DE LAS LEYES DE NEWTON 4
ESTTICA DE LAS MASAS PUNTUALES. 4
DINMICA CON FRICCIN DESPRECIABLE. 7
FRICCIN 11
Algunos valores tpicos de coeficientes de friccin 13
DINMICA DEL MOVIMIENTO CIRCULAR 27
FUERZA CENTRPETA 27
CURVAS EN LAS PISTAS 32
MOVIMIENTO EN MARCOS DE REFERENCIA NO INERCIALES 34
MARCO CON MOVIMIENTO DE TRASLACION NO UNIFORME 34
MARCO DE ROTACIN 37
FUERZA CENTRFUGA 38
FUERZA DE CORIOLIS 39
PREGUNTAS Y PROBLEMAS 40
CAPITULO 5

TRABAJO Y ENERGA

INTRODUCCION 1
TRABAJO 1
ENERGIA CINETICA 4
SISTEMAS CONSERVATIVOS Y NO CONSERVATIVOS 6
LA FUNCION ENERGA POTENCIAL 8
CONSERVACION DE LA ENERGA 9
Observadores en movimiento relativo 13
SISTEMAS NO CONSERVATIVOS 15
LA CONSERVACIN DE LA ENERGA Y LA FRICCIN 16
POTENCIA 16
MAQUINAS 18
PREGUNTAS Y PROBLEMAS 19
CAPTULO 6

SISTEMA DE PARTCULAS

INTRODUCCION 1
SISTEMA DE PARTICULAS 1
SEGUNDA LEY DE NEWTON APLICADA A UN SISTEMA DE PARTICULAS 1
CENTRO DE MASA 2
MOVIMIENTO DEL CENTRO DE MASA. 2
IMPULSO Y CANTIDAD DE MOVIMIENTO 4
CONSERVACIN DE LA CANTIDAD DE MOVIMIENTO 6
SISTEMA DE REFERENCIA CENTRO DE MASA 9
CHOQUES 9
CASOS DE CHOQUE 11
El pndulo balstico 18
MOVIMIENTO CON MASA VARIABLE - PROPULSIN POR REACCIN 20
CANTIDAD DE MOVIMIENTO ANGULAR Y TORQUE 22
MOMENTO DE INERCIA 23
MOMENT0 DE UNA FUERZA o TORQUE 23
CONSERVACION DE LA CANTIDAD DE MOVIMIENTO ANGULAR 24
CONSERVACION DE LA CANTIDAD DE MOVIMIENTO ANGULAR DE UN 26
SISTEMA DE PARTICULAS.
PREGUNTAS Y PROBLEMAS 30
CAPTULO 7
CUERPO RGIDO

INTRODUCCION 1
CUERPO RIGIDO 1
MOVIMIENTO DE UN CUERPO RGIDO 1
TRASLACION 1
ROTACIN 1
CANT1DAD DE MOVIMIENTO ANGULAR DE UN CUERPO RGIDO 2
MOMENTO DE INERCIA DEL CUERPO RGIDO. 2
El teorema de Steiner o de los ejes paralelos. 2
El teorema de la figura plana 2
SEGUNDA LEY DE NEWTON PARA ROTACION 5
Maquina de atwood tomando en cuenta la polea 7
EQUILIBRIO ESTTICO 11
TRABAJO Y ENERGIA EN ROTACIN 15
POTENCIA 16
TRASLACIONES Y ROTACIONES COMBINADAS 24
CONSERVACION DE LA CANTIDAD DE MOVIMIENTO ANGULAR 35
GIROSCOPOS Y TROMPOS - MOVIMIENTO DE PRECESION 43
PREGUNTAS Y PROBLEMAS 44

.
THEORETICAL PHYSICS, Mechanics of p articles, rigid and elastic bodies, fluids and
heat flow. F: Woobridge Constant. Trinity College. Addison Wesley Publishing Company (1959)
THEORETICAL PHYSICS,Thermodinamics, electromagnetism,waves, and particles. F:
Woobridge Constant. Trinity College. Addison Wesley Publishing Company (1959)
The Fey nman LECTURE S ON PHYSICS. V olumenes I, I I y II I. R ichard P. Feynman, R obert B . Lei ghton.
California I nstitute of Tec hnology, M atthew Sa nds, St anford University. A ddison Wesley Publishing
Company (1964)
CORRIENTES, CAMPOS Y PARTCULAS. Francis Bitter. Massachussets Institute of Technology. Editorial
Revert S. A. (1964).
INTRODUCCIN AL EST UDIO DE LA MECNI CA, MATE RIA Y ONDAS. Uno
Ingard, William L. Kraushaar. Editorial Revert. (1966).
FUNDAMENTOS DE ELECTRICIDAD Y MAGNETISMO. Arthur F. Kip. University of
California. Mc Graw Hill Book Company (1967)
CIENCIA FSICA Orgenes y principios Robert T. Langeman, Universidad Vanderbilt. UTEHA, (1968)
PROBLEMS IN ELEMENTARY PHYSICS. B. Bukhotsev, V: Krivchenkov, G.
Myakishev, V.Shalnov. Mir Publishers. Moscow (1971)
PROBLEMES DE PHYSIQUE COMMENTES. Tomos I y II Hubert Lumbroso. Mason et Cie, Pars. (1971)
ELECTRICIDAD Y MAGNETISMO PARA E STUDIANTES DE CI ENCIAS E
INGENIERA. Luis L. Cant. Instituto Tecnolgico y de Estudios Superiores de Monterrey.
Editorial Limusa Mexico (1973)
FSICA P ARA LAS CIENCIAS DE L A VID A Y LA SALU D. Simon G. G . M acDonald / Desmond M .
Burns University of Dundee. Fondo educativo interamericano. (1975)
MECNICA NEWTONIANA, MIT Physics course. A. P. French. Editorial Revert. (1974).
FSICA I y II. Solomon Gartenhaus. Purdue University. INTERAMERICANA. (1977)
TEACHING TIPS. A guidebook for the beginning College Teacher. Wilbert J. McKeachie (Unive rsity of
Michigan). Seventh edition D. C. Heath and Company (1978)
FSICA PARA LAS C IENCIAS DE LA VIDA. Alan H. Crom er. Northeastern University. Editorial Revert.
(1978)
GENERAL P HYSICS W ITH BIOSCIE NCE ESS AYS. Jerr y B. Mari on. University o f M aryland. John
Wiley & Sons Inc. (1979)
Fsica general II: Teora Hugo Medina Guzmn, Miguel Piaggio H. QC 21 M19 (Biblioteca PUCP) (1979)
Fsica general II: Problemas resueltos Hugo Medina Guzmn, Miguel Piaggio H. FIS 111 M364 (Biblioteca
PUCP) (1979)
Fsica ge neral I: pr oblemas resueltos H ugo Medina G uzmn, M iguel Piaggio H . F IS 1 04 M 364 (Biblioteca
PUCP) (1981)
FSICA P ARA E STUDIANTES DE CIE NCIAS E INGENIERA. 1 y 2. John P.
McKelvey, Clem son University Howard Gr otch, Pennsilvania State University. HARLA.
Mexico. (1981)
Fsica 3: electricidad y magnetism o para es tudiantes de ciencias e ingeniera
Hugo Medina Guzmn, FIS 141 M36 (Biblioteca PUCP) (1982)
EXPLORING PHYSICS Concepts and applications. Roger W. Redding North Texas State University, Stuart
Kenter, Wadsworth Publishing Company (1984)
PROBLEMAS DE FISICA. J. Aguilar Peris, Un iversidad Co mplutense de Mad rid - J. Casano va Co las,
Facultad de Ciencias de Valladolid. Alambra (1985)
PROBLEMAS DE FISICA. dirigido por S. Ksel. Editorial Mir Mosc. (1986)
PROBLEMAS DE FISICA Y COMO RESOLVERLOS. Clarence E. Ben ett Main e Un iversity. CECSA
(1986)
PHYSICS for Engineering and Science. Michael E. Browne, Ph. D. (professor of Physics University of Idaho.
Schaums outline series Mcgraw-Hill (1988)
FSICA: VOLUMEN 1 . M ecnica, ondas y term odinmica. Duane E. Ro ller, Ro nald Blum . Ed itorial
Revert. (1990).
FSICA: VOLUME N 2. Electricidad, ma gnetismo y ptica. Duane E. Ro ller, Ro nald Blum . Ed itorial
Revert. (1990).
PROBLEMAS DE FISICA. dirigido por O. Ya. Svchenko. Editorial Mir Mosc. (1989)
MECNICA. Berk eley physics course volumen 1. Charles Kitte l, W alter D . Knight,
Malvin A. Ruderman. Editorial Revert SA. (1992).
ELECTRICIDAD Y MAGNETI SMO. Berkeley physics course volumen 2. Edward
M.Purcell. Editorial Revert SA. (1992).
FSICA. Tomos I y II Te rcera edi cin revisada (Segunda edi cin e n espaol), R aymond S: Ser way, Jam es
Madison University, Mcgraw-Hill, (1993)
PROBLEMAS DE FISICA Santiago Burbano de Ercilla, enri que B urbano de Ercilla, Carlos Grac ia Muoz ,
XXVI edicin, Zaragoza, MIRA editores (1994)
ONDAS. B erkeley physics course volumen 3. Frank S. Crawford, Jr. Editorial Revert S A.
(1994).
FSICA Para las ciencias de la vida, David Jou M irabent Universidad autnoma de Barcelona, Joseph Enric
Llebot Rabagliati, Universidad de Girona, Carlos Prez garca, Universidad de Navarra. Mcgraw-Hill, (1994)
Fsica uno Hugo Medina Guzmn, FIS 104 M365 (Biblioteca PUCP) (1995)
APPLIED PHYSICS. Arthur Beiser, Ph. D. Schaums outline series Mcgraw-Hill (1995)
TEACHING INTRODUCTORY PHISICS A Sourcebook. Clifford E: Swartz (State University of New York,
Stony Brook) and Thomas Miner (Associate Editor The Physics Teacher 1972 198 8). ATP Press Sp ringer.
(1996)
TEACHING IN TRODUCTORY PHYSICS Arnold Aro ns Un iversity of Wash ington JOHN WILEY &
SONS, INC. (1997)
FSICA John Cutnell / Kenneth W. Johnson. Southern Illinois University. LIMUSA (1998)
FSICA EN L A CIENCIA Y EN LA I NDUSTRIA. A . Cro mer. Northeastern University. Editorial Revert.
(2000)
FSICA CONTEMPORANEA Edwin Jones. R ichard C hilders, Univ ersity of Sout h Carolina. Mcgraw-
Hill, (2001)
PROBLEMAS Y CUESTIONES DE FISICA. Atanasio Lle, Begoa Betete, Javier Galeano, Lourdes Lle,
Ildefonso Ruiz Tapiador. Universidad Politcnica de Madrid. Ediciones Mundi prensa (2002)
The PHYSICS of ever y day phenomena. A co nceptual in troduction to Ph ysics. W. Thomas Griffith , Pacific
University. Mcgraw-Hill, (2004)
FSICA UN IVERSITARIA. Fra ncis W .Sears, Mark W. Zem ansky, Hugh D. Y oung (Carnegie Mellon
University) y Roger A. Freedman (University of C alifornia. Santa Barbara) Volumen 1, Volumen 2. Undecima
edicin. Pearson - Addison Wesley (2004)
FIVE EASY LESSONS Strategies for s uccessful Physics teaching. Randall D. Knight California Polytechnic
State University, San Luis Obispo. Addison Wesley (2004)
FUNDAMENTALS OF PHY SICS. David Hallid ay (Un iv. o f Pittsb urgh), Rob ert Resn ick (R ensselaer
Polytechnic Institute), Jearl Walker (Cleveland State Univ.). 7th Edition (2005)
INTRODUCCIN AL CURSO Hugo Medina Guzmn

Capitulo 1. INTRODUCCIN AL CURSO


QUE ES LA FSICA? Actualmente la fsica se divide en dos clases: Fsica
La fsica es una ciencia dedicada a la comprensin de Clsica y Fsica Moderna.
los fenmenos naturales que ocurren en el universo. La fsica clsica se ocupa de los fenmenos y las
El objetivo principal del estudio cientfico es leyes que se conocan hasta la final del siglo XIX. La
desarrollar teoras fsicas basadas en leyes fsica moderna se ocupa de los descubrimientos
fundamentales que permitan predecir los resultados hechos desde entonces.
de algunos experimentos. Las leyes de la fsica tratan La fsica clsica se subdivide en cierto nmero de
de describir los resultados de observaciones ramas que originalmente se consideraban autnomas:
experimentales y de mediciones cuantitativas de los la mecnica, el electromagnetismo, la ptica, la
procesos naturales. acstica y la termodinmica.
La fsica es la ciencia ms simple porque estudia los La mecnica se ocupa del estudio del movimiento
sistemas ms simples. La fsica es la base de todas las efectos fsicos que pueden influir sobre este.
dems ciencias. El electromagnetismo se ocupa del estudio de los
La relacin entre la fsica y la ingeniera es ms fenmenos elctricos y magnticos y las relaciones
directa que la que existe entre la fsica y cualquier entre ellos.
otra ciencia. En la ingeniera se trabaja con sistemas a La ptica se ocupa de los efectos fsicos que se
los que se aplica inmediatamente los principios de la asocian a la luz visible.
fsica. Cualquiera sea la rama de la ingeniera o de la La acstica al estudio de los efectos fsicos
ciencia a la que uno se dedique, va a encontrar a cada relacionados con los sonidos audibles.
paso la aplicacin de las nociones que aprendi en la La termodinmica se ocupa de la generacin, el
fsica. Siempre se encontrarn tiles los conceptos transporte y la disipacin del calor.
especficos de la fsica, las tcnicas que se emplean Estas disciplinas que originalmente se desarrollaron
para resolver los problemas, la forma de pensar que independientemente, estn enlazadas por medio de la
se adquiere en el estudio de la fsica. mecnica y el electromagnetismo.
La fsica moderna se inici a fines del siglo XIX, con
METODOLOGIA DE LA FISICA el descubrimiento de cierto nmero de fenmenos
La metodologa que se usa tiene tres formas fsicos que entraban en conflicto con algunos
caractersticas. conceptos de la fsica clsica.
La primera forma es el anlisis de un sistema fsico Bsicamente, esas alteraciones conceptuales fueron
que se realiza en base a las propiedades de sistemas de dos tipos. Una de ellas estableci el lmite superior
ms sencillos, estos sistemas estn relacionados de para las velocidades de las partculas a las que se
algn modo importante con el sistema original, pero aplicaban las leyes de la fsica clsica, esto se asocia
poseen un nmero menor de factores en su a la Teora de la Relatividad de Einstein. El segundo
comportamiento. Siendo estos ms sencillos se se puede considerar como el establecimiento de un
pueden investigar hasta entender bien sus lmite inferior para las dimensiones lineales y de
propiedades, una vez que se obtenga el conocimiento masa de los sistemas fsicos, para los que son vlidas
de cada sistema se puede hacer una reconstruccin las leyes clsicas, esto se asocia a la Teora de la
hasta lograr entender las propiedades del sistema Mecnica Cuntica. Para poder comprender estas dos
original. teoras modernas y los fenmenos de que se ocupan,
La segunda forma parte del principio de que la fsica es necesario estudiar primeramente las leyes de la
se fundamenta necesariamente en la experimentacin. fsica clsica.
A veces la teora sugiere el experimento, pero ms
frecuentemente un experimentador realiza el trabajo MAGNITUDES FSICAS: ESCALARES Y
inicial en un rea particular de la fsica y luego el VECTORES.
fsico terico sintetiza los resultados de los En la descripcin y estudio de los fenmenos fsicos
experimentos y perfecciona el entendimiento de su se han desarrollado (y se desarrollan) conceptos
significado. abstractos muy especiales llamados magnitudes
La tercera se refiere al uso frecuente de las fsicas. Estas magnitudes se definen por medio de un
matemticas. La fsica estudia las interacciones entre conjunto de operaciones experimentales que permiten
objetos. Los objetos interaccionan de acuerdo a obtener un nmero como medida de la magnitud en
ciertas leyes, sean estas conocidas o no. Como las cualquier situacin.
leyes fsicas son casi siempre cuantitativas, es Esta definicin comprende dos pasos esenciales:
esencial poder establecer relaciones lgicas 1) La eleccin de una unidad de medida con
cuantitativas al estudiar los sistemas fsicos. Las mltiplos y submltiplos y
reglas que gobiernan todas estas relaciones son objeto 2) un proceso para comparar la magnitud a medir con
de las matemticas. Por eso se dice que la matemtica la unidad de medida y establecer un nmero (entero o
es el lenguaje de la fsica. fraccionario) como medida de la magnitud. Son
ejemplos de magnitudes fsicas: la longitud, el rea, el
PARTES DE LA FISICA volumen, el tiempo, la masa, la energa, la

1
INTRODUCCIN AL CURSO Hugo Medina Guzmn

temperatura, la fuerza, la potencia, la velocidad, la medir, que por definicin tiene 1m de largo. Este
aceleracin, etc. estndar define una unidad de la cantidad. El metro es
Llamamos magnitud fsica a aquella propiedad de un una unidad de distancia, y el segundo, de tiempo. Al
cuerpo que puede ser medida. La masa, la longitud, la describir una cantidad fsica con un nmero, siempre
velocidad o la temperatura son todas magnitudes debemos especificar la unidad empleada; describir
fsicas. El aroma o la simpata, puesto que no pueden una distancia como "4,29" no significa nada.
medirse, no son magnitudes fsicas. Las medidas de Las mediciones exactas y fiables exigen unidades
las magnitudes se realizan mediante las unidades de inmutables que los observadores puedan duplicar en
medida, establecidas por la Unin Internacional de distintos lugares. El sistema de unidades empleado
Pesas y Medidas (UIPM), que forman el Sistema por los cientficos e ingenieros se denomina
Internacional de unidades (S. I.), aunque existen otras comnmente "sistema mtrico", pero desde 1960 su
unidades que se siguen usando por tradicin (como el nombre oficial es Sistema Internacional, o SI.
kilate, que se emplea para medir la masa de las Las definiciones de las unidades bsicas del sistema
piedras preciosas). mtrico han evolucionado con los aos. Cuando la
Academia Francesa de Ciencias estableci el sistema
Magnitud escalar. Para muchas magnitudes fsicas mtrico en 1791, el metro se defini como una
basta con indicar su valor para que estn diezmillonsima parte de la distancia entre el Polo
perfectamente definidas. As, por ejemplo, si decimos Norte y el Ecuador (ver figura). El segundo se defini
que Jos Antonio tiene una temperatura de 38 C, como el tiempo que tarda un pndulo de 1m de largo
sabemos perfectamente que tiene fiebre y si Rosa en oscilar de un lado a otro. Estas definiciones eran
mide 165 cm de altura y su masa es de 35 kg, est poco prcticas y difciles de duplicar con precisin,
claro que es sumamente delgada. Cuando una por lo que se han sustituido por otras ms refinadas y
magnitud queda definida por su valor recibe el por acuerdo internacional.
nombre de magnitud escalar.

Magnitudes vectoriales. Otras magnitudes, con su


valor numrico, no nos suministran toda la
informacin. Si nos dicen que Daniel corra a 20
km/h apenas sabemos algo ms que al principio.
Deberan informarnos tambin desde dnde corra y
hacia qu lugar se diriga. Estas magnitudes que,
adems de su valor precisan una direccin se llaman
magnitudes vectoriales, ya que se representan
mediante vectores. En este tema estudiaremos los
vectores y sus propiedades. Unidades fundamentales
Las fuerzas, velocidades, presiones, energas, en
UNIDADES. SISTEMA INTERNACIONAL DE realidad todas las propiedades mecnicas, pueden
UNIDADES. expresarse en trminos de tres cantidades bsicas:
masa, longitud y tiempo. En el sistema SI, las
MEDICIN. La fsica es una ciencia experimental. unidades correspondientes son:
Los experimentos requieren mediciones cuyos Masa Kilogramo
resultados suelen describirse con nmeros. Cualquier Longitud Metro
nmero empleado para describir cuantitativamente un Tiempo Segundo
fenmeno fsico se denomina cantidad fsica. Dos Estas unidades se conocen como unidades
cantidades fsicas que describen a una persona son su fundamentales.
peso y su altura. Algunas cantidades fsicas son tan
bsicas que slo podemos definirlas describiendo la TIEMPO
forma de medirlas, es decir, con una definicin Desde 1889 a 1967, la unidad de tiempo se defini
operativa. Ejemplos de esto son medir una distancia como una cierta fraccin del da solar medio (el
con una regla, o un intervalo de tiempo con un tiempo medio entre llegadas sucesivas del Sol al
cronmetro. En otros casos definimos una cantidad cenit). El estndar actual, adoptado en 1967, es
fsica describiendo la forma de calcularla a partir de mucho ms preciso; se basa en un reloj atmico que
otras cantidades medibles. As, podramos definir la usa la diferencia de energa entre los dos estados
velocidad media de un objeto como la distancia energticos ms bajos del tomo de cesio. Cuando se
recorrida (medida con una regla) dividida por el bombardea con microondas de una determinada
tiempo de recorrido (medido con un cronmetro). frecuencia, los tomos de cesio sufren una transicin
entre dichos estados. Se define un segundo como el
UNIDADES. Al medir una cantidad, siempre la tiempo requerido por 9 192 631 770 ciclos de esta
comparamos con un estndar de referencia. Si radiacin.
decimos que un automvil mide 4,29 m, queremos
decir que es 4,29 veces ms largo que una regla de LONGITUD

2
INTRODUCCIN AL CURSO Hugo Medina Guzmn

En 1960 se estableci tambin un estndar atmico


para el metro, usando la longitud de onda de la luz 1 nanosegundo = 1 ns =10-9 s (tiempo en el que la luz
naranja emitida por tomos de kriptn (86Kr) en un viaja 30 m)
tubo de descarga de luz. En noviembre de 1983 el 1 microsegundo = 1 s = 10-6 s (tiempo en el que
estndar se modific de nuevo, esta vez de forma ms una bala del rifle viaja 1 m)
radical. Se defini que la velocidad de la luz en el
1 milisegundo = 1 ms = 10-3 s (cerca de 14 ms entre
vaco es exactamente 299 792 458 m/s. Por
los latidos del corazn)
definicin, el metro es consecuente con este nmero y
con la definicin anterior del segundo. As, la nueva
CONVERSION DE UNIDADES
definicin de metro es la distancia que recorre la luz
Algunas veces encontramos los datos dados en
en el vaco en 1/299 792458 s. ste es un estndar de
unidades distintas al sistema SI. En este caso
longitud mucho ms preciso que el basado en una
debemos convertir las unidades al sistema SI usando
longitud de onda de la luz.
los factores conocidos de conversin.
La tabla siguiente muestra tales factores.
MASA
El estndar de masa, el kilogramo, se define como la
Factores de Conversin
masa de un determinado cilindro de aleacin platino-
Longitud
iridio que se guarda en la Oficina Internacional de
1 pulgada (in) = 2,54 centmetros (cm)
Pesos y Medidas en Sevres, cerca de Pars. Un
1 pie (ft) = 0,3048 metro (m)
estndar atmico de masa, sera ms fundamental,
1 milla (mi) = 5280 ft = 1,609 kilmetros (km)
pero an no podemos medir masas a escala atmica
1 m = 3,281 ft
con tanta exactitud como a escala macroscpica.
1 km= 0,6214mi
Unidades derivadas o
Las cantidades que interesan a los cientficos no se
1 ngstrom A = 10-10 m

limitan a masa, longitud y tiempo. A menudo el
1 ao luz = 9,461 x 1015 m
comportamiento de objetos se describe en trminos
1 unidad astronmica (AU) = 1,496 x 1011m
de sus velocidades; hay que identificar las fuerzas que
1 prsec (pc) 3,09 x 1016 m
actan sobre los cuerpos; se paga por la energa que
Masa
consumen los aparatos domsticos y nos interesa la
1 slug = 14,59 kilogramos (kg)
potencia que pueda desarrollar un motor; la presin
1 kg = 1000 gramos = 6,852 x 10-2 slug
atmosfrica es un indicador til de las condiciones del
1 unidad de masa atmica (amu) = 1,6605 x 10-27 kg
tiempo. Todas las anteriores propiedades,
(1 kg tiene un peso de 2,205 lb donde la aceleracin
aparentemente dispares, que se miden en metros por
de la gravedad es 32,174 ft/s2)
segundo (velocidad), newton (fuerza), joules
Tiempo
(energa), watts (potencia) y pascales (presin),
1 dia =24 h= 1,44 x 103 min = 8,64 x 104 s
finalmente se pueden expresar como productos de
1 ao = 365,24 das = 3,156 x 107s
potencias de masa, longitud y tiempo. Esas unidades,
1 hora (h) =60min =3600s
por tanto, se conocen como unidades derivadas, para
Velocidad
distinguirlas de las tres unidades fundamentales.
1 mi/h = 1,609 km/h = 1,467 ft/s 0,4470 m/s
1 km/h = 0,6214 mi/h = 0.2778 m/s 0,9113 ft/s
Prefijos comnmente encontrados. Utilizamos con
Volumen
frecuencia prefijos para obtener unidades de un
1 litro (L) = 10 m3 = 1000 cm3 = 0,353 1 ft3
tamao ms conveniente. Ejemplos de prefijos
1 ft3 = 0,02832 m3 = 7,481 U.S. galones (gal)
comnmente encontrados:
1 U.S. gal = 3,785 x 10 m3 = 0,1337 ft3
1 manmetro = 1 nm = 10-9 m (un poco ms grande
Fuerza
que el dimetro del tomo)
1 pound (lb) = 4,448 Newton (N)
1 micrmetro = 1 m =10-6 m (una clula de sangre
1 N = 10 Dinas = 0,2248 lb
humana es aproximadamente de 7 m) Trabajo y Energa
1 milmetro = 1 mm =10-3 m (el carbn del lpiz es 1 joule (J) = 0,7376 ft.lb = 107 ergios
aproximadamente de 0,5 milmetros en dimetro) 1 kilogramo-calora (kcal) = 4186 J
1 centmetro = 1 cm =10-2 m (el dimetro de un 1 Btu (60F) = 1055 J
bolgrafo) 1 kilowatt-hora (kWh) = 3,600 x 106 J
1 kilmetro = 1 km = (1000 m) 1 electron volt (eV) = 1,602 x 10-19 J
Angulo
1 microgramo = 1 g =10-6 g = 1-9 kg (masa de una 1 radian (rad) = 57,30
partcula pequea de polvo) 1 = 0,0 1745 rad
1 miligramo = 1 mg = 10-3 g = 10-6 kg (una gota de Presin
agua es aproximadamente 2 mg) 1 pascal (Pa) 1 N/m2 = 1,450 x 104 lb/in2
1 gramo = l g = 10-3 kg (la masa de un clip para papel 1 lb/in2 = 6.895 x 10-5 Pa
es de aproximadamente 1 g)

3
INTRODUCCIN AL CURSO Hugo Medina Guzmn

l atmsfera (atm)= 1,013 x 10 Pa= 1,013 bar = 14,70 x m a g bt c


lb/in2 = 760 torr
Donde a, b y c son exponentes que deben ser
Potencia
determinados y el smbolo indica
1 horsepower (hp) = 550 ft.lb/s = 745,7 W
1 watt (W) = 0,7376 ft.lb/s proporcionalidad. Esta ecuacin es correcta
nicamente si las dimensiones de ambos lados son
iguales, como la dimensin de x es de longitud, la
ANALISIS DIMENSIONAL
La especificacin numrica de una cantidad fsica dimensin, del lado izquierdo tambin debe ser de
depende de las unidades que se empleen. Por longitud.
ejemplo, aunque una distancia se mida en unidades de [m a
]
g bt c = L
metros o pies o millas siempre ser una distancia. Se
L
b
dice que su dimensin es de longitud, la M 2 Tc = L
a
denominacin no depende del sistema de unidades T
empleado. a b c - 2b
Los smbolos usados para especificar la 1ongitud, la M LT =L
masa y el tiempo son L, M y T, respectivamente. Igualando exponentes en ambos miembros
Para denotar las dimensiones de una cantidad se usan obtendremos
a = 0, b =1, c-2b = 0
[]
corchetes, por ejemplo de distancia l = L, de De aqu a = 0, b = 1 y c = 2
[]
velocidad v = L/T, de rea A = L2. [ ] Por lo tanto la expresin debe tener la forma
Entre sus aplicaciones tenemos: x gt 2 o x = kgt 2
El anlisis dimensional puede describir la forma de la
a) Verificacin de una frmula especfica. El ecuacin pero no indica el valor de la constante k.
anlisis dimensional utiliza el hecho de que las
dimensiones se pueden tratar como cantidades Ejemplo 2. Mediante el anlisis dimensional
algebraicas (se pueden sumar y restar slo si se tienen determinar la expresin para la aceleracin centrpeta
las mismas dimensiones). de una partcula que describe un movimiento circular
Si una ecuacin se lee uniforme.
A=B+C Solucin.
Los trminos A, B, y C deben tener las mismas Supongamos que la aceleracin centrpeta depende de
dimensiones. la velocidad, del radio de curvatura y el peso
Ejemplo 1. Verificar la frmula siguiente ac = kv a R bW c
1 2 L
x = x0 + vt + at , donde x y x0 representan aceleracin centrpeta [ac ] =
2 T2
[v] = L
distancias, v es velocidad, a es aceleracin y t es un
intervalo de tiempo. velocidad
Solucin. T
Como []
radio v = L

[x] = [x0 ] + [vt ] + 1 at 2 = L ML


peso [W ] = 2
2 T
Y las dimensiones de la velocidad son L/T y de la Reemplazando
aceleracin L/T2, tenemos: L L b ML
a c

= (L ) 2
[vt ] = L (T ) = L T 2
T T
T a +b +c
LT -2 = L T a 2c M c
1 2 L 2
( )
2 at = T 2 T = L
Igualando exponentes para L: 1 = a + b + c
para T: 2 = a 2c
Podemos ver que esta frmula es correcta porque para M: 0 = c
todos los trminos tienen la dimensin de longitud.
de donde obtenemos a = 2 , b = 1 y c = 0
por lo tanto
b) Desarrollo de ecuaciones. Esto lo podemos ver en
el ejemplo de encontrar la distancia recorrida por un v2
cuerpo en cada libre. ac = kv 2 R 1 = k
R
Pongamos que esta cada puede depender de la masa,
la aceleracin de la gravedad y del tiempo. c) Convertir un sistema de unidades a otro. Si
x = f (m, g , t ) tenemos una frmula en un sistema de unidades
El procedimiento para el anlisis dimensional es podemos convertirlo a una frmula en otro sistema de
poner la expresin en la forma unidades. Sean L1, M1, T1 y L2, M2, T2 sus unidades.

4
INTRODUCCIN AL CURSO Hugo Medina Guzmn

Si la cantidad G de una ecuacin tiene dimensiones G M 12 M 22


= La Mb Tc. Se mide g1 con la unidad G1, y mide g2 g1 = g 2
con la unidad G2, la relacin es: L21T12 L22 T22
G1 2
g1G1 = g 2 G2 g 2 = g 1 M1
G2
a b c g 2 = g1 M2
L M1 T1 2 2
g 2 = g1 1 L1 T1

L2 M2 T2
L2 T2
Ejemplo 3. Si en el sistema MKS la frmula para el
g2 = 5
(2,2 )
2
= 2,25
clculo de la variable R de unidades kg/ms aparece
1 (3,28)2 (1)2
5p 2 Luego en el Sistema Ingls la ecuacin
como R = correspondiente es
1,782 A + p 1
Donde. p tiene unidades de m/s y A de km/m3. 2,25 p 2
Hallar la frmula en el Sistema Ingls. R =
1 kg = 2,2 1b l m = 3,28 pie 95,75 A + p
Solucin. Para comprobar esta expresin evaluemos
Sean en el sistema MKS, L1, M1, T1, y en el sistema m kg
Ingls, L2, M2, T2. R1 para p1 = 1 , A1 = 1 3 y R2 para
s m
Las relaciones entre estos sistemas son;
pie
M1 L T p 2 = 3,28 ,
= 2,2 , 1 = 3,28 , 1 = 1 s
M2 L2 T2
2,2 lb lb
1 A2 = = 6,23 10 2
5p 2 (3,28 pie ) 3
pie 3
En la ecuacin R = Operando en las ecuaciones respectivas obtenemos
1,782 A + p
kg lb
R1 = 1,34 y R2 = 0,899
[R] = M , [ p] = L , [A] = M3 m.s pie.s
LT T L Realizando la conversin de unidades R1 encontramos
que es equivalente a R2.
La cantidad l,782 A tiene las mismas unidades que p

[1,782 A] = [1,782][A] = [1,782 ] M3 = L CIFRAS SIGNIFICATIVAS


Cuando e realizan mediciones, los valores medidos se
L T
Las unidades de 1,782 son conocen nicamente dentro de los lmites de la
incertidumbre experimental, 1o datos medidos
L4
[1,782 ] = inherentemente no son exactos y si se registran en
MT notacin decimal consisten de un conjunto finito de
Observando la ecuacin de R, concluimos que las dgitos llamados cifras significativas, la ltima de las
unidades de 5 son las correspondientes a (R)2. cuales es conocida como cifra dudosa.
Cuando se mide una longitud mediante una regla se
M2
[5 ] = observa la lectura de un instrumento en el cual hay
L2 T 2 una escala, el punto de observacin para la lectura
Para obtener el valor correspondiente a 1,7132 en el llega a una posicin como la que se indica en la
sistema Ingls figura siguiente.
4
L1

4
L1 L24
L
g1 = g2 g 2 = g1 2
M1T1 M 2 T2 M1 T1

M 2 T2 Se puede leer exactamente hasta 11 y apreciar un
g 2 = 1,7132
(3,28)4 = 95,75 dgito ms, este ltimo depende de cada persona
(2,2)(1) puede ser 11,6 , 11,5 11,7.
Si suponemos que nuestros instrumentos estn
Para obtener el valor correspondiente a 5 en el adecuadamente construidos, entonces las lecturas que
sistema Ingls tomemos tendrn significado y sern reproducibles,
excepto el ltimo digito, el de los dcimos de la

5
INTRODUCCIN AL CURSO Hugo Medina Guzmn

divisin ms pequea, ser aunque con significado un Regla 3: Multiplicacin y Divisin


poco incierto. El nmero de cifras significativas del producto
Por lo que no hay objeto en aadir una segunda cifra cociente ser redondeado a un nmero de
incierta. Una cifra significativa es cualquier dgito Significativas igual a aquel componente de
que denota la magnitud de la cantidad segn el lugar aproximacin como se muestra en los ejemplos:
que ocupa en un nmero. Por ejemplo si escribimos 3,14159 x 21,13 = 66,38179 = 66,38
S/. 10,52, todas las cifras son significativas, el uno 3,14159 / 21,13 = 0,14868 = 0,1487
representa el nmero de decenas en soles, el 0 Esto es porque 21,13 tiene slo cuatro cifras
representa que no hay unidad de sol y es significativo significativas, el resultado se redondea a cuatro cifras
y finalmente sabemos que tenemos 52 cntimos. En significativas
la expresin 0,01052 gr. el primer cero de la
izquierda sirve para llamar la atencin hacia la coma, Regla 4. Potencias y races
el segundo cero muestra que el 1 ocupa el segundo La potencia o raz de un nmero de n cifras
lugar despus de la coma. Estos ceros no son significativas se redondea a n cifras significativas.
significativos, sin embargo el 0 entre 1 y 5 es como se muestra en los ejemplos:
significativo. 2,14 2 = 4,5796 = 4,58 2,14 3 = 9,800344 = 9,80
10,52 tiene cuatro cifras significativas (1, 0, 5 y 2)
0,01052 tiene cuatro cifras significativas (1, 0, 5 y 2) 2,14 = 1,46287 = 1,46 3
2,14 = 1,288658 = 1,29
La incertidumbre ms pequea posible con cualquier
aparato de medicin es mitad del lmite de la lectura. Ejemplo 4. Cules son los resultados en las cifras
Sin embargo, la mayora de las investigaciones correctas de las siguientes operaciones indicadas?
generan una incertidumbre mayor que esto. La tabla a) 2,5 x 10-2 x 20
siguiente enumera la incertidumbre de algunos b) 3,32 x 103 + 3,2 x 10
equipos comunes del laboratorio. c) 4,52 x 108 + - 4,2 x 103
d) 2,801 x 4 x 10-3
Regla de metro 0,05 cm e) 6,2 x 104 / 3,0 x 10
Calibrador vernier 0,005 cm Solucin.
Micrmetro 0,005 mm Aqu todos los nmeros estn expresados en notacin
cientfica.
Reloj de segundos 0,5 s
Por ejemplo:
Cronmetro 0,0005 s
0,025 = 2,5 x10-2 = 2,5(-02), tiene 2 cifras
Dinammetro 0,1 N significativas
20 = 2 x 10 = 2(+1), tiene una cifra significativa.
Cuando se anotan y se manipulan nmeros obtenidos
por medidas, sern de mucha ayuda las siguientes a) 2,5 x 10-2 x 20 = 5 x 10-1
reglas: b) 3,32 x 103 + 3,2 x 10 = 3,35 x 103
c) 4,52 x 108 - 4,2 x 103 = 4,52 x 108
Regla 1: Redondeo de un nmero - d) 2,801 x 4 x 10-3 = 11 x 10-3
En el proceso de rechazo de uno o varios de los e) 6,2 x 104 / 3, 0 x 10 = 2,1 x 103
ltimos dgitos. La ltima cifra retenida se
incrementar en 1 si la cifra rechazada es 5 o mayor. Ejemplo 5. Para determinar la densidad de un lquido
Ejemplo. se toman 10 cm3 de ste. La masa del lquido medida
Nmero Redondeo a en una balanza es 15,38g. Cul es la expresin
dado Cuatro Tres Dos correcta de la densidad?
cifras cifras cifras Solucin.
62,578 62,58 62,6 63 La densidad del lquido es
10 232 10 230 10 200 10 000 m 15,38 g
329 350 329 400 329 000 330 000 = = = 1,538 3
V 10 cm
Regla 2: Suma y Resta Siendo 10 el nmero con menos cifras significativas
El nmero de cifras significativas de la suma o (2), el resultado se redondea a 2 cifras significativas.
diferencia ser redondeado desechando todas las La expresin correcta de la densidad es
cifras a la derecha del lugar ocupado por la cifra g
incierta en cualquiera de las cantidades que est ms = 1,5
cm 3
hacia la izquierda, como se muestra en el ejemplo:
ERRORES
Como hemos indicado las mediciones fsicas
involucran incertidumbre. El valor exacto de una
magnitud medida es algo a lo cual intentamos
aproximarnos pero que nunca conocemos. Un nmero
de lecturas cuando se promedia se considera como el

6
INTRODUCCIN AL CURSO Hugo Medina Guzmn

mejor acercamiento al verdadero valor de una lectura, Si se usan los dos primeros trminos.
y la diferencia entre una lectura y la verdadera lectura
3

o lectura exacta se llama error. Aqu la palabra error


2
no significa equivocacin sino una incertidumbre. et = N - N = 1,00000 + = +0,07516
2 3!
Error absoluto es la diferencia entre el valor (+7,5%)
aceptado N (asumimos conocido) y el valor Si se usan los tres primeros trminos.
3 5
aproximado N , obtenido por mediciones o clculos.

e= N-N 2 2
et = N - N = 1,00000 +
2 3! 5!
Error relativo es la relacin entre el error absoluto e = -0,00453 (-0,5%)
y el valor aceptado N Si se usan los cuatro primeros trminos.
e N et = 0,00015 , el error de truncado ya es
e= = 1 insignificante.
N N
c) Error de redondeo (er ) , es el error introducido
Porcentaje de error es el nmero de partes por cada por redondeo de un decimal. Por ejemplo.
100 en que un nmero est errado Si = 3,14159
N Si redondeamos a = 3,14, entonces:
e% = (100e )% = 1 % er = 3,14159 - 3,14 = 0,00159 y
N
0,00159
er = 100 = 0,05%
Cuando calcule el porcentaje de error en fsica 3,14159
d) Error de interpolacin (e p ) , es el error
elemental no use ms de dos cifras significativas.
Por ejemplo si una pista para carreras de 3500 metros
tiene 17 metros ms. introducido por la aproximacin de un valor por su
El error absoluto o simplemente error es equivalente interpolado. Por ejemplo:
e = 17 m Si conocemos la circunferencia de un crculo de l0
El error relativo es metros de dimetro y de otro circulo de 11 metros.
17 C10 = 10 = 31,42 m y
e=
3500 C11 = 11 = 34,56 m
El porcentaje de error es Por interpolacin lineal la circunferencia de un
17 crculo de 10,6 metros es:
e% = 100% = 0,49%
3500 C10,6 = C10 + (C11 C10 ) 0,6 = 33,30 m
Pero el valor exacto es
Clasificacin de errores. C10,6 = 10,6 = 33,31 m
En los clculos numricos pueden ocurrir cinco tipos De aqu
de errores bsicos. e p = 33,31 33,30 = 0,01 m
a) Error inherente (ei ) . Es el error en los datos
0,01
iniciales debido a mediciones, observaciones o o ep % = 100 = 0,03%
registros inexactos. 33,31
b) Error de truncado et . Es el error creado por e) Error de aproximacin (ea ) , es el error
representar una funcin con slo unos cuantos introducido por la aproximacin de una constante o
trminos de una serie. Por ejemplo: una funcin por un valor elegido. Por ejemplo:
La aceleracin debido a la gravedad g = 9,80665
El valor correcto de N = sen = 1,000 m/s2 puede aproximarse por:
2 51 m
El valor aproximado de N computado por expansin g= 10 = 9,80769 2 ea % = 0,01%
52 s
de series es: mejor por
3 5 7
507 m
g= 10 = 9,80658 2
2 2 2 517 s
N= + ...
2 3! 5! 7! ea % = 0,00%
Si se usa solo el primer trmino. (El error aparece en el cuarto decimal)
= -0,57080 (-57%)
et = N - N = 1,00000
2

7
INTRODUCCIN AL CURSO Hugo Medina Guzmn

Error cuadrtico medio o desviacin normal o mm el error absoluto o incertidumbre de la medida es


estndar l = 0,05 mm.
En general cuando se realiza una medicin cualquiera
siempre se comete error, cuando repetimos las Ejemplo 6. Un estudiante realiza varias mediciones
mediciones varias veces, encontramos casi siempre de la masa de un cuerpo, obteniendo los siguientes
resultados diferentes para cada una, aunque resultados: 35,73 g , 35,76 g , 35,80 g, 35,76 g, 35,70
empleemos el mismo mtodo y el mismo aparato. g
Las mediciones sucesivas de un objeto determinado Cul es el mejor valor estimado de la masa del
presentan discrepancias debido a los errores al azar o cuerpo?
aleatorios de las medidas. Si la longitud verdadera de Solucin.
una varilla es l 0 la media aritmtica de un gran La masa media es:
nmero de medidas sucesivas ser un nmero que 35,73 + 35,76 + 35,80 + 35,76 + 35,70
mm =
representa la longitud media l m . Una medida 5
Individual cualquiera tendr una desviacin de la = 35,75 g
media e = l l m , cantidad que puede ser positiva o La desviacin de la media de cada medicin es:
negativa segn l sea mayor o menor que l m , es m1 mm = 35,73 35,75 = 0,02
decir m2 mm = 35,76 35,75 = 0,01
l = lm e m3 mm = 35,80 35,75 = 0,05
Si elevamos al cuadrado cada uno de los valores de e
m4 mm = 35,76 35,75 = 0,01
y tomamos la media de todos los e 2 , obtenemos
m5 mm = 35,70 35,75 = - 0,05
em2 que es la varianza de las medidas.
La varianza de las medidas es:
( 0,02)2 + (0,01)2 + (0,05)2 + (0,01)2 + ( 0,05)2
n

e 2
i em2 =
5
em2 = 1=1
= 0,0112
n La desviacin normal
A la raz cuadrada de esta meda se la conoce como el
error cuadrtico medio o desviacin normal o = em2 = 0,0112 = 0,0334
estndar . La incertidumbre o error estndar de la medida es:
= e 2

0,0334
m
Cuanto mayor sea el nmero n de medidas, menor m = = = 0,01496 = 0,02
ser la diferencia entre su media l m y la longitud n 5
El mejor valor estimado es:
verdadera l 0 , es decir el error estndar de la media,
m = mm m = 35,75 0,02

, ser menor. Por esto el mejor valor estimado m = (35,75 0,02 ) g
n Si hubiramos realizado una sola medicin con una
de l 0 es: balanza cuya menor divisin es de 0,1 g la
incertidumbre seria 0,05 y el resultado de la medicin
podra expresarse as:
l = lm = l m l
n m = (35,75 0,05) g
En donde l es la incertidumbre o error absoluto Observemos que en ambos casos la incertidumbre
determinado a partir de n mediciones. En el caso de corresponde al segundo orden decimal (0,02 y 0,05
verdaderos errores aleatorios, la media l m cae en un respectivamente) incidiendo por lo tanto en la cifra 5,
que es la cifra dudosa.
68 por ciento de las veces dentro de una distancia l
del valor verdadero pero desconocido l 0 . PROPAGACIN ERRORES
De esta forma podemos presentar el resultado final de La determinacin experimental de algunas cantidades
un experimento en el cual se mide varias veces una fsicas tales como densidad o volumen se obtienen
magnitud. Sin embargo, muchas veces realizamos por medicin directa. Generalmente, la cantidad a
slo una medicin de la magnitud. En este caso se determinar se re1aciona de alguna manera conocida a
considera generalmente que la incertidumbre o error una o ms cantidades medibles. El procedimiento es
absoluto es igual a la mitad de la divisin menor de la medir estas cantidades y con estas calcular por medio
escala del instrumento. Por ejemplo: si para medir de relaciones conocidas la cantidad original. Por
longitudes se usa una regla cuya divisin minina es 1 ejemplo el volumen de un cilindro puede conocerse si
tenemos su longitud y Su dimetro. Estas pueden
medirse directamente, cada una con su intervalo de

8
INTRODUCCIN AL CURSO Hugo Medina Guzmn

error asociada, Estos intervalos de error determinan el magnitudes la incertidumbre en el resultado es la raz
Intervalo de error de la cantidad calculada. Es cuadrada de la suma en cuadratura de las
importante saber como hacer esta determinacin de la incertidumbres en las magnitudes.
propagacin de errores.
A continuacin determinemos los errores para Ejemplo 7. Medimos la masa de un tomillo y
diferentes situaciones. obtenemos m1 m1 = (253 5) g , luego
a) Suma de dos o ms variables.
medimos tambin la masa de una tuerca,
Consideremos z = x + y .
m2 m2 = (48 5) g . Cunto vale la masa M
z z = ( x x ) + ( y y ) del tornillo y la tuerca juntos?
Puesto que x e y tienen las incertidumbres x y Solucin.
y , cul es la incertidumbre z en z? Evidentemente, la masa M es
Los mayores valores posibles para x e y son x + x M = m1 + m2 == 253 + 48 = 301 g
e y + y , respectivamente, dando un valor superior La Incertidumbre en la suma es
de z = x + y . M 2 = m12 + m22 = 50 = 7 g
Los menores valores posibles para x e y son x x y el resultado final es
e y y , respectivamente, dando un valor inferior M = (301 7 ) g
de z = (x + y ) .
Ejemplo 8. Cul es la diferencia M entre las masas
Es decir, los valores lmites para z son
m1 y m2 del tornillo y la tuerca respectivamente?
z = ( x + y ) (x + y )
Solucin.
Sin embargo, no utilizamos los (x + y ) como la Evidentemente, la masa M es
incertidumbre. M ' = m1 m2 == 253 48 = 205 g
La razn es que para que z realmente valga La Incertidumbre en la diferencia tambin es
z = ( x + y ) (x + y ) se necesita que la
incertidumbre en la medicin, tanto de x como de y,
M ' 2 = m12 + m22 = 50 = 7 g
sea tal que los dos resultados experimentales sean y el resultado final es
subestimaciones. M ' = (205 7 ) g
Ms probable es que uno de los resultados sea un
poco bajo y el otro un poco alto. Si ste es el caso, la c) Producto de dos o ms variables.
incertidumbre en una de las mediciones puede Supongamos z = xy
compensar, en parte, la incertidumbre en la otra.
Para tomar en cuenta esta posibilidad, lo que hacemos z z = ( x x )( y y )
no es sumar las incertidumbres, sino que calculamos = xy yx xy + xy
z = x 2 + y 2 el error de z es z = yx + xy
Esta manera de combinar las incertidumbres, considerando el mayor valor posible y no tomando en
sumndolas elevadas al cuadrado, se llama suma en cuenta xy por se el producto de dos cantidades
cuadratura. pequeas.
La incertidumbre z calculada de esta manera es El significado de esto se ms claramente en el error
siempre mayor que las a x y y por separado, relativo.
pero menor que la suma x + y . La diferencia z yx + xy x y
= = +
entre simplemente sumar las incertidumbres y z xy x y
sumarlas en cuadratura es que la suma simple da la
Ejemplo 9. Cul es el producto de (2,6 0,5) cm
incertidumbre mxima en el resultado, mientras que
la suma en cuadratura da la incertidumbre ms
probable. y (2,8 0,5) cm?
b) Diferencia de dos variables Solucin.
Consideremos z = x y . Primero, determinamos el producto de 2,6cm x 2,8cm
z z = ( x x ) ( y y ) = 7,28 cm2
La incertidumbre que queremos es la incertidumbre 0,5
Error relativo 1 = = 0,192
ms probable, que viene a ser la raz cuadrada de la 2,6
suma en cuadratura de las incertidumbres
0,5
z = x 2 + y 2 Error relativo 2 = =0,179
2,8
Por lo tanto, tenemos una regla para la propagacin Suma de los error relativos = 0,371 o 37,1 %
de incertidumbres Cuando sumamos o restamos dos

9
INTRODUCCIN AL CURSO Hugo Medina Guzmn

Error absoluto = 0,37l x 7,28 cm2 o 3,71 % x 7,28 Ejemplo 12. Encontrar el error en el clculo de
cm2 = 2,70cm2 1
Los errores son expresados con una cifra significativa z= 3
= x 3
= 3 cm2 x
El producto es igual a 7,3 3 cm2 Solucin.
x
d) Potencias y races.
z = 3x 31 x = 3 x 4 x = 3
x4
Sea z = x
n
Como los errores son indeterminados debemos elegir
Donde n es el nmero entero o fraccin positivo o el signo de tal manera que ste sea el mximo, por
negativo. esto:
z z = ( x x ) x
n
z = 3
Esto se puede escribir x4
y el error relativo es
x
n

z z = x 1
n
x
x 3 4
z x
= x =3
x
n
z 1 x
Haciendo la expansin binomial de 1 + 3
x x
e) Cocientes.
x =
n

1 + x
x Supongamos z =
x n(n 1) x n(n 1)(n 2) x
2 3 y
1+ n
x
+ +
2! x 3!
+ ...
x
z z =
(x x )
ignorando las potencias mayores que 1 de x ( y y )
x x
n
Esto se puede escribir como:
1 + = 1+ n z z = ( x x )( y y )
1
x x
1
De aqu x 1 y
x = x 1 1

z z = x n 1 n x y y
x
x x y
n 1
El error de z es z = nx x 1 1 m
Y el error relativo es
y x y
z x x x y x y
=n 1 +
z x y x y x y
Ignorando el ltimo trmino por se muy pequeo y
Ejemplo 10. Encontrar el error en el clculo de tomando el valor mximo para z .
z = x2 El error de z es:
Solucin. x x y yx + xy
z = 2 x 21 x = 2 xx z = + =
E error relativo es
y x y y2
z x El error relativo es:
=2
z x yx + xy
Ejemplo 11. Encontrar el error en el clculo de z y2 yx + xy x y
= = = +
z = x = x1 2 z x xy x y
Solucin y
1
1 2 1 1 x
z = x x = Ejemplo 13. Supongamos que queremos calcular la
2 2 x densidad de un cilindro de metal habiendo
E error relativo es
medido su masa M, su longitud L y su dimetro D. Al
z 1 x mismo tiempo queremos calcular el error relativo
=
z 2 x resultante de los errores en las cantidades medidas.
Sabemos que la densidad est dada por la ecuacin

10
INTRODUCCIN AL CURSO Hugo Medina Guzmn

M 4M
= = Ejemplo 16. La medida de los lados de un rectngulo
(D 2) L D 2 L
2
son (1,53 0,06) cm, y (10,2 0,1) cm,
Solucin. respectivamente. Hallar el rea del rectngulo y el
4M 4 error de la medida indirecta.
= = MD 2 L1 Solucin.
D L
2

Como 4 y son cantidades exactas no tienen error. El rea es A = 1,53 10,2 = 15,606 cm2
M Como debe de tener solamente 3 cifras significativas
El error relativo de M es
M A = 15,6 cm 2
2D El error relativo del rea
El error relativo de D es 2 2
D A 0,06 0,1
= + = 0,0404422504
L A 1,53 10,2
El error relativo de L es
L El error absoluto del rea
De aqu A = 0,0404422504(1,53 10,2 ) = 0,63083
El error relativo de es
El error absoluto con una sola cifra significativa es
M 2D L 0,6.
= + + La medida del rea junto con el error y la unidad se
M D L
escribir como
Ejemplo 14. El volumen de un cilindro de base A = (15,6 0,6 ) cm 2
circular es V = R L . Cunto vale la
2

incertidumbre o error en el volumen en trminos de Ejemplo 17. Se mide x con una incertidumbre x
las incertidumbres R y L ? y se calcula y = ln x . Cunto vale y ?
Solucin. Solucin.
Como es cantidad exacta no tienen error. y + y = ln ( x + x )
2R En este caso podemos usar aproximaciones para
El error relativo de R es
R cantidades pequeas, cuando x << 1 , tales como:
L
El error relativo de L es (1 x )n
1 nx , e x 1 + x , ln (1 + x ) x ,
L
De aqu senx x , cos x 1 , tan x x
En nuestro caso
El error relativo de V es x
y + y = ln( x + x ) = ln x1 +
V R L x
=2 +
V R L x x
Y el error absoluto: = ln x + ln1 + ln x +
R L R x x
V = 2 + V = R 2R + L x
R L L Como << 1 podemos aplicar
x
Ejemplo 15. Supongamos que queremos medir el x x
periodo T de un oscilador, es decir, el tiempo que ln1 + , luego:
tarda en efectuar una oscilacin completa, y
x x
disponemos de un cronmetro que aprecia las x x
dcimas de segundo, 0,1 s. Medimos el tiempo que y + y = ln x + ln1 + ln x +
tarda en hacer 10 oscilaciones, por ejemplo 4,6 s,
x x
dividiendo este tiempo entre 10 resulta t =0,46 s, Siendo y = ln x :
cmo se expresa la medida? x
Solucin. y =
x
t t
T= , T =
10 10 PRECISIN Y EXACTITUD
0,1 Los trminos "PRECISION " y "ACCURACY" del
Obtenemos para el error T = = 0,01 s . Por idioma ingls no son sinnimos, para efectos de
10 lenguaje estadstico traduciremos "Precision" como
tanto, la medida la podemos expresar como precisin y "Accuracy" como exactitud, estableciendo
T = (0,46 0,01) s diferencias claras entre las dos palabras.

11
INTRODUCCIN AL CURSO Hugo Medina Guzmn

La precisin es una indicacin de la concordancia


entre un nmero de medidas hechas de la manera
indicada por el error absoluto. Un experimento de
gran precisin tiene un bajo error al azar.
La exactitud es una indicacin de cuan cercana est
una medida al valor aceptado indicado por el error
relativo o del porcentaje de error en la medida. Un
experimento de gran exactitud tiene un error
sistemtico bajo. En la direccin vertical, dibujamos una lnea arriba y
As como la obtencin de una serie de medidas con abajo para que cada punto muestre la gama de
las unidades correctas, se requiere una indicacin del incertidumbre del valor de la fuerza. Entonces
error experimental o el grado de incertidumbre en las ponemos una pequea lnea marcadora horizontal en
medidas y la solucin. Cuanto mayor es la exactitud el lmite del extremo incierto para el punto.
y la precisin en nuestras investigaciones, ms bajo En la direccin horizontal, dibujamos una lnea a la
es el grado de incertidumbre. izquierda y a la derecha para que cada punto muestre
Las cuatro figuras a continuacin ilustran la la gama de incertidumbre del valor de la extensin.
diferencia: Entonces ponemos una pequea lnea marcadora lnea
vertical en el lmite del extremo incierto para el
punto.
Cuando todos los puntos de la tabla se trazan en un
grfico, la lnea del mejor ajuste con las barras
apropiadas de error se muestra en la figura siguiente y
se puede ver que la lnea del mejor ajuste cae dentro
del rango de la incertidumbre de la barra del error.

RANGO DE ERROR O INCERTIDUMBRE


Cuando una respuesta se expresa como valor con
incertidumbre tal como 2,3 0,1 cm, entonces la
gama de la incertidumbre es evidente. El valor cae
entre 2,4 (2,3 + 0,1) y 2,2 (2,3 - 0,1) cm. En la
ESTIMADOS Y CLCULOS DEL ORDEN DE
fsica, determinamos a menudo la relacin que existe
MAGNITUD
entre las variables. Para visin la relacin, podemos Hasta donde hemos visto, es importante cuidar el
realizar una investigacin y trazar un grfico del eje seguimiento de las incertidumbres en la medicin
dependiente) contra la variable independiente (eje x). cuando se calculan las respuestas a los problemas. En
Considere un resorte que tenga varios pesos, unido a algunas ocasiones, tanto en la vida cotidiana como en
l. A mayor peso se une a un resorte, el resorte el quehacer cientfico, es necesario resolver un
extiende ms lejos de su posicin del equilibrio. La problema del que no tenemos informacin suficiente
tabla siguiente muestra algunos valores para esta para obtener una respuesta precisa. A menudo
investigacin de Fuerza/alargamiento. podemos obtener una respuesta til mediante la
estimacin de los valores de las magnitudes
Fuerza 5 N 100 15 0 20 0 25 0 30 0 apropiadas. Estas estimaciones, realizadas
Alargamiento 3,0 4, 4 6, 2 7, 5 9, 1 generalmente a la potencia de diez ms cercana, se
0,2 cm denominan estimaciones del orden de magnitud. El
Cuando se traza un grfico de la fuerza contra el clculo resultante del orden de magnitud no es
alargamiento, la lnea del mejor ajuste no pasa por exacto, pero generalmente es correcto con un factor
cada punto. Una barra del error se puede utilizar para de diez. El conocimiento justo del orden de magnitud
dar una indicacin del rango de la incertidumbre para de las cantidades fsicas con frecuencia nos
cada punto segn se muestra en la figura a proporciona informacin suficiente para obtener una
continuacin Fuerza/alargamiento. comprensin til de la situacin fsica y la capacidad
para formarnos un juicio y hacer clculos para la
construccin de modelos.
Realizar estimaciones de magnitud con frecuencia es
sencillo. Por ejemplo, imagine que va a la escuela por

12
INTRODUCCIN AL CURSO Hugo Medina Guzmn

primera vez y que quiere estimar cunto dinero As, el nmero aproximado de caramelos que hay en
necesitara para comprar libros. Usted conoce que la el frasco es:
carga habitual para la mayor parte de los estudiantes 800cm3
es de cinco materias, y que en cada una se necesita un Nmero de caramelos 240 .
libro de texto. 27 3
cm
Con estos datos puede estimar el costo de un solo 8
libro con el razonamiento siguiente. Sabe por Un conteo realizado de los caramelos que llenan un
experiencia que S/. 1 es demasiado bajo y que S/. 100 frasco de un cuarto (0,95 litros) dio 255 caramelos.
es demasiado alto. Incluso S/. 10 es bajo. Una
estimacin razonable puede ser S/. 50. As, el costo MODELOS IDEALIZADOS
estimado de los libros para un semestre es de 5 x S/. Ordinariamente usamos la palabra "modelo" para
50 = S/. 250. Aunque el resultado no es exacto, est referimos a una rplica en menor escala (digamos, de
dentro del orden de magnitud correcto y proporciona un ferrocarril) o a una persona que exhibe ropa (o se
una estimacin razonable a un problema real. El exhibe sin ropa). En fsica, un modelo es una versin
siguiente ejemplo ilustra la aplicacin de las simplificada de un sistema fsico que sera demasiado
estimaciones del orden de magnitud. complejo si se analizase de forma detallada. Por
ejemplo, supongamos que nos interesa analizar el
Cuando hacemos clculos de este tipo con frecuencia movimiento de una pelota de bisbol lanzada en el
tambin efectuamos otras aproximaciones. Al aire. Qu tan complicado es el problema? La pelota
remplazar por 3 o remplazar 2 por 3/2 no es perfectamente esfrica ni perfectamente rgida:
hacemos pocas diferencias en el orden de magnitud, tiene costuras, est girando y se mueve en el aire. El
pero hacerlo simplifica mucho los clculos. Los viento y la resistencia del aire afectan su movimiento,
ejemplos siguientes ilustran esta tcnica. la Tierra gira, el peso de la pelota vara un poco al
cambiar su distancia respecto al centro de la Tierra,
Ejemplo 18. Una tienda ofrece un premio al cliente etc. Si tratamos de incluir todos estos factores, la
que adivine con la mayor aproximacin el nmero de complejidad del anlisis nos abrumar. En vez de
caramelos de goma que llenan un frasco de un litro ello, inventamos una versin simplificada del
exhibido en un mostrador de la tienda. (Un litro es problema. Omitimos el tamao y la forma de la pelota
igual a 1000 cm3.) Estime cual ser el nmero. representndola como objeto puntual, o partcula.
Despreciamos la resistencia del aire haciendo que la
pelota se mueva en el vaco, nos olvidamos de la
rotacin terrestre y suponemos un peso constante.
Ahora tenemos un problema sencillo de tratar.
Para crear un modelo idealizado del sistema debemos
pasar por alto muchos efectos menores y
concentramos en las caractersticas ms importantes.
Claro que hay que ser cuidadosos para no despreciar
demasiadas cosas. Si ignoramos totalmente los
efectos de la gravedad, nuestro modelo predecir que
Solucin. si lanzamos la pelota hacia arriba sta se mover en
Una revisin cuidadosa del frasco (vase la figura) lnea recta y desaparecer en el espacio. Necesitamos
revela varias cosas. Los caramelos de goma pueden algn criterio y creatividad para crear un modelo que
aproximarse vagamente a pequeos cilindros de casi simplifique lo suficiente un problema sin omitir sus
2 cm de largo por aproximadamente 1,5 cm de caractersticas esenciales.
dimetro. Adems, los caramelos no estn apretados Al usar un modelo para predecir el comportamiento
en el frasco; posiblemente tan s1o se ha llenado 80% de un sistema, la validez de las predicciones est
de ste. Podemos hacer uso de estas observaciones limitada por la validez del modelo. La prediccin de
para estimar el nmero de caramelos que hay en el Galileo respecto a la cada de los cuerpos corresponde
frasco. a un modelo idealizado que no incluye la resistencia
Volumen ocupado del frasco del aire. El modelo funciona bien para una bala de
Nmero de caramelos =
Volumen de un caramelo can, pero no para una pluma.
El concepto de modelos idealizados es muy
EI volumen ocupado del frasco = 0,8 x 1000 = 800 importante en fsica y en tecnologa. Al aplicar
cm3, principios fsicos a sistemas complejos siempre
Volumen de un caramelo = usamos modelos idealizados, y debemos tener
2 presentes las suposiciones que hacemos. De hecho,
3
d
2 cm 27 3
los principios mismos se expresan en trminos de
h 2cm 3 2 cm modelos idealizados; hablamos de masas puntuales,
2 2 8 cuerpos rgidos, aislantes ideales, etc. Estos modelos
desempean un papel crucial en este libro. Trate de

13
INTRODUCCIN AL CURSO Hugo Medina Guzmn

distinguirlos al estudiar las teoras fsicas y sus habilidad para resolver problemas puede ser la
aplicaciones a problemas especficos. principal prueba de los conocimientos. Es esencial
que se comprendan los principios y conceptos bsicos
COMO ESTUDIAR FISICA? antes de intentar resolver problemas.
Para estudiar fsica es necesario dar atencin especial En fsica general los exmenes se componen
a los significados especficos de las palabras para principalmente de problemas a resolver, es muy
poder entender el material, deben estudiarse importante que se entiendan y recuerden las hiptesis
detenidamente los grficos, dibujos, tablas y que sirven de base a una teora o formalismo en
fotografas incluidos para entender claramente los particular.
principios fsicos involucrados. Para la resolucin de problemas se incluyen cinco
Gran parte de lo que se aprender ser en las clases. etapas bsicas:
Debern aprender a tomar apuntes exclusivamente de a) Dibuje un diagrama con ejes coordenados si son
las partes significativas de cada leccin y necesarios y ponga las notaciones identificatorias,
concentrarse por completo en lo que el profesor est con esto podemos eliminar errores de signo.
diciendo, estos apuntes son necesariamente breves y b) Identifique el principio bsico, incgnitas, listando
carentes de relacin. Por lo tanto, es recomendable los datos y las incgnitas.
tener un cuaderno ordenado con las notas de clase c) Seleccione una relacin bsica o encuentre una
completando con apuntes tomados del estudio de los ecuacin que se pueda utilizar para determinar la
libros. Hagan esto tan pronto como sea posible incgnita y resulvala simblicamente. En esta forma
despus de clase, esto permitir tener un conjunto de se evitan errores y ayuda a pensar en trminos fsicos
notas claras e inteligibles para repaso; ayudar a el problema.
detectar las reas dbiles de conocimiento. d) Sustituya los valores dados con las unidades
La parte ms importante de los apuntes son los apropiadas dentro de la ecuacin y obtenga el valor
problemas resueltos. Resulvanse todos los ejemplos numrico de la incgnita.
vistos en clase y los dejados como tarea. e) Verificacin y revisin del resultado por medio de
Richard Feynman premio Nbel en fsica dijo: "usted las siguientes preguntas:
no sabe nada sobre algo hasta que lo ha practicado". Las unidades coinciden?
La habilidad para resolver problemas no es slo una Es razonable el resultado?
prueba del dominio que cada cual posee de la ciencia, Es apropiado el signo? Tiene significado?
sino tambin un ndice del crecimiento de nuestra Una vez que el estudiante ha desarrollado un sistema
propia capacidad como herramienta en las futuras organizado para examinar problemas y extraer la
tareas del intelecto. informacin relevante, tendr confianza y seguridad
Se recomienda desarrollar las habilidades necesarias cuando tenga que resolverlos.
para resolver un amplio rango de problemas. La

PREGUNTAS Y PROBLEMAS

1. Suponga que est planeando un viaje en automvil


a otra ciudad y estima el tiempo que se requiere para 7 Qu modelo describe en la forma ms sencilla las
ir all. Demuestre cmo esta estimacin depende de observaciones siguientes?
un modelo. Cmo se ha descrito en el texto y qu a) Una pelota colocada en cualquier lugar sobre el
tan confiable es? piso permanece en reposo.
b) Una pelota colocada en cualquier lugar sobre el
2. D un ejemplo personal del uso de un modelo para piso empieza a rodar.
el anlisis de los datos medidos. c) D otros modelos ms sencillos para estas
observaciones.
3. Explique la idea bsica detrs de la conversin de Respuesta.
unidades. a) Bola esfrica uniforme sobre un piso horizontal.
b) Bola esfrica uniforme sobre un piso inclinado.
4. Explique la diferencia en significado de las tres c) Para a) la bola tiene una parte plana o no es
cantidades 10 m, 10.0 m y 10.00 m. uniforme y para b) la bola es asimtrica y empieza a
5. Cul de los nmeros siguientes se da con tres rodar hacia su lado ms pesado.
cifras significativas: 0,003 m, 0,32 cm, 0,320 cm,
3,21 mm o 3,213 mm? 8. Se lanza un dado muchas veces con los resultados
siguientes para el nmero que aparece en su cara
6. Un estudiante mide un rectngulo con una regla superior: 1, 63 veces; 2, 58 veces; 3, 62 veces; 4, 63
cuya medida vara 1 mm. Encuentra que la altura veces; 5, 75 veces y 6, 61 veces. Qu modelo puede
es 37 mm y el acho 46 mm. Por qu debe informar hacer para el dado?
que el rea del rectngulo Respuesta.
1700 mm2 en lugar de 1702 mm2? El dado es ms pesado hacia el punto 2.

14
INTRODUCCIN AL CURSO Hugo Medina Guzmn

19. Cul es el rea en centmetros cuadrados de un


9. Un cubo de metal flota en un lquido. Cul es el pedazo de papel de 8 pulg x 14 pulg?
modelo ms sencillo del cubo y del lquido? Hay Respuesta.
otros modelos? 1.25 768 cm2
Respuesta.
El cubo tal vez sea hueco si flota en el agua. 20. Los listones de madera en una cerca estn
Alternativamente, el cubo es slido pero flota en un espaciados 6,0 pulgadas, de centro a centro.
lquido que es ms denso que l. Cuntos listones estn contenidos en un metro de
valla?
10. Un litro (L) es un volumen de 10 cm3. Cuntos Respuesta.
centmetros cbicos hay en 2,5 mililitros? 6,6
Respuesta.
2,5 cm3 21. La Luna gira sobre su eje cada 271/3 das de
modo que la misma cara est siempre hacia la Tierra.
11. Qu tan lejos viaja la luz en un vaco en 1,0 A cuntos grados rotar la Luna respecto a su
nanosegundos (Velocidad de la luz = 3,0 x l08 m/s.) propio eje en una hora?
Respuesta Respuesta.
30cm 0,549

12. Los granos negros en algunos tipos de pelculas 22. Cuntas revoluciones hace el segundero de un
fotogrfica son de aproximadamente 0,8 m de reloj en tres aos? Suponga que no hay ao bisiesto
seccin. Asuma que los granos tienen una seccin en el intervalo.
transversal cuadrada y que todos quedan en un solo Respuesta.
plano de la pelcula. Cuntos granos se requieren 1,58 x 106 revoluciones
para oscurecer completamente 1 cm2 de pelcula?
Respuesta. 23. La Tierra tiene una masa de 5.98 x 1024 kg y un
1,6 x 108 radio de 6,38 x 106 m. a) Cul es la masa por unidad
de volumen de la Tierra en kg/m3? b) Cul es la
13. Una frmula se lee y = at2, donde y est en masa por unidad de volumen de un ncleo de oro que
metros y t en segundos. Cules son las dimensiones tiene una masa de 3,27 x 1025 kg y un radio de 6,98 x
de a? 10-15 m? c) Cul sera el radio de la Tierra si su
Respuesta. masa no cambiara, pero tuviera la misma masa, por
m/s2 unidad de volumen, que el ncleo de oro?
Respuesta.
14. Cul es la altura en centmetros de una persona a) 5,50 x 103 kg/m3, b) 2,30 x 1017 kg/m3, c) 184 m
cuya estatura es 5l1? 24. Calcule el volumen de la tabla rectangular con
Respuesta. altura de 17,5 mm, ancho de 29,4cm y longitud 115,4
180cm cm. Recuerde la regla que se refiere a las cifras
significativas.
15. Cmo es 40,2 mi expresado en kilmetros?
Respuesta
64,7 km

16. Exprese 130 km/h en trminos de millas por


hora.
Respuesta. Respuesta.
80,8 mi/h 5,94 x 103 cm3

17 Una tienda anuncia un tapete que cuesta US


$18,95 por yarda cuadrada. Cunto cuesta el tapete 25. Si usted mide los lados de un cuadrado y son de
por metro cuadrado? diez centmetros con una exactitud de 1 %, cul es
Respuesta. el rea del cuadrado y cul es la incertidumbre?
22,66 dlares/m2 Respuesta.
(100 2) cm2
18. Cuando la gasolina se vende a US $1,609 por
galn, cul es el precio en dlares por litro? (1 gal = 26. Sume los nmeros siguientes: 3,57 x 102, 2,43 x
3,l7853 L) 103 y 4,865 x 102.
Respuesta. Respuesta.
0,282 dlares/L 3,27 x 103

15
INTRODUCCIN AL CURSO Hugo Medina Guzmn

27. Un legajo de papel copia tiene 5,08 cm de 32. En algunos pases el consumo de gasolina de un
espesor. Cul es el espesor de una sola hoja del automvil se expresa en litros consumidos por 100
papel? Exprese su respuesta en m y mm. km de viaje. Si un automvil logra 27 millas/galn,
Respuesta. cul es el consumo de combustible en litros por 100
1,02 x 10-4 m o 0,102 mm km? (1 gal = 3,7853 L)
Respuesta.
28. El piso rectangular de un gimnasio tiene lados de 8,7 L/100 km
longitud de x x por y y donde x y y son
las incertidumbres estimadas en las mediciones y son 33. La velocidad del sonido a la temperatura
pequeas comparadas con x e y. Demuestre por ambiente es 340 m/s. Exprese la velocidad del sonido
clculo directo que el rea del piso y la en unidades de millas por hora.
incertidumbre en esa rea estn dadas por Respuesta.
x y cuando se ignoran trminos 761 mi/h
A = xy xy +
x y
34. a) Cuntos milisegundos hay en un minuto?
muy pequeos, del orden de ( x)2. (En la mayor Cuntos gigasegundos hay en un siglo?
parte de los casos, este resultado sobrestima la Respuesta.
incertidumbre en el rea, porque no toma en a) 1 min = 60000 ms, b) 1 siglo = 3,16 Gs
consideracin que las incertidumbres en las
longitudes, x y y, provienen de una serie de 35. a) Calcule la altura de un cilindro de radio R que
medidas, que tienen una dispersin natural en sus tiene el mismo volumen de una esfera de radio R. b)
valores.) Demuestre que el cilindro tiene un rea superficial
mayor que la esfera.
29. Estime el espesor de las pginas de un libro. D Respuesta.
su resultado en milmetros. 4
Respuesta. h= R
Aproximadamente 0,06 mm 3

30. Alrededor de cuntos ladrillos se requieren para 36. Considere una esfera que se ajusta exactamente
construir una pared de altura hasta el hombro de 100 dentro de un cubo. Cul es la relacin del volumen
pies de largo? Los ladrillos estndar tienen 8 pulg de de la esfera al volumen del cubo?
largo por 2 1/4 pulg de alto y estn separados por 3/8 Respuesta.
de pulgada de mortero. /6
Respuesta.
3,3 x 103 ladrillos 37. Un vaso cilndrico para malteada tiene un radio
interior medido de r r y una altura de h h.
31. Cul es el volumen en milmetros cbicos de un Demuestre que el volumen del vaso es
cubo de 1,00 pulg por lado?
V = r 2 h 2hr r 2 h si se ignoran los
Respuesta.
trminos muy pequeos del orden (r )
2
1,64 x 104 mm3

16
Movimiento rectilneo Hugo Medina Guzmn

CAPITULO 2. Movimiento rectilneo


DEFINICIN DE PARTCULA. Segn la velocidad: Uniforme y uniformemente
El Punto Material variado.
Es una idealizacin de los cuerpos que existen en la Movimiento uniforme: La velocidad de movimiento
naturaleza y que llamamos punto material. Es un es constante
cuerpo cuyas dimensiones son despreciables al Movimiento uniformemente variado: La aceleracin
compararlas con las otras dimensiones que es constante, como es el caso de los cuerpos en cada
intervienen en el movimiento. libre sometidos a la aceleracin de de la gravedad.
La Mecnica comienza con el estudio de los puntos
materiales y despus extiende estos estudios a los SISTEMAS DE REFERENCIA. POSICIN Y
sistemas de puntos materiales, incluyendo cuerpos DESPLAZAMIENTO.
rgidos y deformables.
El punto material, a diferencia de un punto El movimiento es una nocin esencialmente relativa.
geomtrico, est asociado a una masa inercial; esta As resulta que el movimiento como el reposo son
propiedad est ntimamente ligada al movimiento de hechos relativos, no se puede decir que algo se
los cuerpos, como podemos ver cuando tratamos de mueve o que est en reposo sin aadir respecto a
entender cmo se mueven los cuerpos. qu. En consecuencia necesitamos un sistema de
referencia para descubrir el movimiento.
CONCEPTO DE MOVIMIENTO
El movimiento es un fenmeno fsico que se define Sistemas de referencia. Desde el punto de vista
como todo cambio de posicin que experimentan los estrictamente matemtico, un sistema de referencia
cuerpos en el espacio, con respecto al tiempo y a un en un espacio vectorial de dimensin n est formado
punto de referencia, variando la distancia de dicho por n vectores linealmente independientes,
cuerpo con respecto a ese punto o sistema de formando una base del espacio, y por un punto,
referencia, describiendo una trayectoria. Para definido por n coordenadas, que suele llamarse
producir movimiento es necesaria una intensidad de origen del sistema de referencia.
interaccin o intercambio de energa que sobrepase En el dominio de la fsica, el espacio suele ser la
un determinado umbral. base ms habitual la llamada ortonormal ( i , j ,
La parte de la fsica que se encarga del estudio del
movimiento es la cinemtica. k ), y el origen se sita a conveniencia del
observador. Los vectores de la base son
CLASIFICACIN DEL MOVIMIENTO
Segn se mueva un punto o un slido pueden
i = (1,0,0), j = (0,1,0) y k = (0,0,1).
distinguirse distintos tipos de movimiento: Atendiendo a su posible estado de reposo o
movimiento, los sistemas de referencia pueden ser
Segn la trayectoria del punto: Rectilneo y clasificados siempre y cuando hablemos de su
curvilneo relacin respecto a otro sistema de referencia que
Movimiento rectilneo: La trayectoria que describe arbitrariamente supongamos inmvil. En efecto,
el punto es una lnea recta. debe tenerse en cuenta que cualquier sistema de
Movimiento curvilneo: El punto describe una curva referencia est movindose respecto a otro (este
cambiando su direccin a medida que se desplaza. papel gira y se traslada con la Tierra alrededor del
Casos particulares del movimiento curvilneo son la Sol, el cual a su vez se desplaza en la galaxia, que a
rotacin describiendo un crculo en torno a un punto su vez se expande en el Universo...), por lo que no
fijo, y las trayectorias elpticas y parablicas. cabe hablar de un sistema de referencia absoluto.
De acuerdo con lo anterior, un sistema de referencia
Segn la trayectoria del slido: Traslacin y puede estar:
rotacin. a) en reposo respecto a otro

Traslacin: Todos los puntos del slido describen
trayectorias iguales, no necesariamente rectas. b) movindose con velocidad constante v respecto
Rotacin: Todos los puntos del slido describen al supuestamente fijo
trayectorias circulares concntricas. c) con una aceleracin respecto al fijo.

Segn la direccin del movimiento: Un buen ejemplo del primer caso podemos
Alternativo y pendular. encontrarlo en un sistema de referencia como la
Alternativo: Si la direccin del movimiento cambia, pizarra, que se encuentra en reposo relativo respecto
el movimiento descrito se denomina alternativo si es a las paredes del aula (en condiciones normales).
sobre una trayectoria rectilnea o pendular. Un ejemplo de sistema de referencia inercial
Pendular: Si lo es sobre una trayectoria circular (un podemos encontrarlo en un tren que se mueve en un
arco de circunferencia). tramo de va rectilneo con una velocidad
sensiblemente constante.

1
Movimiento rectilneo Hugo Medina Guzmn

Y por ltimo, la propia Tierra constituye un sistema una recta, circunferencia, espiral, parbola o curvas
de referencia no inercial, ya que gira con una tan complicadas como se nos ocurra.
aceleracin normal, que si bien es pequea, en La trayectoria no define el movimiento, pues no
ciertos fenmenos se observa con claridad. sabemos en que instante de tiempo ocup cada
punto. Sabemos dnde estuvo, pero no cuando y si
Vector Posicin.- Para fijar la posicin de un punto estuvo varias veces en cada punto o no. Hace falta la
en el espacio respecto a un origen de coordenadas ecuacin horaria.
bastan tres nmeros que pueden ser las proyecciones Para encontrar la ecuacin horaria debemos medir
sobre los ejes de un sistema cartesiano ortogonal. las distancias en funcin del tiempo.

En la figura P0 es un origen fijo sobre la curva (C)


que porta la trayectoria.
El vector posicin del punto P es: Sea P la posicin de la partcula en el instante t
sobre la trayectoria definida por el arco
OP = r
El movimiento quedar especificado si conocemos P0 P = S
el vector posicin para cada instante, es decir: La ecuacin horaria del movimiento de la partcula P

r = r (t ) es
Esto se conoce como ley de movimiento.
S = S (t )
El vector posicin puede ser expresado a travs de
las ecuaciones paramtricas de sus componentes en Ejemplo experimental. Estudio del movimiento de
funcin del tiempo: la cada libre de un cuerpo.
x = x(t ) , y = y (t ) , z = z (t ) Solucin.
Si dejamos caer un objeto, obtenemos que la

trayectoria sea una recta vertical.


r = x(t )i + y (t ) j + z (t )k Para encontrar la ley del movimiento podemos
intentar medir a partir de dnde la dejamos caer,
Desplazamiento. distancias sucesivas para diferentes tiempos.
La figura muestra una partcula que se est Una forma experimental es usando una pelcula
moviendo a lo largo de la trayectoria curvilnea C. fotogrfica y una flash electrnico que se encienda
por ejemplo cada 1/30 de segundo. En una
habitacin oscura dispondremos el cuerpo, la
pelcula y un disparador que deje caer el cuerpo y
simultneamente accione el flash. Paralelamente a la
trayectoria a seguir por el objeto se fija una regla.

Sean P1 y P2 las posiciones de la partcula en los


instantes t1 y t 2 = t1 + t . Los vectores posicin
correspondientes son

OP 1 y OP 2 = r 2 = r 1 + r .

Siendo r el vector desplazamiento y describe el
desplazamiento de la partcula de la posicin P1 a la
posicin P2.

Trayectoria y Ecuacin Horaria del Movimiento.-


Se llama trayectoria de una partcula en movimiento
al lugar geomtrico de las posiciones efectivamente
ocupadas por la partcula en el transcurso del
tiempo. De acuerdo al tipo de movimiento podr ser

2
Movimiento rectilneo Hugo Medina Guzmn


r = kt 2 k
Las ecuaciones paramtricas son
x = 0 , y = 0 y z = kt 2
En esencia para cualquier movimiento debemos
ingeniarnos para obtener la ecuacin horaria y
conocida su trayectoria, queda determinado el
movimiento.

VELOCIDAD Y RAPIDEZ
La fotografa mostrada permite conocer las cotas de
la foto en los diferentes instantes bien determinados. Rapidez. La rapidez (que en el lenguaje comn se
La tabla muestra los resultados de la fotografa: denomina simplemente velocidad) se define como el
cociente entre la distancia recorrida y el tiempo
Tiempo Cota(m) transcurrido. La distancia s recorrida a lo largo de
0,2480 una trayectoria es una magnitud escalar,
t0 independiente de la direccin. Como el tiempo
0,3250 tambin es un escalar, la rapidez es tambin un
t1
escalar.
t2 0,4130 La rapidez se designa mediante el smbolo v y sus
0,5130 dimensiones son:
[v] = LT -1
t3
t4 0,6235
La unidad en el sistema SI es el metro por segundo
t5 0,7450 (m/s).
La figura muestra una partcula que se est
t6 0,8875 moviendo a lo largo de la trayectoria curva C. En el
t7 1,0215 instante t1 esta en P1, a una distancia S1 de un

t8 1,1760 punto P0 de referencia. En el instante t 2 est en P2 a


1,3405 una distancia S 2 del punto de referencia.
t1
t 10 1,5155

Tracemos la curva representativa del la funcin


z = f (t )
En el tiempo que transcurre entre t1 y t 2 ,
t = t 2 t1 , la partcula ha recorrido una distancia
S es la diferencia entre S 2 y S1 , esto es
S = S 2 S 1 .
Se define como rapidez media dentro de este
intervalo
S 2 S 1 S
vm = =
Esta curva corresponde a una parbola y su t 2 t1 t
expresin matemtica es El smbolo (delta) significa un incremento de una
z = kt 2 magnitud fsica.
z est en segundos Si la rapidez de la partcula vara a lo largo de la
trayectoria, para conocer con mejor precisin el
m movimiento debemos hacer los intervalos S ms
Donde k = 4,9 2
s pequeos y tomar la rapidez media de cada uno de
ellos. La figura a continuacin nos muestra el
t est en segundos grfico distancia recorrida versus tiempo, observen
Luego la ecuacin horaria es que cuando t 2 tiende a t1 , t tiende a cero.
s = kt 2 Mediante este proceso llamamos a la rapidez
Si fijamos el origen del movimiento en z = 0, la ley instantnea v en el instante t. Este proceso se
del movimiento es expresa matemticamente como

3
Movimiento rectilneo Hugo Medina Guzmn

S dS S (t ) = Asen (t )
v = lim =
t 0 t dt Solucin.
En el intervalo de tiempo de t hasta t + t la
partcula que se mueve:
S = S (t + t ) S (t )
= Asen (t + t ) Asent
= Asent cos(t ) + Acostsen(t )
- Asent
La rapidez en un instante t cualquiera es
dS
La cantidad se llama derivada de S con S
dt v = lim
t 0 t
respecto a t y el proceso de encontrarla se llama
= lim Asen t cos (t ) + Acostsen (t ) Asent
derivacin o diferenciacin. La notacin dS , dt , t 0 t
expresa incrementos infinitesimalmente pequeos v = A cos t
que se conocen como diferenciales.
El proceso desarrollado en los dos ejemplos
Ejemplo 1. anteriores se hace simple con la prctica.
a) Hallar una expresin para la rapidez de una Hay muchas reglas o frmulas para derivar
partcula que se mueve de acuerdo a la ley horaria diferentes tipos de funciones. Estas pueden
S = At 2 memorizarse o encontrarse en tablas. La tabla
2
b) Si A = 1,4 m/s , hallar la distancia a la que se siguiente es una pequea muestra de estas.
encuentra la partcula y su rapidez para 10 segundos
despus de iniciado su movimiento. Derivadas de algunas funciones
Solucin.
Funcin Derivada
a) Si en el tiempo t est en S(t ) :
S =t n
dS
= nt n 1
S(t ) = At 2 dt
Transcurrido un tiempo t , la partcula estar en S =c dS
S(t + t ) =0
dt
S(t + t ) = A(t + t ) S = cu
2
dS du
=c
At 2 + 2 At t + A(t ) ,
2 dt dt
=
S =u+v dS du dv
Como = +
S = S(t + t ) St dt dt dt
S = uv dS du dv
= At 2 + 2 At t + A(t ) At 2 =v +u
dt dt dt
= 2 At t + A(t )
2
S = Asent dS
La rapidez en el instante t es: = Acost
dt
S
v(t ) = lim S = Acost dS
t 0 t = Asent
2 Att + A(t )
2 dt
= lim = 2 At
t 0 t
b) Para t = 10 es Ejemplo 3. Hallar una expresin para la rapidez de
una partcula que se mueve de acuerdo a la ley
m
S(10 ) = 1,4 2 (10s ) = 140 m
2
horaria S = At , usando frmulas de la tabla
2

s anterior.
y su rapidez es Solucin.
m m Tenemos que:
v(10 ) = 21,4 2 (10s ) = 28 2
( )
2

s s dS d At 2 dt 2
v= = =A = 2 At
dt dt dt
Ejemplo 2. Hallar una expresin para la rapidez de
una partcula que se mueve segn la ecuacin Ejemplo 4. Hallar una expresin para la rapidez de
horaria una partcula que se mueve de acuerdo a la ley

4
Movimiento rectilneo Hugo Medina Guzmn

horaria S (t ) = Asen (t ) , usando frmulas de la


La magnitud del vector velocidad instantnea, v ,
tabla anterior. r
Solucin. es decir v o simplemente v es igual a la rapidez
Tenemos que instantnea en ese punto.
dS d ( Asent ) dsent La velocidad es la pendiente del grfico de x versus
v= = =A = A cos t
dt dt dt t, como se muestra en la figura.

Velocidad. La velocidad (que ms apropiadamente


sera vector velocidad), a diferencia de la rapidez
debemos incluir el concepto de direccin en nuestro
estudio; para esto debemos emplear vectores.
La figura muestra una partcula que se est
moviendo a lo largo de la trayectoria curvilnea C.

Cuando la pendiente es positiva, el objeto se est


moviendo a la derecha.
Cuando la pendiente es negativa, el objeto se est
moviendo a la izquierda.
Cuando la pendiente es cero, el objeto se detiene.

Sean P1 y P2 las posiciones de la partcula en los Ejemplo 5. Entre dos observadores hay una
instantes t1 y t 2 = t1 + t . Los vectores posicin distancia de 1050 m, uno de ellos dispara un arma de
fuego y el otro cuenta el tiempo que transcurre desde
correspondientes son OP1 = r 1 y que ve el fogonazo hasta que oye el sonido,
obteniendo un valor de 3 s. Despreciando el tiempo
OP 2 = r 2 = r 1 + r . Siendo r el vector empleado por la luz en hacer tal recorrido, calcular
desplazamiento y describe el desplazamiento de la la velocidad de propagacin del sonido.
partcula de la posicin P 1 a la posicin P 2 .

Velocidad media. El cociente entre el vector



desplazamiento r y el intervalo de tiempo t es
el vector velocidad media. Solucin.
La velocidad es:
r c = s/t = 1050/3 = 350 m/s
vm =
t
Como el desplazamiento es un vector y el tiempo es Ejemplo 6. Nos encontramos en una batalla naval,
un escalar positivo, la velocidad es una magnitud en un buque situado entre el enemigo y los
vectorial que tiene la misma direccin y sentido que acantilados de la costa. A los 3 s de ver un fogonazo
el desplazamiento. Esto significa que si una omos el disparo del can, y a los 11 s del fogonazo
partcula sufre un desplazamiento negativo, su percibimos el eco. Calcular la distancia a que estn
velocidad ser tambin negativa. de nosotros el enemigo y la costa. Velocidad del
sonido, 340 m/s.
Velocidad instantnea. Como en el caso de la

rapidez obtendremos la velocidad instantnea v
tomando la velocidad media en un intervalo de
tiempo cada vez menor t medido desde un cierto
tiempo t1 . En el lmite, cuando t tiende a cero:
Solucin.

r 2 r1 r d r
v (t1 ) = lim
Despreciando el tiempo empleado por la luz en su
= lim = recorrido, la distancia a que se encuentra el enemigo
t 2 t1 t t 0 t dt
es:
S = 340 x 3 = 1020 m
La direccin de este vector es la direccin lmite del
El sonido emplea para ir y volver a la costa, desde
vector cuando t 0 de la figura anterior. Es nuestra posicin, un tiempo que es:

t = 11 - 3 = 8 s 2S= 340 x 8 S = 1360 m
evidente que en este lmite la direccin de r es la
de la tangente la trayectoria en P1. La costa est a 1020 + 1360 = 2380m.

5
Movimiento rectilneo Hugo Medina Guzmn

x dx
Ejemplo 7. La posicin de una partcula en b) de la ecuacin v x = lim = , la velocidad
t 0 t dt
coordenadas cartesianas est dada por la ecuacin
instantnea en funcin del tiempo es
r (t ) = x(t )i + y (t ) j + z (t )k v x = 2bt 3ct 2 = (4,80)t (0,360)t 2
donde tal que
x(t ) = 5 + 6t 2 , y (t ) = 3t , z (t ) = 6 i) v x (0) = 0,
t en segundos, x, y, z en metros.
ii) v x (5) = (4,80 )(5) (0,360)(5) = 15,0 y
2
a) Determinar el desplazamiento entre t = 0 y t = 1 s.
b) Determinar la velocidad media iii) v x (10) = ( 4,80)(10) (0,360)(10) = 12,0
2

c) Determinar la velocidad y la rapidez para t = 1 s.


Solucin. c) el auto est en reposo cuando v x = 0 .
a) para t = 0 s, x = 5m, y = 0m, z = 6m
Por consiguiente ( 4,80)t (0,360)t = 0 .
2

r 0 = 5i + 6k El nico tiempo despus de t = 0 en que el auto se
Para t = 1s, x = 11m, y =3m, z = 6m 4,8
encuentra en reposo es t = = 13,3 s
r 1 = 11i + 3 j + 6k 0,360
El desplazamiento es
Ejemplo 9. Un ciclista marcha por una regin
r = r 1 r 0 = (11 5)i + (3 0) j + (6 6 )k donde hay muchas subidas y bajadas
En las cuestas arriba lleva una rapidez constante de 5
= 6i + 3 j
km/h y en las cuestas abajo 20 km/h. Calcular:
b) la velocidad media es a) Cul es su rapidez media si las subidas y bajadas

tienen la misma longitud?
r 6i + 3 j
vm = = = 6i + 3 j b) Cul es su rapidez media si emplea el mismo
t 1 0 tiempo en las subidas que en las bajadas?
c) la velocidad instantnea es c) Cul es su rapidez media si emplea doble tiempo

[( ) ]
en las subidas que en las bajadas?
d r d 5 + 6t 2 i + 3tj + 6k Solucin.
v= =
dt dt stotal ssubida + sbajada
a) vm = =
= 12ti + 3 j
ttotal ttotal
La magnitud de v es 2v1v 2
2s
v = 12 2 + 32 = 153 = = = 8 km / h
s s v1 + v 2
= 12,4 m/s +
Valor que corresponde a la rapidez instantnea para
v1 v 2
t = 1s. v t + v2t v1 + v2
b) vm = 1 =
2t 2
Ejemplo 8. Un auto est parado ante un semforo.
= 12,5 km/h
Despus viaja en lnea recta y su distancia respecto
al semforo est dada por x(t) = bt2 - ct3 , donde b = v1 2t + v2t 2v1 + v2 2 5 + 20
c) vm = = =
2,40 m/s2 y c = 0,120 m/s3. 3t 3 3
a) Calcule la velocidad media del auto entre t = 0 y = 10 km/h
t = 10,0 s. (Obsrvese que la rapidez media es la media
b) Calcule la velocidad instantnea en aritmtica de las rapideces uniformes nicamente en
i) t = 0; ii) t = 5,0 s; iii) t = 10,0 s. el caso de que el tiempo que duran los distintos
c) Cunto tiempo despus de arrancar vuelve a recorridos sea el mismo).
estar parado el auto?
Solucin. ACELERACIN
a) En t1 = 0, x1 = 0 , tal que la ecuacin En el lenguaje ordinario el trmino aceleracin se
refiere slo a incrementos del mdulo de la
x 2 x1 x
vm = = velocidad (rapidez), pero en Fsica se utiliza con un
t 2 t1 t sentido ms amplio para designar un cambio del
vector velocidad. En Fsica se dice que un cuerpo
x2 (2,4)(10) (0,120 )(10)
2 3
est siendo acelerado no slo cuando aumenta su
vm = =
t2 (10) velocidad sino tambin cuando disminuye o cambia
de direccin.
= 12,0 m/s
Se llama aceleracin al cambio de la velocidad
(vector velocidad) en el tiempo.

6
Movimiento rectilneo Hugo Medina Guzmn

Observe que la aceleracin negativa no significa


Aceleracin Media. necesariamente bajar la velocidad.
Cuando la velocidad y la aceleracin ambas tienen el
mismo signo, el objeto aumenta su velocidad.
Cuando la velocidad y la aceleracin tienen signos
opuestos, el objeto disminuye su velocidad.
Los grficos de la figura siguiente ilustran el
desplazamiento, la velocidad, y la aceleracin para
un objeto en movimiento.

La razn en la cual la velocidad cambia se mide por


la aceleracin. As si un objeto tiene la velocidad

v 1 en el t1 del tiempo y velocidad v 2 en el t 2 , su
aceleracin media es

v 2 v 1 v v
am = = = i
t 2 t1 t t
Supongamos que una partcula que se mueve en la
trayectoria C de la figura anterior en el instante t1
est en P1 con una velocidad v1 y en el instante
t 2 = t1 + t est en P2 con una velocidad v 2 . Por
definicin el vector aceleracin media de la partcula Ejemplo 10. Una partcula se mueve a lo largo de
una lnea curva
entre los instantes es t1 y t 2 es
( ) ( )

r (t ) = t 2 + t i + (2t 1) j + t 3 2t 2 k
v 2 v1 v
am = = Encontrar:
t2 t1 t a) La velocidad para t = 1 s y para t = 3 s .
[]
Las dimensiones de la aceleracin son a = LT
2 b) La aceleracin media entre t = 1 s y para t = 3 s .
c) La aceleracin y su magnitud para t = 1 s .
La unidad de la aceleracin en el sistema SI est en Solucin.
metros / segundo por segundo: a) Las ecuaciones paramtricas son:
ms m
= 2 x(t ) = t 2 + 1 , y (t ) = 2t 1 , z (t ) = t 3 2t 2
s s Las componentes de la velocidad son:
dx dy
Aceleracin Instantnea o simplemente vx = = 2t + 1 , v y = = 2.
aceleracin. Cuando t 2 t1 o t 0 dt dt
llegaremos al valor de la aceleracin en el instante dz
vz = = 3t 2 + 4t
t1 . Este proceso para el lmite se expresa dt
matemticamente como La velocidad es:

( )

v (t ) = (2t + 1)i + 2 j + 3t 2 4t k

v 2 v1 v d v
a (t1 ) = lim = lim =
t 2 t1 t t t 0 t
2 1 dt Para t = 1 s : v (1) = 3i + 2 j k



Como v =
dr
, tenemos: Para t = 3 s . v (3 ) = 7i + 2 j + 15k
dt b) La aceleracin media entre t = 1 s y t = 3 s .


d v d d r d2 r v v (3) v (1)
a= = = am = = =
dt dt dt dt 2 t 3 1

Es mejor evitar el uso de la palabra comn (7 3)i + (2 2) j + (15 + 1)k
desaceleracin. 2
Describa la aceleracin simplemente como positiva
o negativa. a m = 2i + 8k

7
Movimiento rectilneo Hugo Medina Guzmn

c) la aceleracin instantnea es

r x x x0
v = vi = = i=

a (t ) =
dv d
dt
=
dt
[ (
(2t + 1)i + 2 j + 3t 2 4t k )] t t t t0
i

x x x0
= 2i + (6t 4 )k v= = = tan
t t t0
para t = 1s

a (1) = 2i + 2k Diagrama velocidad-tiempo


la magnitud de la aceleracin es
a(1) = 2 2 + 2 2 = 2 2 m s 2

Ejemplo 11. Una persona que se asoma por la


ventana de un edificio alto de oficinas observa lo
que sospecha es un ovni. La persona registra la
posicin del objeto en funcin del tiempo y El grfico velocidad-tiempo del movimiento
uniforme es una recta paralela al eje del tiempo.
determina que est dada por
( )

r (t ) = 5,0t i + 10,0t j + 7,0t 3,0t 2 k x x0


a) Obtenga los vectores de: desplazamiento, De v = x = x 0 + v(t t 0 )
velocidad y aceleracin del objeto en t = 5,0 s.
t t0
b) Hay algn tiempo en que la velocidad del objeto Si el instante inicial t 0 = 0 , tenemos
sea cero?
c) La aceleracin del objeto es constante o cambia x = x0 + vt
con el tiempo?
Solucin. Diagrama espacio-tiempo
a) El vector desplazamiento es:
( )

r (t ) = 5,0t i + 10,0t j + 7,0t 3,0t 2 k


El vector velocidad es la derivada del vector
desplazamiento:

d r (t )
= 5,0i + 10,0 j + [7,0 2(3,0)t ]k
dt
y el vector aceleracin es la derivada del vector El grfico indica las posiciones instantneas del
velocidad: mvil en cada instante

d 2 r (t )
= 6,0 k
dt 2 MOVIMIENTO RECTILNEO
en t = 5,0 s: UNIFORMEMENTE VARIADO.

[
r (5 ) = 5,0(5)i + 10,0(5) j + 7,0(5) 3,0(5) k
2
] Para que un movimiento sea rectilneo
= 25,0i + 50,0 j 40,0k uniformemente variado su aceleracin debe ser
constante y diferente de cero.
d 2 r (5)
= 6,0 k
dt 2 Estudio del Movimiento

b) la velocidad en ambas direcciones x e y es Como la aceleracin es constante, a m =a
constante y diferente de cero, luego la velocidad dv v
nunca puede ser cero a= = = constante
c) La aceleracin del objeto es constante, ya que t no dt t

aparece en el vector aceleracin. v v
a = ai = = i
MOVIMIENTO RECTILNEO UNIFORME. t t
Para que un movimiento sea rectilneo uniforme su v v v0
velocidad debe ser constante, es decir, que la a= =
aceleracin sea siempre igual a cero. t t t 0
v v 0 = a(t t 0 )
Estudio del Movimiento
Si el tiempo inicial t 0 =0
Como el movimiento es uniforme v m = v , y
considerando que su trayectoria est en el eje x
v = v0 + at

8
Movimiento rectilneo Hugo Medina Guzmn

Diagrama velocidad-tiempo

v v v0
a= = = tan Ejemplo 12. Demostrar que el rea encerrada bajo la
t t t 0 curva de la velocidad del diagrama velocidad-tiempo
La velocidad media: es igual al mdulo del desplazamiento

Si la posicin en t 0 es r 0 = x 0 i y la posicin en t
x = x x 0 .

es r = xi , la velocidad media en este intervalo es
x x x0
vm = =
t t t0

La posicin.
De lo anterior:
x x0 = vm (t t0 )
y x = x0 + vm (t t0 ) Solucin.
El rea encerrada es igual al rea de un trapecio
Por otra parte como la velocidad es una funcin
cuyas bases son b1 = v y b2 = v 0 con altura
lineal, la velocidad media v m es
h = (t t 0 ) .
v + v0
vm =
2 (b1 + b2 )
Area del trapecio = h
y como v = v 0 + a(t t 0 ) 2
(v + v0 )
resulta = (t t 0 )
v 0 + [v 0 + a (t t 0 )] a(t t 0 ) 2
vm = = v0 +
1
2 2 = v0 (t t 0 ) + (v v 0 )(t t 0 )
finalmente 2
a(t t 0 ) (v v0 )
x = x 0 + v 0 + (t t 0 ) Pero como a = tan =
2 (t t 0 )
1 (v v0 ) = a(t t 0 )
x = x 0 + v0 (t t 0 ) + a (t t 0 )
2

2 Luego
1
Area del trapecio = v0 (t t 0 ) + a (t t 0 )
2
Si el tiempo inicial t 0 =0 2
1 2 Valor que precisamente corresponde al
x = x0 + v0 t + at desplazamiento x = x x 0 .
2
Diagrama espacio -tiempo
LA ECUACIN DE TORRICELLI.
Podemos obtener una relacin muy til eliminando
el tiempo como variable en la ecuacin
1
x = x 0 + v0 (t t 0 ) + a (t t 0 )
2

2
(v v0 ) (v v0 )
Como a = (t t 0 ) =
(t t 0 ) a

9
Movimiento rectilneo Hugo Medina Guzmn

Sustituyendo
(v v0 ) 1 (v v0 )
2

x = x0 + v0 + a
a 2 a2
De donde se puede despejar: Los diagramas aceleracin-tiempo, velocidad-
tiempo y espacio-tiempo correspondientes son los
v = v + 2a ( x x 0 )
2 2
0 siguientes:
Conocida como la ecuacin de Torricheli.

Descripcin del movimiento de una partcula con


aceleracin constante.

Consideramos una aceleracin constante a > 0 en a 0 = constante


el sentido positivo de la trayectoria.

1er Caso:
La partcula tiene una velocidad inicial v0 0.
La partcula se desplaza de P 0 al infinito con un
sentido constante y aumentando su velocidad.

v = v 0 + at

Los diagramas aceleracin-tiempo, velocidad-


tiempo y espacio-tiempo correspondientes son los
siguientes:

1
x = x0 + v0 t + a0 t 2
a 0 = constante 2
Ejemplo 13. Una tortuga camina en lnea recta
sobre lo que llamaremos eje x con la direccin
positiva hacia la derecha. La ecuacin de la posicin
de la tortuga en funcin del tiempo es
x(t) = 50,0 cm + (2,00 cm/s)t - (0,0625 cm/s2)t2 .
v = v 0 + at a) Determine la velocidad inicial, posicin inicial y
aceleracin inicial de la tortuga.
b) En qu instante t la tortuga tiene velocidad cero?
c) Cunto tiempo despus de ponerse en marcha
regresa la tortuga al punto de partida?
d) En qu instantes t la tortuga est a una distancia
de 10,0 m de su punto de partida? Que velocidad
(magnitud y direccin) tiene la tortuga en cada uno
1 de esos instantes?
x = x0 + v0 t + a0 t 2 e) Dibuje las grficas: x-t, vx-t y ax-t para el intervalo
2 de t = 0 a t = 40,0 s.
2do. Caso: Solucin.
La partcula tiene una velocidad inicial v 0 < 0. dx
vx = = 2,00 cm s (0,125 cm s 2 )t
La partcula se desplaza de P 0 en sentido negativo dt
con movimiento retardado (desacelerado) hasta dv
ax = x = 0,125 cm s 2
detenerse en P 1 y cambia de sentido. A partir de ese dt
instante la velocidad aumenta constantemente a) En t = 0, x = 50,0 cm, v x = 2,00 cm s ,
(acelerado) y se desplaza al infinito con un sentido
constante. a x = 0,125 cm s 2 .
b) Hagamos vx = 0 y resolvamos para t:
t = 16,0 s

10
Movimiento rectilneo Hugo Medina Guzmn

c) Hagamos x = 50,0 cm y resolvamos para t. t2 +10t -11 = 0 t = 1 s


Esto da: t = 0 y t = 32,0 s . v D = vC + at = 50 + 10 = 60 cm/s
La tortuga regresa al punto de partida despus de
32,0 s. b) v B = 2aAB
d) La tortuga est a 10,0 cm del punto de partida vB2 400
cuando x = 60,0 cm o x = 40,0 cm. AB = = = 20cm
Hagamos x = 60,0 cm y resolvamos para t: 2a 20
t = 6,20 s y t = 25,8 s c)
En t = 6,20 s, vx = + 1,23 cm/s. v B = at 20
En t = 25,8 s, vx = - 1,23 cm/s. t= =2 s
Hagamos x = 40,0 cm y resolvamos para t :
20 = 10t 10
d) Ser la suma de los tiempos parciales:
t = 36,4 s t = 2 + 3 +1 = 6 s
(la otra raz de la ecuacin cuadrtica es negativa y
por lo tanto sin significado fsico). MOVIMIENTO VERTICAL CON
En t = 36,4 s, vx = - 2,55 cm/s. ACELERACIN DE LA GRAVEDAD.
e)
La variacin de la magnitud de la aceleracin g
debido a la gravedad en la superficie de la tierra con
la latitud est dada por la frmula internacional de la
gravedad adoptada en 1930 por el Congreso
Geofsico Internacional:
g = 978,049000 (1 + 0,0052884 sen 2
- 0,0000059 sen 2 )
2

g en cm/s ,
2
en grados
Donde es la latitud de la tierra medida en el
ecuador
Para = 0 (ecuador), g 0 = 978,0490
Ejemplo 14. Un mvil parte del reposo y de un
punto A, con movimiento rectilneo y Para = 90 (polos), g 90 = 983,2213
uniformemente acelerado (a =10 cm/s2); tarda en
recorrer una distancia BC = 105 cm un tiempo de 3 La variacin de la aceleracin gravitacional con la
s, y, finalmente, llega al punto D (CD = 55 cm). altura sobre el nivel del mar es aproximadamente
Calcular:
a) La velocidad del mvil en los puntos B, C y D. g = g 0,000002860h
b) La distancia AB. h en metros y g en m/s
2
c) El tiempo invertido en el recorrido AB y en el
CD. Donde h 40 000 m
d) El tiempo total en el recorrido AD.
Cerca de la superficie de la tierra la magnitud de la
aceleracin debido a la gravedad vara muy poco
con la altura y en los clculos tcnicos ordinarios se
toma g = 9,81 m/s2 (dirigido verticalmente hacia
abajo).
Solucin.
a) Un cuerpo que se deja caer est sometido a la
1 2 aceleracin de la gravedad y su movimiento
BC = v B t + at corresponde a un movimiento rectilneo
2
vB =20 cm/s uniformemente variado en el eje vertical
1 2 perpendicular a la tierra,
105 = v B 3 + 10 3
2 1 2
y = y 0 + v0 t + at
vC = v B + at = 20 + 30 = 50 cm/s 2
v = v 0 + at
1
CD = vC t + at 2 a = g
2

1 2 a) Cada libre
55 = 50t + 10t
2

11
Movimiento rectilneo Hugo Medina Guzmn

1 2v
h = h + v0 t + gt 2 t P = 0 y por
2 g
supuesto t P = 0 , que corresponde al tiempo inicial.
Observamos que t P = 2t m
La velocidad es
2v
v P = v0 g 0 = v0 2v0 = v 0
Si se deja caer un cuerpo desde una altura h sobre el g
nivel del piso y consideramos despreciable la Finalmente toca piso cuando y = 0
resistencia del aire.
En este caso y 0 = h , v 0 = 0 , luego: 1 2v 2h
h + v 0 t gt 2 = 0 t 2 0 t =0
1 2 2 g g
y =h gt cuya solucin es
2
v = gt v0 v02 + 2h
a = g t=
g g
El cuerpo toca tierra cuando y = 0 toca el piso al tiempo
1 2 2h v02 + 2h
Luego h gt = 0 t = v0
2 g t= +
g g
y la velocidad es v = 2 gh con una velocidad
v = v02 + 2 gh
b) Lanzamiento hacia arriba
Si el mismo cuerpo desde la misma altura h se lanza
c) Lanzamiento hacia abajo
hacia arriba con velocidad v 0 , se mueve con un Si el mismo cuerpo desde la misma altura h se lanza
movimiento rectilneo uniformemente retardado hacia abajo con una velocidad v 0 , el cuerpo se
(desacelerado).
mueve en un movimiento rectilneo uniformemente
acelerado.

1
y = h + v 0 t + gt 2
2
1 2
v = v0 gt y = h v0 t gt
a = g 2
El cuerpo sube hasta que alcanza la altura mxima v = v0 gt
y m . Esta corresponde a cuando la velocidad a = g
disminuye a cero. y = 0.
El cuerpo alcanza el piso cuando
v0 1 2v 2h
v 0 gt = 0 t m = h v0 t gt 2 = 0 t 2 + 0 t =0
g 2 g g
2 cuya solucin es
v 1 v
De aqu y m = h + v0 0 + + g 0 v02 + 2h
g 2 g v0
t=
v02 g g
= h+ toca el piso al tiempo
2
Cuando el cuerpo pasa por el punto de lanzamiento v0 v02 + 2h
y=h t= +
g g
con una velocidad

12
Movimiento rectilneo Hugo Medina Guzmn

y2 = h - 30(t - 2) - 5(t - 2)2 = 0


v = v02 + 2 gh De aqu t = 4 s;
h = 80m
Ejemplo 15. Desde lo alto de un edificio, se lanza
verticalmente hacia arriba una pelota con una Ejemplo 19. Desde el piso, se lanza hacia arriba una
rapidez de 12,5 m/s. La pelota llega a tierra 4,25 s, pelota con una rapidez de 40 m/s. Calcule:
despus. Determine: a) El tiempo transcurrido entre los dos instantes en
a) La altura que alcanz la pelota respecto del que su velocidad tiene una magnitud de 2,5 m/s.
edificio. b) La distancia respecto al piso que se encuentra la
b) La rapidez de la pelota al llegar al suelo. pelota en ese instante.
Solucin. Solucin.
La altura en funcin del tiempo ser 1 2
1 y = v0t gt (1)
y = h + v0 t gt 2 2
2 v y = v0 gt ( 2)
Con g = 10m/s2, v0 = 12,5 m/s
y = h + 12,5t - 5t2
a) Al tiempo t = 4,25 s, y = 0, luego: a) De la ecuacin (2):
h + 12,5(4,25) - 5(4,25)2 = 0, v y = v0 gt1 = 2,5
h = 37,19 m
b) vy = 12,5 - 10t = 12,5 - 10(4,25) v y = v0 gt 2 = 2,5
= -30,0 m/s Restando obtenemos:
5
Ejemplo 16. Se deja caer un cuerpo desde una altura t = t2 t1 = = 0,5s
de y0 = 33 m, y simultneamente se lanza hacia g
abajo otro cuerpo con una rapidez inicial de 1 m/s. b) De la ecuacin (2):
Encontrar el instante en que la distancia entre ellos v y = v0 gt1 = 2,5
es de 18 m.
Solucin. 40 gt1 = 2,5
y1 = 33 - 5t2 37,5
y2 = 33 - t - 5t2 t1 = = 3,83 s.
y1 - y2 = t 9,8
Entonces la distancia entre ellos es 18m a los 18 s Con t1 en (1):
1
h = 40(3,83) g (3,83) = 81,41 m.
2
Ejemplo 17. Un cuerpo que cae, recorre en el ltimo
segundo 68,3 m. Encontrar La altura desde donde
2
Con t2 se obtiene la misma altura, porque es cuando
cae.
la pelota est de bajada.
Solucin. Suponiendo que se solt del reposo
y = h - 5t2
Ejemplo 20. Una roca cae libremente recorriendo la
h segunda mitad de la distancia de cada en 3(s).
El tiempo en que llega al suelo es t= Encuentre
5
La distancia recorrida en el ltimo segundo ser a) la altura desde la cual se solt.
b) El tiempo total de cada.
h h Solucin.
y 1 y =
5 5 1 2
y =h gt
2 2 2
h h
5 5 1 = 68,2 h
5 5 El tiempo en que alcanza h/2 es t1 = y el
g
h = 268,6 m
2h
Ejemplo 18. Desde lo alto de un acantilado, se deja
tiempo en que h = 0 es t2 =
g
caer una piedra, desde la misma altura se lanza una
segunda piedra 2 s ms tarde con una rapidez de 30 a) por lo tanto el tiempo empleado en la segunda
m/s. Si ambas golpean el piso simultneamente. parte de recorrido es
Encuentre: La altura del acantilado. 2h h
Solucin. = 3 h = 524,6 m
g g
y1 = h - 5t2
y2 = h - 30(t - 2) - 5(t - 2)2 2h 524,6
Siendo al mismo tiempo b) t= = = 10,2 s
y1 = h - 5t2 = 0
g 5

13
Movimiento rectilneo Hugo Medina Guzmn

Ejemplo 21. Se deja caer una piedra desde un globo


que asciende con una velocidad de 3 m/s; si llega al
suelo a los 3 s, calcular:
a) Altura a que se encontraba el globo cuando se
solt la piedra.
b) Distancia globo-piedra a los 2 s del lanzamiento.

Solucin.
Primer mtodo:
En el instante en que empieza a caer el cuerpo el
ascensor lleva una velocidad vertical hacia arriba v.

Solucin. Tomaremos el origen de coordenadas en El espacio vertical y hacia abajo que debe recorrer la
el punto en que se suelta la piedra. Magnitudes lmpara es:
positivas son las que tienen direcci6n hacia arriba. 1
a) h vt + at 2
2
v0 = 3m/s (h = altura del ascensor) y (vt + at2/2) ascenso del
1
g 10m/s 2 y = h + 3t 10t 2 suelo de ste. La lmpara al desprenderse lleva una
2 velocidad inicial hacia arriba v. Aplicando la
t = 3s ecuacin:
Cuando la piedra toca suelo, y = 0 1 2
Luego s = vt + at
2
1
h = 3(3) 10(3) Siendo positivas las magnitudes hacia arriba y
2

2 negativas las descendentes, tendremos:


= 36 m 1 2 1
b) h + vt + at = vt gt 2
2 2
t = 2 s.
2h 23
t= = = 0,74 s
h1: distancia al origen del globo en t'. g+a 9,8 + 1
h2: distancia al origen de la piedra en t'.
Segundo mtodo:
h1 = v 0 t ' = 3 2 = 6m
La aceleracin de la lmpara respecto al ascensor,
1 1 considerando magnitudes positivas hacia abajo, es:
h2 = v 0 t '+ gt ' 2 = 3 2 10 40 14m
2 2 aBA = aB - aA = 9,8 (-1) = 10, 8 m/s2
d = 6 + 14 1
= 20 m h= a BA t 2
2
Ejemplo 22. La cabina de un ascensor de altura 3 m 2h 23
asciende con una aceleracin de 1 m/s2. Cuando el t= = = 0,74 s
a BA 10,8
ascensor se encuentra a una cierta altura del suelo, se
desprende la lmpara del techo. Calcular el tiempo
que tarda la lmpara en chocar con el suelo del Ejemplo 23. Una bola es lanzada verticalmente
ascensor. hacia arriba con una velocidad de 20 m/s de la parte
alta de una torre que tiene una altura de 50 m. En su
vuelta pasa rozando la torre y finalmente toca la
tierra.
a) Qu tiempo t1 transcurre a partir del instante en
que la bola fue lanzada hasta que pasa por el borde
de la torre? Qu velocidad v1 tiene en este tiempo?
b) Qu tiempo total t 2 se requiere para que la bola
llegue al piso? Cul es la velocidad v 2 , con la que
toca el piso?

14
Movimiento rectilneo Hugo Medina Guzmn

c) Cul es la mxima altura sobre el suelo Ejemplo 24. Una maceta con flores cae del borde de
alcanzada por la bola? una ventana y pasa frente a la ventana de abajo. Se
d) Los puntos P1 y P2 estn a 15 y 30 m, puede despreciar la resistencia del aire. La maceta
respectivamente, por debajo del techo de la torre. tarda 0,420 s en pasar por esta ventana, cuya altura
Qu tiempo se requiere para que la bola viaje de P1 es de 1,90 m. A qu distancia debajo del punto
a P2? desde el cual cay la maceta est el borde superior
e) Se desea que despus de pasar el borde, la bola de la ventana de abajo?
alcance la tierra en 3s, con qu velocidad se debe Solucin.
lanzar hacia arriba de la azotea? Si la velocidad de la maceta en la parte superior de
la ventana es v 0 , podemos encontrarla en funcin
de la altura h de la ventana y el tiempo que tarda en
pasarla::
1 2 2h gt 2
h = v0 t + gt v 0 =
2 2t
2(1,90 ) (9,8)(0,42)
2
m
Luego: v 0 = = 2,47
2(0,42 ) s
La distancia y desde la azotea al borde superior de
la ventana es:
Solucin. v02 2,47 2
y= = = 0,311 m
2 g 2(9,8)
a) Para el sistema de coordenadas mostrado en la
1 2
figura, y = v 0 t + at . Otra forma de encontrar la distancia es: como
2 t = 0,420 s es la diferencia entre los tiempos
Pero en el borde del techo y = 0, luego tomados en caer la las alturas ( y + h ) e y ,
1 2 tenemos
0 = v0 t1 + at1 ,
2 2( y + h) 2y
De la cual t1 = 0, indica el instante en el cual la bola t=
es lanzada, y tambin t1 = 4,08 s, la cual es el g g
tiempo en que la bola retorna al borde.
gt 2
Luego, de v = v 0 + at + y= y+h
2
v1 = 20 + ( 9,8)(4,08) = 20m / s , que es el Elevando al cuadrado:
negativo de la velocidad inicial.
gt 2
1 + 2 gyt 2 + y = y + h
b) 50 = 20t2 + ( 9,8)t22 t2 = 5,8 s 2
2
v2 = 20 + ( 9,8)(5,8) = 37 m / s gt 2
+ 2 gyt 2 = h
c) Mxima altura sobre tierra: h = y max + 50 .
2
Resolviendo para y :
De v 0 + 2ay max = 0 ,
2
2
1 2h gt 2
(20)
2 y=
ymax = = 20,4 m 2 g 2t
2(9,8) Con los datos
Luego, h = 70,4 m.
1 2(1,9 ) (9,8)(0,42)
2 2
d) Si t1 y t2 son los tiempos para alcanzar P1 y P2,
y= = 0,311 m
respectivamente, 2(9,8) 2(0,42)
15 = 20t1 4,9t12 y 30 = 20t 2 4,9t 22
Resolviendo, t1 = 4,723 s, t2 = 5,248 s, y el tiempo Ejemplo 25. Malabarismo. Un malabarista acta
de P1 a P2 es (t2 - tl) = 0,525 s. en un recinto cuyo cielorraso est 3,0 m arriba del
e) Si v0 es la velocidad inicial deseada, entonces v0 nivel de las manos. Lanza una pelota hacia arriba de
es la velocidad cuando pasa el borde. Luego modo que apenas llega al techo.
1 2 a) Qu velocidad inicial tiene la pelota?
aplicando y = v 0 t + at al viaje hacia abajo de b) Cunto tiempo tarda la pelota en llegar al techo?
2
En el instante en que la primera pelota est en el
la torre, encontramos:
cielorraso, el malabarista lanza una segunda pelota
-50 = (- v0)(3) 4,9(3)2, v0 = 1,96 m/s.
hacia arriba con dos terceras parte de la velocidad
inicial de la primera.

15
Movimiento rectilneo Hugo Medina Guzmn

c) Cunto tiempo despus de lanzada la segunda


pelota se cruzan las dos pelotas en el aire? d) A qu
altura sobre la mano del malabarista se cruzan las
dos pelotas
Solucin.
a) Tomemos el sentido positivo hacia arriba.
Tenemos que v y = v 0 y 2 g ( y y 0 )
2 2

En el cielorraso, v y = 0 , y y 0 = 3,0 m .
Luego: 0 = v 0 y 2(9,8)(3) v 0 y = 7,7 m s .
2

b) Tambin tenemos:
v y = v0 y gt = 0 = 7,7 9,8t
t = 0,78 s .
c) Tomemos el sentido positivo hacia abajo.
La primera bola viaja hacia abajo una distancia d Ejemplo 27. En el salto vertical, un atleta se
agazapa y salta hacia arriba tratando de alcanzar la
en el tiempo t . Como comienza desde su mxima
mayor altura posible. Ni los campeones pasan
altura, v0 y = 0. mucho ms de 1,00 s en el aire (tiempo de
suspensin). Trate al atleta como partcula y sea
d = v0 y t + 12 gt 2 d = (4,9 m s 2 )t 2
y mx su altura mxima sobre el suelo. Para explicar
La segunda bola tiene
por qu parece estar suspendido en el aire, calcule la
v' 0 y = 13 (7,7 m s) = 5,1 m s .
razn del tiempo que est sobre y mx / 2 al tiempo
En el tiempo t habr viajado hacia arriba
que tarda en llegar del suelo a esa altura. Desprecie
(3,0 m d ) y estar en el mismo lugar que la la resistencia del aire.
primera bola. (3 d ) = v' 0 y t 12 gt Solucin.
2

El tiempo al caer para alcanzar y mx es:


(3 d ) = 5,1t 4,9t 2
Tenemos dos ecuaciones con dos incgnitas. 2 y mx
t1 = =1 s .
Resolvindolas obtenemos: g
t = 0,59 s y d = 1,7 m.
El tiempo al caer para alcanzar y mx / 2 es:
d) 3,0 m d = 1,3 m
2 y mx / 2 y mx t 1
t2 = = = 1 = s.
Ejemplo 26. Una manzana cae libremente de un g g 2 2
rbol, estando originalmente en reposo a una altura
H sobre un csped crecido cuyas hojas miden h. 1
El tiempo debajo de y mx / 2 es 1 , de tal
Cuando la manzana llega al csped, se frena con 2
razn constante de modo que su rapidez es 0 al manera que la razn entre el tiempo que est sobre la
llegar al suelo, mitad de la altura mxima y el tiempo que est por
a) Obtenga la rapidez de la manzana justo antes de debajo de la altura mxima es.
tocar el csped.
b) Obtenga la aceleracin de la manzana ya dentro 1/ 2 1
= = 2,4.
del csped. 1 1/ 2 2 1
c) Dibuje las grficas: v-t y a-t para el movimiento
de la manzana. Esto explica porque el atleta parece estar suspendido
Solucin. en el aire.
a) La rapidez de un objeto que cae una distancia
Ejemplo 28. Un excursionista despierto ve caer un
H en cada libre una distancia H h es:
peasco desde un risco lejano y observa que tarda
v = 2 g ( H h). 1,30 s en caer el ltimo tercio de la distancia. Puede
b) La aceleracin para llevar a un objeto desde la despreciarse la resistencia del aire.
rapidez v al reposo sobre una distancia h es: a) Qu altura (en m) tiene el risco?
b) Si en (a) obtiene dos soluciones de una ecuacin
v2 2 g ( H h) H cuadrtica y usa una para su respuesta, qu
a= = = g 1.
2h 2h h representa la otra?
Solucin.
c)
a) Sea h la altura y toma un tiempo t en caer:
h = 12 gt 2

16
Movimiento rectilneo Hugo Medina Guzmn

Si tarda 1,30 s en caer el ltimo tercio h :


Ejemplo 30. Una piedra que cae libremente pasa a
2
3 h = 12 g (t 1,3) 2 las 10 horas frente a un observador situado a 300 m
Eliminando h de estas dos ecuaciones obtenemos: sobre el suelo, y a las 10 horas 2 segundos frente a
1
3 gt 2 = 12 g (t 1,3) 2 un observador situado a 200 m sobre el suelo. Se
pide calcular:
t 7,8t + 5,07 = 0
2
a) La altura desde la que cae.
b) En qu momento llegar al suelo.
t = 7,08s
Resolviendo t = 3,9 3,18 1 c) La velocidad con que llegar al suelo.
t 2 = 0,73s
La primera es la solucin correcta porque es mayor
que 1,30 s,
h= 1
2
(9,8)(7,08)2 = 245,6 m
b) Con la segunda solucin para t encontramos h =
2,6 m. Esto correspondera a un objeto que estaba
inicialmente cerca del fondo de este "acantilado" que
era lanzado hacia arriba y tomando 1,30 s la subida a
la cima y la cada al fondo. Aunque fsicamente es
posible, las condiciones del problema imposibilitan
esta respuesta.

Ejemplo 29. Desde la cornisa de un edificio de 60 m


de alto se lanza verticalmente hacia abajo un Solucin.
proyectil con una velocidad de 10 m/s. Calcular:
a) Velocidad con que llega al suelo. h1 = 300m
b) Tiempo que tarda en llegar al suelo.
h2 = 200m t1 = 2 s
c) Velocidad cuando se encuentra en la mitad de su
recorrido. g 10m/s 2
d) Tiempo que tarda en alcanzar la velocidad del
h3 = 100m
apartado c). a)
v2 = v1 + gt1 v2 = v1 + 10 2
1 2 1
h3 = v1t1 + gt1 100 = 2v1 + 10 4
2 2
2 2
v v
h4 = 2 h4 = 2
2g 2 10
H = h2 + h4
v1 = 40m/s

Solucin. De aqu se obtiene v2 = 60m/s ,
Tomamos corno origen de coordenadas el punto de h = 180m
lanzamiento y como sentido positivo el del eje 4
vertical descendente. Las ecuaciones de este Finalmente H = 200 + 180 = 380 m
movimiento sern:
v = v 0 + gt b) Llamando t2 al tiempo que tarda en recorrer hl:
v0 = 10 m/s
1 2 1 2
s = v0 t + gt g 10m/s 2 h1 = v1t 2 + gt 2
2 2
a) y b) h = 60 m 1
300 = 40t 2 + 10t 22
v = 10 + 10t 2
t = 2,6 s
1 t 2 = 5s
60 = 10t + 10t 2
v = 36m/s Luego llega al suelo a las 10 horas 5 segundos
2
c) y d) h = 30 m c) v = 2 gH = 2 10 380
v' = 10 + 10t ' = 87 m/s
t ' = 1,65 s
1
30 = 10t '+ 10t ' 2
v' = 26,5m/s
2

17
Movimiento rectilneo Hugo Medina Guzmn

PROBLEMA INVERSO - CLCULO El desplazamiento total para el intervalo (t t 0 ) es


INTEGRAL
la suma de todas las reas de todos los rectngulos
Conociendo la ley del movimiento x = x (t ) es de tal modo que:
posible sin mayores dificultades calcular v(t ) y x = v m (t i )t
a(t ) tal como fue mostrado i

La regla para los tiempos es que t i +1 = t i + t .


dx(t ) dv(t ) d 2 x(t )
x(t ) v(t ) = a (t ) = = La distancia que obtenemos con este mtodo no ser
dt dt dt 2 la correcta porque la velocidad cambia durante el
Como hemos visto, el clculo diferencial tiempo del intervalo t .
proporciona la herramienta para determinar la Si tomamos los intervalos muy pequeos la suma
velocidad y aceleracin en cualquier instante del tiene mayor precisin. As es que los hacemos tan
tiempo. pequeos a fin de tener una buena aproximacin.
En esta seccin veremos cmo el clculo integral, Obtendremos la distancia real en el lmite:
que es el inverso del clculo diferencial, puede
utilizarse para deducir las frmulas que ya hemos x = lim v(t i )t
t 0
visto. Por ejemplo, hallar la posicin de una i

partcula en un instante cualquiera, dado su Obsrvese que hemos reemplazado la velocidad


velocidad inicial y su aceleracin conocida. promedio v m por la velocidad instantnea v ,
porque en el lmite esta aproximacin es vlida.
Ya hemos demostramos que el rea encerrada bajo Los matemticos han inventado un smbolo para este
la curva de la velocidad del diagrama velocidad- lmite, anlogo al smbolo para la diferencial. El
smbolo se convierte en d , v(t i ) se llama v(t )
tiempo es igual al desplazamiento.
1
Area del trapecio = v0 (t t 0 ) + a(t t 0 )
2
y el smbolo sumatoria se escribe como una "s
2 grande la cual se conoce el signo integral Luego
1 escribimos
x x 0 = v0 (t t 0 ) + a(t t 0 )
2

x = v(t )dt
t
2
En el caso de un movimiento con velocidad t0

constante el desplazamiento entre los tiempos t y El proceso de integracin es el inverso del proceso
t 0 es de derivacin. Con un diferencial obtenemos una
frmula integral si la invertimos.
x x 0 = v0 (t t 0 )
Ejemplo 31. Encontrar la velocidad de un mvil a
o x = v 0 (t t 0 )
partir de la aceleracin.
Para un movimiento cualquiera con aceleracin Solucin.
variable el diagrama velocidad-tiempo ser el dv
mostrado en la figura siguiente a= dv = adt
dt
v t t

v0
dv = adt = a dt
t0 t0
Integrando obtenemos
v v0 = a(t t0 ) v = v0 + a(t t0 )
Para encontrar la posicin
dx
v= dx = vdt
dt
x t

Si descomponemos el tiempo total desde t 0 hasta t



x0
dx = vdt
t0

dx = [v0 + a(t t0 )]dt


x t
en segmentos pequeos t , entonces cada tramo
x0 t0
vertical que baja desde la curva de velocidades hasta
Integrando obtenemos
el eje de absisas tiene un rea
1
A = v m t x x0 = v0 (t t0 ) + a(t t0 )
2

2
Donde v m es la velocidad media del intervalo. Esta
1
x = x0 + v0 (t t0 ) + a(t t0 )
2
rea corresponde al desplazamiento en ese intervalo
que como se puede observar el rea faltante se 2
complementa con el excedente del otro lado.

18
Movimiento rectilneo Hugo Medina Guzmn

Tambin se puede encontrar la ecuacin del x = [v0 + a(t t 0 )]dt + C 2


movimiento expresando la integral de la siguiente
manera: Integrando:
v = adt + C1 , x = vdt + C2 1 2
x = v0 t + at at 0 t + C 2
2
Los valores de C1 y C2 dependen de las
Como para t = t 0 se tiene x = x 0 , tenemos
condiciones iniciales del movimiento.
1
x0 = v 0 t 0 + at 02 at 02 + C 2
2
Pequea Tabla de Integrales
1
C 2 = x0 v0 t 0 + at 02
2
dx = x Reemplazando el valor de C 2 obtenemos
x n +1
x dx =
n
(n 1) 1 1
x = v0 t + at 2 at 0 t + x0 v0 t 0 + at 02
n +1 2 2

dx
x
= lnx 1
x = x0 + v0 (t t0 ) + a(t t0 )
2

2
e ax
e dx = a
ax
Ejemplo 33. La aceleracin de una motocicleta est
cos(ax ) dada por a (t ) = 1,5t 0,12t , con t en s m/s3. La
2

sen(ax ) = a moto est en reposo en el origen en t = 0.


a) Obtenga su posicin y velocidad en funcin de t.
(u + v )dx = udx + vdx b) Calcule la velocidad mxima que alcanza.
Solucin.
a) Para encontrar v(t ) .

dv = adt = (1,5t 0,12t 2 )dt


Ejemplo 32. Encontrar las ecuaciones del dv
movimiento para una partcula que se mueve con a=

dt
aceleracin constante a = ai y que para el tiempo Integrando con v0 = 0 y t 0 = 0 :

( )
t
inicial t 0 se encontraba en r 0 = x 0 i y tena una v = 1,5t 0,12t 2 dt = 0,75t 2 0,40t 3
0

Para encontrar x(t ) .



velocidad inicial v 0 = v 0 i .
Solucin. dx
v= = 0,75t 2 0,40t 3
El movimiento es en el eje x . dt
La aceleracin es a=
dv dx = (0,75t 2 0,40t 3 )dt
dt Integrando con x0 = 0 y t 0 = 0 :
La velocidad se puede encontrar en trminos de una
integral como t
( )
x = 0,75t 2 0,40t 3 dt = 0,25t 3 0,10t 4
v = adt + C1 v = at + C1 0
b) Para que la velocidad sea mxima la aceleracin
Como para t = t 0 se tiene v = v0 , tenemos debe ser cero,

v0 = at 0 + C1 C1 = v 0 at 0 t = 0

a (t ) = 1,5t 0,12t = 0
2
1,5
Reemplazando el valor de C1 obtendremos la
t = 0,12 = 12,5s
ecuacin de la velocidad:
v = v0 + a(t t 0 ) Para t = 0 la velocidad es mnima
Para t = 12,5 la velocidad
v = 0,75(12,5) 0,40(12,5) = 39,1 m/s
2 3
Ahora consideremos la definicin de la velocidad
dx
v= Ejemplo 34. Salto volador de la pulga. Una
dt pelcula tomada a alta velocidad por M. Rothschild,
Tambin se puede escribir en forma integral Y. Schlein. K. Parker, C. Neville y S. Sternberg
x = vdt + C2 (3500 cuadros por segundo, The Flying Leap of the
Flea, en el ScientificAmerican de noviembre de
Reemplazando el valor de v : 1973) de una pulga saltarina de 210 g produjo los

19
Movimiento rectilneo Hugo Medina Guzmn

datos que se usaron para dibujar la grfica de la 1


figura. La pulga tena una longitud aproximada de 2 = (1,3 10-3 )(133) + (0,2 10-3 )(133)
2
mm y salt con un ngulo de despegue casi vertical.
= 0,11 cm
Use la grfica para contestar estas preguntas.
a) La aceleracin de la pulga es cero en algn
momento? Si lo es, cundo? Justifique su respuesta. Ejemplo 35. La grfica de la figura describe, en
b) Calcule la altura mxima que la pulga alcanz en funcin del tiempo, la aceleracin de una piedra que
los primeros 2,5 ms. baja rodando por una ladera, habiendo partido del
c) Determine la aceleracin de la pulga a los: 0,5 ms, reposo.
1,0 ms y 1,5 ms. a) Determine el cambio de velocidad de la piedra
d) Calcule la altura de la pulga a los: 0,5 ms, 1,0 ms entre t = 2,5 s y t = 7,5 s.
y 1,5 ms. b) Dibuje una grfica de la velocidad de la piedra en
funcin del tiempo.

Solucin.
Solucin.
a) Pendiente de a = 0 para t 1,3 ms dv
a) a= dv = adt
b) La altura mxima corresponde al recorrido hasta dt
cuando la aceleracin se hace cero y llega al tiempo Como a (t ) es la ecuacin de la recta:
t = 2,5 ms, y es el rea bajo la curva v versus t.
a2 84
(Dibujado aproximndolo a Un tringulo y un = = 0,8 a = 0,8t + 2
rectngulo). t 0 7,5 2,5
dv = (0,8t + 2 )dt
dv = (0,8t + 2)dt
v t
Integrando: v0 t0

v v0 = 0,4(t t 02 ) + 2(t t 0 )
2

Con t 0 = 2,5s , t = 7,5s , y v = v v0 :


hmax = rea bajo (v t )
ATringulo + ARectngulo v = 0,4(7,52 2,52 ) + 2(7,5 2,5)
cm
1 = 30
[(1,3)(133) + (2,5 1,3)(133)]10 3 s
2 Otra manera de encontrar el cambio de velocidad es
0,25 cm encontrando el rea bajo la curva a versus t, entre las
c) a = pendiente del grfico v t. lneas en t = 2,5 s y t = 7,5 s. El rea es:
a (0,5 ms) a (1,0 ms) cm
133
1
2 (4 + 8)(7,5 2,5) = 30
= 1,0 105 cm s
2
s
1,3 10 -3
Como la aceleracin es positiva, el cambio de
a (1,5 ms) = 0 porque la pendiente es cero. velocidad es positivo.
b)
d) h = rea bajo el grfico v t.
1
h (0,5) ATringulo = (0,5 10 -3 )(33)
2
= 8,3 10 3 cm
1
h (1,0) ATringulo = (1,0 10 -3 )(100)
2
= 5,0 10 2 cm Ejemplo 36. La velocidad de un punto que se mueve
h (1,5) ATringulo + ARectngulo en trayectoria recta queda expresada, en el SI por la
ecuacin: v = 40 - 8t. Para t = 2 s, el punto dista del
origen 80 m. Determinar:

20
Movimiento rectilneo Hugo Medina Guzmn

a) La expresin general de la distancia al origen.


b) El espacio inicial.
c) La aceleracin.
d) En qu instante tiene el mvil velocidad nula?
e) Cunto dista del origen en tal instante?
f) Distancia al origen y espacio recorrido sobre la
trayectoria a partir de t = 0, cuando t = 7 s, t = 10 s y
t = 15 s.
Solucin.
a) s = vdt = (40 8t )dt = 40t 4t 2 + C
s = s 0 + 40t 4t 2
b) 80 = s0 + 80 - 16 s0 = 16
dv m
c) a = = 8 2 Representacin grfica de la velocidad origen en
dt s funcin del tiempo
d) 0 = 40 - 8t t =5 s
e) s5 =16 + 40x5 - 4x52 = 116 m
f) s7 =16 + 40x7 - 4x72 = 100 m
sl0 =16 + 40x10 - 4x102 = 16 m
s15 = 16 + 40x15 - 4x152 = -284 m
Clculo de caminos sobre la trayectoria a partir de
t = 0:
El mvil cambia el sentido de su velocidad para
t = 5s
El recorrido en los 5 primeros segundos es:
C5 = s s0 = 116 - 16 = 100 m En la grfica de la velocidad frente al tiempo, el rea
A ellos hay que sumar el recorrido en los segundos limitada por el eje de abscisas y la grfica entre dos
restantes que se obtienen de la integral de la instantes coincide numricamente con el camino
ecuacin general de la velocidad, en valor absoluto, recorrido por el mvil entre esos dos instantes.
entre los limites t = 5 s y t = instante final.

(40 8t )dt = 116m


7 Ejemplo 37. El vector velocidad del movimiento de
C7 = 100 + una partcula viene dado por
5

v = (3t - 2)i + (6t 2 - 5) j m/s. Si la posicin del

mvil en el instante t =1 s es r = 3i 2 j m.
Calcular

a) El vector posicin del mvil en cualquier instante.

(40 8t )dt = 200m


10
C10 = 100 + b) El vector aceleracin.
5

c) Las componentes tangencial y normal de la


aceleracin en el instante t = 2 s. Dibujar el vector
velocidad, el vector aceleracin y las componentes
tangencial y normal en dicho instante.

Solucin.

(40 8t )dt = 500m


15
C15 = 100 + a) Para el movimiento horizontal
5

dvx m
vx = 3t - 2 ax = =3 2
dt s

Representacin grfica de la distancia al origen en dx


funcin del tiempo
Como vx = dx = v x dt , integrando
dt

21
Movimiento rectilneo Hugo Medina Guzmn

t
dx = (3t 2 )dt
t
an = asen ( ) = 2m/s 2
3 1

3 7 CINEMTICA DE PARTCULAS LIGADAS.


x = t 2 2t + m MOVIMIENTOS DEPENDIENTES.
2 2
Observemos los sistemas fsicos de la figura.
Podramos decir que estos sistemas se componen de
Para el movimiento vertical varias partculas ligadas (conectadas).

dv y m
v y = 6t 2 - 5 ay = = 12t
dt s2

dy
Como vy = dy = v y dt , integrando
dt

2
t t
(
dy = 6t 2 5 dt
1
) Las partculas podran ser las poleas y los cuerpos a
desplazar (bloques, baldes). La ligadura la tienen a
y = (2t 3 5t + 1) m travs de las cuerdas. Es decir, cuando el hombre
desplaza el extremo de la cuerda con una aceleracin
a, la aceleracin de las poleas y los cuerpos a
3 7
( )
desplazar (bloques, baldes) tendrn una dependencia
r = t 2 2t + i - 2t 3 5t + 1 j de a. Lo mismo se cumplir para las otras variables
2 2 cinemticas (desplazamiento y velocidad).

Ejemplo 38. Anlisis del montaje de la figura


b) a = 3i + 12tj siguiente.

c) Para t = 2 s

vx = 4 m/s, vy = 19 m/s

ax = 3 m/s2, ay = 24 m/s2
a = ax2 + a y2 = 24,2m / s 2

vy 19 Para analizar las relaciones que hay entre las


tan = = = 4,75 = 78 o variables cinemticas del bloque m1 , del balde m 2
vx 4
y de la polea mvil, debemos primero saber cules
son sus posiciones. Para ello elegimos un sistema de
ay 24 coordenadas. En nuestro caso elegimos el eje y
tan = = = 3 = 83o apuntando hacia abajo y con el origen en el techo.
ax 3 Para el sistema de coordenadas escogido las
posiciones del bloque, del balde y de la polea son
respectivamente: y1 , y 2 , y p . Estas se representan
en la figura siguiente.

at = a cos( ) = 24,1m/s 2

22
Movimiento rectilneo Hugo Medina Guzmn

La longitud de la cuerda debe permanecer constante Anlogamente podramos hacer un anlisis para las
en todo instante. Por tanto debe ser siempre vlida la aceleraciones, y concluiramos que:
siguiente relacin: 1
Longitud de la cuerda = constante a p = a1
AB + arco BC + CD +arco DE +EF = constante 2
De la figura podemos concluir que las siguientes Es decir, si el bloque por ejemplo, baja con una
relaciones son vlidas: aceleracin igual a 2,0 m/s2 , la polea subir con una
aceleracin igual a 1,0 m/s2 .
AB = y p
CD = y p c2 De esta figura tambin se deduce la siguiente
relacin entre la posicin del balde y la posicin de
EF = y1 c2 la polea mvil:
Por tanto, y 2 = y p + c1 (3)
y p + arcoBC + ( y p c2 ) + arcoDE + y1 = constante
Si el balde se desplaza una cantidad y 2 , y la polea
Como los arcos BC y DE permanecen constantes
podremos escribir la relacin anterior as: se desplaza una cantidad y p .
2 y p + y1 = k (1) El balde pasa a ocupar la posicin: y 2 + y 2 ,
Siendo k una constante. La polea pasa a ocupar la posicin y p + y p .
Esta ecuacin relaciona las variables cinemticas de
la polea mvil y del bloque. Sin embargo, la relacin anterior se debe seguir
cumplindose.
Si el bloque se desplaza una cantidad y1 y la polea
( )
( y 2 + y 2 ) = y p + y p + c1 (4)
en una cantidad y p .
Restando (3) y (4) obtenemos,
La nueva posicin de la polea: y 2 = y p
y p + y p ,
Los desplazamientos de la polea y el balde son
La nueva posicin del bloque: y1 + y1 . iguales.
Sin embargo, la relacin anterior debe seguir Si dividimos la ecuacin anterior por el intervalo de
cumplindose: tiempo t obtenemos como se relacionan las
( )
2 y p + y p + ( y1 + y1 ) = k (2) velocidades: v2 = v p .
Restando (1) de (2), obtenemos: Las velocidades de la polea y del balde son iguales.
2y p + y1 = 0 Lo mismo podremos concluir para las aceleraciones:
a2 = a p
y
y p = 1 En definitiva si el bloque baja con una aceleracin
2 igual a 4 m/s2, el balde y la polea mvil subirn con
Por ejemplo, si el bloque baja 1,0 m, la polea solo una aceleracin igual a 2 m/s2.
sube 0,50 m. La polea solo se desplaza la mitad de lo
que se desplaza el bloque.

PREGUNTAS Y PROBLEMAS

1. Un acelerador atmico emite partculas que se 3. Una cucaracha se desplaza en lnea recta y su
desplazan con una rapidez de 2,8x108 m/s. cunto posicin con respecto al tiempo se expresa de
demoran estas partculas en recorrer una distancia de acuerdo al siguiente grfico. De acuerdo a la
5,6mm? informacin dada se pide calcular.
Respuesta a) distancia recorrida entre 4s y 9 s
2x10-11 s. b) distancia recorrida entre 9 s y 14s
c) distancia recorrida entre 0 y 16s.
2. Se desea calcular cul es la profundidad de un d) velocidad media entre 0s y 16s.
lago, para tal efecto se usa un instrumento conocido e) velocidad media entre 9s y 16s.
como sonar que mide el tiempo que tarda un pulso
sonoro en ir y volver desde la superficie del agua. Si
se sabe que la rapidez del sonido en el agua es de
1450m/s y el instrumento marc 0,042s cuando se
hizo la medicin, calcule la profundidad del lago.
Respuesta. 30,45m

23
Movimiento rectilneo Hugo Medina Guzmn

a) v = 2t 2 t 4 ; b) x = 2 + 2t 3 / 3 t 5 / 5
9. El movimiento de una partcula se define
mediante la relacin x = t / 3 3t + 8t + 2 ,
3 2

donde x se expresa en metros y t en segundos.


Determinar
a) el momento en que la velocidad es nula;
b) la posicin y la distancia total recorrida cuando la
Respuesta aceleracin es nula.
a) 4m b) 8m c) 22m d) 5/8 m/s e) 0 Respuesta
a) 2s, 4s; b) 8m, 7,33m
4. Un hombre camina con una velocidad v
constante pasa bajo un farol que cuelga a una altura 10. El movimiento de una partcula est dado por la
ecuacin horaria x = t + 4t + 5 x sobre el eje x,
H sobre el suelo. Encontrar la velocidad con la que 3 2

el borde de la sombra de la cabeza del hombre se x en metros t en segundos.


mueve sobre la tierra. El alto del hombre es h. a) Calcular la velocidad y la aceleracin de la
Respuesta partcula en el instante t.

b) Encontrar la posicin, la velocidad y la
Hv
aceleracin de la partcula para t 0 = 2s y t 1 = 3s.
H h
c) Cules son la velocidad media y la aceleracin
5. Un tren arranca en una estacin y acelera media de la partcula entre t 0 y t1 ?
2
uniformemente a razn de 0,6 m/s hasta alcanzar Respuesta.
2 2
una velocidad de 24 m/s. Determinar el tiempo a) v = (3t + 8t)m/s , a = ( 6t + 8 ) m/s
empleado y la distancia recorrida en ese perodo si la b) x0 = 29m, v0 = 27 m/s, a 0 = 20 m/s 2
velocidad media fue: a) 16 m/s, b) 22m/s.
Respuesta x1 = 68 m v1 = 51 m/s, a1 = 26 m/s 2
a) 60s, 960m, b) 240s, 5280m 2
c) v m = 39 m/s , a m = 23 m/s
6. Un ciclista recorre 100 km en 2 horas. El viaje de
vuelta dos das ms tarde lo realiza en el tiempo 11. La posicin de una partcula que se mueve en el
usual de 6 horas. eje x est dada por 8 t + 5, x es la distancia a origen
a) Cul es su rapidez media a la ida? en metros y t es el tiempo en segundos.
b) Cul es su rapidez media al regreso? a) Para t = 2, encontrar la posicin, velocidad y
c) Su rapidez media en e viaje completo? aceleracin
d) Su velocidad media en e} viaje entero? b) Grafique x versus t
Respuesta. a) 50 km/h , b) 16,7 km/h c) Encuentre la ley horaria, la ley del movimiento y
c) 25 km/h d) 0 la trayectoria.
d) Analizar el movimiento.
7. Un automvil que viaja con una velocidad de 50 Respuesta. a) x = -3, v = 0 , a = 4

( 8t + 5)i
km/h hacia el oeste repentinamente empieza a perder
b) s = 2t 8t + 5 , r = 2t
2 2
velocidad a un ritmo constante y 3 segundos ms
tarde su velocidad es de 25 km/h hacia el oeste. Trayectoria rectilnea en el eje x.
a) Cunto tiempo tardar en detenerse el auto,
contando a partir del momento en que empez a 12. Un automvil se encuentra detenido frente a un
desacelerar? semforo, le dan luz verde y arranca de modo que a
b) Cul es la distancia total que recorrer antes de los 4s su rapidez es de 72 km/hora. Si se movi en
detenerse? trayectoria rectilnea, con aceleracin constante,
c) Cul sera el tiempo necesario para detenerse y la I.- Determine:
distancia recorrida el) la frenada con la misma a) La rapidez inicial en metros por segundo.
aceleracin, pero con una velocidad inicial de 100 b) El mdulo de la aceleracin en ese tramo.
km/h? c) La rapidez que lleva a los 3s.
Respuesta. a) t = 6s ; b) 41,7m ; c) 125; 125m d) La distancia que recorre en los tres primeros
segundos
8. La aceleracin de una partcula est dada por: e) La distancia que recorre entre t = 2s y t = 4s.
a = 4t 4t 3 , t 0 . II.- Haga un grfico representativo de posicin
a) Hallar la velocidad de la partcula en funcin del versus tiempo y de la rapidez versus tiempo.
tiempo.
b) Hallar su posicin en funcin del tiempo. Respuesta. a) 20m/s b) 5 m/s2 c) 15m/s
Respuesta d) 22.45m e) 30m

24
Movimiento rectilneo Hugo Medina Guzmn

movimiento para retornar hacia A con una


13. Una partcula A, se mueve en el eje X, de aceleracin constante de mdulo 6(m/s2). Calcular:
acuerdo a la siguiente grfica. a) La distancia total cubierta hasta que la partcula
Determinar a partir del grfico de la partcula: retorne al punto A.
b) El tiempo total para el recorrido completo hasta
a) Velocidad media entre t = 0 y t = 4 s volver a dicho punto A.
b) Velocidad instantnea en t = 2 s c) El intervalo de tiempo que transcurre entre los
c) Aceleracin media entre t = 0 y t = 4 s pasos de la partcula por el punto situado a 1/3 de
d) Intervalos de tiempo en que se acerca al AB, medido desde A.
origen
e) Intervalos de tiempo en que se aleja del 17. Desde una altura de 45m se deja caer un objeto
origen A. simultneamente se lanza un objeto B
f) Ecuacin Itinerario de la partcula A verticalmente desde una altura de 5m. Calcular:
g) Qu tipo de movimiento tiene esta partcula? a) la velocidad inicial de B para que los objetos se
crucen a una altura de 20m.
b) la distancia que separa a los objetos cuando B
alcanza su altura mxima.

18. Sobre un mismo eje x se mueven dos partculas


A y B. En t = 0 la partcula A parte desde P con
aceleracin constate de 15i (m/s2). Un segundo
despus, B pasa por Q con una velocidad de 20i
(m/s). Encuentre las retardaciones constantes que
Respuesta. a) ( -8;0)m/s b) (-8;0)m/s c) 0 deben aplicar A y B a partir de este ltimo instante
para que ambas partculas se detengan
d) (0-3)s e)(3-....) f) x(t ) = 24 8t simultneamente antes de chocar.
g) Movimiento rectilneo uniforme.

14. Un vehculo se mueve en el eje x de acuerdo con


la siguiente ecuacin de itinerario:
x(t ) = 20 36t + 6t 2 . Con x medido en metros y
t en segundos.
a) Identifique a posicin inicial, la velocidad inicial
y la aceleracin. 19. Una partcula se mueve a lo largo del eje x
b) Determine la ecuacin que entregue la velocidad con aceleracin constante. En t = 0 pasa por la

para cualquier instante.
c) Determine el instante en que cambia de sentido posicin x 0 = 10i m con una velocidad

d) La velocidad de la partcula en t = 2 s y en t = 4
s
v 0 = 20i m/s y en t =3s su posicin es

e) Posicin de la partcula en t = 6 segundos
x = 52i m. Calcule:
f) Grfico x versus t. Describa la curva
a) La ecuacin itineraria de la partcula
g) Grfico v x versus t. Describa la curva b) La distancia recorrida en el intervalo (3-6) s.
h) Grfico a versus t. Describa la curva c) La velocidad media en el intervalo (4-7) s.
Respuesta. a) (20,0)m (-36,0)m/s d) Intervalos de tiempo en que la partcula se aleja
(12,0)m/s2 b) v(t ) = 36 + 12t c)3s del origen del sistema.
d) (-12,0)m/s (12,0)m/s e) (20,0)m

15. Se lanza un cuerpo hacia arriba con una rapidez


de 16m/s, 20. Sobre el eje x de un sistema de coordenadas se
a) Qu altura alcanza a subir? mueven dos partculas A y B. El grfico (a) es una
b) Qu tiempo demora en volver al punto de parbola cuadrtica que muestra la variacin de
partida? la componente x de la posicin en funcin del
Respuesta. a) 3,2m b) 6,4s tiempo de la partcula A. El grfico (b) muestra la
variacin de la componente v x de la velocidad en
16. Una partcula se mueve sobre una recta
horizontal; parte hacia la derecha desde un punto A funcin del tiempo de la partcula B. Si en t = 0,
con una rapidez de 28 (m/s) y una retardacin ambas partculas tienen la misma posicin,
constante de mdulo 12(m/s2). En el punto B, es determinar:
donde se anula su rapidez, invierte el sentido de a) Ecuacin horaria de las partculas A y B.

25
Movimiento rectilneo Hugo Medina Guzmn

b) Posicin de B cuando A cambia de sentido de


movimiento. 23. Un cuerpo que se ha dejado caer desde cierta
c) Instante en que se encuentran. altura, recorre 72 m en el ltimo segundo de su
d) Distancia recorrida por A y B entre 3 y 9 s. movimiento.
Calcule la altura desde la cual cay el cuerpo y el
tiempo que emple en llegar al suelo.

24. Un hombre parado en el techo de un edificio tira


un cuerpo verticalmente hacia arriba con una rapidez
de 14m/s. El cuerpo llega al suelo 4,7s ms tarde.
a) Cul es la mxima altura alcanzada por el
cuerpo?
b) Qu altura tiene el edificio?
c) Con qu rapidez llegar el cuerpo al suelo?

21. En el grfico de la figura estn representadas la


componente v x del vector velocidad de dos
partculas, A y B, que se mueven a lo largo del eje
x Calcular:
a) La aceleracin de B.
b) Camino recorrido por A y B cuando B alcanza la 25. Un malabarista mantiene cinco bolas continuamente
en el aire, lanzando cada una de ellas hasta una altura de
velocidad v B = 30i m/s. 3m.
a) Cul es el tiempo que debe transcurrir entre
c) Desplazamiento de B en el intervalo (0-10)s.
lanzamientos sucesivos?
d) Ecuacin horaria de A si en t0 = 0 su posicin es b) Cules son las alturas de las otras pelotas en el

momento en que una de ellas vuelve a su mano?


x 0 = 8i m. Respuesta. a) 0,31s ;
b) 1,91; 2,87; 2,87 y 1,91 m.

26. Dos cuerpos son lanzados uno despus de otro con las
mismas velocidades v0 desde una torre alta. El primer
cuerpo es lanzado verticalmente hacia arriba, y el segundo
verticalmente hacia abajo despus del tiempo .
Determinar las velocidades de los cuerpos una con
respecto al otro y las distancias entre ellos en el instante
t > .
Respuesta. La velocidad del primer cuerpo relativa al

22. Dos partculas A y B se mueven sobre el mismo segundo es: v1 v 2 = 2v0 g .


eje x. En t = 0, B pasa por Q con 1 2
La distancia es S = 2v0 t v0 gt + g
m/s v B (0 ) = ( 5,0 ) m/s y 2s despus A pasa 2
por P a 6i m/s. Encuentre las retardaciones
constantes que deben aplicar A y B a partir de este
ltimo instante para que ambas partculas se
detengan simultneamente justo antes de chocar.
Determine la ecuacin itinerario de A y B (diga cul
es su origen).

26
Movimiento en un plano y en el espacio Hugo Medina Guzmn

CAPITULO 3. Movimiento en un plano y en el espacio

MOVIMIENTO CIRCULAR 1 . La velocidad angular del mvil ha cambiado


es
Se define movimiento circular como aqul cuya
trayectoria es una circunferencia. Una vez situado el = 1 0 en el intervalo de tiempo
origen O de ngulos describimos el movimiento t = t1 t 0 comprendido entre t 0 y t1 .
circular mediante las siguientes magnitudes.

Posicin angular,
En el instante t el mvil se encuentra en el punto P.
Su posicin angular viene dada por el ngulo , que
hace el punto P, el centro de la circunferencia C y el
origen de ngulos O.
El ngulo , es el cociente entre la longitud del arco
S y el radio de la circunferencia r, = S / r . La
Se denomina aceleracin angular media al cociente
posicin angular es el cociente entre dos longitudes
entre el cambio de velocidad angular y el intervalo
y por tanto, no tiene dimensiones.
de tiempo que tarda en efectuar dicho cambio.

m =
t
La aceleracin angular en un instante, se obtiene
calculando la aceleracin angular media en un
intervalo de tiempo que tiende a cero.
d
= lim =
t 0 t dt
RELACIN ENTRE LAS MAGNITUDES
ANGULARES Y LINEALES
Velocidad angular, De la definicin de radin (unidad natural de medida
de ngulos) obtenemos la relacin entre el arco y el
En el instante t1 el mvil se encontrar en la
radio. Como vemos en la figura, el ngulo se obtiene
posicin P1 dada por el ngulo 1 . El mvil se habr dividiendo la longitud del arco entre su radio
desplazado = 1 0 en el intervalo de tiempo s s'
= =
t = t1 t 0 comprendido entre t 0 y t1 . r r'

Derivando s = r respecto del tiempo obtenemos la


Se denomina velocidad angular media al cociente relacin entre la velocidad lineal y la velocidad
entre le desplazamiento y el tiempo. angular
ds d
m = , con las unidades en el SI de rad/s. =r v = r
t dt dt
Como ya se explic en el movimiento rectilneo, la La direccin de la velocidad es tangente a la
0H

velocidad angular en un instante se obtiene trayectoria circular, es decir, perpendicular a la


calculando la velocidad angular media en un direccin radial
intervalo de tiempo que tiende a cero.
d Aceleracin tangencial
= lim = Derivando esta ltima relacin con respecto del
t 0 t dt tiempo obtenemos la relacin entre la aceleracin
tangencial a t y la aceleracin angular.
Aceleracin angular,
Si en el instante t la velocidad angular del mvil es dv d
=r at = r
y en el instante t1 la velocidad angular del mvil dt dt

1
Movimiento en un plano y en el espacio Hugo Medina Guzmn

Existe aceleracin tangencial, siempre que el d


mdulo de la velocidad cambie con el tiempo, es Siendo = d = dt , integrando
decir, en un movimiento circular no uniforme dt
obtenemos el desplazamiento 0 del mvil
Hallar el desplazamiento angular a partir de la
entre los instantes t 0 y t :
velocidad angular.

d = [

+ (t t0 )] dt
Si conocemos un registro de la velocidad angular del t

mvil podemos calcular su desplazamiento 0


0
0 t0

1
entre los instantes t 0 y t , mediante la integral = 0 + 0 (t t0 ) + (t t0 )2
definida. 2
t Habitualmente, el instante inicial t 0 se toma como
0 = dt
t0 cero. Las frmulas del movimiento circular
uniformemente acelerado son anlogas a las del
Hallar el cambio de velocidad angular a partir de movimiento rectilneo uniformemente acelerado.
2H

la aceleracin angular. = constante , = 0 + t ,


Del mismo modo que hemos calculado el
desplazamiento angular del mvil entre los instantes 1
= 0 + 0 t + t 2
t 0 y t , a partir de un registro de la velocidad 2
angular en funcin del tiempo t . Despejando el tiempo t en la segunda ecuacin y
sustituyndola en la tercera, relacionamos la
d d
= = velocidad angular con el desplazamiento 0 .
dt dt
t t
0 = dt 0 = dt 2 = 02 + 2 ( 0 )
t0 t0

MOVIMIENTO CIRCULAR UNIFORME COMPONENTES NORMAL Y TANGENCIAL


Un movimiento circular uniforme es aqul cuya DE LA ACELERACIN.
velocidad angular es constante, por tanto, la Cuando el sistema de referencia se sita sobre la
aceleracin angular es cero. partcula tal como se indica en la figura, pero no de
d cualquier modo. Uno de los ejes siempre est
= d = dt perpendicular a su trayectoria, y el otro siempre es
dt tangente a la misma. As pues,
La posicin angular del mvil en el instante t
podemos calcularla integrando
t
0
d = dt
t0

0 = (t t 0 )
O grficamente, en la representacin de en
funcin de t.
Habitualmente, el instante inicial t 0 se toma como
cero. Las ecuaciones del movimiento circular
uniforme son anlogas a las del movimiento
1H
El primero siempre pasar por el centro de la
rectilneo uniforme circunferencia. Al primer eje se le denomina eje
= 0 = constante = 0 + t normal, con vector unitario (r = n ) y al segundo eje

MOVIMIENTO CIRCULAR
()
tangencial, con vector unitario t . Debemos
estudiar ahora que componentes tienen la velocidad
UNIFORMEMENTE ACELERADO
y la aceleracin en este sistema de referencia.
Un movimiento circular uniformemente acelerado es
aqul cuya aceleracin es constante.
Velocidad.
Dada la aceleracin angular podemos obtener el Con anterioridad se ha deducido que el vector
cambio de velocidad angular 0 entre los velocidad siempre es tangente a la trayectoria
instantes t 0 y t , mediante integracin de la descrita. Por tanto es fcil afirmar que en este

en funcin del tiempo
velocidad angular movimiento la velocidad ser de la forma v = vt
= 0 + (t t0 ) .
Aceleracin.

2
Movimiento en un plano y en el espacio Hugo Medina Guzmn

No es tan obvio que la aceleracin tenga una sola dt


componente, de manera que adoptar la expresin parntesis es efectivamente n , por lo que
dt
general a = at t + an n dt v
quedar como = n = n .
Sabemos por la definicin de aceleracin que dt R

2
dv dv v
a= , luego. Finalmente: a = t n
dt dt R
As, en esta expresin, se denomina aceleracin
d v dvt dv dt dv
a= = = t +v tangencial (at ) al trmino at = y aceleracin
dt dt dt dt dt
dt v2
Estudiemos el ltimo trmino de esta expresin normal (a n ) a la ecuacin an =
dt R
Si se define el ngulo , como el ngulo formado De esta expresin para la aceleracin pueden
por el eje normal con el eje de abscisas (eje x), tal concluirse cosas sustancialmente importantes:
como se muestra en la figura. Existen dos componentes: Una tangente a la
trayectoria y una perpendicular y orientada hacia el
centro de la circunferencia.
La aceleracin tangencial slo se dar en aquellos
movimientos en los que el mdulo de la velocidad
vare con el tiempo. Por tanto, en el caso particular
del MCU, su aceleracin tangencial ser nula.
La aceleracin normal siempre existir, salvo que el
No es difcil darse cuenta que el vector t desde el radio de curvatura fuera muy grande, con lo cual
sistema de referencia situado en el centro de la tendera a cero, que es el caso extremo de los
circunferencia tendr la forma movimientos rectilneos.
t = sen i + cos j , mientras que n al ser Concluyendo pues, en un MCU, la aceleracin
perpendicular a este adoptar la expresin v2
tendr la expresin a = n es decir slo
n = cos i + sen j R
Derivando t presentar aceleracin normal.
dt d d
= cos i sen j Un objeto puede experimentar la aceleracin normal
dt dt dt o centrpeta y la aceleracin tangencial. En las
dt d figuras siguientes se muestran algunas
= ( cos i senj ) combinaciones posibles para v y a para un auto en
dt dt movimiento. Para entender la aceleracin,
descompngala en las componentes paralela y
perpendicular a v . Para decir si el auto est dando
vuelta a la derecha o a la izquierda, imagnese que
usted es el conductor que se sienta con el vector de
la velocidad dirigido hacia adelante de usted. Un
Ahora bien, si tomamos un desplazamiento diminuto componente de la aceleracin hacia adelante
significa que la velocidad est aumentando.
sobre la circunferencia, al que denominamos ds ,
teniendo en cuenta que arco = ngulo x radio, del
esquema adjunto se deduce que ds = Rd , y
adems el mdulo de la velocidad instantnea lo
ds
podemos expresar como v = , utilizando estos
dt
d v
dos ltimos llegamos a = = ,
dt R

dt
reemplazando en :
dt
dt v
= (cos i + sen j ) , si observamos
dt R
detenidamente esta ecuacin, comprobaremos que el

3
Movimiento en un plano y en el espacio Hugo Medina Guzmn

rapidez lineal de un pasajero en el borde es constante


e igual a 7,00 m/s. Qu magnitud y direccin tiene
la aceleracin del pasajero al pasar
a) por el punto ms bajo de su movimiento circular?
b) por el punto ms alto?
c) Cunto tarda una revolucin de la rueda?

Ejemplo 1. Un avin a chorro militar de combate


volando a 180 m/s sale de una picada vertical dando
la vuelta hacia arriba a lo largo de una trayectoria
circular de 860 m de radio cul es la aceleracin del
avin? Exprese la aceleracin como mltiplo de g.
Solucin.
v 2 180 2 m
a= = = 37,7 2
r 860 s Solucin.
g v 2 7,00 2 m
a = 37,7 = 3,8 g a) a = = = 3,50 2 . La aceleracin el
9,8 R 14,0 s
punto ms bajo del crculo es hacia el centro, hacia
Ejemplo 2. Una rueda de 75 cm de dimetro gira arriba.
alrededor de un eje fijo con una velocidad angular de b) a = 3,50 m/s , dirigida hacia abajo., hacia el
2
1 rev/s. La aceleracin es de 1,5 rev/s2.
a) Calclese la velocidad angular al cabo de 6 centro.
segundos. 2R
c) Como v =
b) Cunto habr girado la rueda en ese tiempo? T
c) Cul es la velocidad tangencial en un punto de la
2R 2 (14,0)
periferia de la rueda en t = 6 s? T= = = 12,6 s
d) Cul es la aceleracin resultante de un punto de v 7,00
la de la periferia para t = 6 s?
Solucin. Ejemplo 4. La rueda de la figura del problema
rad rad anterior, que gira en sentido antihorario, se acaba de
R = 37,5 cm , 0 = 2 , = 3 2 poner en movimiento. En un instante dado, un
s s pasajero en el borde de la rueda que est pasando
a) (t ) = 0 + t por el punto ms bajo de su movimiento circular
rad tiene una rapidez de 3,00 m/s, la cual est
(6 ) = 2 + 3 (6) = 20 aumentando a razn de 0,500 m/s2.
s Calcule la magnitud y la direccin de la aceleracin
1 del pasajero en este instante.
b) (t ) = 0 t + t 2 Solucin.
2
v 2 3,00 2 m m
(6 ) = 2 (6) + (3 )(6 2 ) = 66 rad
1 ac = = = 0,643 2 , y at = 0,5 2
2 R 14,0 s s
66 Luego:
Habr girado = 33 vueltas.
2 a = a c n + at t = 0,643 j + 0,5i
c) v(t ) = R(t ) m
a = a c2 + at2 = 0,643 2 + 0,5 2 = 0,814
cm s2
v(6 ) = 37,5(20 ) = 750
s 0,5
= tan 1 = 37,9
d) an = R
2
(6 ) 0,643
an = (20 ) (37,5) = 147894 cm/s2.
2

at = R an = (3 )(37,5) = 353,25 cm/s2.


a = an2 + at2 = 147894,42 cm/s2.

Ejemplo 3. Una rueda de la fortuna de 14,0 m de


radio gira sobre un eje horizontal en el centro. La

4
Movimiento en un plano y en el espacio Hugo Medina Guzmn

2 2
Como H = , M = donde TH = 12 h y
TH TM
TM = 1 h y bajo la condicin que estos formen un

ngulo de 90, es decir, M H =
2
De (2) - (1), con 0 H
= 0M = 0 ,
M H = ( M H )t
Ejemplo 5. Una partcula se mueve sobre una
Se encuentra para t:
circunferencia de radio R con aceleracin angular
constante partiendo del reposo. Si la partcula realiza 3
t= = h,
n vueltas completas a la circunferencia en el primer 2(M H ) 11
segundo, determine la aceleracin angular de la
Es decir, en t = 16,36 min.
partcula. Determine adems el nmero de vueltas
Por lo tanto forman 90 a las 12:16:22 h.
que realiza la partcula durante el siguiente segundo
del movimiento.
Ejemplo 7. Dos partculas describen movimientos
Solucin.
circulares de radio R = 1m, como lo muestra la
1 figura. El primero (1) parte de O con rapidez angular
Aqu = t 2
2 = 10 rad/s constante en sentido antihorario y el
1 segundo (2) parte del reposo del mismo punto en
Entonces 2n = = 4n sentido horario con aceleracin tangencial constante
2 2
de 2 m/s . Determine cuando y donde se cruzan
1
Como = (4n )t = 2nt ,
2 2
ambas partculas.
2
Nmero de vueltas para t = 1
(1)
n(1) =
2

2
Nmero de vueltas para t = 2
(2 )
n(2) =
2

2
Durante el siguiente segundo (dos) realiza Solucin.
(2 ) (1)
= n(2 2 12 ) = 3n
Como el cuerpo (1) se mueve con M.C.U., la
vueltas. posicin angular de este ser:
2
1 = 0 + 1 t = 10 t . (1)
Ejemplo 6. En un reloj anlogo el horario y el El cuerpo (2) posee una aceleracin tangencial
minutero coinciden a las 12:00:00 horas. constante y por lo tanto, se trata de un M.C.U.A.
A qu hora minutero y horario formarn un ngulo Debido que at = R = 2 m/s , = 2rad/s . Por
2 2
de 90?
Solucin. otro lado, como parte del reposo, 0 = 0.
1
2 = 2 t 2 = t 2
2
El recorrido se muestra en la figura siguiente:

Como los movimientos del horario y minutero son


circulares uniformes, encontramos para la posicin
angular del horario:
H = 0 H + H t . (1) El encuentro se produce cuando:
Anlogamente para el minutero se tiene: 1 + 2 = 2
M = 0 M + M t . (2) 10t + t 2 = 2

5
Movimiento en un plano y en el espacio Hugo Medina Guzmn

t = 0,59 s
t 2 + 10t 2 = 0 1 2 = rad / s
2

t 2 = 10,59 s 6
La solucin significativa es: t = 0,59 s
Reemplazando este valor de t en ecuacin (1), se 2 = 4 t rad / s
6
obtiene para el ngulo de encuentro:
2
encuentro = 5,9 rad = 338,04 . 2 = + 4 t t rad
2 12
Ejemplo 8. Dos vehculos describen la misma
trayectoria circular de radio 0,75 m. El primero est
animado de un movimiento uniforme cuya velocidad Los mviles se encontrarn cuando 1 = 2
angular es de 60 rpm. y sale de la posicin A cuando
se empieza a contar el tiempo. El segundo mvil est
animado de un movimiento uniformemente 2 t = + 4 t t2
acelerado cuya aceleracin angular vale - /6 rad/s2, 2 12
pasa por B dos segundos ms tarde llevando una
velocidad angular de 120 rpm. t 2 2 t =0
12 2
a) Escribir las ecuaciones del movimiento de cada
uno de los mviles. Hallar el instante y la posicin
de encuentro por primera vez de ambos mviles. t 2 24t 6 = 0
b) La velocidad lineal, la velocidad angular, las
componentes tangencial y normal de la aceleracin
de cada uno de los mviles en el instante de t = 0,25 s
Resolviendo
encuentro. t = 24,25 s
c) Realcese un esquema en el que se especifique los
vectores velocidad, aceleracin, en dicho instante de
encuentro. La solucin es 24,25 s.

El punto de encuentro es

1 = 2 (24,25) = 48,5 rad


2 = 0,5 + 4 (24,25) (24,25)2 = 48,5 rad
12

Los valores son iguales, tal como esperbamos.

Solucin. Como 1 = 2 = 48,5 rad , equivalente a 24


vueltas mas 1/4 de vuelta, el encuentro es en punto B.
a) Para t = 2 s el mvil 1 como su velocidad angular b) La velocidad lineal, la velocidad angular, las
es 2 rad/s estar en el punto A, y podemos componentes tangencial y normal de la aceleracin
considerar ese instante como tiempo inicial, con lo de cada uno de los mviles en el instante de
que: encuentro.
Mvil 1
Mvil 1: 1 = 2 rad / s
v = r = 1,5 m/s
1 1
1 = 0
1 = 0 at1 = 1 r = 0
1 = 2 rad / s a = 2 r = 3 2 m / s
= 2 t rad n1 1
1 Mvil 2

Mvil 2:

6
Movimiento en un plano y en el espacio Hugo Medina Guzmn

24,25
2 = 4 6 = 0,04 rad / s

v 2 = 2 r = 0,03 m/s
at 2 = 2 r = 0,125 m/s 2

a n 2 = 22 r = 0,0012 2 m/s 2
El mvil 2 tiene velocidad negativa, porque a l
tiempo t = 24 s su velocidad se hizo cero e inicia el
retorno, al tiempo t = 24,25 s se produce el
encuentro.
Como at = a cos m/s y a n = asen m/s ,
2 2
c) Esquema especificando los vectores velocidad,
aceleracin, en el instante de encuentro.
En el instante del encuentro el esquema sera el
siguiente: La aceleracin tangencial en cualquier instante, se
obtiene a partir del producto escalar del vector

aceleracin a y el vector velocidad v .


v a = va cos = vat


v a vx ax + v y a y
at = =
v v x2 + v y2

MOVIMIENTO CURVILNEO La aceleracin normal, se obtiene a partir del


mdulo de la aceleracin a y de la aceleracin
El movimiento curvilneo es aquel en el que pueden tangencial a t .
combinarse tramos rectos y/o curvos. La extensin
de las ecuaciones en el sistema intrnseco es
inmediata sufriendo slo una ligera modificacin a 2 = a x2 + a y2 = a n2 + a t2
respecto a la aceleracin. Esta adopta la expresin
dv v 2
a = t + n donde es el denominado radio a n2 = a x2 + a y2 at2
dt
de curvatura y corresponde al radio de una hipottica
2
circunferencia en cada uno de los puntos de la v a +v a
trayectoria. Es evidente que en el caso del a n2 = a x2 + a y2
x x y y
movimiento circular ste no vara ya que coincide v x + v y2
2
con el radio de la circunferencia en cada uno de esos
dv v2
puntos. a t = y an = v y ax vx a y
dt Finalmente a n =
v x2 + v y2

El radio de curvatura

v2 v2
an = =
an

Ejemplo 9. El vector velocidad del movimiento de


una partcula viene dado por

( )
La figura siguiente muestra la velocidad y la
aceleracin con las coordenadas x e y para un v = (3t 2 )i + 6t 2 5 j m/s. Calcular las
determinado instante.

7
Movimiento en un plano y en el espacio Hugo Medina Guzmn

componentes tangencial y normal de la aceleracin y b) las componentes normal y tangencial de la


el radio de curvatura en el instante t =2 s. velocidad y aceleracin.
c) la ecuacin de la trayectoria en coordenadas
Solucin. cartesianas.
Solucin.
dv x = 3t , y = 2t 5t 2
v x = (3t 2 ) m/s a x = x = 3 m/s 2
dt a) vx = 3; vy = 2 - 10t; ax = 0; ay = -10;

dv y
v y = (6t 2 5) m/s a y =

= 12t m/s 2 v 3i + (2 10t ) j
dt b) t = = ,
v 9 + (2 10t )
2

En el instante t = 2 s 3 j + (2 10t )i
v x = 4 m/s a x = 3 m/s 2 n = t k = entonces
9 + (2 10t )
2

v y = 19 m/s a y = 24 m/s
2

vt = v t = v = 9 + (2 10t )
2

vn = 0
v = 4 2 + 19 2 = 19,49 m/s
10t (2 10t )
a = 3 2 + 24 2 = 24,19 m/s 2 aT = a .t =
9 + (2 10t )
2

La aceleracin tangencial es:



10t (2 10t )
aT = a .t =
9 + (2 10t )

v a v x a x + v y a y 4(3) + 19(24 )
2
at = = = = 24 m/s 2
v vx + v y
2 2 19 , 49 30
a n = a n =
9 + (2 10t )
2

La aceleracin normal es:


v y a x v x a y 19(3) 4(24) 2 5
an = = = 2 m/s 2 c) y = x x2
vx + v y
2 2 19,49 3 9

Ejemplo 11. Una partcula se mueve en el plano xy


El radio de curvatura de acuerdo con la ley ax = 0, ay = 4cos(2t) m/s2. En
el instante t = 0, el mvil se encontraba en x = 0, y =
-1 m, y tena la velocidad vx = 2, vy = 0 m/s.
v2 v2
an = =
an a) Hallar las expresiones de r(t) y v(t).

b) Dibujar y calcular las componentes tangencial y


a = 3 2 + 24 2 = 24,19 m/s 2 normal de la aceleracin en el instante t = /6 s.
v = 4 2 + 19 2 = 19,49 m/s
Solucin.

v 2 = 377 , an = 2 m/s 2 a) En t = 0

v 2 377 m
= = = 188,5 m ax = 0 , vx = 2 , x=0
an 2 s

Ejemplo 10. Una partcula se mueve de modo que m


sus coordenadas cartesianas estn dadas como a y = 4 cos(2t ) , v y = 0 , y = 1 m
s2
funciones del tiempo
x = 3t , y = 2t 5t 2 m
Determine En el eje x el movimiento es uniforme v x = 2 ,
a) las componentes cartesianas de la velocidad y de s
la aceleracin. x = 2t m

8
Movimiento en un plano y en el espacio Hugo Medina Guzmn

Para encontrar el movimiento en y hay que integrar v y = 0


Para t = 2 s
m v x = 4
v y = 4 cos(2t )dt v y = 2sen (2t )
vy t
0 0 s

dy = 2sen (2t )dt y ( 1) = 1 cos(2t )


y t
1 0

y = cos(2t ) m

b) Las componentes tangencial y normal de la


aceleracin en el instante t = /6 s.

vx = 2 , ax = 0

m m
vy = 3 , ay = 2 at = ax = 2 an = a y = 10
s2 s2

v2 v 2 42
an = = = = 1,6m
an 10

Ejemplo 13. El vector velocidad del movimiento


de una partcula viene dado por

( )

v = (3t 2)i + 6t 2 5 j m/s.

Si la posicin del mvil en el instante t = 1 s es



m r = 3i 2 j m. Calcular
at = 2 cos = 1,31 2 ,
s
m v 2 a) El vector posicin del mvil en cualquier instante.
a n = 2sen = 1,51 2 , tan = x =
s vy 3 b) El vector aceleracin.
= 49,1
c) Las componentes tangencial y normal de la
aceleracin en el instante t = 2 s.
Ejemplo 12. Un mvil se mueve en el plano xy
con las siguientes aceleraciones: ax=2 m/s2, ay =10
m/s2. Si en el instante inicial parte del origen con Dibujar el vector velocidad, el vector aceleracin y
velocidad inicial vx = 0 y vy =20 m/s. las componentes tangencial y normal en dicho
instante.
Calcular las componentes tangencial y normal de la
aceleracin, y el radio de curvatura en el instante t = Solucin.
2 s.
a) Para el movimiento horizontal
Solucin.
dv x m
m vx = 3t - 2 ax = =3 2
a y = 10 2 v y = 20 + ( 10 )t dt s
s
dx
m Como v x = dx = v x dt , integrando
ax = 2 v x = 2t dt
s2

9
Movimiento en un plano y en el espacio Hugo Medina Guzmn

Considere un objeto que se desplaza en el aire sin


dx = (3t 2 )dt
t t
3 1 ninguna fuerza con excepcin de la gravedad y de la
resistencia del aire. La fuerza de la gravedad
3 7
x = t 2 2t + m produce una aceleracin constante hacia abajo de
2 2 magnitud 9,80 m/s2. Como primera aproximacin,
no tomemos los efectos del aire y de variaciones en
Para el movimiento vertical g . Asumiremos que la tierra es plana para el rango
horizontal de los proyectiles. A pesar de estas
simplificaciones, podemos an obtener una
dv y m descripcin bastante buena del movimiento del
v y = 6t 2 - 5 ay = = 12t
dt s2 proyectil. El recorrido de un proyectil se llama su
trayectoria.
Si se desprecia la resistencia del aire, no hay
dy entonces aceleracin en la direccin horizontal, y
Como v y = dy = v y dt , integrando
dt a x = 0 . La aceleracin en la direccin de y es
debido a la gravedad. Es constante y dirigida hacia

2
t

1
t
( )
dy = 6t 5 dt y = 2t 5t + 1 m
2
( 3
) abajo, as que a y = g . Es conveniente elegir
x0 = 0 y y 0 = 0 (es decir, poner el origen en el
punto donde el proyectil comienza su movimiento).
3 7
( )

r = t 2 2t + i - 2t 3 5t + 1 j Adems, nos referimos tpicamente a v 0 como la


2 2
rapidez inicial del proyectil. Si el proyectil es
lanzado con un ngulo sobre la horizontal, la

b) a = 3i + 12tj velocidad inicial en la direccin x y la velocidad
inicial en la direccin y se pueden expresar en
trminos de g y de y usando la trigonometra.
c) Para t = 2 s

vx = 4 m/s, vy = 19 m/s

ax = 3 m/s2, ay = 24 m/s2
a = ax2 + a y2 = 24,2m / s 2

vy 19 v0 x = v 0 cos , v0 y = v0 sen
tan = = = 4,75 = 78 o
vx 4 ax = 0 , a y = g
Con esto:
ay 24 v x = v0 cos = constante , v y = v0 sen gt
tan = = = 3 = 83o
ax 3 1 2
x = (v0 cos )t , y = (v0 sen )t gt
2
Ecuacin de la trayectoria.
x
De la ecuacin para x obtenemos t = .
v0 cos
Sustituyendo en la ecuacin para y

g 2
y = (tan )x 2 x

2v 0 cos
2

at = a cos( ) = 24,1m / s 2 Corresponde a la ecuacin de una parbola que pasa
por el origen. Una caracterstica dominante del
movimiento del proyectil es que el movimiento
an = a(sen ) = 2m horizontal es independiente del movimiento vertical.
As un proyectil se mueve a una velocidad constante
MOVIMIENTO PARABLICO.

10
Movimiento en un plano y en el espacio Hugo Medina Guzmn

en la direccin horizontal, independiente de su v02sen (2 )


movimiento vertical. Esto se ilustra en la figura. =
g

La altura mxima que alcanza un proyectil se


obtiene con v y = 0 .

v y = v0 sen gt = 0 , despejando t.

Podemos entender mejor el significado de la v0sen


1 2 t= , como vemos es igual a la mitad del
ecuacin y = (v 0 sen )t gt viendo el g
2 tiempo de vuelo.
movimiento del proyectil de esta manera: Primero, si
no hubiera fuerza de la gravedad y aceleracin hacia
1 2
abajo, en el tiempo t el proyectil movera una y mx = (v0 sen )t gt
distancia v 0 t en una lnea inclinada recta. Si ahora 2
imaginamos con la gravedad el efecto sera hacer 2
que el proyectil se aleje de la trayectoria recta por = (v0sen ) v0sen 1 v0sen
g


una distancia gt2. De la superposicin de estos g
2 g
dos efectos resulta la trayectoria parablica como se
muestra en la figura.
Finalmente:

v02 sen 2
y mx =
2g

Su valor mximo se obtiene para el ngulo de


disparo = 90.
Tiempo de vuelo. Poniendo y = 0
Ejemplo 14. UN BLANCO EN CADA LIBRE
1
y = (v 0 sen )t gt 2 = 0 , despejando t, (Tiro al mono)
2 Se deja caer una botella desde el reposo en el
instante en que una piedra es lanzada desde el origen.
Determinar los valores del ngulo y de la velocidad
2v0sen de disparo para que la piedra rompa la botella.
t2 t=0
g (Tmese g = 9,8 m/s2)

Resolviendo obtenemos dos soluciones t = 0, que


corresponde al disparo del proyectil y

2v0sen
t=
g

El valor mximo de t se obtiene para = 90.


Cuando el proyectil se lanza verticalmente hacia
arriba, describiendo una trayectoria rectilnea a lo
largo del eje y. Solucin.
Movimiento de la piedra: El movimiento
El alcance horizontal de cada uno de los curvilneo de la piedra se realiza bajo la aceleracin
3H

proyectiles se obtiene para y = 0. constante de la gravedad, es decir, es la composicin


de dos movimientos
- Uniforme a lo largo del eje horizontal
2v sen
xmx = (v0 cos )t = (v0 cos ) 0
g

11
Movimiento en un plano y en el espacio Hugo Medina Guzmn

a px = 0 De estas ecuaciones, obtenemos:



Horizontal v px = v0 cos 2x 2y g
= y = x Ecuacin de una
ax g ax
x p = v 0 cos t
- Uniformemente acelerado a lo largo del eje vertical. lnea recta.
b) En tierra, y = 124 , tal que
a px = g
2( 124)
Vertical v px = v 0 sen gt t2 = t = 5,03 s
9,8

y p = v 0 sen t gt / 2 c) v y = v0 y gt = 0 (9,8)(5,03)
2

m
Movimiento de la botella: La botella se mueve = 49,3
verticalmente bajo la aceleracin constante de la s
vx = v0 x + a xt = 0 + (1,10 )(5,03)
4H

gravedad.
abx = g m
= 5,53
vbx = gt s
y b = H gt 2 / 2 v = vx2 + v y2 = (5,53)2 + ( 49,3)2
Choque de la piedra con la botella: Cuando se
m
produce el choque, la posicin de la piedra y de la = 49,6
botella coincide. s
A = v0 cos t Ejemplo 16. Disparamos un proyectil desde el
origen y ste describe una trayectoria parablica
H gt 2 / 2 = v0 sen t gt 2 / 2 como la de la figura. Despreciamos la resistencia del
aire.
H = v0 sen t Dibuja en las posiciones A, B, C, D y E el vector
Dividimos la segunda ecuacin entre la primera. velocidad, el vector aceleracin y las componentes
H normal y tangencial de la aceleracin. (No se trata
tan = de dar el valor numrico de ninguna de las variables,
A slo la direccin y el sentido de las mismas)
Para romper la botella debemos de apuntarla
Qu efecto producen an y at sobre la velocidad?
directamente y en el instante en el que se deja caer,
se debe lanzar la piedra. La velocidad debe tener un
valor mnimo para hacer el recorrido A, mientras la
botella est en el aire.
2H
Esto sucede para el tiempo t = , y el
g
recorrido horizontal de la piedra debe cumplir:
2H A g
v0 cos A v0
cos Solucin.
g 2H

Ejemplo 15. Una bolsa de arena cae del reposo de v es tangente a la trayectoria
un globo de aire caliente desde una altura de 124 m
est soplando un viento horizontal, y el viento da a
bolsa de arena una aceleracin horizontal constante
de 1,10 m/s2.
a) Demuestre que la trayectoria de la bolsa de arena
es una lnea recta.
b) Cuanto tiempo toma para llegar la tierra?
c) Con qu velocidad llega a la tierra?
Solucin.
1 2x
a) x = axt 2 t 2 =
2 ax Cuando sube
1 2y
y = gt 2 t 2 =
2 g

12
Movimiento en un plano y en el espacio Hugo Medina Guzmn

y = 150 + 180(sen /6)t - 5t2


at y v tienen sentidos opuestos. a) Punto de cada
150 + 180(sen /6)t - 5t2 = 0,
Cuando baja t = 19,5 s
x = 180(cos /6)(19,5) = 3039,8m

b) Tiempo para la altura mxima
at y v tienen el mismo sentido 180(sen /6) - 10t = 0, t = 9,0 s
entonces
ymax = 150 + 180(sen /6)(9) - 5(9)2
at modifica el mdulo de la velocidad con el = 555,0m
tiempo. El vector unitario tangente es

v
t = = i cos + jsen
a n modifica la direccin de v v 6 6

a = 10 j
Ejemplo 17. Una bala del rifle se dispara con una
Entonces
velocidad de 280 m/s hacia arriba de una superficie
plana inclinada 30 sobre la horizontal. La bala se

at = a t = 10sen = 5 m/s 2
dispara con un ngulo de elevacin inicial de 45 6
sobre la horizontal (es decir, 15 sobre la superficie
plana). Cul es el alcance de la bala sobre el plano? an = a 2 an2 = 100 25 = 8,66 m/s 2
Solucin.
La ecuacin del plano inclinado es Ejemplo 19. Un can de artillera lanza
y x proyectiles con una rapidez de 300 (m/s). El artillero
= tan 30 y = debe darle a un blanco que se encuentra a 8640 (m)
x 3 detrs de un cerro, cuya altura es de 1000 (m)
La ecuacin de la trayectoria parablica. ubicado a 1200 (m) del can. Demuestre que es
g 2 posible darle al blanco y determine el ngulo de
y = (tan )x 2 x elevacin para cumplir el objetivo.
2v 0 cos
2
Solucin.
La interseccin de la parbola y la lnea recta ocurre Supondremos que damos en el blanco entonces
cuando
gx 2
x g y = x tan 2 =0
= (tan )x 2 x2 2v0 cos 2
3 2v0 cos
2

5(8649)
2
v2 1 8649 tan =0
Para = 45 : x = 0 1
g
(300)2 cos 2
3
Tiene dos races reales
Para un tringulo 30, 60, 90 vemos que 1 = 53,03
3 2 = 36,97
x = S cos 30 = S. Debemos verificar que el disparo pasa sobre el cerro,
2
para ello evaluamos en ambos ngulos y(1200)
De aqu S =
2
3
( )v2 v2
3 1 0 = 0,49 0 , arriba del
g g
y1 (1200) = 1373,0 m
y2 (1200) = 777,95 m
La altura del cerro es excedida en el primer caso.
plano.
Con y 0 = 280 m/s, S = 3,90 km. Ejemplo 20. Se dispara un proyectil de modo que
su alcance horizontal es igual al triple de la altura
Ejemplo 18. Se dispara un proyectil desde la cima mxima. Encuentre el ngulo de lanzamiento.
de una colina de 150 (m) de altura con una rapidez Solucin.
de 180 (m/s) y formando un ngulo de 30 con la Sabemos que
horizontal. Calcule: v02sen 2
(a) La distancia horizontal entre el punto de xmax =
lanzamiento y el punto de cada del proyectil. g
(b) La altura mxima del proyectil con respecto al
v02sen 2
suelo. ymax =
(c) Las componentes normal y tangencial de la 2g
aceleracin al salir en el punto de disparo. Entonces
Solucin:
x = 180(cos /6)t

13
Movimiento en un plano y en el espacio Hugo Medina Guzmn

v02sen 2 v 2sen 2
=3 0
g 2g
2 cos = 3sen
2
tan = =33,69
3
Ejemplo 21. Un lanza granadas tiene un alcance
mximo de 300 m. Para dar en un blanco que se
encuentra a una distancia de 400 m del lanza Solucin.
granadas. Determine: a) y b)
a) La altura mnima que debe subirse el lanza
granadas.
b) La rapidez de lanzamiento.
c) El ngulo de lanzamiento,
Solucin.
La ecuacin de la parbola de seguridad es
v02 gx 2
y = h+
2 g 2v02
Sabemos tambin que para h = 0 la distancia
mxima alcanzable es
v02
x (0 ) = = 300 ax = 0 v x = 45 cos
g
y para una altura h la distancia horizontal mxima a y = 10 v y = 45sen 10t
ser x = 45 cos .t
x(h ) = (v2
0 + 2hg ) vg = 400m
0
1
y = 45sen .t 10t 2
2
de la primera
b) Punto de impacto x = 45 , y = 45 3

v0 = 3000 = 54,77
m 45 = 45 cos .t
s 2

45 3 = 45sen .t 5t
y de (54,77 )2 + 2h(10) 54,77 = 400 1 1
10 45 3 = 45sen . 5
a) cos cos 2
h = 116,701m tan 2 9 tan + 1 9 3 = 0
c) El ngulo de lanzamiento cuando el blanco est
sobre el lmite de la parbola de seguridad es 1 = 84,5 o t1 = 10,45s
v 02 2 = 54,5 o t 2 = 1,72 s
tan = entonces = 36,87o
gx t1 v x = 4,31 ax = 0
c) Para t =
2 v y = 7,46 a y = 10
Ejemplo 22. Un patinador desciende por una pista
helada, alcanzando al finalizar la pista una velocidad vx
de 45 m/s. En una competicin de salto, debera tan = = 30 o
alcanzar 90 m a lo largo de una pista inclinada 60 vy
respecto de la horizontal.
a) Cul ser el ngulo (o los ngulos) que debe
at = g cos 30 o = 5 3 m s 2
formar su vector velocidad inicial con la horizontal?. a n = gsen30 o = 5 m s 2
b) Cunto tiempo tarda en aterrizar?
c) Calcular y dibujar las componentes tangencial y
normal de la aceleracin en el instante t/2. Siendo t
el tiempo de vuelo. Tomar g =10 m/s2

14
Movimiento en un plano y en el espacio Hugo Medina Guzmn

Punto de encuentro
20015 cos .t
1 1
30 9,8t 2 = 15sen t 9,8t 2
2 2
30 = 56,3 o

tan =
20 y = 1,69m

b)

Ejemplo 23. Se deja caer una botella desde el


reposo en la posicin x =20 m e y =30 m. Al mismo
tiempo se lanza desde el origen una piedra con una
velocidad de 15 m/s.

a) Determinar el ngulo con el que tenemos que


lanzar la piedra para que rompa la botella, calcular la
altura a la que ha ocurrido el choque.

b) Dibujar en la misma grfica la trayectoria de la


piedra y de la botella. (Tomar g = 9,8 m/s2). Ejemplo 24. Desde un can que est sobre un
plano inclinado un ngulo con la horizontal se
Solucin: dispara un proyectil. Este sale con una velocidad v 0
formando un ngulo con el plano horizontal.
a) Encontrar.
a) El punto ms alto al que llega el proyectil.
b) El alcance del proyectil.
Solucin.

a) v 0 x = v 0 cos v 0 y = v 0 sen
Movimiento de la botella
v y = v0 sen gt
x = 20 La altura mxima se produce cuando v y = 0
ax = 0 vx = 0
1 v02 sen 2
a y = 9,8 v y = 9,8t y = 30 9,8t 2
2 y mx =
2g
Con ese valor,
Movimiento de la piedra
v sen v 02
x = v0 cos 0 = sen 2
ax = 0 vx = 15 cos g 2g
a y = 9,8 v y = 15sen 9,8t v2
y = x tan = 0 sen 2 tan
x = 15 cos t 2g
v2
h = y mx y = 0 (sen 2 sen 2 tan )
1
y = 15 sen t 9,8t 2
2 2g
b) El alcance mximo S .
x = v0 cos t y = v 0 sen t 12 gt 2
Ecuacin del plano en funcin de t y = x tan
Dividiendo y x :

15
Movimiento en un plano y en el espacio Hugo Medina Guzmn

c) El punto de cada ocurre para z = 0 y la distancia


y v0 sen t 12 gt
2
= = tan v021 + cos 2
x v 0 cos t vale x( ) = tan + sen 2
gsen
v0 sen t 12 gt 2 = tan v 0 cos t
dx( )
Resolviendo encontramos el tiempo para el que el La distancia mxima ocurre para = 0.
proyectil toca tierra: d
2v0
t= (sen cos tan )
g
El valor de x cuando el proyectil toca tierra es:
Ejemplo 26. Se lanza una pelota verticalmente
2v02
x = v0 cos t = cos (sen cos tan ) hacia arriba con una velocidad de 20 m/s desde la
azotea de un edificio de 50 m de altura. La pelota
g
adems es empujada por el viento, produciendo un
Y el alcance S es:
movimiento horizontal con aceleracin de 2 m/s2.
x 2v02 cos
S= = (sen cos tan ) Calcular:
cos g cos
a) La distancia horizontal entre el punto de
Ejemplo 25. La figura muestra una colina inclinada lanzamiento y de impacto.
un ngulo respecto a la vertical y la trayectoria
de un proyectil. El proyectil se lanza desde el origen b) La altura mxima
O con una velocidad inicial de mdulo v 0 y que
c) El valor de las componentes tangencial y normal
forma un ngulo con el eje z (perpendicular al de la aceleracin cuando la pelota se encuentra a 60
plano). El eje x se toma tangente al plano apuntando m de altura sobre el suelo.
hacia abajo.
a) Tome el sistema de referencia indicado en la
Tmese g =10 m/s2.
figura y halle las componentes de los vectores
aceleracin, velocidad y posicin del proyectil en
funcin del tiempo.
b) Halle la mxima separacin entre el proyectil y la
colina.
c) Halle la distancia entre el origen y el punto de
cada del proyectil sobre la colina. Demuestre que
esa distancia es mxima si = / 2 .

Solucin.

1 2
a x = 2 , v x = 2t , x = 2t
2

a y = 10 , v y = 20 + ( 10 )t ,
Solucin. 1
a) a x = g cos , v x = g cos t + v 0 sen , y = 20t + ( 10)t 2
2
1
x= g cos t 2 + v0 sen t a) Punto de impacto
2
a z = gsen , v z = gsen t + v0 cos ,
y = -50 t = 5,74 s x = 32,97 m
1
z = gsen t 2 + v 0 cos t
2 b) altura mxima
b) La mxima separacin ocurre para v z = 0 y vale
v02 cos 2 v y = 0 t = 2 s y = 20 m
z=
2 g 2sen
hmxima = 70 m sobre el suelo.

16
Movimiento en un plano y en el espacio Hugo Medina Guzmn

c) h = 60 y = 10 m t1 = 0,59 s t2 = 3,41 s = 2 1 = 14,5o


a n = a. cos = 9,87 m, /s 2

at = a.sen = 2,55 m/s 2

Ejemplo 27. Nos encontramos en la antigua


Suiza, donde Guillermo Tell va a intentar ensartar
con una flecha una manzana dispuesta en la cabeza
de su hijo a cierta distancia d del punto de disparo
(la manzana est 5 m por debajo del punto de
lanzamiento de la flecha). La flecha sale con una
velocidad inicial de 50 m/s haciendo una inclinacin
de 30 con la horizontal y el viento produce una
aceleracin horizontal opuesta a su velocidad de 2
v x = 1,17 a x = 2 m/s2.
t1 = 0,59 s
v y = 14,14 a y = 10 a) Calcular la distancia horizontal d a la que deber
a = 2 2 + 10 2 estar el hijo para que pueda ensartar la manzana.

b) Hllese la altura mxima que alcanza la flecha


vx 1,17
tan 1 = = = 0,08 1 = 4,7 o medida desde el punto de lanzamiento. (g = 9,8
v y 14,14 m/s2)
ay 10
tan 2 = = = 5 2 = 78,7 o
ax 2

= 2 1 = 73o
a n = a. cos = 2,81 m/s 2

at = a.sen = 9,80 m/s 2


Solucin.

v x = 6,83 m/s a x = 2
t 2 = 3,41s
v y = 14,14 m/s a y = 10 ax = 2 , v x = 50 cos 30 o 2t ,
a = 2 2 + 10 2 1
x = 50 cos 30 o 2t 2
2
vy 14,14
tan 1 = = = 2,07 1 = 64,2 o
vx 6,83 a y = 9,8 , v y = 50sen30 o 9,8t ,
ay 1
10 y = 50sen30o 9,8t 2
tan 2 = = = 5 2 = 78,7 o 2
ax 2
Punto de impacto x = d, y = -5

-5 =25 t -4,9 t2 t = 5,29 s x = 201,23 m

17
Movimiento en un plano y en el espacio Hugo Medina Guzmn

Mxima altura vy = 0

50sen30 - 9,8t = 0 t = 2,55 s y = 31,89 m

Ejemplo 28. Un paraguas abierto mojado se


sostiene hacia arriba como se muestra en la figura y
se gira sobre la manija a razn uniforme de 21
revoluciones en 44 s. Si el borde del paraguas es un
crculo 1 m de dimetro, y la altura del borde sobre Sea un objeto P determinado por un observador en el
el piso es 1,5 m, hallar donde las gotas del agua al
hacer girar del borde tocan el piso. sistema S por r = xi + yj + zk y por un
observador en el sistema S' por

r ' = x' i + y ' j + z ' k como se muestra en la figura.

Las ecuaciones de transformacin de Galileo que


relacionan las observaciones desde los sistemas S y
S' son
x = x'+ Vt , y = y ' , z = z ' t = t '
Aqu se supone que puede establecerse una escala de
tiempo absoluta aplicable a ambos marcos de
referencia de manera que t = t ' . Esto sucedera si la
Solucin. velocidad de la luz fuera infinita (Debemos
La velocidad angular del paraguas es reconocer que las escalas de tiempo asociadas a dos
21 2 rad marcos de referencia no son los mismos si existe
= = 3rad / s movimiento relativo entre ellos es uno de los
44s principios fundamentales de la teora especial de la
La velocidad tangencial de las gotas de agua que relatividad propuesta por Einstein en 1905).
salen del borde del paraguas es Vectorialmente podemos representar la
v0 = r = (0,5)(3) = 1,5m / s transformacin de Galileo como

Para calcular el tiempo en que la gota llega al piso
r = r '+ V t .
1 2
usamos h = gt Derivando las relaciones anteriores podemos obtener
2 la relacin de la velocidad.
2h 2(1,5) dx dx'
= + V v x = v' x ' + V
t= = = 0,553m dt dt
g 9,8
El alcance horizontal de la gota es dy dy '
= v y = v' y '
x = v0t = (1,5)(0,55) = 0,83 m; dt dt
y el locus de las gotas es un crculo de radio dz dz '
= v z = v' z '
R= (0,5)2 + (0,83)2 = 0,97 m. dt dt

VELOCIDAD Y ACELERACIN RELATIVAS. Vectorialmente v = v'+ V
Derivando nuevamente obtenemos la relacin de la
Movimiento Relativo de Traslacin Uniforme. aceleracin
La Relatividad de Galileo dv x dv' x ' dV dV
= + a x = a' x ' +
Consideramos dos sistemas de referencia S y S', S' dt dt dt dt
tiene un movimiento de traslacin rectilneo dv y dv' y '
uniforme con respecto a S; S' se aleja de S con una = a y = a' y '

dt dt
velocidad V = vi

18
Movimiento en un plano y en el espacio Hugo Medina Guzmn

dv z dv' z '
= a z = a' z '
dt dt

Si la velocidad V del sistema S' es constante,

dV
= 0 y a = a'
dt
Estas relaciones encontradas son de aplicacin
general si S y S' estn animadas por un movimiento
relativo cualquiera, como se muestra en la figura b) Si se observa el mismo proyectil desde un sistema
siguiente de referencia situado en el suelo S con un origen en
el lugar de lanzamiento (para t 0 = 0 , O = O'),
entonces las posiciones, las velocidades y las
aceleraciones respecto de O estarn dadas por la
transformacin de Galileo. En este caso la velocidad
inicial v 0 vista desde el suelo ser
r
v0 = Vi + v y 0 j v0 = V 2 + v y20
Las ecuaciones son: v y0

0 = tan 1
V
r = r '+ V t , r ' = r V t La trayectoria ser una parbola tal como se ve en la

v = v'+ V figura siguiente

a = a'
Ejemplo 29. Desde la plataforma de un camin en
r
movimiento horizontal V constante se lanza un
proyectil directamente hacia arriba con una
r
velocidad v 0 . Cmo ser visto el movimiento del
proyectil por: La componente horizontal del movimiento del
a) un observador situado en el camin (sistema S')? proyectil es igual al movimiento del can, de modo
b) un observador situado en el suelo (sistema S)? que cuando cae el proyectil coincidir con el can.
Solucin.
a) El tiempo se mide desde el momento del Ejemplo 30. El observador O suelta una piedra del
lanzamiento t 0 = 0 , cuando el proyectil se eleva trigsimo piso de un rascacielos. El observador O,
con velocidad v 0 . La componente horizontal de la descendiendo en un ascensor a velocidad constante
de V = 5,0 m/s, pasa el trigsimo piso justo cuando
velocidad coincide con la velocidad V del camin. se suelta la piedra. Al tiempo t = 3,0 s despus de
El observador O' en el camin ver nicamente la que se suelta la piedra, hallar:
componente vertical v ' y '0 , la componente horizontal a) La posicin, la velocidad, y la aceleracin de la
piedra relativa a O.
ser v ' x '0 = 0 . b) La posicin, la velocidad, y la aceleracin de la
Para un instante t cualquiera piedra relativa a O.
Solucin.
x' = 0 y ' = v' y '0 t gt 2 a) Para O, la posicin de la piedra est dada por:
v' x ' = 0 v' y ' = v' y '0 gt 1 2
x = x0 + v0 t + at
a' x ' = 0 a' y = g 2
Donde x = 0 en el trigsimo piso con la direccin
hacia abajo como la direccin positiva de x. As, en
t = 3,0 s,
1
x = 0+0+ (9,8)(3,0)2 = + 44 m/s
2
Tambin, v = v0 + at da
v = 0 + 9,8 m/s2 x 3,0 s = +29 m/s.

19
Movimiento en un plano y en el espacio Hugo Medina Guzmn

La aceleracin de un cuerpo en cada libre, segn el Sea W la rapidez del ro y u la rapidez de los botes
observador O que est inmvil con respecto a la respecto al agua, (igual en ambos), entonces
tierra, se sabe que la aceleracin gravitacional es V1 = u -W
constante. V2 = u + W
(De hecho, esto es la base de la validez de los dos de modo que
clculos anteriores.) V2 V1
As tenemos: W =
a = +g = +9,8 m/s2. 2

b) O mide la posicin x', relativa a x por medio de Ejemplo 34. Un bote cruza un ro que mide de
la ecuacin x' = x - Vt. ancho a en el cual la corriente fluye con una rapidez
Luego, despus de 3,0 s, uniforme de u. El botero mantiene una orientacin
x' = 44 m 5,0 m/s x 3,0 s = +29 m. (es decir, la direccin en la cual apunta el bote)
Es decir, la piedra se localiza 29 m debajo del perpendicular al ro y al motor fijo para dar una
observador O despus de 3,0 s. rapidez constante de v m/s con respecto al agua. De
La velocidad de la piedra relativa a O' es v' = v -V; acuerdo a los datos
de aqu, en t =3,0s, (a) Cul es la velocidad del bote respecto a un
v' = 29 m/s 5,0 m/s = +24 m/s observador detenido en la orilla?
Puesto que V es constante, a' = a, y a'= +g = +9,8 (b) Hasta dnde estar el bote, medido corriente
m/s2. abajo paralelamente al ro, desde la posicin inicial
El observador O ve la piedra con la misma hasta cuando alcance la orilla opuesta?
aceleracin vista por O. (en general, las Solucin.
aceleraciones son iguales en todos los sistemas a)

inerciales.)
V = ui + vj
Ejemplo 31. Un automovilista viaja hacia el oeste b) La componente de la velocidad absoluta
sobre la Ruta Interestatal 80 a 80 km/h y es seguido perpendicular al ro determine el tiempo de cruce de
por un auto patrulla que viaja a 95 km/h. a
a) Cul es la velocidad del automovilista respecto acuerdo a t =
al auto patrulla?
v
Por lo tanto el bote avanza paralelamente al ro una
b) Cul es la velocidad del auto patrulla respecto al
distancia
automovilista?
Solucin. u
d = ut = a
Si el Oeste indica el sentido positivo entonces v
a) 80 - 95 = -15 km/h
b) 95 - 80 = 15 km/h Ejemplo 35. Un comprador que est en una tienda
puede caminar sobre una escalera mecnica en 30 s
Ejemplo 32. Un ro tiene una rapidez uniforme de cuando est detenida. Cuando la escalera mecnica,
0,5 m/s. Un estudiante nada corriente arriba una funciona normalmente, puede llevar al comprador
distancia de 1 km y regresa al punto de partida. Si el sin caminar al siguiente piso en 20 s. Cunto
estudiante puede nadar con una rapidez de 1,2 m/s tiempo le tomara al comprador al subir caminando
en agua tranquila, cunto dura el recorrido? con la escalera mecnica en movimiento? Suponga
Compare este resultado con el tiempo que durara el que el comprador hace el mismo esfuerzo al caminar
recorrido si el agua estuviera tranquila. sobre la escalera mecnica en movimiento o cuando
Solucin. est parada.
La rapidez absoluta (respecto a la ribera) cuando Solucin.
nada corriente arriba es 1,2 0,5 = 0,7 y cuando Sea L el largo de la escalera. Entonces la velocidad
nada corriente abajo es 1,2 + 0,5 = 1,7 entonces el de la persona respecto a la escalera es
tiempo de ida y vuelta ser
L
1000 1000 v' = .
t= + = 2016,81 s = 0,56 h 30
0,7 1,7 Sea ve la velocidad de la escalera. Ella corresponde a
la de la persona cuando no camina, es decir
Ejemplo 33. Dos remeros en idnticas canoas L
ejercen el mismo esfuerzo remando en un ro, uno ve =
corriente arriba (y se mueve corriente arriba), 20
mientras que el otro rema directamente corriente Si la escalera funciona y la persona camina, entonces
abajo. Un observador en reposo sobre la orilla del L L L
ro determina sus rapideces que resultan ser de V1 y v = ve + v' = + =
20 30 t
V2 respectivamente. Determine en trminos de los
de donde el tiempo ser
datos la rapidez de las aguas del ro.
t = 12 s
Solucin.

20
Movimiento en un plano y en el espacio Hugo Medina Guzmn

Ejemplo 36. El piloto de un avin observa que la


brjula indica que va dirigindose hacia el oeste. La
rapidez del avin respecto al aire es de 150 km/h. Si
existiera un viento de 30 km/h hacia el norte, calcule
la velocidad del avin respecto a la Tierra.

a) La direccin en la que debe dirigirse el avin est


dada por el ngulo .
Solucin. vv 50
cos = = = 0,25 = 75,5
La velocidad del viento es vv = 30 km/h y la rapidez v 200
del avin respecto al aire es v = 150 km/h. Debe dirigirse 75,5 direccin N-O.
Pero b) Su velocidad respecto a la Tierra es:

v = vj = 30i + v' v = v' 50i
Y su rapidez respecto a tierra es:
De donde v ' = vj 30i
y si tomamos magnitudes v = v'2 502 = 200 2 502
= 193,6 km/h
150 = v 2 + 302
v = 146,969 km/h Ejemplo 38. Un nio en peligro de ahogarse en un
ro est siendo llevado corriente abajo por una
Ejemplo 37. El piloto de un avin desea volar corriente que fluye uniformemente con una rapidez
hacia el oeste en presencia de un viento que sopla de 2,5 km/h. El nio est a 0,6 km de la orilla y a 0,8
hacia el sur a 50 km/h. Si la rapidez del avin km corriente arriba de un embarcadero cuando un
cuando no sopla el viento es de 200 km/h, bote de rescate se pone en camino.
a) en qu direccin debe dirigirse el avin? a) si el bote procede a su rapidez mxima de 20
b) cul debe ser su rapidez respecto a la Tierra? km/h con respecto al agua, cul es la direccin,
relativa a la orilla, que deber tomar el conductor del
bote?
b) Cul es el ngulo que hace la velocidad, v, del
bote con respecto a la orilla?
c) Cunto tiempo le tomar al bote para alcanzar al
nio?
Solucin.
a) Considerando al bote y al nio dentro del ro se
encuentran en un sistema inercial S.
En este sistema el nio esta en reposo y el bote se
Solucin. mueve con su velocidad, para poder alcanzar en el
La velocidad del viento es vv = 50 km/h hacia el sur menor tiempo el bote de enfilar con un ngulo
y la rapidez del avin respecto al aire es v = 200 relativo a la orilla dado por:
km/h.
0,6
Para poder volar directamente hacia el oeste con tan = = 1,5 = 37
respecto a tierra debe compensar el arrastre 0,8
producido por el viento, tal como se muestra en la
figura siguiente.

b) La velocidad del bote v, con respecto a la orilla


vx = 20 cos 37 +2,5 = 13,5 (1)

21
Movimiento en un plano y en el espacio Hugo Medina Guzmn

v y = 20sen37 = 12 (2) V 2 cot 2


h=
Dividiendo (2) : (1) 2g
vx 12
= tan = = 0,89
vy 13,5 Ejemplo 41. La brjula de un avin indica que se
est dirigiendo hacia el este con una velocidad de
= 41 400 km/h. La informacin de tierra indica que el
viento sopla hacia el norte con una velocidad de 300
km/h. cul es la velocidad del avin con respecto a
tierra?
Solucin.
En este caso tenemos dos sistemas, el sistema tierra
(S) y el sistema aire (S') que se mueve con una
velocidad de 300 km/h respecto a tierra.
c) El tiempo que le tomar al bote para alcanzar al
nio:
d
d = vt t =
v
Siendo v = 20 km/h y
d = 0,82 + 0,62 = 1,0 km
1
t= = 0,05 h = 3 min
20

Ejemplo 39. Desde el techo del carro de un tren que V = 300 j
est acelerando hacia el norte a una razn de 2,5
m/s2 se suelta y cae un perno. Cul es la aceleracin v' = 400i
del perno con respecto a:
a) el carro del tren? R =V t
b) la estacin?
Solucin: r ' = v' t
Si y es la vertical hacia arriba y x es la direccin de La posicin del avin visto desde O es
la aceleracin del tren, entonces
a) r = R+ r ' = V t + r '
La velocidad es
a ' = 2,5i 9,8 j .
b)
dr

v= = V + v'
dt
a = 9,8 j
Luego v = 300 j + 400i
Ejemplo 40. Un estudiante de la Facultad de Su magnitud
Ingeniera pasea sobre el vagn de un tren que viaja
km
a lo largo de una va horizontal recta a una rapidez v = 300 2 + 400 2 = 500
constante de V m/s. El estudiante lanza una pelota al h
aire a lo largo de una trayectoria que inicialmente 300
forma un ngulo de con la horizontal y est en = tan -1 = 37 o
lnea con la va. El profesor del estudiante, que est
400
parado cerca sobre la tierra, observa que la pelota El avin se dirige hacia el NE formando un ngulo
sale verticalmente. Qu altura subir la pelota? de 37 con la direccin este, el mdulo de la
Solucin. velocidad es 500 km/h.
Si V es la rapidez inicial de lanzamiento relativa al
tren, entonces en la direccin x tenemos: Ejemplo 42. Un nadador recorre una piscina de 100
m en 2 min. Va a nadar en un ro observando antes
Vx = V cos V = 0
de lanzarse e al agua, que un trozo de madera que
Porque el profesor observa que sale verticalmente.
flota en ella recorre 20 m en 1 minuto. Calcular el
V tiempo que tardar el nadador en recorrer 100 m en
V '=
cos el ro, segn vaya a favor o en contra de la corriente.
Luego Solucin.
Vy = Vy = Vsin= V cot
Subir una altura h dada por

22
Movimiento en un plano y en el espacio Hugo Medina Guzmn

La velocidad del nadador es:


s 100 m
vn = = = 50
t 2 min
m
La velocidad del agua del ro es: vr = 20
min
La velocidad nadando a favor de la corriente es:
v1 = v n + v r = 50 + 20 = 70 m/min Si en la figura y es el ancho del ro y x el avance
Y el que tarda en recorrer 100 m es: producido por la corriente, el camino recorrido por
s 100 la lancha es s.
t1 = = = 1 min 26 s
v1 70 s = x 2 + y 2 = 60 2 + 50 2
La velocidad nadando en contra de la corriente es: = 78,1 m
v 2 = v n v r = 50 - 20 = 30 m/min
Ejemplo 45. La velocidad que provocan unos
Y el que tarda en recorrer 100 m es:
remeros a una barca es de 8 km/h, La velocidad del
s 100 agua de un ro es 6 km/h, y el ancho de tal ro 100 m.
t2 = = = 3 min 20 s
v2 30 a) Suponiendo la posicin de la proa perpendicular a
las orillas, calcular el tiempo que tarda la barca en
cruzar el ro y la distancia a que es arrastrada, aguas
Ejemplo 43. Un acorazado navega con rumbo NE a
abajo, por la corriente.
una velocidad de 50,56 km/h. Suena zafarrancho de
combate y uno de los tripulantes marcha corriendo
de babor a estribor para ocupar su puesto, a una
velocidad de 10 km /h. Calcular el valor de la
velocidad resultante y su direccin.
Solucin.

b) En qu direccin debe colocarse la proa de la


barca para alcanzar el punto de la orilla opuesta
situado enfrente del de partida? (punto de partida y
llegada en la perpendicular comn a las orillas),

km km
v A = 55,56 , vT = 10
h h
V = 55,56 + 10 = 56,45 km/h
2 2

c) Qu velocidad, respecto a la tierra, lleva la barca


55,56 en los dos casos estudiados?
tan = = 79,8
10 d) Cunto tarda en atravesar el ro?.
= 79,8 45 = 34,8 = 34 47 49 Solucin.
a) vx = vr = 6 km/h, vy = vb = 8 km/h
La direccin ser 90 - = 55 12 11
y 0,1
y = vyt t = = h = 45 s
Ejemplo 44. Una pequea lancha atraviesa un ro de vy 8
50 m de. Anchura, al mismo tiempo la corriente lo La distancia a que es arrastrada por la corriente:
arrastra 60 m aguas abajo. Qu camino ha 0,1
recorrido? x = vxt = 6 Km = 75 m
Solucin. 8
b) Para que la barca vaya en la direccin de v2 la
componente horizontal de vb ha de ser igual a 6 km/h.
6
vb sen = vr sen =
8
= 48o 35
c) En el primer caso

23
Movimiento en un plano y en el espacio Hugo Medina Guzmn

b y c) La velocidad del bote con respecto a la orilla


v1 = v x2 + v y2 = 6 2 + 8 2
= 10 km/h es v neta = v B + v R .
En el segundo caso:
v2 = vbcos = 8cos 48 35 Como v B y v R son perpendiculares, tenemos
= 5,3 km/h v neta = v B2 + v R2
d) En el primer caso son 45 s ya calculados.
En el segundo caso: = 3 + 4 = 5m / s .
2 2

y 0,1 El ngulo mostrado en la figura se determina por


t= = h = 68 s
v2 5,3 vR
tan = .
vB
Ejemplo 46. Una canoa de 2,5 m de larga est junto
Para las velocidades dadas encontramos = 53,1 .
o
a la orilla de un ro y perpendicularmente a ella. Se
pone en marcha con una velocidad de 5 m/s y al El bote se mueve a lo largo de una lnea dirigida
llegar a la orilla opuesta ha avanzado en el sentido 53,1 ro abajo.
de la corriente 23,4 m.
a) Calcular la velocidad del agua sabiendo que el ro
tiene una anchura de 100 m.
b) Si la canoa marcha a lo largo del ro, determinar
el camino recorrido en 1 minuto segn vaya en el
sentido de la corriente o en sentido contrario.
Solucin .
a) La proa de la canoa debe recorrer un espacio en
direccin perpendicular al ro: d) Haciendo D = distancia ro abajo, tenemos
y = 100 2,5 = 97,5 m D v 4
siendo y = vc t = 97,5 m = R = , tal que D = 133 m.
el ro arrastra a la canoa x = 23,4 m = vr t 100 v B 3
dividiendo las dos anteriores
97,5 5 Ejemplo 48. Un submarino de propulsin
= vr = 1,2m / s convencional (Diesel) sufri un incendio en el
23,4 v r Atlntico norte despus de salir de Inglaterra.
b) v1 = vc + v r = 5 + 1,2 = 6,2 m/s Debido a un huracn no era posible enviar barcos ni
aviones para ayudar al submarino diesel. La marina
x1 = 6,2 x 60 =372 m decidi enviar un submarino de propulsin nuclear
v 2 = vc v r = 5 - 1,2 = 3,8 m/s para ayudar al de propulsin Diesel. El submarino
x2 = 3,8 x 60 =228 m diesel se encuentra al Sur a 500 km de distancia del
submarino nuclear (ver figura). La rapidez del
Ejemplo 47. Un bote de remos se dirige submarino nuclear respecto al agua es de 54 km/h.
perpendicular a la orilla de un ro. Los remos Adems, hay una corriente marina de 36 km/h que
pueden propulsar el bote con una velocidad de 3,0 se mueve al NE formando un ngulo de 30 respecto
m/s con respecto al agua. El ro tiene una corriente al norte. (Asuma que el eje x es el eje DE, y el eje y
de 4,0 m/s. es el NS).
(a) Construya un diagrama en el cual las dos a) Si V es el mdulo de la velocidad del submarino
velocidades se representen como vectores. nuclear visto desde tierra, escriba en forma vectorial,
(b) Encuentre el vector que representa la velocidad usando el sistema de coordenadas x -y, la velocidad
del bote con respecto a la orilla. del submarino nuclear respecto a tierra para que
(c) Qu ngulo forma este vector con la direccin llegue al submarino diesel y la velocidad de la
en la cual el bote est sealando? corriente marina con respecto a tierra.
(d) Si el ro tiene 100 m de ancho, determnese cuan b) Halle la velocidad del submarino con respecto a la
lejos ro abajo del punto del lanzamiento el bote corriente de agua.
llega al orilla opuesta. c) Calcule el mdulo de la velocidad V.
Solucin. d) Halle el tiempo en el cual los marineros son
Solucin: rescatados.
a) Diagrama.

24
Movimiento en un plano y en el espacio Hugo Medina Guzmn

a) A qu distancia del poste contada a lo largo de la


va, y a qu distancia de esta chocar el cuerpo con
el suelo?

b) Realcese un esquema de la trayectoria seguida


por el cuerpo

Dato: la altura inicial del objeto sobre el suelo es de


2,45 m

Solucin.
a) Si V es el mdulo de la velocidad del submarino
nuclear visto desde tierra, escriba en forma vectorial,
usando el sistema de coordenadas x-y, la velocidad
del submarino nuclear respecto a tierra para que
llegue al submarino diesel y la velocidad de la Solucin.
corriente marina con respecto a tierra.
km m
Velocidad del tren v y = 108 = 30 ,
h s

km m
Velocidad de la piedra vx = 36 = 10
h s
m
g 10
s2

b) Halle la velocidad del submarino con respecto a la a) El movimiento de la piedra lanzada est dada por
corriente de agua. las ecuaciones:

V sR = 54 sen i 54 cos j ,
1
x = 10t , y = 30t , z = 2,45 10t 2
v c = 36sen30 i + 36 cos 30 j = 18i + 31,18 j 2
18 1
54sen + 18 = 0 sen = = Cuando la piedra llega al suelo z = 0
54 3
cos = 0,94
1
z = 0 = 2,45 10t 2 t = 0,7 s
V sT = ( 54 cos + 31,18) j 2
= ( 50,76 + 31,18) j
Distancia del poste medida desde la va:
= 19,18 j y = 30t = 30(0,7) = 21m
c) Calcule el mdulo de la velocidad V.
19,18 km/hora
d) Halle el tiempo en el cual los marineros son Distancia de la va al punto de cada:
rescatados. x = 10t = 10(0,7) = 7m
d 500
t= = b)
V 19,18
= 26 horas

Ejemplo 49. Desde el interior de un tren que viaja a


108 km/h, un nio lanza un objeto por una ventana
con una velocidad de 36 km/h, horizontalmente y
perpendicularmente a la marcha del tren, justo en el
momento en que pasa en frente de un poste
indicador.

25
Movimiento en un plano y en el espacio Hugo Medina Guzmn

PREGUNTAS Y PROBLEMAS

1. La velocidad de la corriente de un ro aumenta en


proporcin a la distancia de la orilla y alcanza su b) r 1 = r 2 = 840i + 36 j , t = 12
valor mximo v 0 en el medio. Cerca de la orilla la
6. Las posiciones de dos partculas P1 y P2 estn
velocidad es cero. Un bote que navega en el ro tiene
( )

una velocidad u relativa al agua, constante y dadas por r 1 = 5 + 3t + 2t i ,
2
perpendicular a la corriente.
r 2 = (t + 5t 2 )i .

a) Encontrar la distancia que fue arrastrando el bote
al cruzar el ro de ancho C.
b) Determinar la trayectoria del bote a) En qu instante chocarn las dos partculas?
b) Cul es la diferencia de velocidades en ese
v0
Respuesta. a) d = C instante?
2u Respuesta: a) t = 2 b) 8

2. Un automovilista entra en una curva de 150 m de 7. El movimiento de una partcula est definido por
radio, una velocidad de 72 km/h. Accionando los el vector posicin
frenos hace disminuir su velocidad de modo
2 r = Rsenb t i + Ct j + Rcosb t k . Determinar.
uniforme a razn de 1,5 m/s .
Determinar el mdulo de la aceleracin del a) La velocidad y aceleracin de la partcula.
automvil cuando su velocidad es de 63 km/h. b) La trayectoria de la partcula.
2 c) El radio de curvatura.
Respuesta: 2,53 m/s
Respuesta. a) v = C 2 + R 2 b 2 , a = Rb 2 ,
3. Las ecuaciones paramtricas del movimiento de
C2
una partcula son x = R cos t , y = Rsen t , b) Helicoide, c) = R +
z = vt . R, , v son constantes. Rb 2
Probar que se trata de un movimiento uniforme,
8. El movimiento de una partcula est definido por
dibujar la trayectoria.
el vector posicin

Respuesta: Movimiento helicoidal con velocidad r = 0,1sen t i + 0,25cos2 t j , r en metros y t
angular y subiendo con velocidad v.
en segundos:
a) Determinar la velocidad y aceleracin para t = l s.
4. Dadas las ecuaciones paramtricas de un
b) Demostrar que la trayectoria de la partcula es una
movimiento x = Asen t , y = A cos t , parbola.
a) Escribir la ecuacin del movimiento.
b) La ley horaria Respuesta. a) v = 0,1 i m/s , a = 0
c) La trayectoria
b) y = 0,025 5 x
2

Respuesta. a) r = Asen t i + Acos t j , b)
9. La aceleracin de un cuerpo es:
s = At , c) x 2 + y 2 = A 2
( )
a = 3i + 2 j + k cm/s 2
5. Dos objetos se mueven en el plano xy de acuerdo a) Si el cuerpo parte del reposo Cul es su
( )
velocidad despus de 3 segundos?
a r 1 = 4t + 3t + 228 i + (2t + 12 ) j y
2
b) Cul es su posicin despus de 10 segundos?

r 2 = (8t 2 + 11t 444 )i + (5t 24) j


c) Cul es su rapidez media durante los primeros 10
segundos?
respectivamente. (
Respuesta. a) 9i + 6 j + 3k cm/s )
( )
a) Cuales son la velocidad y aceleracin de cada
objeto? b) 150i + 100 j + 50k cm
b) Dnde y cuando chocan? c) 18,71 cm/s
Respuesta.

a) v 1 = (8t + 3)i + 2 j , a 1 = 8i
10. Si una partcula que se mueve sobre una
trayectoria curva tiene una aceleracin total en un

v 2 = (16t + 11)i + 5 j , a 1 = 16i ( )

momento dado a = 3t + 2n cm/s . Hallar:
2

26
Movimiento en un plano y en el espacio Hugo Medina Guzmn

a) La aceleracin tangencial.
b) La aceleracin centrpeta. partcula 1 con aceleracin constante a = aj , y la
c) El mdulo de la aceleracin total. partcula 2 con aceleracin angular constante , en
d) El ngulo que la aceleracin total forma con la sentido contrario al movimiento de las agujas de un
tangente a la curva. reloj, describiendo una circunferencia de radio R,
Respuesta: a) at = 3 cm/s
2 como se muestra en la figura. Determine en funcin
de a y R:
b) a c = 4 cm/s
2
a) El tiempo que tardan en encontrarse, suponiendo
que lo hacen sobre el eje de las ordenadas, antes que
c) a = 5cm/s
2
la partcula 2 complete una vuelta completa.
d) = 53,1o Encuentre el valor de que hace esto posible.
b) Halle los vectores velocidad y aceleracin de las
11. Dos cuerpos se lanzan simultneamente desde un dos partculas para el instante del encuentro.
mismo punto con la misma rapidez inicial pero en
distintas direcciones, uno verticalmente hacia arriba
y el otro formando un ngulo = 60 con la
horizontal. Conociendo que la rapidez inicial de
ambos cuerpos es v 0 = 25 m/s, a qu distancia se
encontrarn cuando hayan pasado 1,7 s?

12. Una partcula se mueve en un plano de tal suerte 16. Un nio hace girar uniformemente una piedra en
que su radio vector con respecto a un punto fijo un crculo horizontal por medio de una cuerda de 1
barre ngulos iguales en tiempos iguales mientras m de longitud. El nio se encuentra sobre un
que la distancia al punto fijo es variable con el montculo de tal forma que el plano del movimiento
tiempo. Escriba las componentes radial y tangencial se encuentra a 5 m de altura sobre el suelo. La
de la velocidad y la aceleracin de la partcula cuerda se rompe y la piedra sale disparada
mostrando explcitamente cualquier cantidad que se horizontalmente, golpeando el suelo a 3 m de
mantenga constante durante el movimiento. distancia. Cul fue la aceleracin centrpeta de la
piedra mientras estaba en movimiento circular?
13. Un tren pasa por una estacin con una velocidad
de 30 km/h. En el instante en que la locomotora pasa
junto al guardagujas este lanza una bolsa a uno de
los ingenieros de maquinas. Sabiendo que la rapidez
inicial con que el guardagujas lanz la bolsa fue de
45 km/h
a) Cul tendr que ser el ngulo de lanzamiento
para lograr el objetivo?.
b) Describa la trayectoria de la bolsa en el sistema
de referencia del maquinista.
17. Desde un sistema de referencia situado en el
14. Un arquero est en una colina cuya pendiente suelo, con eje horizontal x y vertical y, se observa el
forma un ngulo con la horizontal. Si el arquero movimiento de un objeto sometido a una aceleracin
dispara la flecha segn una direccin respecto a
a = 2i 6 j (m/s). Si en el instante inicial el
la colina y con velocidad v 0 , encontrar la distancia, objeto se encontraba en el punto P = (-3, 2) (m),
medida a lo largo de la colina, a la cual caer la
movindose con una velocidad v (t = 0 ) = 3 j (m/s):
flecha.
a) Obtenga la ecuacin explcita de la trayectoria del
objeto.
b) Determine el instante en el que la velocidad y la
aceleracin son perpendiculares.
c) Calcule las coordenadas del punto ms alto de la
trayectoria.
d) Calcule el tiempo que tard el mvil desde que
sali del punto P hasta que lleg al suelo.

18. La figura muestra una cuenta p que desliza por


un alambre plano en forma de parbola. La ecuacin
15. Dos partculas se encuentran inicialmente en
de la parbola es y = x2/b, donde b es una constante
reposo en las posiciones que muestra la figura.
positiva con dimensiones de longitud. Llamaremos a
Ambas comienzan a moverse al mismo tiempo, la

27
Movimiento en un plano y en el espacio Hugo Medina Guzmn

al ngulo entre la tangente a la curva y el eje x. en el Respuesta.


punto donde se encuentra la cuenta. Llamaremos E y N a los vectores unitarios en
a) Halle tan en funcin de la coordenada x de P. direccin Este y Norte respectivamente.
b) Suponga que la cuenta tiene rapidez v y se mueve
v = (300 E + 60 N ) km/h, v = 60 26 km/h.

hacia la derecha. Halle las componentes x e y de la
velocidad de la cuenta en funcin de y y de la
coordenada x de P. 21. Un hombre gua su automvil bajo lluvia a una
Ayuda: recuerde que el vector velocidad es tangente velocidad constante respecto a Tierra de mdulo y
a la trayectoria. direccin. Mientras conduce el hombre observa que
la trayectoria de cada gota es una lnea recta que se
aparta un ngulo de la vertical y al detenerse
observa que la lluvia cae verticalmente y
prcticamente con velocidad constante. Halle el
vector velocidad de las gotas de lluvia respecto al
auto en movimiento y respecto a Tierra (tome
vertical hacia arriba).
Respuesta.
Respuesta. v
v gota ,Tierra = j ,
2x tan
a) tan =
b v
v gota , Auto = j vi
bv 2 xv tan
b) v x = , vy =
b + 4x
2 2
b + 4x 2
2
22. Un vagn de ferrocarril motorizado va cuesta
abajo sobre un plano inclinado un ngulo
19. Un ascensor parte del reposo y desciende con
aceleracin constante de 1 m/s2 respecto a Tierra.
. La distancia entre el techo y el piso del vagn es
Dos segundos despus de iniciarse el descenso se H y su aceleracin respecto a Tierra es constante y

cae la lmpara del techo del ascensor. La distancia
vale a = ai , ver figura. Un pasajero del vagn
del techo al piso del ascensor es de 2 m. Definimos
observa que una lmpara, situada en el centro del
el referencial del ascensor como aqul con origen en
techo del vagn, se desprende y choca con el piso en
su techo y direccin y positiva apuntando hacia
el punto O (en el extremo inferior del vagn).
abajo.
a) Halle la aceleracin de la lmpara respecto a
a) Halle los vectores aceleracin, velocidad y
Tierra y respecto al pasajero del vagn. Exprese sus
posicin de la lmpara respecto al ascensor.
b) Determine el tiempo que tarda la lmpara en caer. resultados en trminos de los vectores unitarios i y
c) Encuentre la distancia recorrida por el ascensor j .
mientras cae la lmpara. b) Escriba las componentes cartesianas de la
Respuesta. velocidad y posicin de la lmpara segn el pasajero.
Todas las unidades estn expresadas en el sistema Torne el origen en el punto o solidario al vagn y
MKS. L indica lmpara, A ascensor y T Tierra. llame L a la longitud del vagn.
a) Tomaremos como t = 0 el instante para el cual se c) Halle el tiempo que tarda la lmpara en caer y la
desprende la lmpara. longitud L del vagn.

d) Determine la ecuacin de la trayectoria de la
a LA = a LT a AT = 9 j , v LA = 9tj ,
lmpara, y = y ( x ) , segn el pasajero. Qu clase

9
r LA = t 2 j de curva es la trayectoria de la lmpara vista por el
2 pasajero y vista desde Tierra?
9 2 2
b) y LA = t = 2 t =
2 3
14
c) D =
9
20. Los instrumentos de un aeroplano en vuelo
horizontal indican que se dirige hacia el Este con
una rapidez de 300 km/h respecto al aire. En Tierra
se observa que el aeroplano se encuentra en medio Respuesta.
de una corriente de aire que sopla hacia el Norte con Los subndices L, P y T hacen referencia a la
rapidez de 60 km/h. Halle la velocidad y rapidez del lmpara, al pasajero y al referencial inercial de
avin respecto a Tierra. Tierra respectivamente.

28
Movimiento en un plano y en el espacio Hugo Medina Guzmn

24. El aro de la figura tiene radio R y rueda sobre


(
a) a LT = g sen i cos j , ) una superficie horizontal fija a Tierra.
El aro gira en sentido horario mientras su centro e se
a LP = (gsen a )i g cos j mueve hacia la derecha con rapidez
b) v x = (gsen a )t , v y = g cos t V respecto a la superficie. Considere un observador
con origen en C (se traslada con el aro)
1
x= (gsen a ) t 2 L , y que no rota respecto a Tierra. Suponga que todos
los plintos del aro tienen rapidez V respecto al
2 2 observador (se dice entonces que el aro rueda sin
1 deslizar).
y = g cos t 2 + H
2 En la figura se han marcado cuatro puntos para un
2 H (gsen a )
cierto instante. El punto A es el punto ms alto del
2H
c) t = , L= aro, el B el ms bajo, el D el punto del extremo
g cos g cos izquierdo y el E con un radio vector que forma un
d) Vista por el pasajero la trayectoria es una lnea ngulo con la vertical.
recta de ecuacin a) Halle la velocidad angular w del aro.
g cos b) Halle los vectores velocidad de los puntos A, B y
y= x D respecto a la superficie.
gsen a c) Halle el vector velocidad del punto E respecto a la
Vista desde Tierra la trayectoria es una parbola. superficie y diga para qu ngulo su mdulo es
igual a V.
23. La corriente de un ro fluye de Este a Oeste con
rapidez constante v = 2 m/s respecto a Tierra. Un
bote atraviesa el ro y de acuerdo a sus instrumentos
de a bordo se mueve respecto al ro dirigindose al
Norte con rapidez constante = 10 m/s. Respecto al
bote un pasajero se desplaza sobre la cubierta en
lnea recta desde el punto A hasta el punto G con
una rapidez constante v1 = 10 m/s. Suponga que BA
= 4 m y apunta hacia el Norte y BC = 3 m y apunta
hacia el Este.
Respuesta.
a) Halle el vector unitario u que apunta de A a C y a) La rapidez de cualquier punto del aro respecto a C
las velocidades del bote y del pasajero respecto a
es V = R , luego = V / R .
Tierra. r r r
b) Halle el tiempo que tarda el pasajero en ir de A b) V A = 2Vi , V B = 0 , V D = Vi + Vj
r r
hasta C. Qu distancia recorre el bote en ese tiempo c) V E = V (1 cos ) i + Vsen j , V E = V
segn un observador en Tierra?
= 60
25. Para conocer la rapidez de un avin es necesario
determinar cuanto tiempo toma volar en un rizo
cerrado de longitud conocida. Cunto tiempo
tomar al avin volar alrededor de un cuadrado de
lado a, con el viento soplando con una velocidad u?,
en dos casos:
a) la direccin del viento coincide con uno de los
lados del cuadrado;
b) la direccin del viento coincide con la diagonal
Respuesta. del cuadrado?
Las letras b, p y T designarn respectivamente el Sin viento la rapidez del avin es v, mayor que u.
bote, pasajero y Tierra. Respuesta,
a) u =
3i + 4 j
( )
, v b ,T = 2i 10 j m/s,
a) t1 =
(
2a v + v 2 u 2 )

(
v2 u 2 )
, b)

v p ,T = (4i 2 j ) m/s. 4a v 2 u 2 / 2
t2 =
1
b) t = s , d = 26m
(
v2 u 2 )
2
26. Un hombre que viaja en un camin intenta
golpear un poste con una piedra, y cuando pasa

29
Movimiento en un plano y en el espacio Hugo Medina Guzmn

frente a l arroja la piedra con una velocidad


horizontal de 20 m/s respecto al camin. Sabiendo
que la velocidad del camin es de 40 km/h, Calcular:
a) la direccin en que debe lanzar la piedra.
b) la velocidad horizontal de la piedra respecto al
suelo.
Respuesta. a) 56,3 con relacin a la direccin
trasera del camin
b) 16,63 m/s

27. El piloto A est volando con un avin con una


velocidad de 150 km/h, sobrevolando al piloto B, 30. Un automvil viaja hacia el Este con una
cuyo avin vuela a 135 km/h, 300 m por debajo Con rapidez de 50 km/h. Est lloviendo verticalmente
el mismo rumbo. El piloto A para mandar un con respecto a la Tierra. Las marcas de la lluvia
mensaje a B lo sujeta a una piedra y la arroja a la sobre las ventanas laterales del automvil forman un
cabina de B. Sin tomar en cuenta la resistencia del ngulo de 60 con la vertical, calcule la velocidad de
aire. la lluvia con respecto a:
a) Con qu velocidad deber lanzarla respecto a su a) el automvil y
avin cuando B est directamente debajo de l? b) la Tierra.
b) Cundo B est todava a 300 metros delante de l?
Respuesta, a) v = 15 km/h hacia atrs; b) v = 128 31. La distancia de A a B es l . Un aeroplano vuela
km/h haca adelante. desde A hasta B y vuelve otra vez con una velocidad
constante V relativa al aire. Calcular el tiempo, total
28. Una partcula describe una circunferencia de que emplear en realizar el recorrido si el viento
radio R = 0,5 m con una frecuencia de sopla con una velocidad v en las siguientes
10 r pm. Si en t0= 0 la partcula est en la posicin A direcciones:
movindose en el sentido horario, calcular: a) Sobre la lnea que une A y B.
a) El perodo T y la rapidez del movimiento b) Perpendicular a esta lnea.
b) La velocidad media y aceleracin media en el c) Formando un ngulo con esta lnea.
intervalo (0; 0,75T). Demostrar que la duracin del trayecto siempre
c) La aceleracin en t =T / 2 aumenta con la existencia del viento.
Respuesta.
2l
Poniendo T0 = , los resultados son:
V
T0
a)
v2
1 2
V
T0
b) 12
v2
29. Una partcula P se mueve con aceleracin
1 2
angular constante sobre una circunferencia de radio V
1
R =3m. Parte desde el reposo del punto A y
completa la primera vuelta en un tiempo t = 2s. vsen 2
2

Calcular: 1
a) El mdulo de la aceleracin angular V
c) T0
v2
b) La ecuacin r = r(t ) . 1 2
c) El tiempo que emplea para llegar a la posicin V
definida por = 3/2 .
d) La velocidad lineal en = 32. El bloque deslizante A se mueve hacia la
izquierda a una velocidad constante de 0,3i m/s,
Determinar:
a) La velocidad del bloque B;
b) las velocidades de los tramos de cable C y D;
e) la velocidad relativa de A respecto a D;
d) La velocidad relativa del tramo de cable C
respecto al tramo D.

30
Movimiento en un plano y en el espacio Hugo Medina Guzmn

Respuesta.
a) 0,2i m/s, b) 0,2i m/s, 0,4i m/s,
c) 0,1i m/s, d) 2i m/s,

31
Dinmica de una partcula Hugo Medina Guzmn

CAPTULO 4. Dinmica de una partcula


INTRODUCCIN
En el captulo anterior estudiamos el movimiento de
una partcula con respecto a un sistema de referencia
sin preguntarnos sobre la causa del movimiento. Lo
describimos simplemente en trminos de los vectores

r , v y a.
Nuestra discusin fue geomtrica, en este captulo
discutiremos la causa del movimiento. Seguiremos
tratando a los cuerpos como partculas simples. En ausencia de una fuerza resultante, el objeto se
Posteriormente trataremos sobre sistemas de mantiene en movimiento con velocidad uniforme o
partculas y cuerpos rgidos. permanece en reposo. Esta es la PRIMERA LEY
DE NEWTON DEL MOVIMIENTO
EL ORIGEN DEL MOVIMIENTO Ahora podemos pensar acerca de la situacin cuando
Qu origina el movimiento? Qu detiene el un objeto era empujado sobre un plano. Cuando la
movimiento? Se necesita causa para mover las fuerza era pequea no haba movimiento, pero una
cosas? Por qu un objeto al que se le da un empujn fuerza debera causar movimiento; la conclusin es
pronto se detiene? Por qu los planetas mantienen su que debe haber otra fuerza actuando sobre el cuerpo
movimiento alrededor del sol? la cual anula justamente el efecto de la fuerza que
Aristteles joven filsofo griego (siglo IV a.c.) deca aplicamos. Al incrementar nuestra fuerza, la fuerza
que un cuerpo permaneciera en movimiento era opuesta tambin se incrementa, hasta que en algn
necesario ejercer alguna accin sobre l ya que el valor particular la fuerza opuesta termina de
estado natural es el reposo. Esto parece ser razonable, incrementarse y comienza el movimiento porque hay
cuando dejamos de empujar un cuerpo, este pronto una fuerza resultante actuando sobre el objeto. La
alcanza el reposo. Parece ser necesaria una accin fuerza opuesta es la fuerza de Friccin
exterior o fuerza aplicada al cuerpo para mantener el
movimiento. Sin embargo, observemos esta situacin QU ES FUERZA? En la vida cotidiana se
con mayor detenimiento. La figura siguiente muestra considera fuerza a una sensacin comn asociada con
un bloque de madera sobre un plano. la dificultad para mover o levantar un cuerpo. En
Fsica se identifica una fuerza por el efecto que
produce. Uno de los efectos de una fuerza es cambiar
el estado de reposo o de movimiento del cuerpo, ms
concretamente, una fuerza cambia la velocidad de un
objeto, es decir produce una aceleracin. Cuando se
aplica una fuerza sobre un cuerpo y no se produce
Aplicamos una fuerza pequea al bloque, no pasa movimiento, entonces puede cambiar su forma, an si
nada. Incrementamos la fuerza y a un valor particular el cuerpo es muy rgido. La deformacin puede o no
el bloque se mueve. Si seguimos incrementando la ser permanente. Entonces los efectos de la fuerza neta
fuerza empujando o jalando ms, el objeto se mueve son dos: cambiar el estado de movimiento de un
con mayor rapidez, Cuando dejamos de empujar el cuerpo o producir una deformacin, o ambas cosas.
cuerpo rpidamente vuelve al reposo. Sin embargo si Normalmente sobre un cuerpo pueden actuar varias
ponemos ruedas al bloque el resultado es diferente, fuerzas, entonces el cuerpo acelerar cuando el efecto
una fuerza muy pequea causa el movimiento. La de la fuerza neta que acta sobre l no es cero.
diferencia son las ruedas debido a la friccin. Se llama fuerza neta o fuerza resultante a la suma de
Para hacer un estudio libre de la friccin busquemos todas las fuerzas que actan sobre un cuerpo. Si la
llegar cercanamente a esta condicin, una forma de fuerza neta es cero, la aceleracin es cero, el
lograr esto es con una mesa neumtica, se sopla aire movimiento es con velocidad igual a cero (cuerpo
sopla hacia arriba a travs de pequeos agujeros detenido) o con velocidad constante. Cuando un
manteniendo un disco suspendido sobre un colchn cuerpo est en reposo o se mueve con velocidad
de aire. Qu pasa cuando empujamos un objeto en constante, se dice que est en equilibrio.
ausencia de friccin? Este se mantiene en Se pueden distinguir dos grandes clases de fuerzas:
movimiento a velocidad constante. fuerzas de contacto, representan el resultado del
contacto fsico entre el cuerpo y sus alrededores, por
ejemplo mover un carro o estirar un resorte; y fuerzas
de accin a distancia que actan a travs del espacio
sin que haya contacto fsico entre el cuerpo y sus
alrededores, por ejemplo la fuerza con que la Tierra
atrae a los cuerpos que caen en cada libre. Todas las
diferentes formas de fuerzas se encuentran dentro de
esas dos grandes clasificaciones.

1
Dinmica de una partcula Hugo Medina Guzmn

Para describir el mundo, la fsica contempornea si se estira 2,5 unidades, entonces la fuerza aplicada
recurre a cuatro interacciones o fuerzas es 2,5 veces la unidad de fuerza.
fundamentales, que actan sobre las partculas de Este procedimiento es vlido para pequeos
materia (y sobre las antipartculas), son vehiculadas alargamientos del resorte, ya que si la fuerza es muy
por unas partculas llamadas vectores de interaccin, intensa, se puede deformar y no volver a su forma
que son: fotn (interaccin electromagntica), bosn original.
(interaccin dbil), glun (interaccin fuerte) y
gravitn (interaccin gravitacional). CAMBIO DE VELOCIDAD
1) Fuerzas electromagnticas de atraccin o repulsin Nuestro siguiente problema es encontrar una relacin
entre partculas cargadas en reposo o en movimiento, entre la fuerza y el cambio en el movimiento
explica la cohesin de los tomos, es mucho ms producido por sta.
intensa que la fuerza gravitacional. Para esto necesitamos lo siguiente:
2) Fuerzas nucleares intensas entre partculas 1. Un carro muy ligero que pueda moverse sin
subatmicas, responsable de la existencia del ncleo friccin sobre una superficie horizontal.
atmico asegura la cohesin interna de los 2. Una fuerza constante. Esta podernos obtenerla
constituyentes del ncleo atmico, protones y mediante un resorte (Si mantenemos un resorte
neutrones, y es responsable de un gran nmero de estirado una misma longitud, la fuerza que la estira es
reacciones y de desintegraciones; es la de mayor constante).
magnitud (102 - 103 veces la fuerza
electromagntica).
3) Fuerzas nucleares dbiles de corto alcance, rige
algunos procesos radiactivos, establece la estabilidad
de algunos ncleos, es varios rdenes de magnitud
(1012) menor que la fuerza electromagntica.
4) Fuerza de atraccin gravitacional entre cuerpos 3. Un registrador de tiempo. El movimiento del carro
debido a sus masas, entre otras cosas hace que caigan puede estudiarse si una cinta de papel atada a ste
las manzanas y que suba la marea, es la fuerza de pasa a travs del registrador que produce marcas en la
menor magnitud comparada con las otras. cinta a intervalos de tiempo regulares.
Para que el concepto de fuerza sea exacto se debe
establecer un mtodo para medirla. Una fuerza se
puede medir por el efecto que produce. Por ejemplo
se puede usar la deformacin que una fuerza produce
en un resorte, como en la figura. Si se aplica una
fuerza verticalmente a un resorte y se estira una
unidad, le asignamos a la fuerza una magnitud
unitaria F. Se aplica ahora otra fuerza al mismo
resorte horizontalmente, producindole un
estiramiento de dos unidades, la magnitud de la
fuerza ser de 2F. Si se aplican simultneamente las
dos fuerzas, el resorte se inclina, y se estira 5
veces. La fuerza equivalente que produce ese
estiramiento del resorte es la suma vectorial de F y
2F. Es decir, la fuerza es un vector.

La figura siguiente muestra la cinta de papel


producida por una fuerza constante.

El instrumento para medir fuerzas se llama


dinammetro, es un resorte que se estira sobre una
escala. Si se aplica una fuerza de una unidad sobre el
dinammetro, el resorte se estira hasta que ejerce una
fuerza igual y contraria a la aplicada. En la escala se Con los datos obtenidos en esta experiencia se realiza
mide el alargamiento del resorte y se le asigna una el grfico distancia - tiempo y se obtiene una curva.
unidad de fuerza. De esa manera se calibra el Con los datos tambin se puede obtener la velocidad
dinammetro y se usa para medir fuerzas, por media en cada intervalo de tiempo. El grfico
ejemplo se aplica una fuerza sobre el dinammetro y velocidad - tiempo es una lnea recta que indica que

2
Dinmica de una partcula Hugo Medina Guzmn

el movimiento es con aceleracin constante. De aqu internacional (S.I.) la unidad de aceleracin es m/s.
podemos concluir que una fuerza constante produce Cuales son las unidades de fuerza y de masa? Como
una aceleracin constante. son dos cantidades que se relacionan slo tenernos
que especificar un estndar para una de ellas.
El sistema internacional adopta corno unidad una
pieza de material llamado KILOGRAMO, cuyo
smbolo es kg. El kilogramo es la masa un prototipo
de platino iridiado sancionado por la Conferencia
General de Pesas y Medidas realizada en Pars en
1889 y depositado en el pabelln de Breleuil en
Sevres.
Si duplicamos la fuerza usando dos resortes iguales La unidad de fuerza es el newton, cuyo smbolo es N
estirados la misma longitud, como se muestra en la y se define as:
figura. El newton la fuerza que produce una aceleracin de
un metro por segundo al cuadrado a una masa de un
kilogramo.
kgm
N=
Duplica la fuerza y produce el doble de aceleracin. s2
Si triplicamos la fuerza se obtiene una aceleracin de Otros sistemas:
valor triple. MKS: igual al S.I.
Concluimos que la aceleracin a del cuerpo es CGS: Masa gramo (g), l g = 10-3 kg
directamente proporcional a la fuerza. Aceleracin cm/s2
aF Fuerza dina = g.cm/s2
Podemos escribir esto como F = ma , donde m es la Ingls tcnico: En este sistema la unidad fundamental
constante de proporcionalidad. A esta constante la es la unidad de fuerza.
llamaremos MASA. Fuerza libra (lb), 1 lb = 4,45 N
Para una determinada fuerza a mayor constante m la Aceleracin pie/s2
aceleracin es menor. A mayor valor de la constante Masa slug = lb58
es ms difcil acelerar el cuerpo. s2/pie
Para conocer qu factores cambian esta constante
realicemos el siguiente experimento: en lugar de usar PESO DE UN CUERPO. El peso de un cuerpo es la
un solo carro jalado por el resorte estirado usemos fuerza de atraccin que ejerce la Tierra sobre el
dos carros uno sobre otro y luego tres carros como se cuerpo. Un cuerpo de masa m sometido a cierta
muestra en la figura fuerza cae con la aceleracin de la gravedad g, el
peso P de este cuerpo es

P = mg

La aceleracin que se obtiene con los carros es igual a


la mitad y con tres es igual a un tercio. Como el valor
de F es igual en todos los casos, quiere decir que la
constante con dos carros es igual a 2m y con tres
carros es 3m.
Como la aceleracin es una cantidad vectorial la
fuerza tambin lo es y tiene la misma direccin que la
aceleracin, pero un mdulo m veces mayor, de modo
que la relacin anterior puede escribirse en la forma Su direccin es hacia abajo (hacia el centro de la
Tierra). Como el peso es una fuerza debe medirse en
F = ma Newtons.
Fuerza = masa x aceleracin. Debido a que la aceleracin de la gravedad vara de
Esta expresin constituye la SEGUNDA LEY DE un lugar a otro de la Tierra, el peso de un cuerpo es
NEWTON DEL MOVIMIENTO. diferente en lugares distintos, sin embargo la masa de
La fuerza que acta sobre un cuerpo es igual al un cuerpo es la cantidad fija que no depende del lugar
producto de la masa del cuerpo por la aceleracin que donde est situado el cuerpo,
le imprime. Aunque el peso de un objeto vara de un sitio a otro,
esta variacin es demasiado pequea para ser
UNIDADES DE FUERZA Y MASA observada en la mayor parte de las aplicaciones
La relacin F = ma nos da una relacin entre prcticas, por esto, el peso de un cuerpo parece ser
fuerza, masa y aceleracin. En el sistema una caracterstica constante al igual que su masa. Este

3
Dinmica de una partcula Hugo Medina Guzmn

hecho ha conducido al empleo ordinario de otras dos aplicacin de las leyes de Newton.
medidas:
KILOGRAMO FUERZA, es el peso de un Kilogramo APLICACIONES DE LAS LEYES DE NEWTON
masa. Cuando estudiamos Cinemtica, encontrarnos las
1 kgf = 9,8 N relaciones entre desplazamiento, aceleracin y
LIBRA MASA, es la masa de un cuerpo que pesa una
libra. tiempo. Por ejemplo, conociendo la aceleracin a
1 libra masa = 0,454 kg. las condiciones tales como posicin inicial, velocidad
Estas unidades son prcticas pero incorrectas y no inicial, es decir la posicin y la velocidad en el
deben ser usadas en Fsica. tiempo que llamamos inicial (t = 0), podemos conocer
la velocidad y posicin para cualquier tiempo. Las
ACCION Y REACCION. condiciones iniciales las tenemos pero la aceleracin,
Hagamos una observacin ms detallada cuando
jalamos el carro con un resorte estirado una de dnde? Para esto tenemos F = m a , todo lo que
determinada longitud. tenemos que hacer es conocer las fuerzas sobre el

cuerpo y su masa, y entonces podremos encontrar a .
La mejor forma de estar seguros que comprendemos

el significado de F = m a , es hacerlo con algunos
problemas que involucran las leyes de Newton. Para
Para que el resorte est estirado es necesario jalarlo resolver un problema sugerimos cuatro pasos a
por los dos lados. Se necesitan fuerzas en sentidos seguir:
opuestas y en cada extremo del resorte. 1. Dibujar un esquema del sistema
Cuando jalamos el carro, una fuerza acta sobre el 2. Identificar el cuerpo a cuyo movimiento se refiere
carro y una fuerza en sentido opuesto acta sobre el problema.
nuestra mano. Cules son las magnitudes de estas 3. Dibujar otra figura con solamente el objeto en
fuerzas? particular manteniendo el marco de referencia poner
todas las fuerzas que actan sobre el objeto mediante
flechas. Esto se conoce como DIAGRAMA DEL
CUERPO LIBRE (DCL). Si se comete una
equivocacin todo lo dems fallar, por eso es
conveniente hacerlo bien. Una mejor forma de
comenzar es poner la fuerza de gravedad primero y
Con el objeto de dar respuesta a esta pregunta luego preguntarse:
pongamos dos resortes iguales al primero y jalemos Qu toca al cuerpo?, la accin de tos resortes,
de tal manera que el carro adquiera la misma cuerdas, manos y otros objetos, todos deben ser
aceleracin que antes, esto quiere decir, por la considerados. As como tambin las fuerzas que
segunda ley de newton que siendo la misma masa m actan sin tocar el cuerpo, como la fuerza elctrica,
estamos aplicando la misma fuerza (F = ma) que magntica de las cuales no nos preocupamos en este
antes y observamos que los resortes estiran la misma curso.
longitud, lo que quiere decir que la fuerza sobre la 4. Finalmente, aplicar la segunda ley de Newton a
mano es igual a la fuerza sobre el carro. cada componente de fuerza y aceleracin.

F =ma
Fx = ma x , F y = ma y , Fz = ma z .
y ahora resolver para la aceleracin.
En algunos de los problemas que se presentan ms
Esto constituye la TERCERA LEY DE NEWTON frecuentemente, las acciones se producen por fuerzas
DEL MOVIMIENTO. sin contacto; en otros se usan cuerdas y varillas como
Si un cuerpo ejerce una fuerza sobre un segundo, ste medios de conexin. Cuando las masas de estos
ejerce una fuerza igual y opuesta sobre el primero. La medios de conexin son despreciables su nico efecto
fuerza ejercida por el primer cuerpo sobre el segundo es el de transmitir
es la ACCIN, la fuerza igual y opuesta actuando
sobre el primero es la REACCIN, ESTTICA DE LAS MASAS PUNTUALES.
Expresado en smbolos, es: Los sistemas en los cuales todas sus partes satisfacen
la primera ley son llamados sistemas estticos, es
F sobre 2 debido a 1 = F sobre 1 debido a 2 decir si la suma vectorial de todas las fuerzas que
Fuerza de contacto de un cuerpo a otro con un cambio actan es nula, el cuerpo esta en equilibrio y
de direccin o sin l permanece en reposo, o si est en movimiento, se
A continuacin presentarnos algunos casos tipo de la mantiene con velocidad constante

4
Dinmica de una partcula Hugo Medina Guzmn

La condicin de este equilibrio es



F = 0
y en componentes cartesianas:
F x = 0, F y = 0 , Fz = 0 .
Las fuerzas son ejercidas sobre el objeto o sistemas
por. Medios exteriores al sistema.

Ejemplo 1. La Fuerza gravitacional Dado que la


aceleracin de un cuerpo en cada libre en la tierra es
g, cul es la fuerza de la gravedad?
Solucin.
Como este movimiento es en una sola dimensin,
consideramos que este se realiza en el eje z, tal que

a = gk
Segn la Segunda Ley de Newton
Diagrama del cuerpo libre (DCL)
F = m a = mgk Aplicando la condicin de equilibrio de la masa m
T1 mg = 0
Fx = 0 , Fy = 0 , Fz = mg . Luego T1 = mg
Siendo esta la respuesta que ya conocamos.
Si despreciamos la masa del dinammetro, tenemos
Ejemplo 2. El dinammetro. El dinammetro es un que:
instrumento que se utiliza para medir las fuerzas. T1 T2 = 0 y T1 = T2
Consta de un resorte con una escala que indica su El dinammetro indica en la escala la fuerza
estiramiento, la cual est graduada en Newtons.
Cuando lo utilizamos para pesar se dispone como lo
T2 = mg
muestra la figura.
Ejemplo 3. Se tiene los dispositivos mostrados en la
figura. Cunto indica el dinammetro de la figura (a)
y cunto el dinammetro de la figura (b)?

Se suspende la masa m, el resorte del dinammetro se


estira hasta que alcanza el equilibrio esttico.

Solucin.
a) El diagrama de cuerpo libre de la figura (a) es

5
Dinmica de una partcula Hugo Medina Guzmn

En la masa
T1 mg = 0 T1 = mg
En la polea
T1 = T2
En el dinammetro
T3 = T2 = T1 = mg
El dinammetro es tensionado por la fuerza T1 y su
indicacin ser
Empezando por la derecha
T1 mg = 0 T1 = mg T1 = mg
Como se puede ver esta situacin es completamente
La figura siguiente muestra la polea
anloga a la anterior, slo que hemos sustituido una
de las poleas por la pared.

Ejemplo 4. Un cuerpo de masa m se sostiene por


medio de cuerdas como se muestra en la figura.
Encontrar las tensiones T1, T2 en las tres cuerdas.

Para que el trozo de cuerda este en equilibrio



F = 0
Descomponiendo las fuerzas sobre el trozo de cuerda
en los ejes x e y.
Como la cuerda se considera sin masa la suma de
fuerzas a lo largo del eje x es
T1 cos T2 cos = 0 T1 = T2 Solucin.
En el dinammetro, considerndolo de masa
despreciable.

F = 0
T2 T3 = 0 T2 = T3
En la polea de la izquierda
T4 = T3
En la masa de La izquierda Tomando un sistema de ejes horizontal y vertical
como el mostrado en la figura tenemos:
F = 0
T1 = mgj
T4 mg = 0 T4 = mg
Como conclusin todas las tensiones son iguales a T2 = T2 cos i + T2 sen j
mg

T4 = T3 = T2 = T1 = mg T3 = T3 cos i + T3sen j

El dinammetro es tensionado por la fuerza
indicacin ser:
T1 , y su Con F = 0

T1 = mg T1 + T2 + T3 = 0
b) El diagrama de cuerpo libre de la figura siguiente Obtenemos:
es
F x = T2 cos T3 cos = 0
F y = T2 sen + T3sen mg = 0
Resolviendo estas dos ecuaciones
mg cos mg cos
T2 = , T3 =
sen ( + ) sen ( + )

Ejemplo 5. Un bloque de 50N de peso se ubica sobre


un plano inclinado en un ngulo de 30 con la

6
Dinmica de una partcula Hugo Medina Guzmn

horizontal. El bloque se sujeta con una cuerda ideal Solucin.


que se encuentra fija en la parte superior del plano El D. C. L. del cuerpo:
inclinado, como en la figura. Estudiar el
comportamiento mecnico del bloque.

Del diagrama de cuerpo libre se obtiene:


F x : mgsen = ma x

Solucin.
F y : N mgcos = ma y = 0
El D. C. L. del cuerpo: De estas ecuaciones se obtiene:
a x = gsen y N = mg cos
Se concluye que la aceleracin del bloque en
direccin del plano inclinado es la componente de g
en esa direccin. Estudiando ahora el movimiento del
bloque, considerando que parte del reposo y se
desliza una distancia D, se puede calcular la rapidez
con que llega a la base del plano. Si se considera que
el movimiento del bloque comienza desde el reposo,
se puede usar:
Fuerza de atraccin de la Tierra, que es su peso mg.
Fuerza de la cuerda que lo sostiene, que es la tensin v 2 = v 02 + 2a x x v 2 = 2( gsen )D
T
y v= 2gDsen
Fuerza que el plano ejerce sobre el cuerpo, que es la
normal N
Como el sistema est en equilibrio, se aplica la Ejemplo 7. Para el siguiente sistema mecnico,
primera Ley de Newton: calcular la aceleracin de las masas y la tensin de la
cuerda.

Del diagrama de cuerpo libre se obtiene:


F x : T + mgsen = 0
F y : N mgcos = 0
Despejando T y N, y reemplazando los valores
numricos, se obtiene: Solucin.
T = mgsen = 50sen30 = 25 N Como no se conoce la direccin del movimiento,
N = mgcos = 50cos30 = 43,2 N supongamos que el cuerpo de masa M sube por el
plano inclinado, lo que determina el sentido de la
aceleracin, entonces aplicando la segunda Ley de
DINMICA CON FRICCIN Newton se aplica cada masa:
DESPRECIABLE. El D. C. L. del cuerpo M:
Los sistemas en los cuales todas sus partes satisfacen
la primera ley son llamados sistemas estticos, es
decir si la suma vectorial de todas las fuerzas que
actan no es nula y la friccin se considera
despreciable,
Ejemplo 6. Si un bloque de masa m se ubica sobre un
plano sin roce, inclinado un ngulo con la
horizontal, resbalar una distancia D a lo largo del
plano. Describir su movimiento. Del diagrama de cuerpo libre se obtiene:
F xT Mgsen = Ma
:
T = Mgsen + Ma
Fy : N Mgcos = 0
De estas ecuaciones se obtiene:
El D. C. L. del cuerpo m:

7
Dinmica de una partcula Hugo Medina Guzmn

Del diagrama de cuerpo libre se obtiene:


F y : T mg = ma T = mg ma
De estas ecuaciones se obtiene
Mgsen + Ma = mg ma
(m Msen ) g a) Sumando (1), (2) y (3):
a= F1 (m1 + m2 + m3 )g = (m1 + m2 + m3 )a
(m + M )
Se observa que el signo de a depende del trmino F1
y a= g
(m - M sen ). (m1 + m2 + m3 )
Ahora se calcula el valor de la tensin reemplazando
560
el valor de a en T: a= 9,8 = 8,87 m/s2
m Msen (20 + 8 + 2)
T = mg m g
m+M b) De (3) FB = m2 ( g + a )
mM FB = 8(9,8 + 8,87 ) = 149,4 N
T= (1 + sen )g De (1) FA = F1 m1 ( g + a )
(m + M )
FA = 560 20(9,8 + 8,87 ) = 186,6 N
Ejemplo 8. Dos bloques de masas m1 = 20 kg y
m2 = 8 kg, estn unidos mediante una cuerda Ejemplo 9. La mquina de ATWOOD. Es un aparato
homognea inextensible que pesa 2 kg. Se aplica al que se utiliza para determinar con exactitud la
conjunto una fuerza vertical hacia arriba de 560 N.
Calcular: gravedad y consiste de dos masas m1 y m 2 ,
a) La aceleracin del conjunto; ( m1 > m 2 ), que estn unidas mediante una cuerda
b) Las fuerzas que actan en los extremos de la que pasa sobre una polea. Considerar la cuerda
cuerda. inextensible y sin masa. Asimismo, no tornar en
cuenta la friccin y la masa de la polea. Describir el
movimiento y calcular la tensin en la cuerda.

Solucin.
En el D. C. L. de m1:
F1 FA m1 g = m1 a (1) Solucin.
En el D. C. L. de la cuerda de masa m3: Siendo m1 mayor que m 2 , la masa m1 se mover
FA FB m3 g = m3 a (2) hacia abajo con una aceleracin a y la masa m 2 se
En el D. C. L. de m2: mover hacia arriba con la misma aceleracin a .
FB m2 g = m2 a (3) La figura siguiente muestra los diagramas de cuerpo
libre de cada una de las partes del sistema.

8
Dinmica de una partcula Hugo Medina Guzmn

Ahora la reaccin del piso es R ' .


Aplicando la Segunda Ley de Newton al movimiento
de la persona
R' mg = ma R ' = m( g + a )
Si el ascensor sube el pasajero se siente ms pesado,
como si fuera empujado contra el piso. Si el ascensor
desciende con esta aceleracin,
R ' mg = ma R ' = m( g a ) , el pasajero se
siente ms liviano.

La polea cumple la funcin de cambiar la direccin Ejemplo 11. La figura muestra a un hombre
T1 Considerando el sentido de la aceleracin o como elevndose mediante una fuerza vertical que aplica l
positiva. mismo a la cuerda que tiene en las manos. Si el
hombre y la silla juntos tienen una masa de 100 kg.
Aplicando la Segunda Ley de Newton a la masa m1 Se pregunta:
m1 g T1 = m1 a a) Con qu fuerza debe jalar para, subir con una
Aplicando la Segunda Ley de Newton para la masa velocidad constante?
b) Con qu fuerza debe jalar para subir con una
m2 : aceleracin de l m/s2 (considerar g = 10 m/s2?
T1 m2 g = m2 a
De estas dos ecuaciones obtenemos:

a=
(m1 m2 ) 2m1 m2
g y T1 = g
(m1 + m2 ) (m1 + m2 )
Si las masas m1 y m 2 fueran casi iguales, el valor de
la aceleracin sera pequea y podra determinarse
midiendo el tiempo en que una de las masas sube o
baja una distancia determinada.
La razn (m1 m2 ) se determina pesando los cuerpos.
(m1 + m2 )
Finalmente, la magnitud de g se obtiene a partir de
estas cantidades mediante la ecuacin Solucin.

g=
(m1 + m2 ) a) La figura siguiente muestra los diagramas de
a
(m1 m2 ) cuerpo libre de cada una de las partes del sistema.

Ejemplo 10. El peso de un pasajero en ascensor.


Consideremos un pasajero de peso mg en un ascensor
este peso es equilibrado por la reaccin que el piso
ejerce sobre l, si el ascensor estuviera parado
R = mg .
Si el ascensor sube con aceleracin a. Cul es el
peso de la persona?
Solucin.
La figura muestra el ascensor subiendo con una
aceleracin a

Como se considera la cuerda con masa despreciable


en el D.C.L. del trozo de cuerda
T=F
La polea solo cambia la direccin de la tensin T .
En el D.C.L .del hombre-silla
T + F W = 0 2F = W
W
y F =
2
Como W = 100 10 = 1000 N

9
Dinmica de una partcula Hugo Medina Guzmn

1000
F= = 500 N
2
b) Ahora como el hombre debe subir con una
aceleracin de l m/s2 tenemos:

Aplicando la Segunda Ley de Newton


T1 m1 g = m1 a T1 = m1 (a + g )
T1 = 1100(2 + 9,8)
W W = 12980 N
T + F W = a 2F = W + a
g g b) Consideremos el D.C.L. de la masa m 2 :
W a
y F = 1 +
2 g
Como W = 1000 N , a = 1 m/s 2 y = 1 m/s 2
1000 1
F= 1 + = 550 N
2 10

Ejemplo 12. La figura muestra un ascensor. Este


consiste de la caja con masa m1 = 1100 kg , el Aplicando La Segunda Ley de Newton
contrapeso con masa m 2 = 1000 kg . El cable y m1 g T2 = m2 a T2 = m2 ( g a )
poleas con masa y friccin despreciables. T2 = 1000(9,8 2)
Cuando el ascensor tiene una aceleracin hacia arriba = 7800 N
de 2 m/s2, el contrapeso tiene igual aceleracin pero c) En el motor Fuerza ejercida por el motor (T1 y T2
hacia abajo. pueden considerarse colineales)
a) Cul es el valor de la tensin T1 ?
b) Cul es el valor de la tensin T2 ?
c) Cul es la fuerza ejercida por el motor sobre el
cable?

FM = T1 T2 = 12980 7800
= 5180 N

Ejemplo 13. Demostracin de la tercera ley de


Newton mediante el uso de la segunda ley.
Se tienen dos cuerpos de masas m1 y m 2 los cuales
son empujados sobre un plano sin friccin por una
fuerza de magnitud P . Demostrar que aqu se
Solucin. cumple la tercera ley de Newton.
a) Consideremos el D.C.L de la masa m1 :

Solucin.

10
Dinmica de una partcula Hugo Medina Guzmn

Asumiremos que no hay friccin entre las superficies a) el valor y sentido de la velocidad del carrito,
de contacto de m1 y m 2 . b) el lugar, donde encontrar
c) el desplazamiento del carrito
La figura muestra los D.C.L. para los bloques 1, 2 y
d) el recorrido total del carrito.
para el sistema.
(Usar g = 9,8 m/s2)

Solucin.
N 1 y N 2 son las fuerzas ejercidas por el plano.
F21 es la fuerza que el bloque 2 ejerce sobre el
bloque 1.
F12 es la fuerza que el bloque 1 ejerce sobre el
bloque 2.
La fuerza P solo acta sobre el bloque 1, ya que
solo est en contacto con l.
Como asumimos que no hay friccin entre los
bloques, las fuerzas son normales a la superficie de
contacto.
Para el bloque 1 tenemos:
P F21 = m1 a1x y N 1 m1 g = 0
Para la masa M:
Similarmente para el bloque 2 T = Ma (1)
F12 = m2 a 2 x y N 2 m2 g = 0 Para la masa m:
Para el sistema T mg = ma (2)
P = (m1 + m2 )a x y Sumando (1) y (2)
N 1 + N 2 (m1 + m2 )g = 0 mg = (M + m )a
m 0,2
En este caso no nos interesan las ecuaciones en y pero
a= g = (9,8) = - 2,8 m/s2
si las ecuaciones en x.
Como los bloques se mueven juntos:
(M + m ) 0,7
La aceleracin es en sentido contrario al indicado en
a1x = a 2 x = a x la figura.
Sumamos la ecuacin para el bloque 1 con la a) La velocidad inicial del carrito es v0 = 7 m/s y su
ecuacin para el bloque 2. aceleracin es a = - 2,8m/s2.
P F21 + F12 = m1 a1x + m2 a 2 x = (m1 + m2 )a x De las ecuaciones de cinemtica
Comparando con la ecuacin para el sistema 1 2
x = v0 t + at , v = v0 + at ,
tenemos: 2
P F21 + F12 = P Hallamos:
Esto dice que la magnitud de la fuerza de 1 sobre 2 es x = 7t 1,4t 2 , v = 7 2,8t
igual a la fuerza de2 sobre 1. Como ellas son opuestas Dentro de 5 s el carrito tendr una velocidad
resulta ser precisamente la tercera ley de Newton. v = - 7 m/s (dirigida a la izquierda).
F21 = F12 , Accin y reaccin. b) x = 7(5) 1,4(5) = 35 35 = 0
2

El carrito se encontrar en la posicin inicial.


Ejemplo 14.. Un carrito de masa M = 500 gramos c) El desplazamiento es cero.
est unido a una carga de masa m = 200 gramos
mediante una cuerda. En el momento inicial el carrito d) El carrito se detiene cuando v = 0 e inicia el
tenia la velocidad inicial v0 = 7 m/s y se mova a la camino de vuelta.
derecha por un plano horizontal. Determinar para t =
5 s:

11
Dinmica de una partcula Hugo Medina Guzmn

0 = 7 2,8t t =
7
= 2,5 s
(F F )fk
a=
2,8 m
[
Recorrido total s = 2 7(2,5) 1,4(2,5)
2
] Si incrementamos la fuerza F, punto C, la fuerza neta
sobre el bloque F F fk se incrementa y tambin se
= 17,5 m
Recorrer un trayecto igual a 17,5 m. incrementa la aceleracin.

FRICCIN Observacin. Encontramos que con fuerzas menores


Cuando un cuerpo sobre una superficie se empuja o que 10 N no se produce movimiento.
se jala ste puede permanecer inmvil, esto sucede Con 10 N el bloque comienza a moverse.
porque la fuerza aplicada no ha sido suficiente para Para fuerzas mayores a 10 N el bloque se acelera.
vencer la fuerza de friccin. Cuando lograrnos que el Si medimos la aceleracin podemos conocer la fuerza
cuerpo deslice sobre la superficie es necesario aplicar resultante sobre el bloque aplicando la segunda ley de
una fuerza para que ste contine en movimiento. Newton, F = ma .
Cuando el dinammetro indica 12 N la fuerza
Comportamiento de un cuerpo que descansa sobre resultante a partir de la aceleracin medida es 4 N,
un plano horizontal esto significa que se necesita 12 N 4 N = 8 N, para
Supongamos que jalamos un bloque con un vencer la fuerza de friccin Si aplicamos 10 N al
dinammetro, como se muestra en la figura. bloque para que inicie el movimiento, despus de esto
es posible reducir la fuerza a 8 N y an mantener el
Comportamiento de un cuerpo que descansa sobre bloque en movimiento.
un plano horizontal En resumen:
Una fuerza de 10 N inicia el movimiento del bloque.
Una fuerza de 8 N mantiene el movimiento del
bloque.

El origen de este fenmeno se debe a la existencia de


fuerzas entre las molculas del cuerpo y la superficie;
Dibujemos una grfica de la fuerza F aplicada sobre si la superficie de contacto del cuerpo con la
el bloque versus el tiempo t . superficie fuera perfectamente plana, la fuerza de
atraccin podra ser considerable, como es el caso de
dos placas de vidrio perfectamente limpias que una
vez puestas en contacto, difcilmente pueden ser
separadas.
Las superficies nunca son perfectamente lisas y las
imperfecciones constituyen verdaderos obstculos al
desplazamiento como se muestra en la figura. Es
preciso vencer estos obstculos para iniciar el
movimiento y tambin para mantenerlo.

1. Desde el origen hasta el punto A la fuerza F


aplicada sobre el bloque no es suficientemente grande
como para moverlo. Estamos en una situacin de
equilibrio esttico
F = Ffs = s N
En el punto A, la fuerza de rozamiento F fs alcanza
su mximo valor smx N A esta fuerza se le conoce como FUERZA DE
F = Ffsmx = smx N FRICCION O ROZAMIENTO F f . ( )
2. Si la fuerza F aplicada se incrementa un poquito Con la finalidad de conocer la dependencia de esta
ms, el bloque comienza a moverse. La fuerza de fuerza de rozamiento realicemos la siguiente
rozamiento disminuye rpidamente a un valor menor experiencia.
e igual a la fuerza de rozamiento dinmico, Supongamos un plano inclinado con un bloque de
F = F fk = k N masa ni descansando sobre l.
Si la fuerza F no cambia, punto B, y permanece igual
a F fsmx , el bloque comienza movindose con una
aceleracin

12
Dinmica de una partcula Hugo Medina Guzmn

intermoleculares son tanto mayores, cuanto mayor es


la superficie de contacto. En realidad se deba esperar
que F f fuera proporcional a la superficie, lo que
suceder es que si el cuerpo pesa muy poco,
prcticamente no hay puntos de contacto entre las dos
superficies (el rea de contacto es despreciable).
Cuando N aumenta, la superficie aumenta y F f
Encontramos que el bloque empieza a resbalar para tambin, por lo tanto F f = N donde se est
un determinado ngulo . Si colocamos dos bloques
juntos, el ngulo con el cual inician el movimiento incluyendo ya el aumento de superficie. Es decir, la
sigue siendo , lo mismo ocurre con tres bloques. La fuerza de friccin F f es proporcional a la fuerza
fuerza que jala al cuerpo es la componente del peso normal N porque la verdadera superficie de contacto
mgsen , paralela al plano. La otra componente es es proporcional a la fuerza normal.
perpendicular al plano mg cos . Esta es la fuerza
Ejemplo 15. Cul es la fuerza mnima F necesaria
que sostiene al bloque sobre la superficie (Fuerza para mover la masa m , siendo el coeficiente de
Normal). Si duplicarnos el peso mg a 2mg,
rozamiento esttico entre el piso y el bloque en cada
duplicamos la fuerza que jale al bloque y la fuerza
uno de los casos siguientes?
normal tal que:
Fuerza que inicia el movimiento
= Constante
Fuerza normal
O
mg sen
= tan = s = Constante
mg cos Solucin.
Ff a) La figura muestra el D.C.L.
= s
N
A esta constante s se le llama coeficiente de
friccin esttica.
Si se toman los datos con el bloque en movimiento, el
ngulo para que el movimiento contine es
generalmente menor y obtenemos
Fuerza para continuar el movimiento
= k
Fuerza normal
A esta constante se le llama coeficiente de friccin F y : N mg = 0 N = mg
cintica k . F x : F N = 0 F = N
es una constante que depende de la superficie y se Luego:
puede escribir simplemente. F = mg
F f = N . b) La figura muestra el D.C.L.
Algunos valores tpicos de coeficientes de friccin.

Material Sobre s k
material
Acero Acero 0,78 0,42
Cuero Cuero 0,64 0,56
Cuero Roble 0,60 0,50
Bronce Hierro 0,40 0,30
Aluminio Aluminio 1,05 1,40
Vidrio Vidrio 0,92 0,40
Caucho Asfalto 0,60 0,40 F y : N + Fsen mg = 0
Caucho Concreto 0,80 0,70
N = mg Fsen
Caucho Hielo 0,02 0,005
Piedra Piedra 0,65 0,60 Fx : F cos N = 0
El hecho que la fuerza de friccin es independiente
F cos = N
del rea de contacto parece absurdo ya que las fuerzas De estas dos ecuaciones obtenemos:

13
Dinmica de una partcula Hugo Medina Guzmn

mg une los bloques m1 y m2. El coeficiente de rozamiento


F= entre los bloques y el plano inclinado es .
cos + sen
c) La figura muestra el D.C.L.

Solucin.
F y : N Fsen mg = 0
N = mg + Fsen
Fx : F cos N = 0
F cos = N
De estas dos ecuaciones obtenemos:
mg
F=
cos sen

Ejemplo 16. Cul es el valor mnimo de F para


Para m0 : {m0 g T1 = m0 a
sostener el bloque de masa m sobre una pared T1 T2 N 2 = m2 a
vertical, como se muestra en la figura, es el Para m 2 :
coeficiente de friccin esttico entre la pared y el N 2 m2 g = 0
bloque? T2 N 1 = m1 a
Para m1 :
N 1 m1 g = 0
De estas ecuaciones obtenemos:
N 2 = m2 g , N 1 = m1 g
y m0 g (m1 + m2 )g = (m0 + m1 + m2 )a
De aqu:
Solucin. a=
[m0 (m1 + m2 )] g
La figura siguiente muestra el D.C.L. (m0 + m1 + m2 )
La tensin del cable que une los bloques m1 y m2:
m1 m0
T2 = m1 (a + g ) = (1 + )g
(m0 + m1 + m2 )
Ejemplo 18. Se tiene una masa m 2 sobre una masa
m1 sobre un piso horizontal, tal como muestra la
Fy : N F = 0 N = F figura. Se aplica una fuerza horizontal F sobre la
mg masa m1 . La masa carece de friccin. Cul es el
F x : N mg = 0 N =
valor mximo de F para que la masa m1 no resbale
Por consiguiente sobre m 2 . Cul es la aceleracin resultante de los
mg bloques?
F=

Ejemplo 17. En el esquema de la figura las masas de
la polea y del cable son despreciables y no hay
rozamiento entre el cable y la polea. Hallar la
aceleracin del bloque m0 y la tensin del cable que

14
Dinmica de una partcula Hugo Medina Guzmn

Solucin.
La figura muestra el D.C.L. de las masas m1 y m 2 .

Solucin.
La figura muestra el D.C.L. para este caso

Aplicando la Segunda Ley de Newton a la masa m 2 ,


la que suponemos se mueve con aceleracin a 2 .

F y : N 2 m2 g = 0
Las ecuaciones para la masa m 2 son
F : N 2 = m2 a 2
F
x
y : N 2 m2 g = 0
Aplicando la Segunda Ley de Newton a la masa m1 ,
la que suponemos se mueve con aceleracin a1 . F x : F N 2 = m2 a 2
Las ecuaciones para la masa m1 son.
F y : N 1 N 2 m1 g = 0
F x : F N 2 = m1 a1 F y : N 1 N 2 m1 g = 0
Trabajando con estas ecuaciones encontramos que F x : N 2 = m1 a1
F = m1 a1 + m2 a 2 Trabajando con estas ecuaciones encontramos que
La aceleracin de la masa m 2 es: F = m1 a1 + m2 a 2
N 2 m2 g La aceleracin de la masa m1 es:
a2 = = = g N 2 m2 g m
m2 m2 a1 = = = g 2
Como el valor de vara desde 0 hasta el valor m1 m1 m1
mximo mx : Como el valor de vara desde 0 hasta el valor
mximo mx :
a 2 = mx g o simplemente a 2 = g .
Pero como queremos encontrar el valor mximo m2
posible de F para que las masas vayan juntas, es
a1 = mx g
m1
decir, para que m1 no se quede, se tiene como
Como la condicin de que las masas m1 y m 2 vayan
condicin que;
juntas es,
a1 = a 2 = g
a1 = a 2
Luego: Fmx = (m1 + m 2 ) mx g
Luego el valor mximo de F pera que m1 y m 2
Si aplicamos una fuerza mayor el bloque m1 vayan juntas es,
avanzar dejando atrs al bloque m 2 . (m1 + m2 )m2
Fmx = mx g
m1
Ejemplo 19. Usando el dispositivo del ejemplo
anterior discuta el caso en ci que la fuerza F se aplica
Ejemplo 20. En el dispositivo de la figura
a la masa m 2 . encontramos el valor mnimo de F para sacar la masa
m1 .
El coeficiente de friccin entre m1 y la mesa es 1 y
el coeficiente de friccin entre m1 y m 2 es 2 .

15
Dinmica de una partcula Hugo Medina Guzmn

Solucin.
Solucin.
La figura muestra los D.C.L. de las masas m1 y m 2
La figura muestra el D.C.L.de las masas m1 y m 2

Considerando que el equilibrio es la condicin


Considerando que el equilibrio es la condicin mnima de inicio del movimiento.
mnima de inicio de movimiento Aplicando la segunda ley de Newton a la masa m 2 :
Aplicando la Segunda ley de Newton para la masa
m2 .
F y : N 2 m2 g = 0

F y : N 2 m2 g = 0 F x : 2 N2 T = 0
Aplicando la segunda ley de Newton para la masa
F x : 2 N2 T = 0 m1 :
Aplicando la Segunda Ley de Newton para la masa
m1
F y : N 2 N 1 + m1 g = 0

F y : N 2 N 1 + m1 g = 0 F x : F 1 N1 2 N 2 T = 0
Resolviendo estas ecuaciones
F x : F 1 N 1 2 N 2 = 0 N 2 = m2 g
Resolviendo estas ecuaciones T = 2 N 2 = 2 m2 g
N 2 = m2 g
N 1 = N 2 + m1 g = (m1 + m 2 )g
T = 2 N 2 = 2 m2 g
F = 1 N1 + 2 N 2 + T
N 1 = N 2 + m1 g = (m1 + m 2 )g
= 1 (m1 + m2 )g + 2 m2 g
F = 1 N 1 + 2 N 2
= [1 m1 + m2 (1 + 2 )]g
= 1 (m1 + m 2 )g + 2 m2 g Siendo este valor de F el mnimo para iniciar el
Siendo este valor de F el mnimo para iniciar el movimiento.
movimiento de la masa m1 .
Ejemplo 22. Los bloques m1 y m 2 de 20 y 60 kg,
Ejemplo 21. En el dispositivo de la figura, encontrar respectivamente, estn unidos por una cuerda de masa
el valor mnimo de F para sacar la masa m1 . El despreciable que pasa por una polea sin rozamiento.
El coeficiente de rozamiento cintico entre las masas
coeficiente de friccin entre m1 y la mesa es 1 , el y la superficie es 0,3. Determinar la velocidad del
coeficiente de friccin entre m1 y m 2 es 2 . sistema 4 segundos despus de partir del reposo.

16
Dinmica de una partcula Hugo Medina Guzmn

Solucin. Ejemplo 23. En una mesa un plato descansa sobre el


La figura muestra el D.C.L. de la masa m1 . mantel, cuyo centro est a 0,25m del borde de la
mesa. El mantel se jala sbitamente en forma
Consideremos que el movimiento es de izquierda a
horizontal con una aceleracin constante de 10 m/s2.
derecha con aceleracin a
El coeficiente de friccin cintico entre el mantel y el
plato es k = 0,75 . Asumiendo que el mantel llega
justo al borde de la mesa.
Cuando el extremo del mantel pasa bajo el centro del
plato, encontrar:
a) La aceleracin del plato
b) La velocidad de! plato
c) La distancia del plato al borde de la mesa.
Solucin.
a) Aplicando la segunda ley de Newton para el plato,
F y : N 1 m1 g cos 30 = 0 la masa del plato es m y su aceleracin a p .

F x : T F f 1 m1 gsen30 = m1 a
De estas ecuaciones
3
N 1 = m1 g cos 30 = 20 10 = 173 N
2
F f 1 = N 1 = 0,3 173 = 51,9 N
1
y T 51,9 20 10 = 20a
2
T = 151,9 + 20a F V = 0 mg N = 0
La figura muestra D.C.L. de la masa m 2 . F H = ma p F f = ma p
De aqu obtenemos:
N = mg y k mg = ma p
De donde:
a p = k g = 0,75 x 9,8 = 7,35 m/s2
El plato resbala ya que a p es menor que 10 m/s2
b) En el instante en que el extremo del mantel
coincide con el centro del plato estn a la misma

F y : N 2 m 2 g cos 60 = 0 distancia del borde de la mesa

F x : m2 gsen 60 F f 2 T = m2 a
De estas ecuaciones
1
N 2 = m 2 g cos 60 = 20 10 = 150 N
2
Ff 2 = N 2 = 0,3 150 = 45 N
3 x p = xm
y 30 10 45 T = 30a
2
T = 214,5 30a 1 1
Igualando los valores de T: x p = 0,25 + a p t 2 = 0,25 + 7,35t 2
2 2
m
151,9 + 20a = 214,5 30a a = 1,25 1 1
s2 x m = a m t 2 = 10t 2
2 2
Como v = v 0 + at , Igualando
Siendo v0 = 0 v = 1,25t 2
1 1
0,25 + 7,35t 2 = 10t 2
m 2 2
Para t = 4 s v = 1,25 4 = 5 Resolviendo:
s
t = 0,58 s y

17
Dinmica de una partcula Hugo Medina Guzmn

v p = v0 + a p t = 0 + 7,35 0,58 horizontal con coeficiente de friccin . La polea por


donde cuelga otro bloque de masa M no tiene roce y
= 4,26 m/s. la cuerda se considera inextensible y de masa
1 despreciable. Calcular la aceleracin y la tensin de
c) x p = 0,25 + a pt 2 la cuerda.
2
1
= 0,25 + 7,35 0,58 = 1,49 m
2

2
Ejemplo 24. El plano inclinado mostrado en la figura
tiene una aceleracin a hacia la derecha. Si el
coeficiente de friccin esttico entre el plano y el
bloque es , encontrar la condicin para que el
bloque resbale.
Solucin.
Se hacen los DCL y se aplica la segunda ley de
Newton, suponiendo que el cuerpo de masa M
desciende y tira a m hacia la derecha, lo que define el
sentido de la aceleracin.
Para m

Solucin.
Consideremos que el bloque tiene masa m , la figura
a continuacin muestra su DCL.

F V = 0 N + F sen mg = 0
N = mg F sen (1)
y FH = ma
T F cos F f = ma (2)
Para M
Para que el bloque no resbale debe tener la misma
aceleracin a .
Aplicando la segunda ley de Newton
F = 0 N cos + N sen mg = 0
V

y F = ma N sen + N cos = ma
H
De estas ecuaciones
mg
N= y
cos + sen
mg
( sen + cos ) = ma F = Ma
((cos + sen )) V

Finalmente T Mg = Ma (3)

a=
( cos sen ) g Adems: F f = N
(cos + sen ) De la ecuacin (1):
Este es el va1or crtico de a para que no resbale; el F f = (mg Fsen ) (4)
bloque resbalar para valores menores que el
De (3) se despeja T:
indicado.
T = Mg Ma (5)
Ejemplo 25. En el siguiente sistema mecnico, se
aplica una fuerza F inclinada un ngulo sobre el Ahora 4) y (5) se reemplazan en (2), lo que permite
cuerpo de masa m, ubicado sobre la superficie despejar la aceleracin

18
Dinmica de una partcula Hugo Medina Guzmn

Mg Ma F cos (mg Fsen ) = ma deslizarse sobre la viga? Dentro do cunto tiempo el

a=
(M m )g F (cos sen ) cuerpo caer de la viga? La longitud do la viga es l .

M +m
y la tensin T

T = Mg M
(M m )g F (cos sen )
M +m
Solucin.
Ejemplo 26. Una viga de masa M est situada en un
plano horizontal. Sobre la viga se encuentra un
cuerpo do masa m. El coeficiente de rozamiento entre
el cuerpo y la viga, as como entre la viga y el plano
es k . Analizar el movimiento para diferentes
valores do la fuerza F. Las ecuaciones del movimiento de la viga y del
cuerpo tienen la siguiente forma:
F fm = ma m , (1)
F k mg = Ma M (2)
Donde F fm es la fuerza do rozamiento, am y aM son
las aceleraciones.
Solucin. Supongamos que no hay deslizamiento, entonces
Si F k (m + M )g , no hay movimiento. am = aM
F > k (m + M )g . Analicemos
De las ecuaciones del movimiento podemos
Supongamos que
determinar la aceleracin y la fuerza de rozamiento.
el caso de ausencia de deslizamiento del cuerpo por la mF
viga. Las ecuaciones del movimiento, en este caso, La fuerza de rozamiento es F fm =
tendran la siguiente forma: (m + M )
F fm = ma , Para que no haya deslizamiento la fuerza de
rozamiento debe satisfacer la siguiente desigualdad:
Ma = F F fm F fM = F F fm k (m + M )g ; F
F fm k mg , es decir, g.
F fm k mg (m + M ) k
de donde Si F > k (M + m) g, entonces surge el deslizamiento.
F Las ecuaciones (1) y (2) en este caso deben escribirse
a= g,
(m + M ) k en la siguiente forma:
ma m = k mg , Ma M = F k mg
mF
F fm = mg k mg De estas ecuaciones obtenemos am y aM:
(m + M ) k (F k mg )
que es posible, si am = k g , aM = .
k (m + M) g < F < 2k (m + M) g. M
Si F > 2k(m + M)g, entonces el cuerpo deslizar por Es evidente que aM > am.
la barra. En este caso las ecuaciones del movimiento
tendrn la siguiente forma:
ma m = k mg ,
Ma M = F k mg k (M + m )g
1 1
de donde xm = am t 2 , xM = aM t 2
F (2m + M ) g 2 2
am = k g , aM = k 1 1
M M xM xm = l = a M t 2 am t 2
2 2
Que es fcilmente verificar en el caso de a M > a m
2l 2l
t= =
Ejemplo 27. Una viga do masa M est sobre un plano aM am (F k mg )
horizontal liso, por el cual puede moverse sin k g
M
friccin. Sobre la viga hay un cuerpo do masa m. El
coeficiente de rozamiento entre el cuerpo y la viga es 2lM
=
k . Con qu valor de la fuerza F que acta sobre la F k g (M + m )
viga en direccin horizontal, el cuerpo comienza a

19
Dinmica de una partcula Hugo Medina Guzmn

Ejemplo 28. En la figura, encontrar la aceleracin


del carro requerida para evitar que caiga el bloque B.
El coeficiente de rozamiento esttico entre el bloque
y el carro es k .

Cuerpo 1: F verticales = m1 g T = m1 a
Cuerpo 2: F horizontales = T = m2 a
Solucin. F verticales = N 3 N 2 T m3 g = 0
Mesa:
Si el bloque no cae, la fuerza de friccin, Ff, debe F horizontales = T Ff 3 = 0
balancear el peso del bloque:
Donde N3 y Ff3 (friccin) las componentes verticales
Ff = mg. y horizontales de la fuerza ejercida por el piso sobre
Pero el movimiento horizontal del bloque est dado la mesa.
por y N = ma. (Asumimos que las patas de la izquierda y de la
derecha comparten la carga igualmente. Esto no
Luego, afecta nuestro anlisis)
Ff g g De las primeras dos ecuaciones,
= a= m1 g
N a Ff a=
N (m1 + m2 )
Ff m1 m2 g
Como el valor mximo de es s , debemos Luego, F f 3 = T = m 2 a =
N (m1 + m2 )
g Finalmente,
tener a si el bloque no cae.
s N 3 = T + m2 g + m3 g
m1m2
Ejemplo 29. Dos cuerpos, de las masas m1 y m2, se = + m2 + m3
liberan de la posicin mostrada en la figura. Si la (m1 + m2 )
masa de la mesa de superficie lisa (sin friccin) es
m3, encuentre la reaccin del piso sobre la mesa Ejemplo 30. Se tiene un bloque de 20 kg sobre un
mientras los dos cuerpos estn en movimiento. plano inclinado que est sujeto a una cuerda (ver
Asuma que la mesa permanence inmvil. figura). Las superficies de contacto entre el bloque y
el plano inclinado son rugosas con coeficiente de
friccin cintica k = 0,5 y el de friccin esttica
s = 0,7.
a) Si la tensin de la cuerda es de 150 N, determine la
magnitud y sentido de la fuerza de rozamiento.
b) Si por un accidente se corta la cuerda, determine
la aceleracin del bloque.

Solucin. La figura muestra los diagramas de cuerpo


libre de cada uno de los elementos.

Solucin.
a)

20
Dinmica de una partcula Hugo Medina Guzmn

T mgsen30 F f = 0
F f = T mgse30 = 150 100 = 50 N
La segunda ley de Newton para m1 es
en el sentido indicado en la figura (hacia abajo).
b) T m1 a = 0 , N 1 m1 g = 0
De aqu T = m1 a (2)
La segunda ley de Newton para m2 es
N 2 m2 a = 0 , T m2 g = 0
De aqu T = m2 g (3)
De (2) y (3) se tiene
m2
a= g (4)
Cuando se rompe la cuerda para iniciar el m1
movimiento debe vencerse a la mxima fuerza de Sustituyendo (4) en (1) se obtiene la fuerza aplicada a
friccin esttica: M
3 m2
F fs = s mg cos 30 = 0,7 20 g = 173 N
F= (M + m1 + m2 )g
2 m1
Como 20g sen 30 = 100 N
100 N < 173 N, el movimiento no se inicia , por lo Ejemplo 32. Determinar la aceleracin mnima con
tanto la aceleracin del bloque es cero. que debe desplazarse el bloque de masa M en sentido
horizontal para que los bloques de masas m1 y m2 no
Ejemplo 31. Determinar la fuerza F aplicada al se muevan respecto de M, siendo el coeficiente de
bloque de masa M de la figura adjunta, para que los rozamiento entre los bloques. La polea y el cable
bloques de masas m1 y m2 apoyados en M, no se tienen masa despreciable.
muevan respecto de M. Todas las superficies son
lisas, la polea y el cable tienen masa despreciable.

Solucin.
Consideremos un sistema de referencia fijo en el
suelo con el eje x paralelo a la fuerza aplicada
Solucin.
Consideremos un sistema de referencia fijo en el F.
suelo con el eje x paralelo a la fuerza aplicada De la segunda ley de Newton aplicada al conjunto se

tiene:
F.
De la primera ley de Newton aplicada al conjunto se F = (M + m1 + m2 ) a (1)
tiene:


F = (M + m1 + m2 ) a (1) Siendo a la aceleracin del conjunto.
Las masas m1 y m2 estn en reposo sobre el bloque M,

luego en la referencia O su aceleracin es del
Siendo a la aceleracin del conjunto. Las masas m1
conjunto.
y m2 estn en reposo sobre el bloque M, luego en la
La fuerza que ejerce el cable sobre m1 y la que ejerce
referencia O su aceleracin es del conjunto. La fuerza
sobre m2 tiene el mismo mdulo T.
que ejerce el cable sobre m1 y la que ejerce sobre m2
tiene el mismo mdulo T.

21
Dinmica de una partcula Hugo Medina Guzmn

Diagrama del cuerpo libre masas separadas

La segunda ley de Newton para m1 es


T m1 a F f 1 = 0 , N1 m1 g = 0
F f 1 = N 1 = m1 g
T = m1 a + m1 g (2) a) Consideremos un sistema de referencia fijo en el

La segunda ley de Newton para m2 es
suelo con el eje x paralelo a la fuerza aplicada F .
N 2 m2 a = o , T + F f 2 m2 g = 0 Sea el instante en que m empieza a deslizar sobre
F f 2 = N 2 = m2 a M. Hasta dicho instante t , el conjunto se mueve

T = m2 g m2 g (3) con una aceleracin comn a .
De (2) y (3) se tiene La segunda ley de Newton aplicada al conjunto en el
instante t = es
m1 a + m1 g = m2 g m2 g
k = (M + m )a( ) , N 2 (M + m )g = 0
(m m1 )
a= 2 g (4) k
(m1 + m2 ) a ( ) = (1)
Sustituyendo (4) en (1) se obtiene la fuerza aplicada a
(M + m )
M La segunda ley de Newton aplicada a la masa m en el
instante t = es, ( la fuerza de rozamiento sobre m
m2
F= (M + m1 + m2 )g tiene, en ese instante, su valor mximo Ff = m g )
m1 F f = N 1 = ma( ) , N 1 = mg
Ejemplo 33. Un bloque de masa m se encuentra mg
a ( ) = = g (2)
sobre otro bloque de masa M que est apoyado sobre m
una superficie horizontal lisa. El coeficiente de
De (1) y (2) queda =
(M + m ) g
rozamiento entre los dos bloques es . Al bloque M s
se le aplica una fuerza horizontal dirigida hacia la k
derecha que depende del tiempo segn la ley F = k t.
Determinar: b) De (1) se tiene que la aceleracin del conjunto para
a) El instante en que m empieza a deslizar sobre M. t < es
b) La aceleracin de cada uno de los bloques. k
a1(t ) = a(t ) = t
(M + m )
Para t > . Las fuerzas que actan sobre m son
constantes, luego la aceleracin de m es
a1 = a( ) = g
La segunda ley de Newton aplicada a la masa M es
kt F f = kt N 1 = Ma 2 (t ) , como N 1 = mg
Solucin. kt mg = Ma 2(t ) y
Diagrama del cuerpo libre del conjunto
m k m
a2(t ) = g + t 2
M M s

Grfica de las aceleraciones en funcin del tiempo

22
Dinmica de una partcula Hugo Medina Guzmn

El movimiento de B es hacia arriba, luego


mB g < T
El movimiento de A es hacia abajo, luego
T + m A cos < m A gsen
El movimiento de los bloques es el indicado si
m B g < m A gsen m A cos

Ejemplo 34. Dos bloques A y B de masas mA y mB


mB
< sen cos
estn unidos mediante un cable que pasa a travs de mA
una polea tal como se muestra en la figura adjunta. El Los bloques no se mueven si
coeficiente de rozamiento entre el bloque A y el plano
inclinado es . Determinar el sentido del movimiento
mB
sen cos < < sen + cos
cuando se dejan en libertad a partir del reposo. El mA
cable es inextensible y las masas del cable y la polea
Ejemplo 35. Dos bloques A y B de masas m A = 10
despreciables.
kg y m B = 7 kg, estn unidos mediante un cable que
pasa a travs de las poleas tal como se muestra en la
figura adjunta. El coeficiente de rozamiento entre el
bloque A y el plano inclinado es = 0,10 y =
30. El cable es inextensible y las masas del cable y
las poleas son despreciables. Determinar:
a) Las aceleraciones de los bloques;
b) La tensin del cable.
Solucin.
Supongamos que el bloque A sube sobre el plano
inclinado. Sea T la fuerza que ejercen los extremos
del cable sobre los bloques dirigida, en ambos
bloques, tal como se indica.

Solucin.

El movimiento de B es hacia abajo, luego


mB g > T
El movimiento de A es hacia arriba, luego
T > m A gsen + m A cos
Supongamos que el movimiento de A es hacia abajo,
El movimiento de los bloques es el indicado si
luego:
m B g > m A gsen + m A cos
T + m A g cos < m A gsen
m
B > sen + cos T < m A gsen m A g cos
mA El movimiento de B es hacia arriba, luego:
m B g < 2T
Supongamos que el bloque A desciende sobre el
De ambas expresiones queda
plano inclinado.
1
m B g < m A gsen m A g cos
2
1
(7 ) < (10)sen30 0,10(10) cos 30
2
Con los valores 3,5 < 4,13
Desigualdad que se cumple, luego el movimiento es
el previsto.

23
Dinmica de una partcula Hugo Medina Guzmn

a) Consideremos un sistema de referencia con el eje x


horizontal. Las posiciones de los bloques estn
relacionadas por la condicin de ligadura
s A + 2 y B = constante ,
Luego sus aceleraciones cumplen
1
a A + 2a B = 0 a B = a A = a (1) Supongamos que el bloque A asciende por el plano
2 inclinado. Consideremos un sistema de referencia con
Fuerzas sobre los bloques
el eje x horizontal.
La segunda ley de Newton aplicada al bloque A es
Las posiciones, por una parte, del bloque A y de la
m A a A = m A gsen T N A , polea mvil, estn relacionadas por las condiciones
N A m A g cos = 0 de ligadura
De estas dos obtenemos: s A + h y p = constante
T = m A g (sen cos ) m A a A (2) Las posiciones de la polea y el bloque B, estn
La segunda ley de Newton aplicada al bloque B es relacionadas por las condiciones de ligadura
2T m B g = m B a B 2 y p y B = constante
1 De estas dos ecuaciones obtenemos:
T = m B (a B + g ) (3) 2 s A + 2h y B = constante
2
Igualando las ecuaciones (2) y (3), Las componentes de las aceleraciones de los bloques
satisfacen la condicin
m B (a B + g ) = 2m A g (sen cos ) 2m A a A
Teniendo en cuenta la ecuacin (1) y los valores:
2a A = a B (1)
7(a + 9,8) = 2(10 )(9,8)(0,5 0,1 0,87 ) 20(2a )
Resolviendo:
a = 0,26 m/s2
Las aceleraciones de los bloques son :
a A = 0,26 m/s 2 para arriba.
a B = 0,52 m/s 2 para abajo.
b) La magnitud de la tensin del cable es el valor de
la fuerza que el cable ejerce sobre los bloques. De la
Sean T A y TB las fuerzas que los cables ejercen
ecuacin (3) se tiene
1 sobre los respectivos bloques. Fuerzas sobre los
T= (7 )(0,26 + 9,8) = 35,2 N bloques y sobre la polea mvil.
2 Como la polea superior tiene masa despreciable solo
cambia el sentido de la fuerza.
Ejemplo 36. Dos bloques A y B de masas m A y La masa de la polea mvil es cero, luego
m B estn unidos mediante un cable que pasa a La tensin en ambos lados son iguales (TB ) y
travs de las poleas tal como se muestra en la figura T A = 2TB (2)
adjunta. El coeficiente de rozamiento entre el bloque De la segunda ley de Newton aplicada al bloque A se
A y el plano inclinado es . El cable es inextensible tiene:
y las masas del cable y la polea son despreciables. TA mA gsen N A = mAa A
Estudiar el sentido del movimiento de los bloques.
N A m A g cos = 0
De estas ecuaciones obtenemos:
T A = m A g (sen + cos ) + m A a A (3)
De la segunda ley de Newton aplicada al bloque B se
tiene
m B g TB = m B a B
TB = m B ( g a B ) (4)
Solucin. De las ecuaciones (1), (2), (3) y (4) obtenemos:
m A g (sen + cos ) + m A a A = 2m B ( g 2a A )
2m B g m A g (sen + cos )
aA =
m A + 4m B

24
Dinmica de una partcula Hugo Medina Guzmn

El movimiento es el indicado, si se cumple: Las fuerzas exteriores que actan sobre la m1 son la
2m B
> (sen + cos ) tensin del cable T y el peso m1 g , y sobre m 2 son
mA la tensin del cable T y el peso m2 g .
El movimiento es de sentido opuesto, si se cumple: De la ecuacin fundamental de la dinmica en la
2 mB
< (sen cos ) referencia no inercial se tiene
mA g
El signo menos es porque en este caso el peso de la
m1 a '1 = T m1 g m1
2
masa A es el que mueve al sistema y la fuerza de
3
rozamiento est en sentido contrario a ste. m1 a '1 = T m1 g (1)
2
Ejemplo 37. A los extremos de un hilo que pasa a g
travs de una polea fija al techo de la cabina de un m1 a ' 2 = T m2 g m2
2
ascensor se atan los cuerpos de masa m1 y m 2
3
(m1 < m2 ) . La cabina comienza a subir con una m1 a ' 2 = T m2 g (2)
2
aceleracin constante g / 2. Despreciando la masa de
De la condicin de ligadura para los bloques se tiene
la polea y la del hilo, as como el rozamiento,
calcular: a '1 +a' 2 = 0 a '1 = a ' 2 = a ' (3)
a) La aceleracin de m1 y m 2 respecto de la cabina De las ecuaciones (1), (2) y (3) se obtiene
y con relacin al foso del ascensor. 3 3
m1 a' = T m1 g y m1 a ' = T + m2 g
b) La fuerza con la cual la polea acta sobre el techo 2 2
de la cabina. Sumando estas ecuaciones:

(m2 + m1 )a' = 3 (m2 m1 )g


2
Despejando a '
3 (m2 m1 )
a' = g
2 (m2 + m1 )
Finalmente:

a'1 = a ' j y a' 2 = a' j

En la referencia fija, las aceleraciones de m1 y de


m 2 se obtienen de sumar a las anteriores la
Solucin. aceleracin del ascensor
a) El ascensor constituye una referencia no inercial en
traslacin que se mueve con una aceleracin a1 =
g
+ a' =
(2m2 m1 )
g y
constante en sentido ascendente respecto de una 2 (m2 + m1 )
referencia fija.
Seleccionemos una referencia con origen O en un g (2m1 m2 )
a 2 = a' = g
punto del ascensor. La aceleracin del origen 2 (m2 + m1 )
O respecto de la referencia fija O es la aceleracin b)
1
del ascensor gj . Sean a '1 j la aceleracin de m1
2
y a ' 2 j la aceleracin de m 2 en la referencia O.

La fuerza que la polea ejerce sobre el techo de la


cabina es
F 2T = 0 F = 2T
De la ecuacin (1) y (3) se tiene
3 3m1m2
T = m1 a '1 + g = g
2 (m2 + m1 )

25
Dinmica de una partcula Hugo Medina Guzmn

Luego
6m1 m2
F x = F f ma cos 30 = 0 (1)
F = 2T =
(m2 + m1 ) F y = N mg + masen30 = 0 (2)
g 3
de (1) F f = 45 = 191N
Ejemplo 38. Un nio de masa m = 45 kg se pesa en 2 2
una bscula de resorte situada sobre una plataforma
especial que se desplaza por un plano inclinado de de (2)
ngulo = 30 como muestra la figura (no hay g
rozamiento entre la plataforma y el plano inclinado). N = mg masen30 = 45 g
Cul ser la lectura de la bscula en estas 4
condiciones? = 33,45 Kg.
Siendo N la cantidad que marca la bscula.

Solucin en una referencia no inercial .


Seleccionemos una referencia con origen O (x,y) en
un punto de la plataforma. El nio est en reposo
sobre la plataforma.

Solucin.
Sea M la masa del conjunto nio - cua., y a la
aceleracin con la que desliza hacia abajo el
conjunto.

Aplicando la segunda ley de Newton al DCL del


nio.
F x = F f = ma cos 30 (1)
F y = N mg = masen30 (2)
g 3
Aplicando la segunda ley de Newton al conjunto nio de (1) F f = 45 = 191N
- cua. 2 2
F // = Ma Mgsen30 = Ma de (2)
g g
a = gsen 30 = N = mg masen30 = 45 g
2 4
1 = 33,45 kg
La aceleracin del conjunto es a = g Siendo N la cantidad que marca la bscula.
2
Solucin en una referencia inercial. Ejemplo 39. Un ascensor de masa total 3M es
Sobre el nio actan: su peso mg y la reaccin Ff en levantado bajo la accin de una fuerza F. El piso del
el apoyo. La indicacin de la bscula el valor de la
ascensor est inclinado un ngulo , con respecto a
normal.
la horizontal. Adems, un bloque de masa M se apoya
sobre el centro del piso rugoso del ascensor (con
coeficiente de friccin esttica ).
a) Hallar la aceleracin del ascensor.
b) Haga el diagrama de cuerpo libre de la masa M.
c) Cul es el valor mximo de F para que el bloque
dentro del ascensor no resbale respecto del piso del
ascensor?
d) Si el ascensor pierde contacto con la fuerza F y
empieza a caer libremente, calcule el valor de la
fuerza normal entre el bloque y el piso del ascensor, y
Aplicando la segunda ley de Newton al DCL del la fuerza de friccin sobre el bloque.
nio. Solucin.
a) Para hallar la aceleracin del ascensor.

26
Dinmica de una partcula Hugo Medina Guzmn

Por ser proporcional a la aceleracin centrpeta, la


fuerza Fc se llama fuerza centrpeta. Su efecto es
cambiar la direccin de la velocidad de un cuerpo. Se
puede sentir esta fuerza cuando se hace girar a un
objeto atado a una cuerda, ya que se nota el tirn del
objeto. Las fuerzas centrpetas no son diferentes de
otras fuerzas ya conocidas, su nombre se debe a que
apunta hacia el centro de una trayectoria
circunferencial. Cualquiera de las fuerzas ya conocida
F 3Mg Mg = (3M + M )a pueden actuar como fuerza centrpeta si producen el
efecto correspondiente, como ser la tensin de una
F 4 Mg F cuerda, una fuerza de roce, alguna componente de la
a= = g normal, la fuerza gravitacional en el caso de
4M 4M
b) Diagrama de cuerpo libre de la masa M. movimientos de planetas y satlites, etc.

Ejemplo 40. Un cuerpo de masa m, sujeto al extremo


de una cuerda de longitud L, que describe una
trayectoria circular en el plano horizontal, genera una
superficie cnica, por lo que se llama pndulo cnico.
Determinar la rapidez y el perodo de revolucin de la
c) Para que el bloque dentro del ascensor no resbale masa.
respecto del piso del ascensor se debe cumplir

M ( g + a )sen M ( g + a ) cos
tan .

Como a depende de F, y a esta en miembros de la


igualdad, el que el bloque resbale dentro del ascensor
solamente depende del coeficiente de friccin.
d) Si el ascensor pierde contacto con la fuerza F y Solucin.
empieza a caer libremente, La partcula est sometida a una aceleracin
centrpeta, y la fuerza centrpeta correspondiente est
N = 0, por lo tanto Ff = 0 dada por la componente de la tensin de la cuerda en
direccin radial hacia el centro de la circunferencia.
DINMICA DEL MOVIMIENTO CIRCULAR El D. C. L. de la masa m.
La primera ley de Newton dice que un objeto
permanecer en movimiento uniforme en lnea recta
con velocidad constante o en reposo si no acta una
tuerza sobre l. Entonces cuando un objeto se mueve
en trayectoria circular, debe haber una fuerza sobre l
cambindole la trayectoria recta. Esta fuerza puede
ser proporcionada por la tensin en una cuerda, para
un objeto que se hace girar en una circunferencia
horizontal al extremo de una cuerda; por la fuerza de
la gravedad para un satlite orbitando la tierra.
Los objetos en movimiento circular no estn en Aplicando la segunda ley de Newton:
equilibrio, debe haber una fuerza resultante, de otro
modo slo habra un movimiento en lnea recta. F y =0
T cos mg = 0
FUERZA CENTRPETA.
Tcos = mg (1)
Una partcula que se mueve sobre una trayectoria
circular de radio R con rapidez constante, se y Fx = ma
encuentra sometida a una aceleracin radial de
magnitud v2/R. Por la segunda ley de Newton, sobre v2
Tsen = ma = m (2)
la partcula acta una fuerza en la direccin de hacia r
el centro de la circunferencia, cuya magnitud es: Dividiendo (2) entre (1):
v2 v2
Fc = mac = m tan = v 2 = rg tan
R rg

27
Dinmica de una partcula Hugo Medina Guzmn

De la geometra de la figura, r = Lsen ,


reemplazando se obtiene la rapidez de m:
v 2 = (Lsen )g tan
v = Lg tan sen

Para calcular el periodo T, esto es el tiempo que


demora en dar una vuelta. Solucin. La figura muestra el D.C.L.
Se sabe que
x = vt , con x = 2r , entonces:
2r 2L sen
t = =
v Lg tan sen
L cos
= 2
g
Aplicando la segunda ley de Newton.
L cos
T = 2 F n = mac T mgsen = mR 2
g
F t = mat mg cos = mR
Ejemplo 41. Una bola de masa m, atada al extremo La tensin en la cuerda es
de una cuerda se hace ir en un plano horizontal T = mR 2 mgsen
formando una circunferencia de radio R. Si tiene una
La fuerza tangencial es mg cos y la aceleracin
velocidad angular , cul es la tensin en la
cuerda? angular es
g
= cos
R
d 2
Como = , obtenemos la ecuacin
dt
diferencial:
d 2 g
= cos
Solucin. dt R
La figura muestra el D.C.L. cuya solucin esta fuera del alcance de este curso.
Pero podramos encontrar la tensin y fuerza
tangencial para posiciones determinadas, es decir
para valores dados de .
T = mR 2
Para = 0 ,
Ft = mg
T = mR 2 mg
Aplicando la segunda ley de Newton a la masa m . Para = 90 ,
Ft = 0
F n = mac T = mR 2
T = mR 2
F t = mat 0 = mR Para = 180
Ft = mg
,

La tensin en la cuerda es T = mR . La fuerza


2

tangencial es cero y la aceleracin tangencial T = mR 2 + mg


Para = 270 ,
tambin es cero, ya que la velocidad angular es
Ft = 0
constante.
T = mR 2
Ejemplo 42. Resolver el problema anterior pero en Para = 360
el caso que el giro sea en el plano vertical. Ft = mg

28
Dinmica de una partcula Hugo Medina Guzmn

b) Cuando el bloque est en A se dirige a B, su


velocidad es en el sentido antihorario y su aceleracin
en el sentido horario. Luego su rapidez disminuye.
c) Si en "B" su velocidad es nula, cul es la
trayectoria que seguir la masa m?

Ejemplo 43. Un pequeo bloque de masa m se


v2 v2
desliza sobre una superficie lisa circular de radio R N + mg = ma c = m N = mg + m
como se muestra en la figura. (La pista est sobre un R R
plano vertical y g = aceleracin de la gravedad) Si v = 0, el valor de N es negativo, lo que no permite
a) Trace el diagrama de cuerpo libre del bloque al bloque sostenerse sobre la circunferencia, por
cuando se encuentra en "A" y muestre (dibujando los consiguiente el bloque caer verticalmente.
vectores) la direccin de la fuerza resultante y su
aceleracin.
d) Si en "B" su velocidad es gR , qu trayectoria
b) Cuando est en "A", su rapidez aumenta o seguir la masa m?
disminuye? (Justifique) v2 gR
c) Si en "B" su velocidad es nula, cul es la N = mg + m N = mg + m = 0,
trayectoria que seguir la masa m? R R
el bloque tiene suficiente velocidad para seguir en la
d) Si en "B" su velocidad es gR , qu trayectoria trayectoria circular.
seguir la masa m?
Ejemplo 44. Un avin describe un rizo (un camino
circular en un plano vertical) de 150 m de radio. La
cabeza del piloto siempre apunta al centro del rizo. La
rapidez del avin no es constante; es mnima en el
cenit del rizo y mxima en el nadir.
a) En el cenit el piloto experimenta ingravidez. Qu
rapidez tiene el avin en ese punto?
b) En el nadir, la rapidez del avin es de 280 km/h.
Qu peso aparente tiene el piloto aqu? Su peso real
Solucin. es de 700 N.
a) Trace el diagrama de cuerpo libre del bloque Solucin.
cuando se encuentra en "A" y muestre (dibujando los a) S el piloto siente ingravidez, est en cada libre, y
vectores) la direccin de la fuerza resultante y su v2
aceleracin. a=g= , luego
R
v = Rg = (150 )(9,80) = 38,3 m s , o
138 km h .
b) El peso aparente es la suma de la fuerza neta hacia
adentro (arriba) y el peso del piloto, o
v2
P' = P + ma = P + m
R
Aqu:
P = 700 N
P 700
m= = = 71,43 kg
g 9,8
v = 280 km/h = 77,6 m/s
R = 150 m
Luego:
77,6 2
P' = 700 + 71,43 = 3579 N
150

29
Dinmica de una partcula Hugo Medina Guzmn


Ejemplo 45. Una partcula de masa m que est unida a = at t + a n n
al extremo de un cable de longitud l , cuyo otro at = gsen ,
extremo est fijo, se mueve en un plano vertical, a
partir de un punto A tal que el cable forma con la T mg cos = ma n = 2mg (cos 0 cos )
vertical un ngulo 0 , iniciando el movimiento con a n = 2 g (cos 0 cos )
velocidad cero. Determinar: Ejemplo 46. Una partcula de masa m se encuentra en
a) La velocidad de v de la esfera en funcin de . el polo de una semiesfera de radio R, la cual est
b) La tensin del cable en funcin de . apoyada sobre una superficie horizontal. Desplazada
ligeramente de su posicin de equilibrio, la partcula
c) La aceleracin a en funcin de .
desliza sobre la superficie, la cual se supone lisa.
Determinar:
a) La velocidad v de la partcula en funcin del
ngulo que forma su radio posicin con el radio
inicial.
b) El valor de la normal N en funcin de .
c) El valor de , en el instante en que la partcula se
despega de la superficie.
Solucin.

Solucin.
En la referencia de origen O, la esfera recorre una
circunferencia de radio l con velocidad variable v(t).
Las componentes intrnsecas la aceleracin son:
dv v2 En la referencia de origen O, la partcula m tiene un
at = , an = movimiento circular no uniforme de radio R. Las
dt l componentes de la aceleracin son:
Sobre la masa m actan la tensin del cable T y su dv v2
peso mg . at = , an =
dt R
De la segunda ley de Newton en componentes n y t Sobre la masa m actan el peso mg y la reaccin en
se tiene:
el apoyo N.
F t = mat mgsen = mat Aplicando la segunda ley de Newton:

F n = ma n T mg cos = ma n F t = mat mgsen = mat


a) Para la componente tangencial se tiene: F n = ma n N mgcos = ma n
dv
mgsen = m a) De la componente tangencial se tiene:
dt dv ds dv
ds dv mgsen = m = gsen
= gsen dt dt ds
dt ds vdv = gsen ds = Rgsen d
vdv = gsen ds = gsen ld Integrando y teniendo en cuenta las condiciones
Integrando y teniendo en cuenta las condiciones iniciales queda
iniciales queda v 2 = 2 Rg (1 cos )
v = 2 gl(cos 0 cos )
2
Finalmente:
v = 2 gl(cos 0 cos ) v = 2 Rg (1 cos )
b) Para la componente normal: b) De la componente normal se tiene:
v 2 N = mgcos ma n =
T mg cos = m = 2mg (cos 0 cos )
l v2
La tensin del cable es mgcos m = mg cos 2mg (1 cos )
R
T = mg (2 cos 0 3 cos ) La normal es N = mg (3 cos 2 )
c) De las ecuaciones anteriores se tiene la c) La masa m deja de estar en contacto con la
aceleracin: superficie cuando N = 0

30
Dinmica de una partcula Hugo Medina Guzmn

N = mg (3 cos 2 ) = 0 m 2 . Los bloques estn a la distancia R del eje de


2 rotacin. El coeficiente de rozamiento esttico entre
cos = = 48,19 las masas y entre m 2 y la tornamesa es
3
Considerando el rozamiento y la masa de la polea
Ejemplo 47. En un parque de diversiones hay un despreciables, encontrar la velocidad angular de la
cilindro grande vertical, de radio R que rota alrededor tornamesa para la cual los bloques justamente
de su eje, con velocidad angular constante . comienzan a resbalar.
Explicar cmo es posible que las personas que estn
dentro, al retirrseles el piso permanezcan pegadas
a la pared interior del cilindro.

Solucin.
Solucin.
En este problema todo depende de tomar
correctamente la direccin de la fuerza de friccin
entre m1 y m 2 . Consideremos m 2 > m1 , por lo
tanto m 2 tender a moverse hacia afuera, jalando a
m1 hacia adentro. La fuerza de friccin actuar en
oposicin a su movimiento relativo.
La figura muestra los D.C.L. de los componentes del
La figura muestra el D.C.L del hombre. sistema.

Aplicando La segunda ley de Newton:


Como el hombre no cae, radialmente est en reposo
(R = constante) Aplicando la segunda Ley de Newton
Fr = mac N = m 2 R F z = ma z , Fr = ma r y F t = mat
F z = 0 mg N = 0 A la masa m1 :
De estas ecuaciones: mg m R N 1 m1 g = 0 , T + F1 = m1 2 R , Ft = 0
2

g A la masa m 2 :
y =
R N 2 N1 m2 g = 0 ,
Esto quiere decir que para que suceda el efecto de T F1 F2 = m2 2 R , Ft = 0
suspensin de las personas, la velocidad angular
De las ecuaciones obtenemos:
tiene que tener un valor relacionado con el radio R y
el coeficiente de friccin . N 1 = m1 , N 2 = (m1 + m2 )g
F1 m1 g , F2 (m1 + m2 )g
Ejemplo 48. En la tornamesa mostrada en la figura el
y 2 F1 + F2 = (m2 m1 ) R
2
bloque de masa m1 descansa sobre el bloque de masa

31
Dinmica de una partcula Hugo Medina Guzmn

Corno puede incrementarse hasta que F1 y F2 Curvas sin peraltar


En estos casos la fuerza de rozamiento es la que nos
alcancen sus valores mximos
proporciona toda la componente normal que servir
2m1 g + (m1 + m2 )g = (m2 m1 ) 2 R para tomar la curva. Siempre que tengamos que sta
Finalmente es mayor que la aceleracin normal el automvil ser
(3m1 + m2 ) capaz de tomar la curva, es decir, el caso lmite se
= alcanza cuando
R(m2 m1 ) v2
Fr = ma c = m
Ejemplo 49. Cmo afectar la rotacin de la tierra
R
al peso aparente de un cuerpo en el ecuador?
Ejemplo 50. Cul es la velocidad a que puede ir un
Solucin.
automvil por una curva sin peralte, de radio R, sin
La figura muestra la situacin de un cuerpo situado
derrapar?, el coeficiente de rozamiento entre las
en la lnea ecuatorial
ruedas y el suelo vale .
Solucin.

Aplicando la segunda ley de Newton


F z = maz Fz = 0
v2
F r = mar N mg = m R 2 F h = mac F V = 0 ac =
R
F t = mat Ft = 0
F f = N = mg = m
v2
v = gR
El peso de la masa es representado por la reaccin N R
N = mg m 2 R
Ejemplo 51. El ciclista tiene que inclinarse al
Para tener una idea de cunto afecta la rotacin de la
desplazarse por una pista circular (o para pasar por
tierra es necesario hacer el clculo numrico para esta
una curva), Encontrar la relacin de la velocidad con
consideracin:
el radio de curvatura, el ngulo de inclinacin y
El radio de la tierra en el ecuador: R = 6,378 x l06m
La velocidad angular de la tierra coeficiente de friccin.
2 rad 5 rad
= = 7,27 10
24 3600 s s
La aceleracin de la gravedad en el
Ecuador: g = 9,780490 m/s2
2R
Porcentaje = 100 = 0,34%
g

CURVAS EN LAS PISTAS.


Para un cuerpo como un vehculo o un vagn de tren
que se mueven describiendo una trayectoria curva de
radio r, sobre el vehculo debe actuar una fuerza
centrpeta para evitar que contine movindose en Solucin.
lnea recta y se salga de la pista; esta es la fuerza para
hacer que el vehculo gire por la pista curva.
La fuerza centrpeta necesaria la da el roce de las
llantas o las pestaas de las ruedas del tren.

La figura muestra el D.C.L.

32
Dinmica de una partcula Hugo Medina Guzmn

Ejemplo 52. Cul es la velocidad a que puede ir un


automvil por una curva con peralte, de radio R, sin
derrapar, el peralte es de grados?
Solucin.

Aplicando la segunda ley de Newton:


F z = maz N mg = 0
v2
v2
F = 0 ac =
F r = mar N = m
R
R
v2
De las ecuaciones obtenemos F// = mac cos mgsen = m R
v2
N = mg y mg = m v = gR tan
R
Finalmente v = gR Curvas peraltadas con rozamiento
Del D.C.L. tambin obtenemos: Este es un caso bastante ms complejo de analizar.
N
tan = = Ejemplo 53. Cul es la velocidad a la que puede ir
N un automvil por una curva con peralte, de radio R,
Esto quiere decir que si el motociclista al realizar una para que no se deslice hacia el exterior, el coeficiente
curva no se reclina y el piso no es lo suficientemente de rozamiento entre las ruedas y el suelo vale ., el
spero (friccin), ste caer. peralte es de grados?
Solucin.
Curvas peraltadas sin rozamiento

Para no tener que confiar en el roce o reducir el


desgaste de los rieles y pestaas, la carretera o la va
pueden inclinarse, como en la figura. En este caso la
componente de la normal dirigida hacia el centro de
curvatura proporciona la fuerza necesaria para
mantener al mvil en la pista. A la inclinacin de la
pista o va se le llama ngulo de peralte, . v2
F f = N , a c =
R
F // = ma c cos
v2
mgsen + N = m cos
R
v2
F = 0 N mg cos = m
R
sen

v2
N = mg cos + m sen
R
v2 v2
mgsen + mg cos + m sen = m cos
R R
En estos casos se toma la proyeccin de la normal
2
sobre la horizontal como causante de la fuerza v v2
mgsen + mg cos + m sen = m cos
centrpeta. Este caso se tiene, que: R R
v2 v2
m mg (sen + cos ) = m (cos sen )
R v2 R
tan = =
mg Rg
v = gR
(sen + cos )
Siendo , la inclinacin de la carretera. (cos sen )

33
Dinmica de una partcula Hugo Medina Guzmn

v = gR
(tan + ) Para que no se vaya
conoce como MARCOS DE REFERENCIA
INERCIALES.
(1 tan ) En los problemas trabajados hasta esta parte el primer
paso era dibujar un sistema de coordenadas. Elegimos
Ejemplo 54. Cul es la velocidad a laque puede ir un sistema fijo a tierra, pero no pusimos atencin al
un automvil por una curva con peralte, de radio R, hecho que la tierra no es un marco inercial debido a
para que no se deslice hacia el interior, el coeficiente que la tierra al viajar en su orbita casi circular
de rozamiento entre las ruedas y el suelo vale ., el alrededor del sol experimenta una aceleracin
peralte es de grados? centrpeta hacia el centro de la tierra. Sin embargo,
Solucin. estas aceleraciones son pequeas comparadas con la
aceleracin de la gravedad y a menudo se pueden
despreciar. En la mayora de los casos se supondr
que la tierra es un marco inercial.
Ahora veremos cmo cambian los resultados cuando
se trabaja en un MARCO DE REFERENCIA NO
INERCIAL, que es el nombre que se da a un marco
de referencia acelerado.

MARCO CON MOVIMIENTO DE


v2 TRASLACION NO UNIFORME.
F f = N , a c =
R Consideremos los sistemas S y S tal corno se
muestra en la Figura siguiente. El sistema S es
F// = mac cos inercial y el sistema S se mueve con respecto a S con

v2 aceleracin constante A = Ai , tal que
mgsen N = m cos
R 1 2
v2 D= At .
F = 0 N mg cos = m R sen 2

v2
N = mg cos + m sen
R
v2 v2
mgsen mg cos + m sen = m cos
R R
2 2
v v De la figura obtenemos que la posicin de la partcula
mgsen mg cos m sen = m cos P es:
R R
v2 1 2
mg (sen cos ) = m (cos + sen ) x = x'+ At , y = y ' , z = z '
R 2
1
v = gR
(sen cos ) r = r '+ At 2 i
2
(cos + sen ) Derivando con respecto al tiempo encontramos

v= gR
(tan ) Para que no se caiga
v x = v' x ' + At , v y = v' y , v z = v' z '
(1 + tan )
v = v'+ Ati
Derivando nuevamente encontramos
La velocidad debe de estar entre esos valores para
permanecer en la carretera. a x = a' x + A , a y = a' y ' , a z = a' z '

gR
(tan + ) v gR
(tan )
a = a '+ Ai o a = a'+ A

(1 tan ) (1 + tan ) Si la partcula P tiene una masa m y aplicarnos la


segunda ley de Newton del movimiento en el sistema
MOVIMIENTO EN MARCOS DE inercial S obtenemos
REFERENCIA NO INERCIALES
Hasta este momento nuestro estudio de mecnica F = ma
clsica lo hemos realizado en sistemas de referencia Donde P es la suma de todas las fuerzas de
que estn en reposo o con movimiento con velocidad interaccin que actan sobre las partculas.
constante con respecto a un sistema considerado en Para relacionar con el sistema no inercial S
reposo. A este conjunto de marcos de referencia se le

34
Dinmica de una partcula Hugo Medina Guzmn


F = m a '+ Ai o m a' = F m A

Aqu vemos que para que el observador segn S
pueda aplicar la segunda ley de Newton debemos

introducir una fuerza extra FA a la llamaremos El observador ve que el resorte se estira l . La
fuerza es
fuerza de arrastre y debemos incluirla en los
diagramas de fuerzas: F = kl
Aplicando la segunda ley de Newton:
FA = m A F x = ma x kl = mA

m a ' = F + FA mA
l =
De este modo, en el sistema S: F
Observador en el vagn:
Donde F ' es la suma de las fuerzas reales ms la de La figura a continuacin muestra el D.C.L. de la masa
arrastre m que no se mueve para el observador en el vagn.
Como es sistema no inercial tenemos que aplicar la
F ' = F + FA fuerza ficticia mA .

Recalquemos el carcter ficticio de FA . Para aplicar
una fuerza real sobre un cuerpo debemos ponerlo en
interaccin con otro, de manera que, segn la tercera

ley de Newton, si A ejerce una fuerza sobre B, FAB , Aplicando la segunda ley de Newton

a su vez B ejercer una fuerza sobre A, FBA , tal que
F x' = 0 mA = kl
mA

FAB = FBA .

l =
F
Ahora, es la reaccin de la fuerza de arrastre?, cul
es el otro cuerpo que est ejerciendo la fuerza ?. No Ejemplo 56. Analizar el caso de masa m colgada
existe tal cuerpo, la fuerza no tiene reaccin, es una mediante un hilo del techo de un vagn, que se
fuerza ficticia que agrega un observador ubicado en mueve con una aceleracin A.
un sistema acelerado (respecto a uno inercial) para a) Desde el punto de vista de un observador en tierra
justificar los fenmenos que observa. (S).
b) para un observador dentro del vagn (S).
Ejemplo 55. La fuerza para estirar o comprimir un
resorte es proporcional a su deformacin lineal,
F = kl , donde k es la constante del resorte y el
signo menos significa que la fuerza es en oposicin a
la deformacin. Si sobre una mesa sin friccin que se
encuentra en un vagn se coloca una masa. m sujeta
a un resorte de constante k y largo l , como se Solucin.
muestra en la figura. El tren arranca con una a) Para un observador en S:
aceleracin A que se mantiene constante en la El D.C.L. de la masa m
direccin x. Calcular la deformacin del resorte desde
el punto de vista del observador en tierra y desde el
punto de vista del observador en el vagn.

Aplicando la segunda ley de Newton:

Solucin.
F x = ma x Tsen = mA (1)
Observador en tierra: F y = 0 T cos mg = 0
La figura muestra el D. C. L. de la masa m. T cos = mg (2)
Dividiendo (1) : (2)

35
Dinmica de una partcula Hugo Medina Guzmn

A g 2
tan = tan = = = 33,7
g 3 3
g
b) Para un observador en S 2
El D.C.L..de la masa m
Ejemplo 58. Resolver el caso del peso del hombre
en un ascensor cuando asciende con una aceleracin
constante A, desde el punto de vista del hombre en el
ascensor.
Solucin.

Aplicando la segunda ley de Newton


F x' = 0 Tsen mA = 0
Tsen = mA = 0 (1)
Fy ' = 0 T cos mg = 0
T cos = mg (2)
Dividiendo (1) : (2) obtenemos:
A
tan = Aplicamos la segunda ley de Newton,
g
Ejemplo 57. Desde el techo de un carrito de juguete
F y' = ma y ' N mg ma = 0
cuelga una masa m unida al cielorraso mediante una N = m( g + a )
cuerda ideal. El carrito se encuentra en el piso de un El peso del hombre ser la reaccin N
ascensor que sube con aceleracin g/2. A su vez el En caso de subir con aceleracin a:
carrito tiene una aceleracin horizontal de magnitud g
N = m( g + a )
respecto al ascensor. Encuentre el ngulo que forma
la cuerda con la vertical, resuelva para un observador En caso de bajar con aceleracin a:
situado dentro del ascensor. N = m( g a )

Ejemplo 59. El pasajero de un tren deja caer una


piedra en diversos estados de movimiento del tren.
Hallar la trayectoria de dicha piedra que ve el
pasajero y la trayectoria vista por un observador en
tierra.
a) El tren acelera con aceleracin A constante.
b) El tren frena con aceleracin A constante.
Solucin.
Solucin.
Para un observador en el ascensor.
El tiempo en que la piedra esta en movimiento, es el
El D.C.L..de la masa m
mismo para todo sistema puesto que el movimiento
vertical es independiente del horizontal.
1 2
y = y' = h gt , para y = 0 la piedra lega al
2
piso:
1 2 2h
h gt = 0 t =
Aplicando la segunda ley de Newton 2 g
F x' = ma x '
a) Cuando el tren va con aceleracin A, deja caer una
Tsen = mg (1)
piedra.
g Considerando que en el momento que suelta la piedra
F = 0 T cos m g + = 0
y'
2 el tren tiene una velocidad v 0 .
3
T cos = m g (2)
2
Dividiendo (1) / (2)

36
Dinmica de una partcula Hugo Medina Guzmn

1 2
xtren = v0 t At
2
La piedra cae a una distancia
1 2
x = xtren x piedra = At , detrs del punto de
2
plomada.
Observador en tierra Observador en el tren
Las ecuaciones del movimiento en el sistema S. La ecuacin del movimiento en el sistema S
Movimiento de la piedra Movimiento de la piedra
x piedra = v0 t
Movimiento del tren
1 2
xtren = v0 t + At
2
La piedra cae a una distancia
1 2 1 2
x = xtren x piedra = At , detrs del punto de x piedra = At
2 2
plomada. 1 2
La piedra cae a una distancia x = At , detrs
Observador en el tren 2
La ecuacin del movimiento en el sistema S del punto de plomada.
Movimiento de la piedra El grfico siguiente muestra el moviendo visto por un
observador en el sistema S y en el sistema S.

1 2
x piedra = At
2
1 2
La piedra cae a una distancia x = At , detrs MARCO DE ROTACIN
2
Veamos el caso de un marco de referencia que est
del punto de plomada.
rotando con velocidad angular con respecto a otro
El grfico siguiente muestra el moviendo visto por un
observador en el sistema S y en el sistema S. marco de referencia. Supongamos que tenemos un
objeto movindose alrededor de un punto arbitrario;
este es un caso especfico, sin embargo tiene todos los
efectos en l.
La posicin de la partcula con respecto a un sistema

inercial est determinada por un vector r .
Consideremos un nuevo sistema de coordenadas tal
que siga al objeto, el nuevo origen est determinado
b) Cuando el tren desacelera con aceleracin A, deja
caer una piedra. por R contenido en r tal que la posicin de la

Considerando que en el momento que suelta la piedra
partcula en este nuevo sistema est ciada por r ' .
el tren tiene una velocidad v 0 .

Observador en tierra
Las ecuaciones del movimiento en el sistema S.
Movimiento de la piedra De la figura tenemos.

x piedra = v0 t r = R + r ' = Rr + r ' r = (R + r ')r
Movimiento del tren Derivando:

37
Dinmica de una partcula Hugo Medina Guzmn

Si la partcula tiene una masa m y aplicamos la


dr d d ( R + r ') dr
= (R + r ')r = r + (R + r ') segunda ley de Newton en el sistema inercial
dt dt dt dt
F = ma
dr
Como = t
dt donde F es la suma de todas las fuerzas de
interaccin que actan sobre la partcula.
d r dR dr ' Para relacionar con el sistema inercia!
= r + r + (R + r ')t
dt dt dt


F = m a '+ A o m a' = F m A
dr
= v es la velocidad de la partcula vista en el Para que el observador pueda aplicar la segunda ley
dt de Newton debemos introducir aqu tambin una

d r' fuerza extra FA y debemos incluirla en los
sistema inercial y = v' es la velocidad de la
dt diagramas de fuerzas
partcula vista en el sistema no inercial.
Tal que FA = m A
dR
v= r + v'+ (R + r ')t FA = FAr r + FAt t
dt d 2 ( R + r ') d (R + r ') dr d (R + r ') d dt
Para encontrar la aceleracin es necesario derivar r + + t + (R + r ') t + (R + r ')
dt 2 dt dt dt dt dt
nuevamente: d 2 ( R + r ')

FAr = m (R + r ') 2 r
d d (R + r ')
2 2
d r d 2
= 2 (R + r ')r = r + (R + r ')t dt
dt 2
y FAt = 2m d (R + r ') + (R + r ') t
dt dt dt

dr dt dt
Como = t y = r
dt dt De este modo, en el sistema no inercial

d 2 r d 2 ( R + r ') d ( R + r ') F ' = m a ' = F + FA
2
= 2
r + t
dt dt dt
d ( R + r ') d Recalquemos el carcter ficticio de FA Con el objeto
+ t + (R + r ') t (R + r ') 2 r
dt dt de clarificar esta idea veamos dos casos especiales:

d 2 (R + r ') d ( R + r ') a) El origen O rota con velocidad angular constante


= 2
r + 2 t a una distancia constante b, tal
dt dt
R + r ' = b , R y r son constantes.
+ (R + r ')t (R + r ') 2 r
d ( R + r ') d 2 ( R + r ')
d 2 ( R + r ') d ( R + r ') =0 y =0
= (R + r ') 2 r + 2 + (R + r ') t
dt
2
dt dt dt 2
d
d2 r


= constante, = =0
donde a = es la aceleracin de la partcula dt
dt 2 Slo nos queda
vista en el sistema inercial y
FAr = m(R + r ') 2 r = mb 2 r

d 2 r' Que es la fuerza ficticia del centro hacia afuera y se le


a' = es la aceleracin de la partcula vista en
dt 2 da el nombre de FUERZA CENTRFUGA, debemos
e1 sistema no inercial. insistir que solo aparece en el marco no inercial.
Llamando a
b) El origen O rota con velocidad angular constante
d 2 ( R + r ') y tambin se est alejando del origen fijo en O
Ar = (R + r ') 2 r
d (R + r ')
2
dt
con una velocidad constante V = .

d ( R + r ')
y At = 2
+ (R + r ') t dt
dt d

Con esto, = =0
Tenemos: A = Ar r + At t
dt
y nos queda

Tal que: a = a '+ A FAr = m(R + r ') 2 r

38
Dinmica de una partcula Hugo Medina Guzmn


y FAt = 2mVt F' z' = ma' z ' , F' r' = ma' r ' ,
Esta ltima fuerza ficticia, cuya direccin es F' t' = ma' t '
transversal, se conoce como FUERZA DE
CORIOLIS. Como a ' z ' = 0 , a ' r ' = 0 , F' t' = ma' t '
Tenemos
Ejemplo 60. Un cuerpo de masa de masa m unido a N mg = 0 , T + m 2 (l + l ) = 0 , Ft = 0
un resorte de constante k y longitud l que gira con
Como T = kl
ve1ocidad angular constante en un plano
horizontal sin friccin. Se quiere calcular el kl + m 2 (l + l ) = 0
estiramiento l del resorte. m 2 l
y l =
k m 2
Visto por un observador no inercial colocado sobre la
misma masa Este caso es idntico al caso anterior.

Ejemplo 61. Se tiene una plataforma circular de


radio R a la cual se le ha pintado un radio y gira con
velocidad angular constante . Un hombre camina
Solucin. de afuera hacia adentro de la plataforma siguiendo la
Visto por el observador inercial. lnea con una velocidad de mdulo constante v .
La figura muestra el D.C. L. de la masa
Cul es la fuerza que la plataforma ejerce sobre el
hombre, en funcin de su posicin?

Aplicando la segunda ley de Newton, el resorte estira


l , luego su longitud es (l + l )
F z = ma z , F
r = ma r , F t = mat Solucin.
La figura muestra el D.C.L. del hombre
Como: a z = 0 , a r =
2
(l + l ) , at =0
Tenemos
N mg = 0 , T = m 2 (l + l ) , Ft = 0
De aqu obtenemos:
N = mg y T = m 2 (l + l )
Como T = kl
kl = m 2 (l + l )
m 2 l Aplicando la segunda ley de Newton:
y l =
k m 2 F r = ma r Rr = ma r mr 2
F t = mat Rt = m( 2v + r )
Visto por un observador no inercial colocado en el
centro de rotacin y girando con la misma velocidad Como: a r = 0 y = 0:
angular.
Rr = mr y 2
Rt = 2mv
Rt es debido a la aceleracin de coriolis.
Rr es el sentido indicado en la figura y Rt en el
sentido contrario.

Aplicando la segunda ley de Newton:

39
Dinmica de una partcula Hugo Medina Guzmn

PREGUNTAS Y PROBLEMAS

1. Sobre una partcula de masa m que parte del Respuesta. 20N opuesta a la velocidad.
reposo en origen de coordenadas. acta una fuerza
5. Qu fuerza en adicin a F1 = 4i N y F2 = 2 j
( )

F = 2i + 3 j Despus de l0s la posicin de la N debe aplicarse al cuerpo en la figura, tal que:


partcula viene dada por las coordenadas (3m; 4,5 m). a) no acelere?
Cul es su masa? b) tenga una aceleracin 4i m/s2
Respuesta. m = 33,3 kg.

2 Hallar las fuerzas que actan sobre cada una de las


seis barras rgidas de peso despreciable. Si estn
unidas mediante pivotes lisos y cada una de las barras
cortas tiene una longitud l .

( )

Respuesta. a) F = 4i 2 j N, b)

( )

F = 16i 2 j N

6. Cul es la mnima aceleracin con la que puede


deslizarse hacia abajo un hombre de 75 kg por una
cuerda que solo soporta una tensin de 490N, Cul
Respuesta. AD = DB = mg ; CB = CA = mg/2, BC = ser la velocidad de la persona despus de deslizarse
2mg; CD = 0. la distancia de 20m?
CD se puede retirar y no pasa nada. Respuesta. a = 3,27 m/s2 ; v = 11,4 m/s

3. Dos cubos de masa m estn unidos mediante una 7. El libro de Fsica I, est apoyado en el extremo
cuerda y uno de ellos est sujeto al techo mediante superior de un resorte vertical, que a su vez esta
otra cuerda igual. parado sobre una mesa. Para cada componente del
a) Si en el cubo inferior se hace presin suavemente sistema libro-resorte-mesa-tierra:
hacia abajo. Cul de las cuerdas se romper antes? a) dibujar el diagrama de cuerpo libre,
porqu? b) identificar todos los pares de fuerzas de accin y
b) Si la masa interior se golpea hacia abajo con un reaccin.
martillo, se rompe la cuerda de abajo porqu?
8. De acuerdo con la leyenda, un caballo aprendi las
leyes de Newton. Cuando se le dijo que tirara una
carreta, se neg argumentando que si l tiraba la
carreta hacia delante, de acuerdo con la tercera ley de
Newton habra una fuerza igual hacia atrs. De esta
manera, las fuerzas estaran balanceadas y de acuerdo
con la segunda ley de Newton, la carreta no
acelerara. Cmo podra usted razonar con este
misterioso caballo?
Respuesta. a) La cuerda superior debido a que la
9. Dos alumnos de forestal ubicados en los bordes
tensin es mayor.
opuestos de un camino recto tiran a un carro por el
b) La tuerza de reaccin inercial de la masa superior
camino, con fuerzas de 160 N y 200 N, que forman
aumenta la resistencia frente a una aceleracin rpida.
un ngulo de 30 y 60 respectivamente, con la
direccin del camino.
4. Una caja de 40 kg que est resbalando en el piso
Calcular la magnitud de la fuerza resultante y la
disminuye su velocidad de 5 m/s a 2 m/s. Asumiendo
direccin en la que se mover el carro.
que la fuerza sobre la caja es constante, encontrar su
Respuesta. 256,1N, -21,3
magnitud y direccin relativa a la velocidad de la
caja.

40
Dinmica de una partcula Hugo Medina Guzmn

10. Una masa de 5kg cuelga de una cuerda de 1m de


longitud que se encuentra sujeta a un techo. Calcular
la fuerza horizontal que aplicada a la masa la desve
30 cm de la vertical y la mantenga en esa posicin.
Respuesta. 15,7 N.

( )

11. Tres fuerzas F1 = 2i + 2 j N,
Respuesta: a) a A = a B = 0,738 m/s2, b) 5,68 N
( ) = ( 45i ) N que actan

F2 = 5i 3 j N y F3
sobre un objeto le producen una aceleracin de valor 16. Un bloque A de 100 kg est unido a un contrapeo
3 m/s2. 8 de 25 kg mediante un cable dispuesto como muestra
a) Cul es la direccin de la aceleracin? la figura. Si el sistema se abandona en reposo,
b) Cul es la masa del objeto? determinar:
c) Si el objeto esta inicialmente en reposo, calcular su a) la tensin en el cable.
velocidad despus de 10s? b) la velocidad de B transcurridos 3 s,
c) la velocidad de A cuando ha recorrido 1,2 m.
12. Una mano ejerce una fuerza horizontal de 5 N
para mover hacia la derecha a dos bloques en
contacto entre s uno al lado del otro, sobre una
superficie horizontal sin roce. El bloque de la
izquierda tiene una masa de 2 kg y el de la derecha de
1 kg.
a) Dibujar el diagrama de cuerpo libre para cada
bloque.
b) Calcular la aceleracin del sistema,
c) Calcular la aceleracin y fuerza sobre el bloque de
1 kg,
d) Calcular la fuerza neta actuando sobre cada Respuesta. a) 302 N, b) 6,79 j m/s, c) -1,346 j m/s
cuerpo.
Respuesta. b) 5/3 m/s2, c) 5/3 m/s2, 5N, d) 5 N. 17. Determinar la aceleracin de cada uno de los
bloques de la figura, Que bloque llega primero al
13. Una fuerza F aplicada a un objeto de masa m1 suelo?
produce una aceleracin de 3 m/s2. La misma fuerza mA=5kg, mB = 15 kg, mC = 10kg
aplicada a una masa m2 produce una aceleracin 1
m/s2.
a) Cul es el valor de la proporcin m1/m2?
b) Si se combinan m1 y m2, encuentre su aceleracin
bajo la accin de F.
Respuesta. a) 1/3, b) 0,75 m/s2.

14. Dos bloques de masas M y 3M ubicado a la


derecha de M, que estn sobre una mesa horizontal
lisa se unen entre s con una varilla de alambre
horizontal, de masa despreciable. Una fuerza
horizontal de magnitud 2Mg se aplica sobre M hacia
la izquierda.
a) Hacer los diagrama de cuerpo libre.
b) Calcular la aceleracin del sistema.
c) Calcular la tensin del alambre. Respuesta. a A == 4,04 j m/s2,
Respuesta. b) 5 m/s2, c) 15Mg N.
a B == 0,577 j m/s2, a C == 2,89 j m/s2
15. Dos paquetes se colocan sobre un plano inclinado C llega primero.
como muestra la figura. El coeficiente de rozamiento
entre el plano y el paquete A es 0,25 y entre el plano
y el paquete B es 0,15. Sabiendo que los paquetes
18. En la figura = 0,45 , 5 kg . m A = 5 kg, m B =
estn en contacto cuando se dejan libres, determinar: 20 kg mC = 15 Kg. determinar la aceleracin de cada
a) la aceleracin de cada paquete, bloque.
b) la fuerza ejercida por el paquete A sobre el B.
c) Resolver el problema invirtiendo las posiciones de
los paquetes.

41
Dinmica de una partcula Hugo Medina Guzmn

b) 28 N y 37 N

21. Pepe anda esquiando, cuando en algn momento


sube 5 m deslizndose por la pendiente de un cerrito
nevado en sus esques, saliendo desde la cima
ubicada a 3 m de altura respecto a la horizontal, con
una rapidez de 10 m/s. El coeficiente de roce entre la
nieve y los esques es 0,1.
a) Calcular la rapidez con la cual el esquiador
comienza a subir la pendiente.
b) Determine la distancia horizontal que vuela Pepe
cuando sale de la punta del cerro.
Respuesta. a A = 4,91 j m/s2, a B = 2,45 j m/s2, Respuesta. a) 13 m/s, b) 16,6 m.

aC = 0 22. El bloque de masa m de la figura parte del
reposo, deslizndose desde la parte superior del plano
inclinado 30 con la horizontal. El coeficiente de roce
19. Determinar la aceleracin del cilindro B de la cintico es 0,3.
figura, si a) T = 1500 N, b) T = 4000 N. a) Calcular la aceleracin del bloque mientras se
mA=250 kg, mB = 100 kg, mueve sobre el plano.
b) Calcular la longitud del plano si el bloque sale con
una rapidez de 5 m/s.
c) Si el bloque cae al suelo a una distancia horizontal
de 3 m desde el borde del plano, determine el tiempo
total del movimiento.

Respuesta. a) 2,4 m/s2, b) 5,2 m, c) 2,8 s.

23. En el sistema de la figura, se aplica una fuerza F


Respuesta. a) -3,11 j N b) -9,81 j N sobre m. El coeficiente de roce es entre cada cuerpo
y los planos. Deducir la expresin de la magnitud de
20. Se tiene un sistema formado por tres bloques y F para que el sistema se mueva:
una polea sin friccin. El bloque A tiene una masa de a) con rapidez constante,
6,0 kilogramos y est en una superficie spera ( = b) con aceleracin a constante.
0,40). El bloque C tiene una masa de 4,0 kilogramos.
Una fuerza externa P = 80 N, se aplica verticalmente
al bloque A, la que mantiene el sistema en equilibrio
esttico segn como se muestra.

Respuesta. b)
Mg ( cos + sen ) + mg + a(m + M ) .

24. En el sistema de la figura, la fuerza F paralela al


plano inclinado empuja al bloque de masa m
hacindolo subir una distancia D sobre el plano, de
a) Cul es la masa del bloque B? Cul es la fuerza coeficiente de roce . Calcular en funcin de m, F, g,
de friccin sobre el bloque A? D, y , la aceleracin del bloque.
b) se quita la fuerza externa de 8,0 N. Las masas de
los bloques B y C se ajustan, de modo el sistema siga
en reposo tal como se muestra, pero estn justo por
iniciar el movimiento. La masa del bloque A no se
cambia. Las tensiones en las dos cuerdas verticales
son:
Respuesta.
a) 3,1 kg 25.2 N

42
Dinmica de una partcula Hugo Medina Guzmn

25. Una fuerza F se aplica a un pequeo bloque de


masa m para hacerlo moverse a lo largo de la parte 30. Sobre el planeta X un objeto pesa 12 N. En el
superior de un bloque de masa M y largo L. El planeta Y, donde la magnitud de la aceleracin de
coeficiente de roce es entre los bloques. El bloque cada libre es 1,6g, el objeto pesa 27 N. Cul es la
M desliza sin roce en la superficie horizontal. Los masa del objeto y cul es la aceleracin de cada libre
bloques parten del reposo con el pequeo en un en el planeta X?
extremo del grande, como se ve en la figura. Respuesta. 1,7 kg, 7m/s2.
a) Calcular la aceleracin de cada bloque relativa a la
superficie horizontal. 31. Dos bloques de 1 y 2 kg, ubicados sobre planos
b) Calcular el tiempo que el bloque m demora en lisos inclinados en 30, se conectan por una cuerda
llegar al otro extremo de M, en funcin de L y las ligera que pasa por una polea sin roce, como se
aceleraciones. muestra en la figura. Calcular:
a) la aceleracin de cada bloque,
b) la tensin en la cuerda.
c) si la aceleracin cuando los planos son rugosos
fuera de la calculada en ese problema, calcular: el
coeficiente de roce y la tensin en la cuerda.
Respuesta. a) (F- mg)/m, mg/(m+M),
b) [2L/(a1-a2)]1/2.

26. Un bloque de masa M se ubica sobre un pequeo


plano inclinado un ngulo sin roce, que tiene su
extremo inferior fijo a un eje vertical que puede girar.
En algn momento el eje gira con el plano con
rapidez constante.
Demostrar que si la masa asciende desde la base del
plano, su rapidez cuando ha subido una distancia L es 32. Un trineo de 50 kg de masa se empuja a lo largo
v = gLsen . de una superficie plana cubierta de nieve. El
coeficiente de rozamiento esttico es 0,3, y el
coeficiente de rozamiento cintico es 0,1.
27. Una fuerza dependiente del tiempo, a) Cul es el peso del trineo?
( )

b) Qu fuerza se requiere para que el trineo


F = 8i 4tj N (donde t est en segundos), se
comience a moverse?
aplica a un objeto de 2 kg inicialmente en reposo. c) Qu fuerza se requiere para que el trineo se
a) En qu tiempo el objeto se mover con una mueva con velocidad constante?
velocidad de 15 m/s? d) Una vez en movimiento, qu fuerza total debe
b) A qu distancia est de su posicin inicial cuando aplicrsele al trineo para acelerarlo a 3 m/s2?
su velocidad es 15 m/s?
c) Cul es la posicin del objeto en este tiempo? 33. La masa m1 sobre una mesa horizontal sin friccin
Respuesta. a) 3s, b) 20,1m, c) (18i 9 j ) m se conecta a la masa m2 por medio de una polea mvil
y una polea fija sin masas. Si a1 y a2 son magnitudes
28. Una araa de 2 x 10-4 kg est suspendida de una de las aceleraciones de m1 y m2, respectivamente.
hebra delgada de telaraa. La tensin mxima que Determinar:
soporta la hebra antes de romperse es 2,1 x 10-3 N. a) una relacin entre estas aceleraciones.
Cul es la aceleracin mxima con la cual la araa b) las tensiones en las cuerdas, y
puede subir por la hebra con toda seguridad? c) las aceleraciones a1 y a2 en funcin de m1, m2 y g.
Respuesta. 0,5m/s2.

29. Los instrumentos de un globo meteorolgico


tienen una masa de 1 kg.
a) El globo se suelta y ejerce una fuerza hacia arriba
de 5 N sobre los instrumentos. Cul es la aceleracin
del globo y de los instrumentos?
b) Despus de que el globo ha acelerado durante 10 34. Calcular la fuerza F que debe aplicarse sobre un
segundos, los instrumentos se sueltan. Cul es bloque A de 20 kg para evitar que el bloque B de 2 kg
velocidad de los instrumentos en el momento en que caiga. El coeficiente de friccin esttico entre los
se sueltan? bloques A y B es 0,5, y la superficie horizontal no
c) cul es la fuerza neta que acta sobre los presenta friccin.
instrumentos despus de que se sueltan?
d) En qu momento la direccin de su velocidad
comienza a ser hacia abajo?

43
Dinmica de una partcula Hugo Medina Guzmn

b) Calcular la tensin de la cuerda.


c) Si el sistema da una vuelta en 30 s, determinar
El ngulo que forma la cuerda con la vertical.

35. Una bola de masa m se suelta sin velocidad inicial


desde un punto A y oscila en un plano vertical al
extremo de una cuerda de longitud L. Determinar:
a) la componente tangencial de la aceleracin en el
punto B. Respuesta. a) v = g (l + Lsen ) tan ,
b) la velocidad en el punto B. b) mg/cos .
c) la tensin en la cuerda cuando la bola para por el
punto mas bajo. 39. Una bola pequea da vueltas con una rapidez y
d) el valor de si la tensin en la cuerda es 2 mg recorriendo una circunferencia horizontal en el
cuando la bola pasa por el punto C interior de un cono recto de base circular. Expresar la
rapidez y en funcin de la altura y de la trayectoria
sobre el vrtice del cono.

Respuesta. a) gsen , b) 2 gL(cos cos 0 ) ,


c) mg (3 2 cos 0 ) , d) 60.

Respuesta. v = gy
36. Tres automviles circulan a la velocidad de 80
km/h por la carretera representada en la figura.
Sabiendo que el coeficiente de rozamiento entre las 40. Cul es el mnimo radio que un motociclista con
llantas y la carretera es 0,60, determinar la velocidad de 21 m/s puede hacer en una pista que
desaceleracin tangencial de cada automvil s sus tiene un coeficiente de friccin con las llantas igual a
respectivos frenos sen repentinamente accionados y 0,3? Cul es el ngulo que har la motocicleta con la
las ruedas deslizan. horizontal?
Respuesta: 147 m; 73 20

41. Un estudiante hace girar un balde que contiene 2


kg de agua en una circunferencia vertical de l,2m de
radio, considerar
a) Cul es la mxima velocidad para que el agua
Respuesta. a A =3,91 m/s2, a B = 7,86 m/s2, aC = permanezca en el balde?
b) Cul es la fuerza ejercida por el balde sobre el
5.89 m/s2.
agua en el punto inferior de la circunferencia?
c) a la altura de los hombros?
37. Con qu ngulo debe peraltarse una carretera en
d) Si el balde pesa 10k, hallar cada una de las fuerzas
una curva de 50 m de radio, para que un vehculo
que actan sobre el balde en el punto inferior de la
pueda tomar la curva a 72 km/h, con un coeficiente
circunferencia.
de rozamiento 0,30?
Respuesta: 22,5 55,9 r
Respuesta. a) 2 , b) 2mg , c) 2mg
g
38. En el sistema de la figura, el brazo del pndulo es d) 10 N debido a la tierra, 40 N debido al agua, 100 N
de longitud l y la cuerda de largo L. debido al hombre.
a) Calcular la rapidez tangencial para que el sistema
gire en torno al eje de rotacin que pasa por la barra 42. Una mesa giratoria horizontal tiene una
vertical, de modo que la cuerda que sostiene a la aceleracin angular de = 3 rad/s2. En el instante en
masa m forme un ngulo de con la vertical. que la velocidad angular vale 2,4 rad/s, una partcula

44
Dinmica de una partcula Hugo Medina Guzmn

de masa 1,8 kg descansa sin deslizar sobre la mesa, c) a = 5 cm/s2, d) = 53,1; e) Ft = 15 x 10-5 N,
con tal que est situada a una distancia inferior a 50
cm del eje vertical de rotacin de la mesa, f) Fr = 20 x l0-5 N, g) F = 25 x 10-5 N.
a) Cul es el valor de la tuerza de rozamiento?
b) Hallar el coeficiente de rozamiento esttico entre el 44. Describir e interpretar las fuerzas que realmente
objeto y la mesa. se apreciaran si nos encontrramos con los ojos
Respuesta: a) Ff = 7,9 N b) s = 0,45 vendados y:
a) de pie sobre una plataforma elevada.
b) cayendo libremente en el aire.
43. Se tiene una partcula de masa 5g que se mueve
c) estando sentado en el suelo de una plataforma en
sobre una trayectoria curva y su aceleracin en un

rotacin, como la de un carrusel a una cierta distancia
momento dado vale a = (3t + 4n ) cm/s2. Hallar: de su centro.
Respuesta. a) Una fuerza de reaccin de la
a) la aceleracin tangencial, plataforma hacia arriba.
b) la aceleracin centrpeta, b) Ninguna fuerza.
c) el mdulo de la aceleracin total, c) Una fuerza de reaccin de la plataforma y una
d) el ngulo que la aceleracin total forma con la fuerza hacia afuera (radial).
tangente a la curva,
e) la componente tangencial de la fuerza aceleradora, 45. Calcular el ngulo de peralte de una carretera en
f) la componente centrpeta de la fuerza aceleradora, una curva de radio 150 m, para que un camin de 15
g) la fuerza aceleradora total. toneladas pueda girar con una rapidez de 70 km/hr,
Respuesta. a) a t = 3 cm/s2 , b) a t = - 4 cm/s2 ; sobre un pavimento cubierto de escarcha.
Respuesta. 14

45
TRABAJO Y ENERGA Hugo Medina Guzmn

CAPTULO 5. TRABAJO Y ENERGA

INTRODUCCIN Sin embargo, es importante notar que los conceptos


Con lo que hemos visto hasta el momento estamos en de Trabajo y Energa se fundamentan en las leyes de
condiciones de analizar un movimiento en situaciones Newton y por lo tanto no requieren ningn principio
en que la fuerza es constante. Una vez aplicada La nuevo.
segunda ley de Newton, determinamos la aceleracin
a = F / m . De aqu podemos determinar la TRABAJO
velocidad y la posicin. Pero en el caso en que la El trmino trabajo que se usa en la vida cotidiana es
fuerza no es constante, por ejemplo cuando se jala para definir una actividad de algn tipo que incluye
una masa situada en un extremo de un resorte, el un esfuerzo fsico o mental y cuya finalidad sea el
problema se complica. alcance de algn objetivo definido y bien establecido.
En el estudio de la mecnica tiene un significado ms
restringido, por ejemplo si subimos cierta altura h con
una masa m decimos que hemos realizado un trabajo
W, si subimos la misma altura h pero con una masa
2m, se habr realizado un trabajo 2W, igual a que si
se hubiese transportado una masa m una altura 2h, o
si se hubiese transportado dos veces la masa m, la
altura h. Estas observaciones sugieren que el trabajo
es una magnitud fsica proporcional a la fuerza y a la
distancia, pero que puede sumarse como un escalar.
Cuando una fuerza constante Fx mueve un cuerpo
La figura muestra un cuerpo de masa m sobre una realizando un desplazamiento x que tiene la misma
superficie horizontal lisa, conectado a un resorte direccin que la fuerza, se define la cantidad de
helicoidal. Si el resorte se estira o se comprime una trabajo realizado por esta fuerza como:
longitud pequea desde su posicin no deformada o W = Fx x
de equilibrio, el resorte ejercer una fuerza sobre el
cuerpo F = kx , donde x es el desplazamiento del
cuerpo desde la posicin de equilibrio ( x = 0 ) , k es
la constante del resorte, el signo negativo (-) significa
que la fuerza es en sentido opuesto al sentido del
desplazamiento. Esta ley de fuerza se conoce como la
ley de Hooke, de la cual nos ocuparemos en el
Captulo de Elasticidad

Apliquemos la segunda ley de Newton: Ahora consideremos que sobre la misma masa m
F = ma acta una fuerza vertical Fy , menor que el peso mg
dv d 2 x del bloque, como tal no dar origen a ningn
Con F = kx y a = = , movimiento vertical y por lo tanto no estar
dt dt 2 realizando trabajo.
Obtenemos:
d 2x
kx = m 2
dt
2
d x k
+ x=0
dt 2 m
A pesar de ser una ecuacin simple esta ltima,
todava no tenernos el conocimiento matemtico para
resolverla. Es decir, estamos en condiciones de
Si ahora aplicamos al mismo tiempo las dos fuerzas,
plantear las ecuaciones del movimiento, pero no
la fuerza aplicada es:
sabemos resolverlas.

Veremos aqu que se puede tomar un atajo y resolver F = Fx i + Fy j
de otra forma el problema. En este capitulo se vern
los conceptos de Trabajo y Energa que se pueden Si el desplazamiento del bloque es nicamente en la
aplicar a la dinmica de un sistema mecnico sin direccin x,
recurrir a las leyes de Newton.

1
TRABAJO Y ENERGA Hugo Medina Guzmn


r = xi dW = F d r cos

dW = F ds cos
dW = Ft ds
Es el trabajo realizado por la componente tangencial
de la fuerza Ft .
El trabajo de la componente normal Fn es nulo.

El trabajo realizado es el producto escalar de la tuerza Para evaluar el trabajo realizado para ir desde el
por el desplazamiento es: punto P1(x1, y1, z1) a un punto P2(x2, y2, z2) tenemos
que integrar el trabajo diferencial.
W = F r = (Fx i + Fy j ) xi = Fx x

P2 P2
WP1P2 = dW = F d r
O W = Fx cos P1 P1
Donde
2 2 Para esto tenemos que conocer como vara F
F = F +F x y y

es el ngulo formado entre la fuerza aplicada y el F = Fxi + Fy j + Fz k


desplazamiento.
Siendo d r = dxi + dyj + dzk
Consideremos el caso general de una fuerza F
cualquiera que mueve a una partcula sobre una Tenemos: F d r = Fx dx + Fy dy + Fz dz
trayectoria curva como se muestra en la siguiente x2 y2 z2
figura. Luego: WP1 P2 = x1
Fx dx + Fy dy + Fz dz
y1 z1
La unidad de trabajo es una unidad derivada de las
unidades de fuerza y de longitud.
[W ] = FL = ML2T 2
En el sistema Internacional la unidad de trabajo es el
Joule (J).
1 Joule = (1 Newton)(1 metro)

Ejemplo 1. Un hombre levanta una masa m con una


fuerza tal que la coloca a una altura h sobre el piso a
Sea P la posicin de la partcula en un instante t , la velocidad constante.
posicin con respecto al origen de coordenadas O est a) Cunto trabajo realiza la gravedad?
dada por b) Cul es la magnitud de la fuerza que ejerce el
hombre?
OP = r Solucin.
La partcula en el tiempo t describe la trayectoria a)

PP' , si esta es suficientemente pequea se puede

asimilar como la cuerda PP' , el desplazamiento de la

partcula en el tiempo t es PP' = r
Cuando P tiende a P (t 0 ) .

La direccin de la cuerda

PP' es el de la tangente PT
Wgravedad =
y =h

y =0

(
F d r = mgk dyk
h

0
)
en P, r es d r , la fuerza es constante en direccin h
y sentido.

= mg 0
dy = mgh
El trabajo de la fuerza F para el desplazamiento b) Podramos hacerlo directamente por la ley de
Newton, pero lo haremos con los conceptos de
d r es un trabajo diferencial. trabajo. Como la masa se mueve con velocidad
constante, el trabajo realizado es cero.
dW = F d r Whombre + Wgravedad = 0

2
TRABAJO Y ENERGA Hugo Medina Guzmn

Whombre = Wgravedad = mgh Calcule el trabajo cuando el cuerpo se mueve desde x


= 0 hasta x = 8 m.
Tambin tenemos:
y=h
Fhombre d r = Fk dyk
h
Whombre =
y =0 0
h
= F
0
dy = Fh
Luego: Fh = mgh F = mg

Y F = mgk Solucin.
El trabajo realizado por la fuerza es exactamente
Ejemplo 2. Se arrastra una caja de masa m sobre un igual al rea bajo la curva desde x = 0 hasta x = 8.
piso horizontal, el coeficiente de friccin cintico 1
entre la caja el piso es , mediante una fuerza que
W = (5N )(2 0)m + (5N )(5 2)m + 1 (5N )(8 5)m
2 2
forma un ngulo con la horizontal, la caja se = (5 + 15 + 7,5) Nm
desplaza un distancia s haca la derecha, = 27,5 J
a) Calcule el trabajo realizado por la fuerza
b) Calcule el trabajo efectuado por La fuerza de Ejemplo 4. Trabajo realizado por un resorte.
friccin.
e) Determine el trabajo neto efectuado sobre la caja
por todas las fuerzas que actan sobre ella.

El resorte de la figura, cuando se deforma o estira


hasta una cierta posicin x, ejercer una fuerza
Solucin.
restauradora F = kx .
a) El trabajo efectuado por F es: Solucin.
x=s Supongamos que el objeto se empuja hacia la
WF = F d r izquierda una distancia x respecto a la posicin de
x =0
equilibrio y se deja libre.

Como F = F cos i + Fsen j y d r = dxi El trabajo realizado desde x1 = x hasta x 2 = 0
por la fuerza del resorte a medida que el objeto se
WF =
x =0
x=s
(F cos i + Fsen j ) dxi mueve es
x2 = 0 0 1
= Fs cos = Fx s W = Fx dx = ( kx )dx = kx 2

x1 = x x 2
La componente vertical de F no realiza trabajo. Y si consideramos el trabajo realizado por el resorte a
medida que se estira de x1 = 0 a x2 = x el trabajo
b) Como F f = Ni
1 2
Y N = mg Fsen es W = kx
2

Obtenemos F f = (mg Fsen )i
Este resultado podemos obtenerlo tambin de La
grfica F versus x, como se muestra en la figura
siguiente.
El trabajo efectuado por F f es
s
W f = F f d r = (mg Fsen )i dxi
s

0 0

= (mg Fsen )s
c) El trabajo neto sobre la caja es la suma de los
resultados obtenidos en a) y b).
Wneto = WF + W f = F cos s (mg Fsen )s
[
= F cos (mg Fsen ) s ]
Ejemplo 5. La posicin de una partcula en el plano

Ejemplo 3. Una fuerza que acta sobre un cuerpo est dada por r = 3ti 2t j (t en segundos, r en
2
vara con respecto a x como se muestra en la figura.

3
TRABAJO Y ENERGA Hugo Medina Guzmn

metros), la fuerza ejercida sobre la misma es P2 v2 1


W12 = dW = d mv 2
F = 4i 5 j (en Newton). P1 v1
2
Qu trabajo se realiza sobre la partcula en el 1 2 1 2
intervalo de t = l s a t = 3 s? = mv 2 mv1
Solucin.
2 2

Aqu tenemos una medida para el trabajo realizado
r = 3ti 2t 2 j d r = 3dti 4tdtj sobre la partcula expresada en funcin de la
Luego 1 2
variacin de la magnitud mv .
( )( ) 2

dW = F d r = 4i 5 j 3dti 4tdtj
Esta magnitud se define como la ENERGIA
= 12dt + 20tdt CINETICA K de la partcula.
El trabajo W realizado sobre la partcula entre t = 1 y 1 2
t = 3. Entonces: K = mv
t =3 3 2
W = dW = (12 + 20t )dt La energa cintica es una propiedad general del
xt =1 1
3
movimiento de la partcula es la ENERGIA DEL
1 MOVIMIENTO. Sus dimensiones son las de trabajo.
= 12t + 20t 2 = 126 22 = 104 J
2 1 [K ] = ML2 T -2
El trabajo realizado sobre la partcula es 104 Joules. Su unidad es la misma que la del trabajo.

Resulta conveniente escribir:


ENERGIA CINETICA W12 = K 2 K 1 = K
Consideremos una partcula de masa m bajo la accin El trabajo realizado por la fuerza al desplazar una

partcula es igual al cambio de energa cintica de la
de la fuerza F .
partcula.
La segunda ley de Newton afirma que:


dv Ejemplo 6. Encontrar la variacin de la energa
F = ma = m cintica de un proyectil en funcin de su altura. Se
dt lanza un proyectil de masa m desde el punto P0 (x0,

Tambin sabemos que d r = v dt . y0) con una velocidad inicial v 0 = v 0 x i + v 0 y j .
Multiplicando escalarmente:
Solucin.
dv
F d r = m v dt = m v d v
dt

Como F d r es el trabajo diferencial dW y
d 1 2 1 d
mv = m v d v
dt 2 2 dt

1 dv 1 dv
= m v + m v Para un proyectil la posicin en funcin del tiempo
2 dt 2 dt es;

dv 1 2
= m v x = x0 + v0 x t , y = y 0 + v0 y t gt
dt 2
De aqu: Y la velocidad
1 v x = v0 x , v y = v0 y gt
d mv 2 = m v d v La energa cintica en P0 es
2
1 2 1
Reemplazando obtenemos: K0 =
2
(
mv0 = m v 02x + v 02y
2
)
1
dW = d mv 2 La energa cintica en P es
2 1 2 1
El trabajo para ir de P1 donde la velocidad es v1 al K=
2 2
(
mv = m v x2 + v y2 )
punto P2 donde la velocidad es v 2 ser: 1 2
( 2 2 2
= m v0 x + v0 y 2v0 y gt + g t
2
)

4
TRABAJO Y ENERGA Hugo Medina Guzmn

La variacin de energa entre P y P0 es: dirigida hacia abajo y la bola se mueve lo ms


1 1 rpidamente.
K = K K 0 = mv 2 mv02
2 2 mv 2
fuerza radial =
1
(
= m 2v 0 y gt + g t
2
2 2
) mv 2
r
1 2
T mg = y mhg = mv
1 2 r 2
= mg v 0 y t gt
2 h = r r cos T mg = 2mg (1 cos )
1 2 T mg 120 80
Como y y 0 = v 0 y t gt cos = 1 = 1 = 0,75
2 2mg 2(80 )
1 2 1 2 = 41,4
Resulta K = mv mv0 = mg ( y y 0 )
2 2
Ejemplo 8. Se arrastra una caja de masa m sobre un
Ejemplo 7. En una demostracin experimental para piso horizontal, el coeficiente de friccin cintico
ilustrar la conservacin de la energa por medio del entre la caja el piso es , mediante una fuerza que
dispositivo siguiente. Se ata una bola del bowling a forma un ngulo con la horizontal. Si se empieza a
un extremo de una cuerda, y se sujeta el otro extremo jalar desde el reposo y considerando que ya se inici
al techo de la sala de conferencias. Se sostiene la el movimiento Cul es la velocidad del bloque
bola parado en una escala tijeras alta, Para la despus que recorre una distancia s?
demostracin se suelta del reposo en el extremo de la
nariz, la bola volver de la oscilacin ms arriba y
golpear violentamente la cara, (intente esto alguna
vez si usted desea experimentar un juego para
asustar)
La demostracin impresiona a la clase, pero no por la
razn esperada. Aunque la cuerda es bastante fuerte
para sostener la bola cuando est inmvil, cuando la Solucin.
dej ir, la cuerda se rompi en el fondo del arco y la En este caso como la fuerza F es constante, por la ley
bola fue despedida alrededor del saln "Boing boing, de Newton podramos encontrar la aceleracin, que es
boing" y dispers a los presentes en todas las constante, pero vamos a hacerlo por conceptos de
direcciones. Energa Cintica y Trabajo.
Una bola de bowling realmente rebota en el concreto. Encontramos que
Suponga que la bola pesa 80 N y la cuerda tena 4,0 WNeto = [F cos (mg Fsen )]s
m de largo y tena una resistencia a ruptura de 120 N. Sabemos que
Cul es el mximo ngulo con la vertical con el que
1 2 1 2
se habra podido lanzar la bola sin tener la rotura de WNeto = K 2 K1 = mv2 mv1
la cuerda? 2 2
Como: v1 = 0 y v2 = v
Finalmente:
2
v= [F cos (mg Fsen )]s
m

Ejemplo 9. Para el caso de la masa m atada a un


resorte con constante de rigidez k . Cul es la
velocidad cuando pasa por la posicin de equilibrio
Solucin. despus de estirarlo una longitud L y soltarlo?
Solucin.

La cuerda debe proporcionar suficiente fuerza


ascendente para balancear el peso ms la fuerza radial
necesaria para que la bola haga la curva hacia arriba.
La tensin en la cuerda ser as la mayor en el punto El trabajo realizado desde x = L a x = 0 por la fuerza
ms bajo del arco, donde la fuerza de la gravedad est restauradora del resorte F = - kx Es:

5
TRABAJO Y ENERGA Hugo Medina Guzmn

1 2
WR = kL
2
1 2 1 2
Tambin WR = K 2 K1 = mv2 mv1
2 2
Siendo v2 = v0 y v1 = 0 Si en un sistema conservativo el trabajo efectuado por
1 2 1 2 la fuerza para desplazar la partcula de A a B es
Tenemos kL = mv0 0 independiente del camino entre A y B, se puede
2 2 escribir:
k WAB = WBA
v0 = L
m En un circuito cerrado
Para el caso que mostramos la respuesta correcta es la WAA = WAB + WBA
negativa. Como WAB = WBA WAA = WAB WAB = 0
Ejemplo 10. Un objeto de masa m se mueve en el eje El trabajo total efectuado por una fuerza conservativa
sobre una partcula es cero cuando la partcula se
A mueve alrededor de cualquier trayectoria cerrada y
x sujeto a la fuerza F = m i donde A es una
x2 regresa a su posicin inicial.
constante y x es la distancia desde el origen. Naturalmente la definicin de un sistema no
a) Cunto trabajo realiza esta fuerza si el objeto se conservativo es aquel que no satisface las condiciones
mueve de x = a a x = b? anteriores.
b) Si la masa tena una velocidad v en la direccin
positiva de x, Cul es su velocidad en b? Ejemplo 11. Sistema no Conservativo. - La fuerza
Solucin. de friccin. Supongamos que un bloque se mueve del
a) El trabajo que realiza la fuerza para mover la masa punto P1 (x1, y1) al punto P2 (x2, y1), siguiendo Las
desde x = a a x = b es: trayectorias mostradas en las figuras siguientes, el
x =b A coeficiente de friccin entre el bloque y la superficie
Wab = F d r , F = m i , d r = dxi es . Calcular el trabajo realizado por la friccin en
x=a x2 ambos casos.

A b dx
i dxi = mA 2 =
b
Luego Wab = m 2
a x a x

1 1 1
b

mA = mA
x a a b

1 2 1 2 Solucin.
b) Como Wab = K b K a = mvb mva Por la trayectoria (a)
2 2 x2
WP1 P2 = F f d r
va = v0
x1
Siendo

1 1 1 2 1 2 Aqu F f = Ni , d r = dxi
Tenemos mA = mvb mv0
a b 2 2 Luego
( N )dx = N (x2 x1 )
x2

1 1 WP1 P2 =
vb = 2 A + v02 x1

a b
Por la trayectoria (b)
y2 y1
SISTEMAS CONSERVATIVOS Y NO x2
WP1P2 = F f 1 d r1 + F f 2 d r2 + F f 3 d r3
CONSERVATIVOS y1 x1 y2

Un sistema conservativo es aquel en el que el trabajo Aqu


realizado por las fuerzas del sistema es independiente
de la trayectoria seguida por el mvil desde una F f 1 = Nj , d r1 = dyj
posicin a otra, no existen fuerzas de rozamiento, ni
dispositivos que puedan producir prdida de la F f 2 = Ni , d r2 = dxi
energa cintica.
F f 3 = Nj , d r3 = dyj
Luego

6
TRABAJO Y ENERGA Hugo Medina Guzmn

( N )dy + x ( N )dx + y (N )dy r2 r2


y2 x2 y1
WP1 P2 = WP1P2 = Fg d r + Fg d r
y1 1 2

= N ( y2 y1 ) N ( x2 x1 ) + N ( y1 y2 )
r1 r3
Aqu

= N ( x2 x1 ) 2 N ( y2 y1 ) Fg = mgj , d r = dxi + dyj


Obviamente el trabajo realizado por la fuerza de

friccin por las dos trayectorias a) y b) no son Fg d r = ( mgj ) (dxi + dyj ) = ( mg )dy
iguales, por consiguiente cuando hay fuerza de Luego
friccin el sistema no es conservativo. (La friccin no
es conservativa).
( mg )dy + y ( mg )dy
y3 y2
WP1P2 =
y1
Ejemplo 12. Sistema Conservativo. La fuerza de 3

la gravedad Supongamos que un bloque de masa m


se mueve del punto P1(x1 ,y2) al punto P2(x2 ,y2) donde = mg ( y3 y1 ) mg ( y2 y3 )
y es la direccin vertical. Calcular el trabajo realizado
por la fuerza gravitacional con los tres casos
= mg ( y 2 y1 )
mostrados en la figura.
Resultado igual que en a) y b)
Luego la fuerza de la gravedad es una fuerza
conservativa.

Trabajo en una trayectoria cerrada.


Si completamos la trayectoria volviendo al punto
inicial, tenemos una trayectoria cerrada y el trabajo es
cero.

El trabajo para ir de 1 a 2 es
x2
(F i + F j ) (dxi + dyj )
x2
Wr1r2 = F d r1 = x y
Solucin. x1 x1

Por la trayectoria a) x2 y2

y2
= x1
Fx dx + Fy dy
y1
WP1P2 = Fg d r Como
y1

Aqu

F = mgj : Fy = mg . Fx = 0
Fg = mgj , d r = dyj Wr1r2 = 0 mg ( y2 y1 )
= mg ( y2 y1 )
Luego
WP1 P2 = mgdy = mg ( y2 y1 )
y2

y1
El trabajo para ir 2 a 1 es
x1 x1 y1
Por la trayectoria b) Wr2 r1 = F d r = Fx dx + Fy dy
x2 y2 x1 x2 x2 y2
WP1P2 = Fg d r1 + Fg d r2 + Fg d r3
x1 y1 x2 = 0 mg ( y1 y2 )
= mg ( y1 y2 )
Aqu

Fg = mgj , d r1 = dxi , d r2 = dyj , d r3 = dxi
El trabajo total es
Luego
Wr1r1 = Wr1r2 + Wr2 r1
( mg )dy + 0 = mg ( y2 y1 )
y2
WP1P2 = 0 +
y1 = mg ( y2 y1 ) mg ( y1 y2 )
Igual que en a) =0

Por la trayectoria c). Esto no sucedera en el caso de una fuerza no


WP1P2 = WP1P3 + WP3 P2 conservativa, como la fuerza de friccin.

7
TRABAJO Y ENERGA Hugo Medina Guzmn

LA FUNCION ENERGA POTENCIAL Si consideramos la energa potencial igual a cero en


El trabajo realizado por la fuerza el nivel de referencia y = 0, la energa potencial a
cualquier altura con respecto a y = 0 es:
F = Fxi + Fy j + Fz k U ( y ) = mgy
Para mover una partcula de Tambin podamos haber determinado esta funcin a
P1(x1. y1, z1) a P2(x2, y2, z2) es igual a: partir de:

(F dx + F dy + F dz ) dU = Fg dy dU = ( mg )dy = mgdy
P2 P2
W12 = F d r = x y z
P1 P1
Integrando
Para un sistema conservativo el trabajo es
independiente de la trayectoria seguida.
dU = mgdy + C
Su integral debe ser un diferencial exacto, U ( y ) = mgy + C
digamos - dU, tal que integrndolo, solamente los Donde C es una constante relacionada con las
lmites determinan el valor de la integral. condiciones de cada caso, por ejemplo aqu
Esto es: consideramos para y = 0
W12 =
P2
( dU ) = ( U )PP 2
= (U 2 U1 ) = U U (0 ) = 0 .
P1 1

Aqu llamamos a U, energa potencial, cuyas La constante es C = 0


unidades son las mismas que las de trabajo. U ( y ) = mgy
Como comprobacin, a partir de esta energa
Hemos determinado la funcin energa potencial a potencial podemos encontrar la fuerza.
partir de una fuerza dada.
U mgy
Fx = = =0
Consideremos ahora el problema inverso, a partir de x x
una funcin energa potencial determinar la fuerza U mgy
Fy = = = mg
dU = F d r = Fx dx Fy dy Fz dz y y
Como U es funcin de x, y y z, podemos escribir esta U mgy
derivada en funcin de sus derivadas parciales:
Fz = = =0
z z
U U U
dU ( x , y , z ) = dx + dy + dz Luego: F = mgj
x y z

Relacionando con los componentes de la fuerza Ejemplo 14. Determinar la funcin energa potencial
obtenemos asociada a un resorte de constante de rigidez k.
Solucin.
U U U Consideremos que el resorte est en el eje x, y se
Fx = , Fy = , Fz =
x y z estira en esa direccin.

U U U F = kxi
F = i+ j+ k
x y z Tenemos que:
dU = Fx dx = ( kx )dx = kxdx
Ejemplo 13. La fuerza de la gravedad es un ejemplo Integrando
de fuerza conservativa. 1 2
Solucin. U= kx + C
Tomemos la vertical a la tierra como el eje y, tal que: 2
Si para la posicin de equilibrio x = 0, la energa
Fg = Fg j = mgj potencial es cero, C es igual a cero y
El trabajo realizado por la gravedad cuando la 1 2
U= kx
partcula se desplaza desde el punto y1 al punto y2 es: 2
Ahora realicemos el problema inverso:
W12 = mgj dyj = mg dy
y2 y2

y1 y1 1 2
Dado U = kx encontrar la fuerza
= mg ( y2 y1 ) 2
Como W12 U : correspondiente:
mg ( y2 y1 ) = U = U ( y1 ) U ( y 2 ) U 1
Fx = = kx 2 = kx
x x 2
O U = U ( y 2 ) U ( y1 ) = mgy2 mgy1

8
TRABAJO Y ENERGA Hugo Medina Guzmn

U Colocando las energas iniciales a un lado y las


Fy = =0 finales al otro tenemos:
y
K1 + U1 = K 2 + U 2
U Esta ecuacin es la forma matemtica de El
Fz = =0
z principio de conservacin de la energa mecnica.
Si definimos la energa mecnica total del sistema E
Luego F = kxi como la suma de la energa cintica y potencial se
puede expresar la conservacin de la energa
Ejemplo 15. Energa potencial gravitatoria cerca mecnica como:
de la tierra. Por la ley de Newton de la gravitacin E = K + U = Constante
universal, la fuerza de atraccin de dos masas es
directamente proporcional al producto de estas e Ejemplo 16. Fuerza de la gravedad: Se suelta una
inversamente proporcional al cuadrado de la partcula de masa m desde la altura h sobre el suelo.
distancia. Cuando la partcula est a una altura y del suelo, su
mM velocidad es v.
F = G r Su energa potencial es U = mgy
r2
Donde m es la masa e un cuerpo, M la masa de la 1 2
tierra, r la distancia entre las masas, G es la constante Su energa cintica es K = mv
gravitatoria universal. 2
Si r = R (radio de la tierra), la masa m est sobre la La energa mecnica total es:
superficie de la tierra y 1 2
E = K +U = mv + mgy
mM 2
F = G r = mgr
R2 Para y = h , v = 0
La energa potencial es E = 0 + mgh = mgh

mM Para y = 0 , v = v 0
dU = F d r = G dr
r2 1 1
mM E = mv02 + 0 = mv02
U = G dr + C 2 2
r2 Para cualquier instante
U = G
mM
+C 1 2
E= mv + mgy = mgh
r 2
Para evaluar la constante C consideremos que el
De aqu v = 2 g (h y )
2
potencial U es cero para r infinito, de aqu C es igual
a cero, v = 2 g (h y )
Luego
El grfico de la variacin de energa potencial y
mM cintica es:
U ( r ) = G
r

Ejemplo 17. Una masa pequea m se suelta desde el


CONSERVACION DE LA ENERGA reposo de la parte ms alta de una superficie esfrica
Hasta esta parte tenemos dos formas de encontrar el de radio R, sin friccin. A qu ngulo con vertical
trabajo realizado sobre un objeto por una fuerza, la dejar el contacto con la esfera?
primera vlida para todo caso ya sea fuerza
conservativa o no conservativa
1 2 1 2
W12 = mv2 mv1 = K 2 K1 = K .
2 2
Y la segunda para el caso de fuerzas conservativas
W12 = U1 U 2 = U Solucin.
Cuando la masa est a una altura h su energa es igual
Luego podemos escribir a cuando est en el punto ms alto.
W12 = K 2 K1 = U1 U 2

9
TRABAJO Y ENERGA Hugo Medina Guzmn

1 2 1
E= mv0 + 0 = mv02
2 2
Para cualquier instante
1 2 1 2 1 2
1 E= mv + kx = kL
mgR + 0 = mgh + mv 2 2 2 2
2
Con h = R cos
2
De aqu: v =
m
k
(
L2 x 2 )
1
mgR = mgR cos + mv 2
2 v= (
k 2
L x2 )
v = gR(2 2 cos )
2 m
El grfico de la variacin de la energa potencial y
La segunda ecuacin de Newton cuando la masa esta cintica es:
en la posicin del ngulo :

v2
Con ac = :
R
Ejemplo 19. Calcular la velocidad necesaria para
v2 que una partcula pueda escapar de la atraccin de la
N mg cos = m
R tierra. La energa total E de una partcula de masa m
La masa deja la superficie esfrica cuando: que est a una distancia r del centro de la tierra y que
N =0 tiene una velocidad v es:
1 2
mv 2 mgR(2 2 cos ) E = K + U , donde K = mv y
mg cos = = 2
R R
mM
v 2
U = G
cos = = 2 2 cos r
Rg 1 2 mM
2 Luego: E = mv G = Constante
cos = 2 r
3 Si la partcula escapa de la atraccin de la tierra y se
= 48,2 sita a una distancia infinita de sta su potencial es
cero.
Ejemplo 18. Fuerza de un resorte: Se jala una masa mM
a sujeta a un resorte de constante k sobre una
r , U = G 0
r
superficie sin friccin, desde la posicin de equilibrio En esta regin con la velocidad menor posible
x = 0 hasta una distancia L y se suelta.
A una distancia x de la posicin de equilibrio la v = 0 Tenemos K = 0
velocidad de la masa es v. Luego: E = K + U = 0
1 2 Como E es constante E = 0
Su energa potencial es U = kx
2 La energa E de la partcula en la superficie de la
1 2 tierra con la velocidad ve para que pueda escapar:
Su energa cintica es K = mv
2
1 2 mM
Su energa mecnica total es: E= mve G =0
1 2 1 2 2 R
E = K +U = mv + kx
2 2 2GM
ve =
Para x = L , v = 0 R
1 1
E = 0 + kL2 = kL2 Como en la superficie de la tierra
2 2
mM GM
Para x = 0 , v = v 0 F = G 2
= mg g = 2
R R
Tenemos: v e = 2 gR

10
TRABAJO Y ENERGA Hugo Medina Guzmn

Siendo
m 6
g = 9,81 2
y R = 6,4 10 m
s
4 m
Obtenemos; ve = 1,12 10
s
Ejemplo 20. Se tiene un resorte de longitud L y Solucin.
constante k conectado a la base de un bloque de masa Velocidad de la partcula:
m, Se suelta el bloque desde la altura H. Cul ser la Tenemos que
distancia mas cercana al piso que alcanzar el bloque 1 2
antes de rebotar?
ETotal = K + U = mv + U ( x )
2
Solucin.
1 2
mv = E U ( x )
2
2
v= (E U ( x ) )
m

La energa cintica:
- Es igual a cero en x1 y x5.
- Tiene su valor mximo donde U(x) es mnimo, el
punto x2
En el instante inicial la energa es solamente la
potencia1 gravitatoria es U = mgH , la energa La partcula se mueve entre x1 y x5, fuera de estos
cintica es cero, tal que la energa total es valores la velocidad sera imaginaria.
E = mgH .
En el instante final: La energa potencial es la dU ( x )
correspondiente a la masa a una altura y, ms la del Como Fx = , la pendiente del grfico de
resorte comprimido una longitud (L y ) , es decir:
dt
U(x) en determinado punto corresponde a La fuerza
1 actuante, tal que la fuerza se hace cero donde la
U = U g + U r = mgy + k (L y )
2
pendiente es cero, como en x2, x3 y x4.
2
Como en ese instante ha cesado el movimiento, la La fuerza es positiva entre x1 y x2.entre x3 y x4. La
energa cintica es cero, fuerza es negativa entre x2 y x3, entre x4 y x5.
La energa total es: Los puntos en que U es mnimo, son posiciones de
1
E = mgy + k (L y ) equilibrio estable, como son x2 y x4.
2

2
Ejemplo 22. En la figura, un auto de juguete de
Por la conservacin de la energa masa m se libera del reposo en la pista circular. Si
se suelta a una altura 2R sobre el piso, cun arriba
1
mgH = mgy + k (L y )
2
sobre el piso estar cuando sale de la pista, desprecie
2 la friccin?

La solucin de esta ecuacin es:


mg
mg
y = L 1+
2k
(H L )
R k mg

Siendo el valor positivo de y la solucin significativa.

Ejemplo 21. El grfico de la figura muestra la Solucin.


funcin potencial y la energa total de un En la figura de arriba:
movimiento. Qu podemos decir acerca del h = R (1 + sen )
movimiento? Despreciando las prdidas por friccin la energa total
es constante, de tal manera que:
Siendo v la velocidad del auto a la altura h.

11
TRABAJO Y ENERGA Hugo Medina Guzmn

1 Solucin.
mg (2 R ) = mg (h ) + mv 2 a) Siendo v la velocidad de la masa en la parte
2 superior del rizo.
1 Por conservacin de la energa:
mg (2 R ) = mgR(1 + sen ) + mv 2
2 1
mg (H ) = mg (2 R ) + mv 2
1 2
gR = gRsen + v 2 (1)
2 1 2
gH = 2 gR + v (1)
Aplicando la segunda ley de Newton en la altura h: 2
Aplicando la segunda ley de Newton en ese punto:

mgsen N = mac
N = 0, condicin de cada.
mg N = mac
v2
ac = N = 0, condicin de cada.
R
Luego: v2
ac =
v2 R
mgsen = m Luego:
R
v2
v = gRsen (2)
2
mg = m
Reemplazando (2) en (1):
R
1 v 2 = gR (2)
gR = gR sen + gR sen Reemplazando (2) en (1):
2 1
3 2 gH = 2 gR + gR
gR = gRsen sen = 2
2 3 5
Finalmente: H= R H = 2,5 R
2
`2
h = R (1 + sen ) = R1 +
3 b) Sea v la velocidad en el punto A su altura es
5 h = R(1 + cos )
= R = 1,67 R
3
Ejemplo 23. Una masa pequea resbala sobre una
superficie inclinada pasando por un rizo de radio R.
a) Cul es la altura mnima H de la que debe soltarse
a fin de que el cuerpo no deje la superficie interior del
rizo al dar la vuelta? Por conservacin de la energa:
b) Con que velocidad llega la masa al punto A? 1
c) Cul es el valor del ngulo , con el que se
mg (H ) = mgR(h ) + mv 2

2
puede retirar el segmento AB de la circunferencia 1
de tal modo que la masa que sale de A alcance el
mg (2,5R ) = mgR(1 + cos ) + mv 2
2
punto B despus de viajar una cierta distancia en el
aire. v 2 = 2 g (2,5R ) 2 gR(1 + cos )
v 2 = 3gR 2 gR cos
v = gR(3 2 cos )

c) La masa sale del punto A, como un proyectil con



velocidad inicial v = vxi + v y j

12
TRABAJO Y ENERGA Hugo Medina Guzmn

Solucin.
F 220
a) k = = = 344 N/m
x 0,64
b) U 1 + K 1 = U 2 + K 2
1 2 1 k
kx + 0 = 0 + mv 2 v = x
En el tiempo t de su recorrido vertical debe alcanzar 2 2 m
al punto B. 344
Recorrido vertical: v= (0,64) = 76,6 m/s
1 2 0,024
y = vsen t gt
2 Ejemplo 24. Puenting. Un saltador que pesa 800 N
Cuando llega a B, y = 0: se ata con una cuerda elstica al tobillo y se salta de
1 2 2v una torre alta. La cuerda tiene una longitud si estirar
0 = vsen t gt t = sen
2 g de 30 m, y un extremo se une al punto donde el salto
comienza. La constante del resorte de la cuerda
Su recorrido horizontal es
elstica es 200 N/m. Cunto recorrer el saltador
x = vxt = v cos t antes de que la cuerda detenga su descenso?
2v
Para t = sen debe de estar en B, luego:
g
2v
2 Rsen = v cos sen
g
gR
v2 =
cos
Igualando esta expresin de la velocidad con la
encontrada anteriormente: Solucin.
Sea el punto ms bajo del salto h = 0. La energa
gR(3 2 cos ) =
gR
cintica inicial y la energa cintica en el punto ms
cos
bajo son ambas igual a cero.
3 1 Tal que por la conservacin de la energa:
cos 2 cos + = 0
2 2 1 2
Resolviendo: mgh = 0 + kx , donde x = h 30 .
2
1 Sustituyendo
cos = mg = 800 N y k = 200 N/m, y resolviendo:
1 2
En nuestro caso tomamos la solucin , con la que h 2 68h + 900 = 0
obtenemos = 60 h = 68 (68)2 4(900) = 50 m, o 18 m.
La solucin correcta es h = 50 m. La solucin h = 18
Ejemplo 24. Un arco del tiro al arco ejerce la fuerza
m corresponde al rebote que comprime la cuerda
kx de la ley de Hooke en una flecha cuando la cuerda
amortiguador auxiliar, pero una cuerda no se
se jala una distancia x. Se supone que un arquero
comprime como un resorte.
ejerce una fuerza de 220 N jalando a la flecha una
distancia de 64 cm.
Ejemplo 25. En la figura mostrada, el hombre y la
a) Cul es la constante del resorte del arco?
plataforma tienen una masa m, el hombre se eleva una
b) Cul es la velocidad de una flecha de masa 24 g
distancia h tirando la cuerda del lado derecho.
cuando deja el arco?
a) En cunto aumenta su energa potencial
gravitatoria?
b) Qu fuerza debe ejercer para elevarse?
c) Qu longitud de cuerda debe tirar para llegar a la
posicin superior?
d) Despreciando el rozamiento Qu trabajo habr
realizado?

13
TRABAJO Y ENERGA Hugo Medina Guzmn

W = Fd
F
La aceleracin de la caja es a =
m
Como la caja parte del reposo su velocidad en la
posicin final es:
2 Fd
v2 = 2ad =
m
El observador determina que el cambio de energa:
Solucin.
a) La energa potencial gravitatoria es 1 2 1 2
K = K 2 K1 = mv2 mv1
U ( y ) = mgy + C 2 2
Para la posicin inicial 2 Fd
Como v1 = 0 y v2 =
U1 = mgy1 + C m
Para la posicin final 1 2 Fd
U 2 = mgy2 + C K = m = Fd
2 m
El aumento de la energa potencial gravitatoria es:
El observador sobre la plataforma concluye que:
U = U 2 U1 = mg ( y2 y1 ) = mgh
1 2 Fd
b) La fuerza para elevar el sistema, siendo esta K = m = Fd
conservativa, 2 m
U W = K
F = = mg b) Observador situado en tierra:
y
Como la polea divide en dos, la fuerza Fh que debe
mg
ejercer el hombre es: Fh = .
2
c) Para llegar a la posicin superior la cuerda debe ser
tirada en una longitud dos veces h El observador en tierra ve que la caja se mueve bajo
d = 2h. la accin de la fuerza F, en este caso la caja se mueve
d) EL trabajo realizado por el hombre es: la distancia d ' = Vt+ d ,. Siendo t el tiempo que
demora el recorrido de la distancia d sobre la
mg
Wh = Fh d = (2h ) = mgh plataforma,
2 2d 2dm 2dm
Justamente igual al cambio de energa. t= = , luego d ' = V +d
a F F
Ejemplo 26. Observadores en movimiento El trabajo es:
relativo. Sobre una plataforma en movimiento 2dm
horizontal con una velocidad constante V. un hombre W ' = Fd ' = F V + d
empuja una caja de masa m con una fuerza F una F
distancia d partiendo del reposo. Demostrar la validez
de la conservacin de la energa desde los puntos de W ' = Fd + V 2 Fdm
vista de observadores en marcos inerciales diferentes. El observador ve que la caja tiene una velocidad
Solucin. inicial
Las leyes de Newton se cumplen slo en marcos de v'1 = V
referencia inerciales. Si se cumplen en uno en y una velocidad final
particular entonces se cumplen en todos los marcos
de referencia que se muevan a velocidad constante en 2 Fd
v'2 = V + v2 = V +
relacin a este mareo. m
a) Observador en la plataforma.
El observador en tierra determina que el cambio de
energa es:
1 1
K ' = K '2 K '1 = mv'22 mv'12
2 2
2
El observador en la plataforma ve que la caja, de 1 2 Fd 1
K ' = mV + mV 2
masa m, se mueve bajo la accin de la fuerza F. El 2 m 2
trabajo realizado para mover la distancia d es:

14
TRABAJO Y ENERGA Hugo Medina Guzmn

K ' = Fd + V 2 Fdm La energa inicial es solo la energa cintica de la


masa:
Aqu se cumple tambin la conservacin de la 1 2
Ei = mv0
energa: 2
W ' = K ' La energa final es solo la energa potencial del
resorte:
SISTEMAS NO CONSERVATIVOS. 1 2
Supongamos que tambin intervienen fuerzas no Ef = kx
conservativas, como la friccin. 2
El trabajo hecho por la friccin
x2
El trabajo total para mover la partcula de r1 a r2 es W f = F f dx , x1 = 0 , x 2 = L + x ,
x1
1 1
W12 = mv22 mv12 = K 2 K1 F f = N = mg
2 2
Luego:
L+ x
Este trabajo es tambin igual a la suma del trabajo W12 = ( mg )dx = mg (L + x )
realizado por las fuerzas conservativas y del trabajo 0

realizado por las fuerzas no conservativas, es decir: Como en un Sistema no Conservativo.


W12 NO CONSERVATIVAS = E2 E1
W12 = W12 CONSERVATIVAS + W12 NO CONSERVATIVAS
1 2 1 2
mg (L + x ) = kx mv0
Como: 2 2
W12 CONSERVATIVAS = U 1 U 2 Ecuacin de segundo grado cuya solucin es:

W12 = U 1 U 2 + W12 NO CONSERVATIVAS


mg 2m2 g 2
x=
k

k 2

m
k
(
2gL v02 )
De las expresiones de trabajo total tenemos:
K 2 K1 = U 1 U 2 + W12 NO CONSERVATIVAS
Ejemplo 28. Un cuerpo de masa 10 kilogramos cae
(K 2 U 2 ) (K1 U 1 ) = W12 NO CONSERVATIVAS desde una altura de 15 metros y alcanza el suelo en 2
segundos. Considerando constante la fuerza de
E 2 E1 = W12 NO CONSERVATIVAS resistencia del aire.
a) Cul era la magnitud de la fuerza de resistencia?
b) Cunta energa mecnica se ha perdido?
A diferencia que en un Sistema conservativo, no es
c) Qu velocidad tena el cuerpo inmediatamente
igual a cero.
antes de chocar Contra el suelo?
Esta ltima expresin nos permite calcular el trabajo
Solucin.
de fuerzas no conservativas, fuerzas que en general
a) Siendo el peso y la fuerza de resistencia del aire las
son complicadas y que en principio deberamos de
fuerzas que intervienen y siendo ambas constantes
calcular resolviendo integrales curvilneas.
tenemos que la aceleracin a del cuerpo es constante.
Ejemplo 27. A un bloque de masa m se le da un 1 2
Como h = at
empujn tal que adquiere la velocidad v 0 a lo largo 2
del eje x. Despus de resbalar distancia L golpea un 2h 2(15) m
resorte de constante k. Si el coeficiente de friccin
La aceleracin es a = 2
= 2 = 7,5 2
t 2 s
entre el bloque y la masa es . Cunto se Aplicando la segunda ley de Newton:
comprime el resorte? mg Fg = ma
Solucin.
Fg = m( g a ) = 10(9,8 7,5) = 23 N
b) La energa que se ha perdido es el trabajo realizado
por las fuerzas no conservativas.
W NO CONSERVATIVAS = Fg d = (23)(15) = 345 J
c) Como W NO CONSERVATIVAS = E 2 E1
Siendo
E1 = K1 + U1 = 0 + mgh = (10)(9,8)(15)
Sea x La longitud que se comprime el resorte. = 1470 J
La distancia recorrida por la masa es 1 2
(L + x). E2 = K 2 + U 21 = mv2 + 0 = 5v22
2

15
TRABAJO Y ENERGA Hugo Medina Guzmn

Tenemos: v2
1470 345 s=
5v22 1470 = 345 v22 = = 225 2 g (sen + cos )
5
m =
(40)2 = 87,5 m
Finalmente: v2 = 15 2(9,8)(sen30 +0,5 cos 30 )
s
Una manera directa de llegar al mismo resultado es
considerar que la aceleracin efectiva de salida es POTENCIA
m Tan importante como saber cual es el trabajo
a = 7,5 , la velocidad despus de 2 segundos es: realizado es conocer tambin la rapidez con la cual se
s2 realiza. Para proporcionar una medida cuantitativa de
m m este concepto que incluye tanto el trabajo como el
v2 = at = 7,5 2 = 15 tiempo necesario para realizarlo se tiene a la
s s Potencia.
La potencia mide la rapidez con la que el trabajo se
LA CONSERVACIN DE LA ENERGA Y LA est realizando.
FRICCIN
La ley de la conservacin de la energa se puede Si se realiza un trabajo W en un intervalo de tiempo
aplicar a los sistemas donde las fuerzas no (de t1 a t2) la Potencia media es:
conservativas como actan las fuerzas de la friccin. W12 W
Si un sistema trabaja contra la friccin, la energa Pm = =
mecnica del sistema disminuir. t2 t1 t
As si Wf es el trabajo hecho contra la friccin, Cuando t 2 t1 , t 0 , tendremos
entonces energa inicial - la energa perdida por la
La Potencia instantnea en el instante t.
friccin
W dW
E1 W f = E 2 P = lim =
t 0 t dt
1 2 1 Tambin como
U1 + mv1 W f = U 2 + mv 22
2 2
dW = F d r
Ejemplo 29. Cerca de Lewiston, Idaho, hay una Tenemos

carretera muy inclinada donde circulan camiones
cargados con madera. Han ocurrido varios accidentes
dW d r
P= = F
serios cuando los carros perdieron sus frenos yendo dt dt
para abajo de la colina a gran velocidad. Se han
construido rampas de contencin que se espera P = F v
puedan detener a los vehculos sin frenos. Suponga
que un carro que viaja a 40 m/s encuentra una rampa El anlisis dimensional
inclinada para arriba 30 sobre horizontal. La grava
floja en la rampa proporciona una fuerza friccional
[P] = [F ][L][T ]1 = [M ][L]2 [T ]1
para ayudar a detener al carro mientras sube la rampa.
La grava tiene un coeficiente eficaz de friccin de Su unidad en el sistema internacional es J/s llamado
0,50. Cun lejos a lo largo de la rampa el carro Watt Vatio cuyo smbolo es W.
viajara antes de detenerse? Un mltiplo muy usado es el kilowatt (kW)
Solucin. 1 kW = 103 W
Existe una unidad de energa o trabajo en trminos de
N = mg cos
la unidad de potencia el kilowatt-hora (kwh), es la
F f = N = mg cos energa convertida o consumida en una hora a una
U1 + K1 W f = U 2 + K 2 razn constante de 1 kW.
1 kWh (103W)(3600s) = 3,6 x l06 .J

Para tener una idea de cuanto es 1 Watt, imaginemos


que tenemos que levantar una masa de 50 kg. a una
altura de 1 metro, cada 5 minutos y realizar este
trabajo durante una jornada de 8 horas. Si levanta
cada 5 minutos, sern 12 veces por hora, siendo 8
1 2 horas por da, har un total de 12 x 8 = 96 veces al
0+ mv F f s = mgh + 0 h = s sen da.
2 El trabajo realizado es:
1 2 W = 96mgh
mv (mg cos )s = mgs sen
2 = 96(50Kg )(9,8m / s )(1m ) = 47040 J

16
TRABAJO Y ENERGA Hugo Medina Guzmn

Para determinar la potencia tenemos que dividirlo por Luego: P = (mg sen )( g sen t )
el nmero de segundos en un da.
47040J = mg 2 sen 2 t
P= = 1,63 W
8 3600 s 2(h y )
Como t=
Comparemos esta potencia con la potencia de un gsen 2
motor pequeo de 1 hp (horse power).
El hp es la unidad de potencia en el sistema ingls Obtenemos:
1 hp = 746 W 2(h y )
P = mg 2 sen 2
gsen 2
Ejemplo 30. Si un objeto que parte del reposo se
desliza por un piso liso inclinado un ngulo con = mgsen 2 g (h y )
respecto a la horizontal de altura h, hallar la potencia
P gastada por la gravedad en funcin de la posicin y Ejemplo 31. El flujo de agua de un ro es de 50 m3
del objeto con respecto a la parte inferior plano por segundo, se tiene un desnivel de 200 metros y se
inclinado. quiere aprovechar construyendo una hidroelctrica
Solucin. a) Si la energa del agua que cae se utilizase
totalmente Que potencia se podra obtener?
b) Si toda la energa procedente de la cada del ro se
convirtiese en energa elctrica y se vendiese a un sol
el kilowatt-hora Cunto dinero se cobrara en un
da?
Solucin.
a) El trabajo realizado por una masa m que cae desde
La potencia es:
una altura h es:
dW W = mgh
P= , siendo W = Fd
dt Como m = V,
Con Donde es la densidad del agua. V es el volumen.
1 2 1 W = Vgh
F = mg sen y d = at = g sen t 2
2 2
Tenemos La potencia que se obtiene al pie de la salida es
1 dW d
W = (mg sen ) g sen t 2 P= = Vgh
dt dt
2 De estas cantidades la que vara con el tiempo es V.
1
= mg sen t y
2 2 2
dV m3
2 = 50
dt s
dW d 1 2 Luego
mg sen t
2 2
P= =
dt dt 2 dV
P = gh
= mg sen
2 2
dt
Como ha recorrido la distancia s: Como

s=
(h y ) = 1 gsen t 2 = 1000
kg m
, g = 9,8 2 , h = 200m
3
sen 2 m s
Obtenemos: Obtenemos
2(h y ) kg m m3
P = 1000 3 9,8 2 (200m ) 50
t=
gsen 2 m s s
Luego = 9,8 x 107 W
2(h y )
P = mg 2 sen 2 b) Si tenemos una potencia P = 9,8 x 107 = 9,8 x 104
gsen 2 kW y consideramos que se consume las 24 horas del
= mgsen 2 g (h y )
da. La energa obtenida es igual a todo el trabajo
realizado.
dW = Pdt
Otra manera de obtener es considerar que:
W = P dt = P(t 2 t1 ) = Pt
t2
P = Fv t1
Donde
F = mg sen y v = at = gsen t W = (9,8 104 kW )(24h ) = 235,2 x 104 kW-h

17
TRABAJO Y ENERGA Hugo Medina Guzmn

si el precio de cada kW-h es 1 sol, cada da se MQUINAS


obtendrn 2,352 millones de soles. Una mquina simple es un dispositivo usado para
magnificar una fuerza o para cambiar una
Ejemplo 32. En la figura, un bloque de masa m desplazamiento pequeo en grande. Las mquinas
descansa sobre una faja que se mueve con velocidad comunes son la palanca, el plano inclinado, el gato
constante v. El coeficiente de friccin entre el bloque hidrulico, o una combinacin de engranajes.
y la faja es k. El trabajo se hace tpicamente en la mquina (el
Tomando como tiempo inicial t = 0, una fuerza trabajo W1 de entrada), y entonces la mquina
horizontal F aplicada al bloque le produce una alternadamente hace un cierto trabajo W2 de salida.
aceleracin constante a. El estado de la energa de la mquina no cambia
a) Determinar la fuerza F y la potencia disipada en apreciable durante este proceso, as que si la friccin
friccin como funcin del tiempo. es insignificante, W1 = W2, basado en la idea de la
b) Si la fuerza F es ejercida por un hombre que se conservacin de energa. Muy a menudo las fuerzas
encuentra sobre la faja. Determinar la potencia que de entrada y de salida son constantes, en las cuales el
este libera en funcin del tiempo. caso W1 = W2, lo que lleva a:
e) Si la fuerza F es ejercida por un hombre que d1
camina sobre el piso al costado de la faja. Determinar F1 d1 = F2 d 2 F2 = F1
la potencia que este libera en funcin del tiempo. d2
Aqu F1 acta sobre una distancia d1 y F2 acta sobre
una distancia d2. La ventaja mecnica de la mquina
se define como
F2
VM =
F1
Solucin.
a) Aplicando la segunda ley de Newton a la masa m Ejemplo 33. La palanca de barra es un dispositivo
en la figura usado para levantar objetos pesados (por ejemplo, un
piano o una pieza grande de maquinaria). Consiste en
una barra larga que se apoya en un fulcro una
distancia corta del extremo de levantar de la barra.
Suponga que el fulcro de una barra de la palanca est
a 3 centmetros de la carga, y el punto donde usted
empuja hacia abajo en el otro extremo est a 1,50 m
F F f = ma del fulcro.
Como F f = k N = k mg , obtenemos: Qu fuerza mnima tendra que ejercer para levantar
una carga de 2000 N?
F = ma + k mg Si mueve el extremo de la barra 4 centmetros hacia
y la potencia disipada en friccin es abajo, cunto levantar la carga?
P = F f v0 = ( k mg )v0 , siendo v0 = at Solucin.

P = k mgat

b) La fuerza que hace el hombre sobre la faja es


F = ma + k mg
Su velocidad en funcin del tiempo es
v' = v + v0 = v + at Si la barra rota con un ngulo pequeo , entonces
y la potencia que debe dar el hombre es d1 = L1 y d 2 = L2
P = Fv' = (ma + k mg )at F1 L1 = F2 L2
L
c) La tuerza que har el hombre sobre el piso es igual F1 = 2 F2
al caso anterior: L1
F ' = ma + k mg F1 =
(0,03) (2000)
La velocidad del hombre en funcin del tiempo en 1,50
este caso es: = 40 N
v' = v + at
Luego la potencia que debe dar el hombre es: Para tringulos semejantes
P' = F ' v' = (ma + k mg )(v + at ) d1 L1 L
= d 2 = 2 d1
d 2 L2 L1

18
TRABAJO Y ENERGA Hugo Medina Guzmn

d2 =
(0,03) (0,04) As el trabajo de la salida es W2 = F(L).
Despreciando la friccin.
L(1,50)
W1 = W2 o F1(18 L) = F2(L)
= 0,008 m = 8 mm.
Observe que una fuerza pequea de entrada da lugar a La ventaja mecnica del polipasto es VM = 18.
una fuerza grande de salida, pero el precio que se
paga es que un desplazamiento grande de la entrada Ejemplo 35. Un trailer est equipado de un sistema
produce solamente un desplazamiento pequeo de para sacar barcos del agua. Consiste en una manija
salida. larga de 30 centmetros unido al eje de un engranaje
pequeo con 12 dientes. Este engranaje pequeo
Ejemplo 34. Se bosqueja aqu un polipasto endienta con un engranaje ms grande con 36 dientes.
diferenciado de la clase usada para levantar un motor Se une a este engranaje grande un tambor del radio 2
de auto. Las poleas tienen dientes que engranan con centmetros en el cual se enrolla la lnea atada al
una cadena continua. Las poleas estn soldadas barco (la lnea es una cuerda.)
juntas, hay 18 dientes en la polea externa y 16 dientes Qu tensin se puede aplicar a la lnea cuando la
en la polea interna. As cuando la polea hace una manivela se empuja con una fuerza de 80 N?
revolucin, 18 acoplamientos de la cadena se So1ucin.
levantan y 16 acoplamientos bajan, dando por Considere que pasa cuando la manivela hace una
resultado la elevacin de la carga. Cul es la ventaja revolucin. La mano mueve una distancia d1 = 2R1.
mecnica de esta mquina? El engranaje grande mueve 12/36 = 1/3 revoluciones.
La lnea es jalada una distancia d2 = 2R2/3.

F1 d1 = F2 d 2
d 2R1 R
F2 = 1 F1 = F1 = 3 1 F1
d2 2R2 / 3 R2
Solucin. 30
Considere qu pasa cuando la polea superior hace una F2 = 3 (80 ) = 3600 N.
revolucin, es decir, cuando el trabajador jala 18 2
eslabones de la cadena hacia l con fuerza F1. La ventaja mecnica:
Sea L = longitud de un eslabn. 3600
El trabajo de la entrada es W1 = F1(18 L). VM = = 45
80
El lazo de la cadena que va bajo de la carga es
La ventaja mecnica del torno (despreciando la
acortado as por 18 eslabones y alargado por 16
friccin) es 45.
eslabones, con un acortamiento neto de 18L - 16L =
2L que acorta al lazo 2L y levanta la carga L (intente
esto con un pedazo de cuerda para convencerse de
esta caracterstica).

PREGUNTAS Y PROBLEMAS

1 Defina primero en palabras y luego en una


expresin matemtica.
a) El trabajo realizado por una fuerza cualquiera.
b) La energa cintica de una partcula.

2 Una partcula P en el plano xy est sometida a la



accin de la fuerza F = y i x j . Calcular el
2 2
Respuesta. W=
ab
(a + b )
trabajo efectuado por la fuerza para desplazar P sin 3
friccin desde B (0,.b) a A (a, 0). 3. Un depsito cilndrico de altura H tiene una masa
m de agua que lo llena hasta la mitad, que ha de
bombearse en su totalidad por encima del borde del
mimo. Cunto trabajo ha de realizar la bomba?

19
TRABAJO Y ENERGA Hugo Medina Guzmn

3 cos = 1 x 2 . El
fuerza tambin vara tal que
Respuesta. W = mgH
4 objeto se mueve entre 2 < x < 2 .
Cul es el trabajo realizado cuando el objeto se
4. Qu fuerza horizontal, constante debe aplicarse a mueve de x = 0 a x = a?
un carro de masa 500 kg que viaja en una carretera
horizontal a 36 km/h para que se detenga en 30
a2 a4
Respuesta. Aa + B B
metros? Quin proporciona la fuerza? 2 4
Respuesta. 2500 N, proporcionada por la carretera.
9. Un bloque que se mueve a lo largo del eje x
5. Un resorte est unido en A a un plano vertical fijo comienza del reposo en x = A y se mueve a x = B
y a un bloque B que resbala sobre una varilla lisa luego vuelve a x = A donde queda en reposo
horizontal Ox. La longitud del resorte no estirado es nuevamente. Si una de las fuerzas actuante sobre el
45 cm y la constante del resorte es k = I000 N/m. bloque es opuesta en direccin y proporcional a la
Cul es el trabajo realizado por el resorte sobre B
cuando se mueve 60 cm desde O por efecto de la magnitud de la velocidad, tal que Fv = b v con b
fuerza F? Constante. Demostrar que el trabajo realizado por
esta fuerza no es cero para una trayectoria cerrada.


10. La fuerza F = x yi + xy j acta sobre la
2 2

partcula .P (x,y) que se mueve en el plano xy.


a) Demostrar que F no es una fuerza conservativa.
b) Determinar el trabajo de F cuando se mueve de A
a C, a lo largo de los caminos ABC, ADC y AC.
Respuesta: 99,38 J

6. Un resorte de masa despreciable y constante k


cuelga del cielorraso de un ascensor y lleva
suspendido una masa in. Cuando el ascensor se
mueve hacia arriba durante t segundos con una
1
aceleracin uniforme a= g . la reaccin inercial
2 Fx Fy a4
hace que el resorte se alargue. Respuesta. a) Si , b) WABC = ,
a) Cunto trabajo realiza el ascensor sobre el sistema y x 3
resorte-masa? a4 a4
b) Cunto trabajo realiza sobre el resorte? WADC = . WAC =
3 2
1 1 m2 g 2
Respuesta. a) mg 2 t 2 b)
4 8 k 11. El tubo de la figura se halla en un plano
horizontal, su resorte comprimido inicialmente 10
7. En la figura se mueve el cuerpo A a lo largo de un cm.. y al dispararse una bolita entra en una canaleta
plano horizontal liso por medio de la fuerza constante circular de radio R, la friccin es constante igual a 1
F = 5 N aplicada al extremo de una cuerda unida a A Newton. Cuntas vueltas dar la bolita antes de
y que pasa por una pequea polea sin rozamiento en detenerse?
B. Calcular el trabajo realizado sobre A por la cuerda R= 50 cm k = 62 N/m
mientras A se desplaza 3 m,

Respuesta. W = 120 J
Respuesta. Una vuelta.
8. Una fuerza cuya magnitud vara con x de acuerdo a
F = A + Bx acta sobre objeto que puede moverse 12. Se aplica una fuerza de 1 N a una partcula de 50
solamente en el eje x. El ngulo con el que acta la g que est inicialmente en reposo sobre una
superficie.

20
TRABAJO Y ENERGA Hugo Medina Guzmn

a) Cunto trabajo realiza sobre la partcula en l0 s si zy


la superficie es lisa y la fuerza es horizontal? U (r ) = 3 x 2 y + y2
b) El mismo caso de a) pero la fuerza hace un ngulo x
de 60 con la horizontal. Respuesta.
c) El caso b) pero con rozamiento entre la partcula y
z z y
la superficie 0,25 y Cunto trabajo se consume en F = y 2 6 x i + 2 y 3x 2 j k
vencer el rozamiento? x x x
Respuesta. a) W = 1000 J , b) W = 2505 J ,
17. Una partcula de masa 4y penetra en una regin
c) W = 143 J , W = 46 J en la cual su energa potencial es la indicada en la
figura y pasa valores grandes de x, a los cuales su
13. Encontrar la funcin energa potencial de un energa potencial es cero, tiene una energa cintica
resorte si el origen se coloca en la pared y la longitud de 16 x 10-7 J .
del resorte sin estirar es L. a) Cul es su energa cintica en los puntos A, B y
C?
b) Estando en el punto A, la partcula pierde
bruscamente la mitad de su energa total. (la grfica
de la energa potencial no se altera). Describe
cualitativamente el movimiento subsiguiente, dando
el dominio de valores de x en el cual puede moverse
1 2 la partcula.
Respuesta. U(x) = kx kLx + C
2
1 2
Si C= kL
2
1 1
2
( 2
)
U ( x ) = k x 2 2 Lx + L2 = k ( x L )
2

14. Una partcula que se mueve a lo largo del eje x


est sometida a la accin de una fuerza en un sistema
conservativo a la que le corresponde la siguiente Respuesta.E A = 8 10 -7 J , EB = 12 10-7 J ,
funcin energa potencial. EC = 6 10-7 J
U ( x ) = a + bx 2 cx 4
Determinar los coeficientes a. b y c, si se sabe que el 18. Un bloque de masa m es lanzado hacia arriba en
potencial se anula en el origen, que x = 2 m en una un plano inclinado con una velocidad de magnitud
posicin de equilibrio y que una partcula de 5 kg con v0 . El ngulo del plano es y el coeficiente de
una velocidad en el origen de 2 m/s queda en reposo
en x = l m. friccin del bloque y el plano es . Si el bloque
Respuesta. a = 0, b = 80/7 J/m2 , c = 10/7 J/m4 viaja una distancia L hasta detenerse y comienza a
bajar volviendo a su posicin original. Calcular,
15. La energa potencial entre dos molculas vecinas a) El trabajo realizado por la fuerza normal durante el
viene dada por: movimiento.
A B b) El trabajo realizado por la fuerza de friccin
U (r ) = + durante el movimiento.
r 6 r12 c) El trabajo realizado por la fuerza de gravedad
siendo r la separacin entre las molculas. durante el movimiento.
a) Cul es la fuerza entre ellas en funcin de r?
d) Encontrar L en funcin de v0 , y .
b) Cul es la posicin de equilibrio de las dos
molculas? e) Cul es la velocidad del bloque cuando vuelve al
c) Qu energa seria necesaria para alejarlas de su punto inicial?
posicin de equilibrio indefinidamente? Respuesta. a) 0, b) 2 mgL cos , c) 0,

Respuesta. a) F(r ) = 6
A B
+ 12 13 , b) v02
7 d) L = ,
r r 2 g ( cos + sen )
1
2B 6 A2 e) v = v02 4 gL cos
r = , c) E =
A 4B
19. Se dispara un proyectil con una velocidad inicial
16. Hallar la fuerza conservativa que da origen a la
de magnitud v0 y formando un ngulo con la
funcin energa potencial.

21
TRABAJO Y ENERGA Hugo Medina Guzmn

horizontal. Usando la conservacin de la energa


encontrar.
a) La altura mxima alcanzada.
b) La magnitud de la velocidad cuando el proyectil
est a la mitad de su mxima altura.
v02sen 2
Respuesta. a) h = ,
2g Respuesta. a) 0,5 mg , b) 0.3 mg , c) 2,5 mg d) 1,1
mg , e) 5,4 mg
b) v = v0
(1 + cos )
2

24. Un bloque pequeo de masa m resbala partiendo


2 de la parte superior de una esfera sin friccin de radio
R. Cul es el ngulo en el que el bloque pierde
20. Una fuerza F = 8t (t en segundos, F en Newton), contacto con la esfera.
acta la partcula P de masa m = 4kg durante un
tiempo t = 6 s.
S parte del reposo a partir del origen.
a) Calcular el trabajo efectuado.
b) Calcular la energa cintica al instante t.
Respuesta. a) W = 2592 J, b) K = 2t4 J.

21. Un resorte de longitud l y constante k se sujeta a


un bloque de masa m y al piso. Si el bloque se levanta
a una altura 3 l y soltado desde el reposo.
2
a) Cul ser la velocidad del bloque cuando est a Respuesta. a) cos =
una altura 2 l ? 3
b) Cul ser la mxima compresin del resorte?
25. 1n saco se empuja suavemente por el borde de
k 2 una pared en A y oscila en un plano vertical colgado
Respuesta. a) v = 3 l + 2 gl ,
m del extremo de una cuerda de 4m que puede soportar
b) una tensin mxima igual a dos veces el peso del
saco.
y=
kl mg kl mg
+ +
2
3kl 2 + 6mgl ( ) a) Determinar la altura a la que se rompe la cuerda.
b) A qu distancia de la pared vertical caer al .suelo
k k k
el saco?
22. Dos placas cuyas masas son m1 y m2,
respectivamente, estn conectadas por un resorte.
Qu fuerza deber aplicarse a la placa superior para
elevar la placa inferior despus que se retira la
presin? No tomar en cuenta la masa del resorte.

Respuesta. a) y = 1,33 m
Respuesta. a) F > (m1 + m 2 )g
26. Una bola pequea de masa m = l g desliza hacia el
23. Una bolita de masa m desliza a partir del reposo fondo de un valle movindose sin rozamiento como
hacia abajo por un carril doblado como se muestra en se indica en la figura. Partiendo del reposo, la bola
la figura, el rozamiento es despreciable, hallar: cae desde una altura h = 2m y abandona el fondo del
a) La reaccin normal del carril en A. valle formando un ngulo con la horizontal. En el
b) La energa cintica de la bolita en B. punto ms elevado de su trayectoria la bola choca con
c) La reaccin normal del carril en 8. un resorte montado sobre una pared y lo comprime 2
d) La energa cintica de la bolita en C. cm. La constante del resorte es k = 49 N/m.
e) La reaccin normal del carril en C. a) A qu altura y est el resorte? b) Cual es el
ngulo ?

22
TRABAJO Y ENERGA Hugo Medina Guzmn

a) Considerando que la fuerza de rozamiento es


independiente de la velocidad, calcule su valor
medio.
b) Qu potencia debe consumirse para mantener el
automvil en movimiento con una velocidad de 48
km/h?
Respuesta. a) Ff = 110 N b), P = 2 hp

Respuesta. a) y =1 m, b) = 45 33. Un automvil de 1500 kg se desplaza 200 m


mientras es acelerado uniformemente desde 50 hasta
73 km/h. Durante todo el movimiento el automvil se
27. Una bola de acero de masa 1 kg est unida a un desplaza sobre una carretera horizontal, y la
extremo de un alambre de 1m de largo y gira resistencia al movimiento es igual al 2 por ciento del
alrededor del otro extremo con una velocidad angular peso del automvil. Determinar:
de 120 rpm. Cul es la energa cintica de la bola? a) La mxima potencia requerida.
Respuesta. 78,88 J b) La potencia requerida para mantener la velocidad
constante de 75 km/h.
28. La faja transportadora de la figura se mueve con Respuesta. a) 25 kW , b) 6,13 kW
una velocidad constante v 0 y descarga los paquetes
34. Un peso D y el contrapeso C tienen cada uno una
sobre la rampa AB. El coeficiente de rozamiento masa de 350 kg. Determinar la potencia requerida
entre los paquetes y la rampa es 0,30. Sabiendo que cuando el peso:
los paquetes deben alcanzar el punto B con una a) Se mueve hacia arriba con velocidad constante de
velocidad de 4 m/s, determinar la velocidad v 0 4m/s.
requerida en la faja transportadora. b) Tiene una velocidad instantnea de 4m/s hacia
arriba y una aceleracin hacia arriba de 0,9 rn/s2.

Respuesta. 3,02 m/s

29. Una locomotora ejerce un tiro constante en la


barra de traccin de 160000 N mientras aumenta la
velocidad de 48 a 72 km/h. Cul es la potencia que
desarrolla la locomotora:
a) al comienzo del periodo?
b) al final del periodo?
c) Cules la potencia .media durante el periodo?
Respuesta. a) 2859 hp , b) 4290 hp c) 3574 hp
Respuesta. 6,86 kW , 8.44 kW
30. Una gra industrial puede levantar su mxima
permitida de 25 toneladas a la velocidad de 20mm/s. 35. Un bloque de 0,50 kilogramos es sujetado contra
Sabiendo que la gra es movida por un motor de 10 el resorte por una fuerza externa horizontal de 36 N.
kW. Determinar su rendimiento. Se quita la fuerza externa, y el bloque se proyecta con
Respuesta. 49% una velocidad v1 = 1,2 m/s a partir de la separacin
del resorte. El bloque desciende una rampa y tiene
31. Cul es la velocidad mxima la que un motor una velocidad v2 = 1,8 m/s en la base. La pista es sin
capaz de suministrar 10 kW puede elevar un ascensor friccin entre los puntos A y B. El bloque ingresa a
de masa 500kg, sin tomar en cuenta las fuerzas de una seccin rugosa en B, extendiendo hasta E. El
rozamiento? coeficiente de friccin cintica es 0,30. La velocidad
Respuesta. v = 2,0 m/s del bloque es v3 = 1,4 m/s en C. El bloque se mueve
hasta C donde se detiene.
32. Si a un automvil de masa 1000 kg que se mueve
sobre una carretera horizontal con una velocidad de
48 km/h se le apaga al motor, este recorre an 0,8 km
antes de detenerse. carga

23
TRABAJO Y ENERGA Hugo Medina Guzmn

38. Un cajn de 100 kilogramos est en una


superficie spera inclinada 30. Una fuerza externa
constante P de 800 N se aplica horizontalmente al
cajn. La fuerza empuja el cajn una distancia de 3,0
m arriba de la pendiente, en un intervalo del tiempo
de 2,0 s, y la velocidad cambia de v1 = 0,8 m/s a
v2 = 2,2 m/s.
a) La constante del resorte es:
b) La compresin inicial del resorte en cm es:
c) La altura h de la rampa en cm es:
d) El trabajo realizado por la friccin entre los puntos
B y C es:
e) La distancia s que el bloque viaja entre los puntos
B y D es: a) El trabajo realizado por el peso es:
Respuesta. b) El trabajo realizado por la fuerza de friccin es:
a) 1800 N/m, b) 2,0, c) 9, d) -0.32 J e) 0,55 m c) El trabajo realizado por la fuerza normal es:
d) La potencia media producida por la fuerza externa
36. Una barra sin masa de 1,5 m se fija libremente a P durante los 2,0 segundos es:
un pivote sin friccin en O. Una bola de 3,0 Respuesta.
kilogramos se une al otro extremo de la barra. La a) -1500 J, b) - 400 J c) Cero , d) 1050 W
bola se sostiene en A, donde la barra hace un ngulo
30 sobre el horizontal, y se lanza. El montaje de la 39. Una muchacha lanza una piedra de un puente.
bola-barra puede girar libremente en un crculo Considere las maneras siguientes que ella puede
vertical entre A y B lanzar la piedra. La velocidad de la piedra con la que
lanza es igual en cada caso.
Caso A: Lanzada derecho para arriba.
Caso B: Lanzada derecho para abajo.
Caso C: Lanzada con ngulo de 45 sobre horizontal.
Caso D: lanzada horizontalmente.
En qu caso la velocidad de la piedra ser mayor
cuando llega al agua?
Respuesta. la rapidez es la misma en todos los
casos.

40. Para hacer el trabajo sobre un objeto,


A) es necesario que haya friccin.
a) La bola pasa a travs de C, donde la barra forma un B) es necesario que no haya friccin.
ngulo de 30 debajo de la horizontal. La rapidez de C) el objeto debe moverse.
la bola cuando pasa por C es: D) la fuerza que hace el trabajo debe estar dirigida
b) la tensin en la barra cuando la bola pasa por el perpendicularmente al movimiento del objeto.
punto ms bajo D es: E) la fuerza aplicada debe ser mayor que la fuerza de
Respuesta. a) 5,4 m/ s, b) 120 N la reaccin del objeto.
Respuesta. C) el objeto debe moverse.
37. Una fuerza externa constante P =120 N se aplica
a una caja de 20 kilogramos, que est en una 41. Un bloque de 8,0 kilogramos se lanza del reposo,
superficie horizontal spera. La fuerza empuja la caja vl = 0 m/s, en una pendiente rugosa. El bloque se
una distancia de 8,0 m, en un intervalo del tiempo de mueve una distancia de 1,6 m abajo de la pendiente,
4,0 s, y la velocidad cambia de v1 = 0,5 m/s a v2 = 3,5 en un tiempo de 0,80 s, y adquiere una velocidad de
m/s. v2 = 4,0 m/s.

a) El trabajo realizado por la fuerza externa es:


b) El trabajo realizado por la friccin es: a)) El trabajo realizado por el peso es:
c) La razn de cambio promedio de la energa b) La razn promedio a la cual la fuerza de friccin
cintica de la caja, en los 4,0 segundos es: realiza trabajo en el intervalo de tiempo de 0,80 s es:
Respuesta. a) 830 J, b) -700 J, c) 30W

24
TRABAJO Y ENERGA Hugo Medina Guzmn

c) La razn promedio a la cual la fuerza normal


realiza trabajo en el intervalo de tiempo de 0,80 s es:
d) La razn promedio a la cual el bloque gana energa
cintica durante el intervalo de tiempo de 0,80 s es:
Respuesta.
a)) + 80 J, b) - 20 W, c) Cero, d) 80 W

42. Una persona de 60 kilogramo cae desde el


reposo uno distancia 1,20 m sobre una plataforma de
masa insignificante apoyada sobre un resorte duro.
La plataforma baja 6 cm antes de que persona vuelva a) El resorte 2 estira 0,06 m. La constante de fuerza
al reposo. Cul es la constante del resorte? del resorte 2 es:
Respuesta. 4,12 x 105 N/m b) La constante de fuerza del resorte 1 es igual a 30
N/ m. La longitud sin estirar del resorte 1 es:
43. Un objeto est sujeto a una fuerza restauradora F Respuesta. a) 20 N/m, b) 0,53 m
= 6x3, donde x es el desplazamiento del objeto desde
su posicin de equilibrio. Qu trabajo debe 46. Una barra ligera de 0,80 m se fija libremente a un
realizarse para mover al objeto desde x = 0 x = 0,15 eje vertical en A. Un disco de 2,0 kilogramos se une a
m? la barra en B. Un resorte se une a la masa en B y a la
Respuesta. 7,59 x 10-4 J manga en el eje en C. A La manga es sin friccin,
permitiendo que se baje y suba libremente, de modo
44. Dos resortes idnticos tienen longitudes sin que el resorte sea siempre horizontal cuando est
estirar de 0,25 m y las constantes de la fuerza de 200 estirado. La longitud del resorte sin estirar es 0,45 m
N/m. Los resortes se unen a un bloque pequeo y se y la constante es 210 N/m.
estiran a una longitud de 0,30 m como en la figura A.
Una fuerza externa P tira del bloque 0,02 m a la
derecha y lo sostiene all. (Vase La Figura B)

a) El eje est girando y el resorte estirado tiene una


longitud de 0,48 m. La aceleracin radial del disco es:
a) El trabajo requerido para ensamblar los resortes y b) El eje est girando y la varilla forma un ngulo de
el bloque (figura A) es : 40 con el eje. El resorte est estirado y horizontal.
b) La fuerza externa P, que mantiene al bloque en su La aceleracin radial del disco es:
lugar (figura B) es: c) El eje est girando y el resorte tiene una longitud
c) El trabajo realizado por la fuerza externa P en jalar de 0,45 m. La aceleracin radial del disco es:
el bloque 0,02 m es: Respuesta.
Respuesta. a) 10,5 m/s2 b) 15,0 m/s2, c) 6,7 m/ s2
a) 0,50 J, b) E) 8 N, c) 0,08 W
47. Cierto coche que viaja 20 resbalones del mph a
45. El bloque A (0,40 kg) y el bloque B (0,30 kg) una parada en 20 metros del punto donde los frenos
estn sobre una mesa sin friccin. El resorte 1 conecta fueron aplicados. En qu distancia el coche parara
al bloque A a una varilla sin fricin O y el resorte 2 aproximadamente la tena que va 40 mph?
conecta el bloque Ay el bloque B. Los bloques estn Respuesta. 80 metros
en movimiento circular uniforme alrededor de o, y los
resortes tienen longitudes de 0,60 m y 0,40 m, como 48. Un motor de la arena en una mina levanta 2.000
se muestra. La velocidad lineal del bloque B es 2.0 kilogramos de la arena por minuto una distancia
m/s. vertical de 12 metros. La arena est inicialmente en
el resto y se descarga en la tapa del motor de la arena
con la velocidad 5 m/s en un canal inclinado de
cargamento. En qu tarifa mnima se debe la
energa proveer a esta mquina?
Respuesta. 4,34 kW

49. La constante de un resorte es 500 N/m y su


longitud sin estirar es 0,60 m. Un bloque de 4,0
kilogramos se suspende del resorte. Una fuerza

25
TRABAJO Y ENERGA Hugo Medina Guzmn

externa tira hacia abajo lentamente el bloque, hasta


que el resorte se ha estirado a una longitud de 0,72 m.
se quita y el bloque sube.
a) La fuerza externa sobre el bloque es:
b) Cuando el resorte se ha contrado una longitud de
0.60 m, la velocidad del bloque hacia arriba es: :
c) Cuando el resorte se ha contrado una longitud de
0.66 m, la aceleracin del bloque incluyendo su
direccin es:
Respuesta. a) 20 N, b) 0,4 m/ s, c) 2 m/s2, haca a) La compresin del resorte en la figura b es:
abajo b) La fuerza externa P en la figura c es:
c) La energa potencial elstica del resorte en la
50. la constante de un resorte es 200 N/m y su figura c es:
longitud sin estirar es 10 centmetros. El resorte se d) La aceleracin inicial del disco cuando la fuerza
pone dentro de un tubo liso de 10 centmetros de alto externa es removida es:
(la figura a). Un disco de 0,40 kilogramos se coloca e) La velocidad v del disco cuando emerge del tubo
sobre el resorte (figura b). Una fuerza externa P en la figura d es:
empuja el disco hacia abajo, hasta que el resorte tiene Respuesta. a) 2,0 , b) 8N, c) 0,36 J, d) 20 m/s2,
4 centmetros de largo (la figura c). Se quita la fuerza e) 0,80m/s
externa, el disco se proyecta hacia arriba y emerge del
tubo (figura d).

26
Sistema de partculas Hugo Medina Guzmn

CAPTULO 6. SISTEMA DE PARTCULAS

La seleccin del contorno de un sistema es similar a


INTRODUCCIN seleccionar un sistema de coordenadas.

Hasta ahora hemos estado estudiando el movimiento SEGUNDA LEY DE NEWTON APLICADA A
de los objetos cualquiera que sea sin considerar su UN SISTEMA DE PARTICULAS
estructura. Ahora demostraremos que lo estuvimos
haciendo bien considerando al objeto sin tomar en La figura siguiente muestra un sistema de n partculas
cuenta las fuerzas que actan sobre sus partes. de masas m1, m2, ..mn, con posiciones especificadas
Introduciremos el concepto de centro de masa de un
sistema de partculas, tambin se introducir el por r 1 , r 2 , . r n ,, respectivamente.
concepto de cantidad de movimiento y se demostrar
que este se conserva cuando el sistema se encuentra
aislado de los alrededores,

SISTEMA DE PARTICULAS

La segunda ley de Newton para la partcula mi es:



F i = m a i = F iexter + F i int

Donde:


La figura muestra un sistema de partculas compuesto F i int = suma de las fuerzas internas sobre mi
de tres masas. En el sistema existen dos tipos de
fuerzas,

F i ext = suma de las fuerzas externas sobre mi
a) Las fuerzas externas como la atraccin
gravitacional de la tierra por ejemplo.
La suma de las fuerzas internas sobre la masa mi es:
b) Las fuerzas internas que las partculas ejercen unas
n
sobre otras (estas fuerzas pueden ser gravitacionales,
F 1int = F 12 + F 13 + ............ F 12 = Fij
e1ctricas, etc.)
( j i )

En general para la partcula i es:

n
F i int = Fij
( j i )

La fuerza total para el sistema es:

i =n i =n i =n n n
Fi = m a i = F i ext + Fij
i =1 i =1 i =1 i =1 ( j i )

En la figura hemos cambiado el contorno del sistema, Por la tercera ley de Newton cada una de las fuerzas
excluyendo la masa m3. Como Una Consecuencia de
esto las fuerzas internas Sobre m1 y m2 debido a m3 ya

no son internas, se han sumado a las fuerzas externas
Fij tiene un F ji igual, pero de sentido contrario
previas, produciendo una nueva fuerza resultante.

1
Sistema de partculas Hugo Medina Guzmn


1 n
1
Fij = F ji xCM = lim
mi 0 M
x m
i =1
i i =
M
xdm

n n
De modo que ( Fij = 0
i = 0 j i )=1
De igual forma se obtiene:

1 n 1
Consecuentemente solo queda y CM = lim
mi 0 M
i =1
y i mi =
M
ydm ,

1 n 1
n
mi ai = Fi ext o
n
d2 n
ii
m

r =
n
F

i ext
z CM = lim
mi 0 M

i =1
z i mi =
M
zdm y
i =1 i =1 dt 2 i =1 i =1
1
M
rCM = r dm
CENTRO DE MASA

Frecuentemente es muy prctico reemplazar un MOVIMIENTO DEL CENTRO DE MASA.


sistema de muchas partculas con una partcula
simple equivalente de masa igual. La pregunta es d2 n n

2 i i
donde colocar esta partcula simple con respecto al Si en la ecuacin: m r = Fiext
origen de x e y. dt i = 0 i =1

Definamos el vector posicin del centro de masa por n


la ecuacin: Sustituimos m
i =1
i ri = M rCM
n

m i ri Obtendremos la ecuacin del movimiento del centro


rCM = i =1 de masa
n

m i
d2 n n
i =1

dt 2
M rCM =
i =1
Fiext M a CM =
i =1
Fiext

n
Llamando a m
i =1
i = M (masa total de las n
El punto indicado por rCM , vector posicin del
partculas).
centro de masa, se mueve se mueve como si en el
estuviera concentrada toda la masa y las fuerzas
n

m i ri externas del sistema.

rCM = i =1
Ejemplo 1. Centro de masa de tres masas
M
puntuales.

Como rCM = xCM i + y CM j + z CM k

1 n
Tenemos que: xCM =
M
m x ,
i =1
i i

1 n
1 n
yCM =
M
mi yi , zCM =
i =1 M
m z
i =1
i i

El centro de masa esta dado por:


Si hacemos que el nmero de elementos n, se
aproximen al infinito, la sumatoria se reemplaza por
1 n
una integral y m por el elemento diferencial dm. xCM =
M
m x
i =1
i i

Luego.
m(1) + 2m(1) + 3m(2)
=
m + 2m + 3m
2
Sistema de partculas Hugo Medina Guzmn

9m 3 Sustituyendo:
= = 2 a b 2
6m 2 x (a x )dx = 2 x(a x )dx
a
xCM =
ab 0 a a 0
a
1 2 2 x2 x3
( )
n

mi yi
a
yCM = = 2
2
ax x dx = 2 a +
M i =1 a 0 a 2 3 0

m(1) + 2m(3) + 3m(2 ) a3 a3 a


= 2
m + 2m + 3m = + =
a2 2 3 3
13m 13
= = Realizando clculos similares encontramos:
6m 6 b
y CM =
3
3 13
rCM = i+ j
2 6 a b
Finalmente: rCM = i+ j
Ejemplo 2. Centro de masa de un tringulo. 3 3

Ejemplo 3. Centro de masa de un arco


semicircular.

1
xCM =
M xdm
Por el sistema de coordenadas escogido, xCM = 0 ,
Para evaluar porque por cada elemento de masa a la derecha (+),
existe otro elemento igual a la izquierda (-). Sin
masa total embargo para y CM es diferente.
dm = rea de la lmina
rea total
1
M 2M
y CM =
M ydm , en este caso dm = dl
= ydx = ydx
1 ab
ab M
2 Donde = y dl = Rd
R
1 1 2M
Luego: xCM =
M xdm = M x ydx
ab
Como y = Rsen , tenemos:

1 R 2
2 a y CM = 0 (Rsen )Rd =
send
ab 0
= xydx M M 0

R 2 R2 M
Para poder integrar tenemos que expresar la variable = [ cos ]0
= (2 )
y en funcin de x. M M R
Por semejanza de tringulos:
2R
= = 0,64 R
y ax
y = (a x )
b
=
b a a

3
Sistema de partculas Hugo Medina Guzmn

El centro de masa no se encuentra dentro del cuerpo.


Las figuras siguientes muestran como localizar
experimentalmente el centro de masa primero
colgndolo de la parte superior y luego de otro punto
cualquiera.

Despus que P1 y P2 se separan, las velocidades



respectivas son v'1 y v' 2 diferentes de v1 , y v 2 .

Ejemplo 4. Explosin de una granada

Ahora nos preguntamos. Qu pasa durante el choque?

El tiempo de contacto total t es muy pequeo,


quizs solo de aproximadamente 0,001 segundos. La
fuerza de contacto inicialmente es cero, aumenta
Una granada lanzada al aire que explota en varios hasta un valor muy grande y. finalmente disminuye
fragmentos. La nica fuerza externa sobre la granada hasta cero, cuando dejan de estar en contacto. La
es la fuerza de la gravedad, entonces la granada sigue figura siguiente muestra una variacin tpica de la
una trayectoria parablica. Si la granada no estallara fuerza en el tiempo de contacto.
continuara movindose a lo largo de la trayectoria
parab1ica indicada en la figura. Como las fuerzas de
la explosin son internas, no afectan al movimiento
del centro de masa. Entonces. Despus de La
explosin el centro de masa de los fragmentos sigue
la misma trayectoria que tendra la granada s! no
hubiera habido explosin.

IMPULSO Y CANTIDAD DE MOVIMIENTO

Supongamos el caso de dos partculas esfricas P1 y


Sea t f t i = t el tiempo que dura el choque,
P2 de masas m1 y m2 con trayectorias contenidas en la
misma recta, se aproximan una a otra con velocidades aplicando la segunda ley de Newton a las partculas
P1 y P2.
v1 , y v 2 respectivamente.

dv
F12 = m1 a1 = m1 1 y
dt

d v2
F21 = m2 a 2 = m2
dt


Cuando P1 y P2 entran en contacto, P1 ejerce sobre P2
la fuerza F12 y P2 ejerce sobre P1 la fuerza F21. De O F12 dt = m1 d v1 y F21 dt = m2 d v 2

acuerdo con la tercera ley de Newton F12 = F21 . Integrando las dos relaciones durante el choque,

4
Sistema de partculas Hugo Medina Guzmn

tf v '1 La partcula P1 ha sufrido en el intervalo


ti
F12 dt = m1 d v1 y
v1 t f t i = t , un cambio de la cantidad de

tf v '2

F21 dt = m2 d v2
tf

ti v2
movimiento ti
F12 dt = m1 v'1 v1 = p f pi

Finalmente
y esta cantidad es tambin igual al impulso J
tf

recibido en ese instante por la partcula
ti
F12 dt = m1 v'1 v1 y


tf


ti
F21 dt = m2 v' 2 v 2

J = p f pi

Luego: El cambio de la cantidad de movimiento


Trabajando con el primer miembro es igual al impulso.

tf
Ejemplo 5. Una pelota de 100 gramos est en reposo
ti
Fdt corresponde al rea bajo la curva mostrada
sobre el piso, cuando recibe un puntapi que la lanza
en la figura anterior, a sta cantidad la llamaremos con una velocidad de 30 m/s.

a) Qu impulso se dio a la pelota?



IMPULSO J
b) Si el tiempo que el pie est en contacto con la
pelota es 10-3 segundos. Cul es la magnitud
tf aproximada de la fuerza impulsiva?
J = F(t ) dt
ti
Solucin.
Sus dimensiones son: [F] [T] = [M][L][T]-1
a) El impulso es igual al cambio de la cantidad de
movimiento:
En el sistema internacional sus unidades son:

Newton.segundo (N.s) J = p f p i = m v f m vi

Trabajando con el segundo miembro


En este caso


m1 v'1 v1 y m2 v' 2 v 2 m = 0,1 kg, vi = 0 , v f = 30i m/s

kg m
Llamaremos a la cantidad mv = p, J = (0,1)(30&i&) 0 = 3i
s
CANTIDAD DE MOVIMIENTO LINEAL o b) Se puede obtener un estimado de la fuerza que
Momentum lineal de la partcula (lo designaremos
en la prctica simplemente como cantidad de acta sobre la pelota, dividiendo e1 impulso J por el
movimiento), cuyas dimensiones son:
tiempo t = t f t i en que acta la fuerza :

[M] [[L]] = [M] [L] [T]-1 J
F=
En el sistema internacional sus unidades son: t

kg.m.s-1 kg m
Como J = 3i y t = 0,001 s
s

5
Sistema de partculas Hugo Medina Guzmn

3i Usando la expresin de centro de masa


F= = 3000i N
0,001 n
m v i i = M v CMi
Ejemplo 6. Se deja caer una pelota de masa m de una i =1
altura h sobre el nivel del suelo y rebota hasta una
altura h1 n
De aqu p total = mi vi = M v CMi
i =1
a) Cul es la velocidad vi inmediatamente antes de
chocar con el suelo?
La cantidad de movimiento total de un sistema es
igual a la cantidad de movimiento de la masa total
b) Cul es la velocidad v f inmediatamente despus concentrada en el centro de masa del sistema.
de chocar con el suelo?
Derivando nuevamente la expresin anterior:

c) Cul es el impulso J que se le da a la pelota en d d
el impacto con el suelo? p total = M v i CM = M a iCM = F i ext
dt dt
Solucin.
Esta cantidad es muy importante, ya que si no hay
fuerza externa,

a) Como v 0 = 0 , x = 0 , y = h0
d
F i ext = 0 p total = 0
dt
vi = 2 gh0 j

p total = CONSTANTE
b) Como despus de chocar y = h1 , la velocidad

v f despus de chocar es: Esto es la conservacin de la cantidad de
movimiento. Si no hay fuerzas externas sobre un
sistema. La cantidad de movimiento total del
sistema es constante.
v f = 2 gh1 j
Ejemplo 7. Tres partculas de masas 2 kg, 1 kg y 3 kg
c) El impulso de la pelota es: respectivamente con vectores posicin


J = m v f m vi = m 2 g ( )
h1 h0 j

[ ]
r1 = 5ti 5t 2 j + (3t 2)k cm ,
r = [(2t 3)i (12 5t ) j + (4 + 6t 3t )k ]cm

2 3
2
CONSERVACIN DE LA CANTIDAD DE
y r = [(12t 1)i (t + 2 ) j t k ]cm

MOVIMIENTO 2 3
3

La cantidad de movimiento de una partcula de masa


Donde t es el tiempo en segundos.
m y velocidad v es:
Encontrar: a) La velocidad del centro de masa en t =
1 s y t = 2 s.
p = mv
b) La cantidad de movimiento lineal total del sistema
La cantidad de movimiento de n partculas es la suma en t = 1 s y t = 2 s.
de las cantidades de movimiento individuales,
c) Analizar si el sistema de tres partculas es sistema
n n aislado
p total = p i = mi vi
i =1 i =1 Solucin.

6
Sistema de partculas Hugo Medina Guzmn

a) La posicin del centro de masa esta dada por la ayudante ahogndose se encontrar el pescador
expresin: cuando alcance el extremo del bote?

Solucin.
m r + m2 r2 + m3 r3
rCM = 1 1
m1 + m2 + m3 Consideremos aislado el sistema bote, pescador,
ayudante, por lo tanto su cantidad de movimiento es
constante.
Reemplazando valores, obtenemos:

[ ( ) (
rCM = (3t 1)i + t 2 + 3 j + t 3 + 2t k cm )]
p = M v cm = CONSTANTE
La velocidad del centro de masa es
Como en inicio el sistema est en reposo:

vCM =
d
dt
[ (
rCM = 3i 2tj + 3t 2 + 2 k
cm
s
)]
p = 0 v cm = 0

Para t = l s
d r cm
Como v cm = =0

v1M [
= 3i 2 j + k
cm
s
]
dt
r cm = CONSTANTE , la posicin del centro de
masa permanece constante
Para t = 2 s
En ste problema que es en una sola dimensin:

v2 M [
= 3i 4 j + 8k
cm
s
] xcm = CONSTANTE

Tomemos como punto de referencia la posicin del


b) La cantidad de movimiento del sistema es: ayudante en el extremo del bote, al soltarse seguir en
la misma posicin.

p = + m1 v1 + m2 v 2 + m3 v3 = M vCM Analicemos la posicin inicial.

[ (
p = 6 3i 2tj + 3t 2 + 2 k
kg cm
s
)]
Para t = l s

[
p1 = 6 3i 2 j k
kg cm
s
]
El centro de masa del sistema pescador-bote est en:
Para t = 2 s
mb xb + m p (2,5)
xcm =

[
p2 = 6 3i 4 j 8k
kg cm
s
] mb + m p

Analicemos la posicin final.



c) Como , p1 p 2 , p no es constante, luego el
sistema no es aislado.

Ejemplo 8. Un pescador de masa 70 kg est en un


bote estacionario de masa 200 kg, cuando su
ayudante que no sabe nadar y est en el agua cogido
del extremo opuesto, se suelta. El pescador corre 2,5
m hasta alcanzar este extremo. A que distancia del
El centro de masa esta en:

7
Sistema de partculas Hugo Medina Guzmn

mb ( x b + x ) + m p x EI muchacho sale con una velocidad de mdulo


xcm =
mb + m p
m2
v1 = v dirigida hacia el oeste,
m1
Como la posicin del centro de masa del sistema es
invariante, se tiene:
Ejemplo 10. Dos personas de masa m ca da una, se
mb xb + m p (2,5) mb ( xb + x ) + m p x encuentran paradas e n l os ex tremos op uestos de un
= bote de l ongitud d y m asa 3 m que se e ncuentra e n
mb + m p mb + m p reposo sobre un lqu ido sin fr iccin, tal como se
muestra en la figura. Las personas caminan una hacia
(mb + m p )x = m p (2,5) la otra con rapidez constante y se encuentran a d/4 del
extremo izquierdo del bote.
a) Si la persona de la izquierda se mueve con
Reemplazando valores:
velocidad v 0 respecto al bote, cul es la velocidad

70(2,5) que tiene la otra persona, respecto al bote?


x= = 0,65m b) Cul es la velocidad del bote, respecto a tierra,
(200 + 70) durante el movimiento de ambas personas?
c) C unto a vanzo el b ote hast a el momento del
La posicin del pescador estar a 0,65 metros del encuentro?
ayudante.

Ejemplo 9. Un muchacho de masa m1 y una


muchacha de masa m2 , ambos con patines, se
encuentran en reposo uno en frente del otro, El
muchacho empuja a la muchacha, mandndola hacia
Solucin.
el este con una velocidad v . Describa el movimiento
del muchacho.

Solucin.

Siendo un sistema cerrado la cantidad de movimiento


se conserva, a)
El tiempo empleado para encontrarse es el mismo
para las dos personas
p antes = p despus = 0 , d 3d
4= 4 v1 = 3v 0 Hacia la izquierda

v0 v1
Si v 1 y v 2 son las velocidades del muchacho y la b) Por conservacin de la cantidad de movimiento
muchacha despus del empujn, respectivamente:
p antes = p despus

m1 v 1 + m2 v 2 = 0 p antes = 0

Considerando el movimiento en el eje x, y la p despus = m(v 0 + vb )i + m( 3v0 + vb )i + 3mvb i = 0
direccin al este como sentido positivo
2
vb = v0 i
v 2 = v = vi 5
c)
De aqu d
El t iempo de caminata de l as pe rsonas es t = ,
4v0
m2
m1 v 1 + m2 vi = 0 v 1 = vi luego el bote se habr movido
m1

8
Sistema de partculas Hugo Medina Guzmn


2 d d m v + m2 v 2
x = vb t = v0 = Como vCM = 1 1
5 4v 0 10 m1 + m2

SISTEMA DE REFERENCIA CENTRO DE


MASA m v + m2 v 2
u1 = v1 1 1
Cuando la fuerza externa resultante que acta sobre
m1 + m2
un sistema es cero, la cantidad de movimiento total es
constante. Muchas veces es conveniente escoger un m2
sistema de coordenada., con el origen situado en el = v1 v 2
centro de masa. Este sistema se denomina m1 + m2
SISTEMA DE REFERENCIA CENTRO DE
MASA
m v + m2 v2

Con respecto a este sistema la velocidad del centro de y u 2 = v2 1 1


m1 + m2
masa por supuesto es cero y la cantidad de
movimiento total es cero.
m1
El anlisis de la mayor parte de los choques es ms = v1 v 2
m1 + m2
sencillo en el sistema de referencia centro de masa.

La transformacin de un sistema de referencia Como comprobacin, calculemos la cantidad de


cualquiera a un sistema centro de masa no es difcil. movimiento total con respecto al centro de masa, el
Consideremos un sistema do dos partculas m1 y m2 cual debe ser igual a cero.

con velocidades v1 y v 2 respectivamente cuyo
p = m1 u 1 + m2 u 2
centro de masa se mueve con velocidad vCM , como
se muestra en la figura. m2
= p = m1 v1 v 2
m1 + m2

m1
= m2 v1 v2 = 0
m1 + m2

La cantidad de movimiento es: En la seccin siguiente veremos ejemplos de


aplicacin usando el sistema de referencia centro de
masa.
p = m1 v 1 + m2 v 2 = (m1 + m2 ) v CM
CHOQUES
Para transformar esta expresin al sistema Centro de
masa, las velocidades de las partculas con respecto al Se llama choque o colisin entre dos cuerpos a un
centro de masa son como se muestra en la figura fenmeno en el que los cuerpos Participantes son
siguiente. libres antes y despus de la interaccin, sobre los que
no actan fuerzas resultantes.

La interaccin dura un tiempo muy corto, durante el


cual los cuerpos ejercen entre si fuerzas de cierta
intensidad.

Por lo general en los choques s1o participan dos


Las velocidades relativas al centro de masa son:
cuerpos, aunque esto no es estrictamente necesario.

u1 = v1 vCM y u 2 = v 2 vCM

9
Sistema de partculas Hugo Medina Guzmn

Sean dos cuerpos de masas m1 y m2 con velocidades Para la masa m 2 :



t0
v1 y v 2 antes del choque y velocidades v'1 y v'2
despus del choque respectivamente. ti
F2 dt = m2 v0 m2 v 2 = J 2 D = J 1D

En todo choque entre dos cuerpos se conserva la


cantidad de movimiento, esto es: Resolviendo para v1 y v 2 .


p = p' p1 + p 2 = p'1 + p' 2 J J
v1 = 1D + v 0 , v 2 = 1D + v0
m1 m2

m1 v1 + m2 v 2 = m1 v'1 + m2 v' 2
La diferencia de estas velocidades es:
Ahora nos introduciremos en el proceso complejo que
1 1 m +m
acompaa al choque, el instante t = t f t i , en el
v 2 v1 = J 1D + = J 1D 1 2

que aparece la fuerza de interaccin, este periodo 1
m m 2 1 2
m m
vamos a dividirlo en dos partes, los periodos de
deformacin y restitucin. La figura muestra el
Ahora aplicaremos la ecuacin Impulso-cantidad de
grfico de la fuerza de interaccin en funcin del movimiento por el periodo de restitucin (R).
tiempo entre las masas m1 y m2.
t0 t f .

Para la masa m1 :

t0

ti
F1 dt = m1 v'1 m1 v0 = J R

Para la masa m 2 :
t0

ti
F2 dt = m2 v' 2 m2 v0 = J 2 R = J 1R


Resolviendo para v '1 y v ' 2 .
E1 tiempo t 0 es el instante de mxima deformacin
en el que empieza la restitucin y las dos masas
J J
poseen la misma velocidad v'1 = 1R + v0 , v' 2 = 1R + v 0
m1 m2

v01 = v02 = v0
La diferencia de estas velocidades es:

Vamos a aplicar la ecuacin impulso - cantidad de


1 1
movimiento para el periodo de deformacin (D),
v' 2 v'1 = J 1R +
ti t 0 : m1 m2

Para la masa m1 : m1 + m2
= J 1R
t0
m1 m2
ti
F1 dt = m1 v 0 m1 v1 = J D
De lo visto encontramos la relacin entre el impulso
de restitucin y el impulso de deformacin.

10
Sistema de partculas Hugo Medina Guzmn

a) Choque perfectamente elstico. En este caso no


J 1R (v' v' )
= 2 1 =
hay prdida en la energa mecnica asociada al

J 1D (v 2 v1 ) impacto, la energa cintica permanece constante.

K 1 + K 2 = K '1 + K ' 2
A esta relacin se le conoce como coeficiente de
restitucin ( ) .
1 1 1 1
m1v12 + m2 v 2 = m1v'12 + m2 v' 22
Esta relacin fue propuesta por Newton y tiene
2 2 2 2
validez solamente aproximada.
( ) (
m1 v'12 v12 = m2 v' 22 v 22 )
EI valor de esta relacin depende de muchos factores
tales como la geometra, las propiedades de los Por conservacin de la cantidad de movimiento
materiales, la velocidad, por ello debemos tenemos:
contentarnos con una determinacin experimental.

Ejemplo 11. Una pelota de bisbol de 0,15 kg de p1 + p 2 = p '1 + p' 2 ,
masa se est moviendo con una velocidad de 40 m/s
cuando es golpeada por un bate que invierte su m1 v1 + m2 v 2 = m1 v'1 + m2 v' 2
direccin adquiriendo una velocidad de 60 m/s, qu
fuerza promedio ejerci el bate sobre la pelota si
estuvo en contacto con ella 5 ms?.
m1 v1 v'1 = m2 v' 2 v 2
Solucin.
Datos: m = 0,15 kg
vi = 40 m/s Asumiendo que el movimiento es en una sola
vf = - 60 m/s (el signo es negativo ya que cambia el direccin
sentido)
t = 5 ms = 0,005 s m1 (v1 v'1 ) = m2 (v' 2 v 2 )
p = J
pf - pi = J mvf - mvi = F t Dividiendo entre si las expresiones halladas por
energa y por cantidad de movimiento obtenemos.
F = m(vf - vi)/t
F = 0,15 kg.(- 60 m/s - 40 m/s)/0,005 s
v1 + v'1 = v' 2 +v 2 (v' 2 v'1 ) = (v 2 v1 )
= 0,15 kg.(- 100 m/s)/0,005 s
= - 3000 N
(v' 2 v'1 )
=1
CASOS DE CHOQUE (v 2 v1 )

Perfectamente elstico El cual es por supuesto el coeficiente de restitucin


de un choque perfectamente elstico = 1 .
= 1 , (v' 2 v'1 ) = (v 2 v1 )
b) Choque perfectamente plstico. En un choque
Inelstico <1 perfectamente Plstico, despus del choque las masas
quedan juntas, es decir tienen la misma velocidad, tal
que
El coeficiente de restitucin y tiene un valor entre 0 y
1.

v' 2 = v'1 , por lo tanto:
Perfectamente plstico

= 0 , (v' 2 v'1 ) = 0 v' 2 v'1 = 0 y = 0

Explosivo >1 Ejemplo 13. Medicin del coeficiente de


restitucin .
Ejemplo 12.

11
Sistema de partculas Hugo Medina Guzmn

Si se quiere medir el coeficiente de restitucin de 1os sentido contrario. Luego del choque ambos cuerpos
materiales, se realiza mediante una bola hecha con continan juntos con una velocidad final comn a
uno de los materiales y una superficie plana hecha ambos.
con el otro material, la que se coloca sobre el suelo.
Se suelta verticalmente la bola sobre la superficie La velocidad final ser:
desde una altura h1 .
m1v1i + m2v2i = m1v1f + m2v2f
Conocemos la velocidad de la bola al momento del
choque Como v1f y v2f son iguales porque ambos cuerpos
siguen juntos:
v1 = 2gh1
v1f = v2f = vf

La bola rebota verticalmente hasta una altura h2 , tal m1v1i + m2v2i = (m1 + m2)vf
que la velocidad v '1 de la bola despus del choque es:
vf =
(m1v1i + m2v2i )
v'1 = 2 gh2 (m1 + m2 )
b) Velocidades de igual direccin y sentido
Como la superficie no tiene velocidad inicial ni
contrario.
velocidad final v 2 = 0 y v ' 2 = 0 .

Encontramos que:

(v' 2 v'1 ) v'1


= =
(v2 v1 ) v1

Reemplazando valores:

En este caso los cuerpos posean velocidades de igual


2 gh2 h2 direccin pero de sentido contrario antes del choque,
= =
2 gh1 h1 como en el caso anterior luego del impacto continan
juntos, con una velocidad final que estar dada por la
diferencia de las cantidades de movimiento. La
Ejemplo 14. Choque plstico o inelstico velocidad final ser:

a) Velocidades de igual direccin y sentido. m1v1i - m2v2i = m1v1f + m2v2f

Igualmente:

v1f = v2f = vf

m1v1i - m2v2i = (m1 + m2)vf

vf =
(m1v1i m2v2i )
(m1 + m2 )
Supongamos un cuerpo 1 de masa m1 y velocidad v1 La velocidad final mantendr la misma direccin pero
que se dirige a hacia el cuerpo 2 de masa m2 y tendr el sentido de la velocidad del cuerpo que antes
velocidad v2, siendo ambas velocidades de igual del choque tena mayor cantidad de movimiento.
direccin y sentido. Sobre cada cuerpo actu en el
momento del choque, el impulso que le provoc el Ejemplo 15. Choque elstico
otro cuerpo, entonces hay dos acciones de igual
intensidad y sentido contrario, en consecuencia a) Velocidades de igual sentido
ambas cantidades de movimiento sern iguales y de

12
Sistema de partculas Hugo Medina Guzmn

Ejemplo 16. Choque plstico. Las dos partculas


quedan en contacto despus del choque.

Estudiar desde dos puntos de vista:

a) Observado desde tierra, sistema laboratorio y

b) Observado desde el centro de masa.

Solucin.
Durante el choque cada cuerpo recibe una cantidad de
movimiento que es igual a la velocidad perdida por el
otro. Al recuperar su forma inicial, cada uno pierde o a) Sistema laboratorio.
gana respectivamente, la cantidad de movimiento
ganada o perdida en el momento del choque, la La figura muestra las dos partculas antes y despus
velocidad final de cada uno ser: del choque.

v1 f =
m2
(v2i v2 f ) + v1i
m1

Si las masas son iguales

v1 f = v2i v2 f + v1i

b) Velocidades de distinto sentido Por conservacin de la cantidad de movimiento


m1 v1 + m2 v2 = (m1 + m2 )v' y


m v + m2 v2 (m1v1 + m2v2 )
v' = 1 1 = i
(m1 + m2 ) (m1 + m2 )
La energa mecnica antes del choque es:
En este caso los cuerpos literalmente rebotan, y la
velocidad final de cada uno ser: 1 1
K = K1 + K 2 = m1v12 + m2 v 22
2 2
v1 f =
m2
m1
(v2i + v2 f ) v1i La energa mecnica despus del choque es:

(m1 + m2 )v'2 = 1 (m1v1 + m2 v 2 )


2
Si las masas son iguales 1
K '=
2 2 (m1 + m2 )
v1 f = v2i + v2 f v1i
La relacin de la energa es:
El principio de conservacin del impulso es el mismo
que el de conservacin de la cantidad de movimiento. 2
m
v1 + 2 v 2
Cabe aclarar que en la prctica podemos aplicar el K' m1 m1
=
principio de conservacin de la cantidad de
K (m1 + m2 ) 2 m2 2
movimiento durante los choques, siempre que el v1 + v2
tiempo que el tiempo de duracin del impacto sea m1
muy pequeo.

13
Sistema de partculas Hugo Medina Guzmn

Por ejemplo la cada de un meteorito a la tierra, la que La figura muestra las dos partculas antes y despus
suponemos inmvil (v2 = 0) y m1 << m2, obtenemos del choque.
K'
K = 0 y = 0 , ste es un choque perfectamente
K
plstico. Si K fuera diferente de cero, la totalidad de
la energa se transformara en calor.

b) Sistema centro de masa.

La figura muestra las dos partculas antes y despus


del choque.

Por conservacin de la cantidad de movimiento:


m1 v1 = m1 v'1 + m2 v' 2 (1)

En sus componentes:
En ste caso:
m1v1 = m1v'1 cos 1 + m2 v' 2 cos 2
m2
u1 = v1 v 2 y 0 = m1v'1 sen1 m2 v' 2 sen 2
m1 + m2
m1 Como es un choque elstico la energa mecnica se
u2 = v1 v 2
m1 + m2 conserva:

1 1 1
Con v 2 = 0 , m1v12 = m1v'12 + m2 v' 22 (2)
2 2 2
Obtenemos: En las ecuaciones (1) y (2) conocidas las masas m1 y
m2, tenemos como incgnitas v1, v1, v2, 1 y 2.
m2 m1 Contamos con tres ecuaciones. Para resolver
u1 = v1 y u 2 = v1 necesitamos conocer al menos dos de las cantidades
m1 + m2 m1 + m2 anteriores.

Despus del choque m1 y m2 entran en contacto En el caso particular en que m1 = m2, podemos llegar
constituyendo una sola partcula de masa (m1 + m2) a la relacin;
que est en reposo en el sistema centro de masa,

u1 = u2 = 0. v1 = v'1 + v' 2

Aqu tambin K = 0, = 0. Elevndola al cuadrado:


Ejemplo 17. Choque elstico. Consideremos dos

v12 = v'12 +v' 22 + 2v'1 v' 2
partculas, una con masa m1 y velocidad v1 , la
Por la conservacin de la energa:
segunda con masa m 2 y velocidad v 2 = 0
v12 = v'12 +v' 22
Solucin.
Luego, obtenemos:
a) Sistema laboratorio.

14
Sistema de partculas Hugo Medina Guzmn


m1u1 = m1u '1 u1 = u '1 y
v'1 v'2 = 0

m2 u 2 = m2 u ' 2 u 2 = u ' 2
Las velocidades v'1 y v'2 son ortogonales, esto nos
dice que las trayectorias de las partculas despus del Para un choque elstico = 1 , como se espera.
choque son perpendiculares entre s, tal que:
1 + 2 = 2 Ejemplo 18. Reflexin de partcula sobre un plano.
Consideremos dos partculas, una con masa m1 , que
b) Sistema centro de masa.
incide sobre una masa m 2 de superficie plana como

La figura muestra las dos partculas antes y despus se muestra en la figura. La masa m1 tiene
del choque.
velocidades v1 y v'1 antes y despus del choque, la

superficie inicialmente est inmvil v 2 = 0 y tiene

una velocidad v' 2 despus del choque.

Solucin.
Por conservacin de la cantidad de movimiento:
Por conservacin de la cantidad de movimiento:

m1 u1 + m2 u 2 = m1 u '1 + m2 u ' 2 = 0
m1 v1 = m1 v'1 + m2 v' 2
De aqu:
Para la energa tenemos que tomar en cuenta si el
2 2 choque es elstico o no.
m m
u = 1 u1 , u '22 = 1 u '1
2
2
m2 m2 a) Choque elstico.

En ste caso la energa mecnica se conserva


Como es un choque elstico la energa mecnica se
conserva: K = K'

1 1 1 1 1 1 1
m1u12 + m2 u 22 = m1u '12 + m2 u ' 22 m1v12 = m1v'12 + m2v'22
2 2 2 2 2 2 2

Reemplazando u 2 y u ' 2 en funcin de u1 y u '1 2


De aqu obtenemos: v '1 v1 =
2 m2 2
v' 2
respectivamente. m1

1 1 1 1 Expresin que podemos escribir como;


(m1u1 )2 + = (m1u '1 )2 +
2m1 2m2 2m1 2m2 m2
v1 + v'1 v1 v'1 = v 2 v' 2
m1
De aqu se deduce:
De la conservacin de la cantidad de movimiento

15
Sistema de partculas Hugo Medina Guzmn



m
m1 v1 v'1 = m2 v' 2 v1 v'1 = 2 v' 2 el otro con velocidad v 2 de tal modo que chocan.
m1
Despus del choque el auto 1 sale con velocidad v'1
Reemplazando sta expresin en la de la energa, cuya direccin forma un ngulo , tal como se
obtenemos: indica en la figura.


a) Hallar la velocidad del auto 1 luego del impacto.
v1 + v'1 = v' 2
b) Determinar la posicin del centro de masa y las
Como ecuaciones paramtricas del mismo.

c) Determinar si el choque es elstico o no.


v1 = v1sen1i v1 cos 1 j ,

v'1 = v'1 sen '1 i + v'1 cos '1 j , m1 = m2 = 200 kg , v01 = 3 m/s ,
v02 = 1 m/s , v'1 = 2 m/s , = 53 , = 37 ,

d = 3m
v' 2 = v' 2 j

Reemplazando obtenemos:

v1sen1i v1 cos 1 j + v'1 sen '1 i


+ v'1 cos '1 j = v'2 j

De aqu:

v1sen 1 + v'1 sen '1 = 0 y


v1 cos 1 v'1 cos '1 = v' 2 Solucin.

En el caso en que v ' 2 = 0 (la superficie no se mueve) a) por conservacin de la cantidad de movimiento


1 = '1 y v1 = v '1 p antes = p despus

b) Choque inelstico. m1 v1 + m2 v 2 = m1 v'1 + m2 v' 2

En ste caso K > K ' Aqu

1 1 1
m1v12 > m1v'12 + m1v' 22 v1 = 3i ,
2 2 2

Para encontrar la relacin de K y K podemos usar el v 2 = (1) cos 53 i + (1)sen53 j
coeficiente de restitucin .

= 0,6i + 0,8 j ,
K'
= , siendo 0 1
K
v'1 = (2)cos 37 i + (2)sen37 j
Ejemplo 19. En un parque de diversiones dos amigos
juegan con los autitos chocones. En cierto
momento las direcciones de ambos vehculos forman = 1,6i + 1,2 j

un ngulo . Un auto se dirige con velocidad v1 y

16
Sistema de partculas Hugo Medina Guzmn

Reemplazando: Como m1 = m 2 = 200kg



3i + 0,6i + 0,8 j = 1,6i + 1,2 j + v'2
1
xCM = (x1 + x 2 ) , yCM = 1 ( y1 + y 2 )
2 2
v'2 = 2i 0,4 j
Antes del choque:
2 2
v'2 = 2 + 0,4 = 4,16
x1 = 3 + 3t , y1 = 0
= 2,04 m/s
x 2 = 0,6 + 0,6t , y 2 = 0,8 + 0,8t
2
tan = = 5 = 79 Luego
0,4
xCM = 1,8 + 1,8t , y CM = 0,4 + 0,4t

Despus del choque:

x1 = 1,6(t 1) , y1 = 1,2(t 1)

x2 = 2(t 1) , y 2 = 0,4(t 1)

Luego

xCM = 1,8(t 1) , yCM = 0,4(t 1)

c) Para saber si es elstico o no, tenemos que analizar


b) Para determinar la posicin del centro de masa es si la energa se conserva o no.
necesario conocer la posicin inicial de la masa m 2 .
La energa cintica antes del choque es:
Como m1 y m 2 emplean el mismo tiempo desde el
inicio hasta el choque: 1 1
K= m1v12 + m2 v 22
2 2
d 3m
t= = = 1s
v1 3 m s 1
= (200)(3)2 + 1 (200)(1)2
2 2
La posicin inicial de m 2 es:
= 900 +100 = 1000 J
x20 = v2 xt = 0,6(1) = 0,6 m ,
La energa cintica despus del choque es:
y20 = v2 yt = 0,8(1) = 0,8 m
1 1
K'= m1v'12 + m2 v' 22
Siendo la posicin inicial de m1 2 2

x10 = 3m , y10 = 0 1
= (200)(2)2 + 1 (200)(2,04)2
2 2
El centro de masa est dado por:
= 400 + 416 = 816 J
x1 m1 + x 2 m2 y m + y 2 m2
xCM = , y CM = 1 1
m1 + m2 m1 + m2 Hay una disminucin de la energa cintica:

17
Sistema de partculas Hugo Medina Guzmn

K = 816 1000 = - 184 J


V = 2 gy = 2(9,80 )(0,45 102 )
Luego el choque es inelstico.
= 0,297 m/s
Ejemplo 20. El pndulo balstico. Este es el caso de La rapidez final v de la bala es
un choque perfectamente plstico, se utiliza para
medir la velocidad de un proyectil. Un proyectil de p mv0 MV M
masa m y velocidad v se incrusta en el bloque de v= = = v0 V
madera de masa M. m m m
1,00
= 450 (0,297)
5,00 10 3
= 390,6 m/s .

Ejemplo 22. Un satlite artificial en vuelo explota en


tres partes iguales. Una parte contina a lo largo de su
lnea original de vuelo y las otras dos van en
direcciones cada una inclinada 60 a la trayectoria
original. La energa liberada en la explosin es dos
veces ms que la energa que tena el sat1ite en el
Aplicando la conservacin de la cantidad de momento de la explosin. Determinar la energa
movimiento. cintica de cada fragmento Inmediatamente despus
de la explosin.

m v = (m + M )V Solucin.

La energa cintica despus del choque es: La figura muestra el satlite antes y despus de la
explosin.
1
K'= (m + M )V 2 , sta se convierte en energa
2
potencial U = (m + M )gh

Luego (m + M )V = (m + M )gh
2

V = 2 gh

La velocidad del proyectil es:


Por conservacin de la cantidad de movimiento.

v=
(m + M ) V =
(m + M ) 2 gh
m m p antes = p despus
Ejemplo 21. Una bala de 5,00 g se dispara contra un
bloque de madera de 1,00 kg suspendido de un hilo La cantidad de movimiento debe conservarse en las
de 2,000 m, atravesndolo. El centro de masa del tres dimensiones x, y, z , independientemente, de all
bloque se eleva 0,45 cm. Calcule la rapidez de la bala que v1, v2, v3 y V deben ser coplanares.
al salir del bloque si su rapidez inicial es de 450 m/s.
As obtenemos:
Solucin. Mv =
M
v1 +
M M
v 2 cos 60 + v3 cos 60
3 3 3
La rapidez del bloque de madera despus de que la
bala ha atravesado (pero antes de que el bloque M M
comience a elevarse; esto asume una gran fuerza y v 2 sen 60 v3sen 60 = 0
aplicada por un tiempo corto, una situacin 3 3
caracterstica de las balas) es
De estas dos ecuaciones encontramos que:

18
Sistema de partculas Hugo Medina Guzmn

v 2 = v3 (1) b) W f = mgd
Wf 0,5
3v = v1 + v 2 (2) = = = 0,41
mgd 2,5 9,8 0,05
1 2 1 2
1 c) kx Wf = mv
La energa inicial es: Ei = Mv 2 2 2
2 1
(1040)(0,05)2 0,5 = 1 (2,5)v 2
La energa final es: 2 2
1 1 3 v = 0,80 m/s
Ef = Mv 2 + 2 Mv 2 = Mv 2
2 2 2 Ejemplo 24. Una pelota de masa m = 100 g se deja
caer desde una altura h = 2m. La pelota rebota
Esta energa es igual a la suma de las energas de los verticalmente hasta h despus de golpear el suelo.
tres fragmentos. a) Calcular la cantidad de movimiento de la pelota
antes y despus de golpear el suelo,
b) si la duracin del golpe fue de 0,01 s, calcular la
3 1M 2 1M 2 1M 2 fuerza media ejercida por el piso sobre la pelota.
Mv 2 = v1 + v2 + v3
2 2 3 2 3 2 3 Solucin.
En la figura se muestra el esquema de la situacin.
9v 2 = v12 + v 22 + v32 (3)

De las ecuaciones (1), (2) y (3) obtenemos:

v1 = v , v 2 = 2v , v3 = 2v

La energa cintica de cada uno de los fragmentos


inmediatamente despus de la explosin es:
a) Cantidad de movimiento inicial:

1 2 2 pi = mvi j
K1 = Mv 2 , K 2 = Mv 2 , K 3 = Mv 2
6 3 3 Cantidad de movimiento final:

Ejemplo 23. Un bloque de 2,5 kg, se desliza sobre p f = mv f j
una superficie rugosa, cuando contacta con el resorte Los valores de las velocidades inicial y final se
tiene una velocidad de 1,2 m/s. el bloque se detiene pueden calcular usando el principio de conservacin
momentneamente cuando el resorte se ha de la energa.
comprimido 5,0 cm. El trabajo realizado por la Inicial:
friccin, desde el instante en que el bloque hace
1 2
contacto con el resorte hasta el instante en que hace el 0 + mghi = mvi + 0 vi = 2 ghi
alto es 0,50 J. 2
a) Cul es la constante del resorte (k)? Final:
b) Cul es el coeficiente de friccin? 1 2
c) Despus de la compresin del resorte, el bloque se mv f + 0 = 0 + mgh f
aleja de l, cual es la velocidad del bloque, despus 2
de separarse del resorte?
3 3
v f = 2 gh f = 2 g hi = ghi
4 2

Por lo tanto, las cantidades de movimiento inicial y


final son:
Solucin.
3
a) Energa antes = Energa despus pi = m 2 ghi j , p f = m ghi j
2
1
(2,5)(1,2)2 = 1 k (0,05)2 + 0,5 Reemplazando los valores, se tiene:
2 2 pi = 0,63 kgm/s , p f = 0,54 kgm/s
k = 1040N/m

19
Sistema de partculas Hugo Medina Guzmn

b) Usando la aproximacin del impulso:




p pf
J = Fm t = p Fm = p = f

t t

Fm =
(
0,54 j 0,63 j )
0,01
= 118 j N

Ejemplo 25. Una bala de la masa m y con velocidad


v pasa a travs de un pndulo de masa M. La bala
emerge con una velocidad de v/2. El pndulo est
suspendido por una barra rgida de longitud l y masa
insignificante. Cul es el valor mnimo de v tal que
la masa del pndulo gire un crculo completo?
Solucin.
En esta colisin, se conserva la cantidad de T + Mg = Mac
movimiento pero la energa no. Condicin mnima para hacer movimiento circular
Este es un ejemplo de una colisin inelstica que no T =0
es perfectamente inelstica. Luego
Para la colisin:
V '2
Mg = M V ' 2 = gl (3)
l
Reemplazando (3) en (2):
1 1
MV 2 = Mgl + Mg (2l )
2 2
1 5
MV 2 = Mgl V = 5 gl
2 2
Reemplazando el valor de V en (1):
2M
v v= 5 gl
p antes = p despus mv = m + MV m
2
De aqu:
2M MOVIMIENTO CON MASA VARIABLE -
v= V (1) PROPULSIN POR REACCIN
m
Despus de la colisin se conserva la energa para el
pndulo (la conservacin de la energa para la bala Por la conservacin de la cantidad de movimiento si
despus de la colisin no es til desde que su energa un cuerpo en reposo puede expulsar una parte de su
no cambia). Este tratamiento nos da la velocidad del masa en cierta direccin, el resto de la masa se
pndulo el momento despus de la colisin: mover en sentido opuesto, con igual cantidad de
movimiento. Si este proceso puede mantenerse
1 1
MV 2 = MV ' 2 + Mg (2l ) (2) durante un tiempo, el resto de la masa, como es el
2 2 caso de un cohete, aparecer para un observador
Condicin para que pueda dar la vuelta externo en reposo. Como si se estuviese acelerando.
Esto se expresa mediante la forma ms general de la
segunda ley de Newton.

Como:


dp
F= , p = mv
dt

Siendo la masa variable

20
Sistema de partculas Hugo Medina Guzmn


d v dm
dm v Como el infinitsimo dmd v es de segundo orden,
F= = m +v podemos despreciar ste trmino; luego
dt dt dt

Expresin que nos permite determinar el movimiento
md v dm u = 0
de un cuerpo cuya masa cambia durante su
movimiento.

d v dm
Cuando aplicamos al caso de un cohete aparecen los Dividiendo por dt se obtiene: m u =0
problemas, evidentemente m es la masa del cohete dt dt
que va cambiando. Cul es la velocidad de escape

del combustible? No es igual a v , la velocidad del dv
Como v es la velocidad del cohete, es la
cohete. Si no existe fuerza externa, F debe ser cero? dt

Entonces no se mover el cohete. Analicemos el
aceleracin a .
problema desde el punto de vista de la conservacin
de la cantidad de movimiento.
De ste modo:
Sea el cohete mostrado en la figura siguiente, en el


instante t, tiene una masa m y una velocidad v = vi , dv dm
m = ma = u
dt dt
con una cantidad de movimiento lineal m v .
Por la segunda ley de Newton se puede identificar la
dm
cantidad u como una fuerza, tal que la fuerza
dt
de empuje es:
En la figura siguiente se muestra el cohete en el

instante t + dt , a expulsado una masa dm que sale F =u
dm
dt
con una velocidad u = ui relativa al cohete.
Ahora la masa del cohete es (m dm ) y su dm
Como u es negativa y por se prdida de mas


velocidad es v + d v . dt

tambin es negativa, F es positiva, como
esperbamos. Esta es una fuerza externa que produce
aceleracin al cohete, al que ahora consideraremos
como un sistema aislado sobre el que hay una fuerza
externa.

Velocidad del cohete.


La cantidad de movimiento lineal total es:

De la expresin md v dm u = 0 , obtenemos:
(m dm ) v + d v + dm v u


dv=u
dm
m
Por conservacin de la cantidad de movimiento lineal
dm m m
= u [ln ]m0 = u ln
m
tenemos: p antes = p despus v (t ) = u
m0 m m0



m v = (m dm ) v + d v + dm v u m
Como u es negativa y m < m0 , ln es negativa.
m0

m v = m v + md v dm v + dmd v + dm v dm u Luego la velocidad es positiva.

21
Sistema de partculas Hugo Medina Guzmn

Velocidad lmite del cohete. (m0 Ct )


v(t ) = gt uln
Una vez que se haya terminado el combustible la
m0
masa se reduce a m1 , y llegamos a la velocidad
b) Antes de encontrar la altura que alcanza en el
lmite.
momento que se acaba el combustible, encontraremos
la altura para el tiempo t.
m1
v m = u ln
m0 dz
Como v(t ) = , tenemos:
dt
Una vez alcanzada sta velocidad, sta permanecer
constante.
(m0 Ct )
dz = gtdt uln dt
Ejemplo 26. Un cohete y su combustible tienen una m0
masa inicial m0 . El combustible se quema a una
Integrando
dm
razn constante = C .Los gases son expulsados
m z t t (m0 Ct )

con una velocidad constante u respecto al cohete. 0
dz = gtdt uln
0 0 m0
dt

a) Despreciando la resistencia del aire, hallar la


1 (m Ct )
z = gt 2 (m0 Ct )ln 0
velocidad del cohete en el instante t, despus de u
1
despegar de la tierra. 2 C m0
b) Cul es la altura que alcanza en el momento que
se acaba el combustible? El tiempo t1 en que se acaba el combustible es
cuando m = m1 . Como m = m1 Ct , obtenemos:
Solucin.
m0 m1
a) Las fuerzas que actan sobre el cohete son la t1 =
fuerza de empuje y la fuerza de la gravedad. C
Aplicando la segunda ley de Newton.
Reemplazando t1 en la expresin anterior
dm
F = m a u dt + m g = m a encontramos la altura que alcanza en el momento en
que el combustible se acaba.

CANTIDAD DE MOVIMIENTO ANGULAR Y


Donde u = uk , g = gk , a = d v = dv k

MOMENTO DE UNA FUERZA O TORQUE
dt dt
Consideremos una partcula P de masa m, su posicin
dm con respecto al origen O en un instante dado est
y = C m = m0 Ct
dt determinada por el vector r .

Reemplazando uC (m0 Ct )g = (m0 Ct )


dv

dt La partcula tiene una velocidad v y su cantidad de



movimiento lineal es p = mv .
dv uC
o = g +
dt (m0 Ct )
v t t uC
Integrando
0
dv = gdt +
0 0 (m0 Ct )
dt

La velocidad en el instante t es:

22
Sistema de partculas Hugo Medina Guzmn


dL d dr d p
= r p = p+ r
dt dt dt dt

Como


dr
= v y p = mv
dt

LA CANTIDAD DE MOVIMIENTO ANGULAR
dr

p = v m v = 0
O MOMENTUM ANGULAR L dt

Se define como el producto vectorial de r y p ,
Luego



L = r p dL d p
= r
dt dt

La direccin de L es perpendicular al plano definido

por r y p , su sentido lo da la regla de la mano Por otra parte si F es la fuerza que produce el
movimiento de la partcula, por la Segunda Ley de
derecha, su mdulo es:
Newton tenemos:

L = rp sen = rmv sen


dv dp
F = ma = m =
Como vsen es la velocidad tangencial ( vt ) y dt dt
vt = r , siendo la velocidad angular. Podemos
escribir: dL
Luego = r F
dt
L = mr 2

A esta cantidad que produce un cambio en la cantidad


MOMENTO DE INERCIA (I ) . de movimiento angular se le conoce como
2
Llamando Momento de Inercia al producto mr ,

MOMENT0 DE UNA FUERZA o TORQUE
Tenemos:

L = I , vectorialmente L = I dL
= = r F
dt
Las dimensiones de la cantidad de movimiento
angular son:
Tiene como mdulo = rF sen

[L] = [M ][L]2 [T ]1 Su sentido est dado por la regla de la mano derecha.

Sus unidades en el sistema internacional: Ejemplo 27. Una partcula de masa m se mueve en el

2 plano xy con una velocidad v a lo largo de una lnea
Kg m
o Js recta. Cul es la magnitud y direccin de su cantidad
s de movimiento angular con respecto al origen O?


Derivando la cantidad de movimiento angular L con
respecto al tiempo:

23
Sistema de partculas Hugo Medina Guzmn

Ejemplo 29. Un cilindro slido Puede girar alrededor


de un eje sin friccin como se ve en la figura. Una
cuerda enrollada alrededor del radio exterior R1 ejerce
una fuerza F1 hacia la derecha. Una segunda cuerda
enrollada alrededor de la otra seccin cuyo radio es
R2 ejerce una fuerza F2 hacia abajo. Cul es el torque
que acta sobre el cilindro alrededor del eje z que
pasa por O?

Solucin.


La posici6n de la partcula es r .


La velocidad de la partcula es v .

Su cantidad de movimiento lineal es



p = mv
Solucin.

Su cantidad de movimiento angular es L = r p Sobre el cilindro actan:



L = r m v = rmvsen k ( ) F 1 = F1i en r 1 = R1 j y F 2 = F2 j en

r 2 = R2 i
La magnitud es: L = rmvsen = mvd , donde
d = rsen El torque neto sobre el cilindro es:


Luego L = (mvd )k

= 1+ 2
Podemos ver que la cantidad de movimiento angular
con respecto a O' es cero. = r 1 F 1 + r 2 F 2 = R1 F1 k + R2 F2 k

Ejemplo 28. En determinado instante, la Posicin de


= (R2 F2 R1 F1 )k
una partcula con respecto a un origen O de

coordenadas est dada por el vector r = 3i + 4 j Si F1 = 10 N, R1 = 2 m y F2 = 5 n, R2 = 1 m:
(en metros) . En ella acta una fuerza

F = 16i + 32 j (en Newton) . Encontrar el torque = [(5 N )(1m ) (10 N )(2m )]k = 15k N m

originado por la fuerza F que acta sobre la
CONSERVACION DE LA CANTIDAD DE
partcula. Con referencia al origen O.
MOVIMIENTO ANGULAR

Solucin.
En el caso de una partcula come en la seccin
anterior, si el torque aplicado con relacin a un punto

El torque es: = r F = (3i + 4 j ) (16i + 32 j ) dado de referencia es cero, tenemos que:


= (3)(32)k + (4)(16)( k ) = 96k 64k dL
= 0 , por consiguiente: L = CONSTANTE
dt
= 32k Nm
La cantidad de movimiento angular con respecto al
punto de referencia es constante.

24
Sistema de partculas Hugo Medina Guzmn

Ejemplo 30. Una partcula de, masa M en el extremo Ejemplo 31. Clculo de la desviacin de un cuerpo
de un hilo gira con velocidad v1 cuando el radio es r1, situado en la lnea ecuatorial y que cae desde una
si disminuimos el radio de r1 a r2, qu sucede con la altura h .
velocidad?
Solucin.
Solucin.

En la figura, sea una partcula de masa m a una altura


h sobre la superficie de la Tierra en un punto que,
para simplificar, consideramos que se encuentra sobre
el ecuador. Para los casos de inters fsico, la altura h
ser por lo comn muy pequea en comparacin con
el radio, R de la Tierra. Si se supone que la partcula
La figura indica la forma como se puede realizar esta parte del reposo en relacin a un punto de la
experiencia, para disminuir el radio basta jalar el hilo. superficie de la Tierra verticalmente por debajo de l
entonces, inicialmente, el componente radial de la
velocidad vr de la partcula desaparece y su
Aplicando la conservacin de la cantidad de
componente tangencial v ser (R + h), en donde
movimiento angular:
es la velocidad angular de la Tierra. Al soltarse,
debido a la atraccin gravitacional de la Tierra, la
Lantes = Ldespus partcula comienza a caer verticalmente haca abajo y,
por ende, su distancia radial r del centro de la Tierra
r1 comienza a disminuir. De L = mrv se deduce que
r1 Mv1 = r2 Mv 2 v 2 = v1 el componente tangencial de su velocidad v debe
r2
aumentar este proceso y de modo tal que haga que el
producto rv sea constante. En trminos ms
Tambin podemos hallar el trabajo realizado para cuantitativos, esto quiere decir que durante su
acortar el radio. descenso hacia el suelo, la distancia radial r y la
velocidad tangencial v se deben relacionar por
1 medio de
K antes = Mv12 ,
2 mrv = m ( R + h) 2 (1)
1 1 r
2
1 r
2
2 Puesto que, inicialmente, la velocidad de la partcula
K despus = Mv 22 = M v1 1 = M 1 v1 es (R + h), de tal modo que su cantidad de
2 2 r2 2 r2 movimiento angular L en relacin al centro de la
Tierra es m(R + h)2 Anticipndonos al hecho de que
El trabajo realizado es igual al cambio de energa la desviacin hacia el este ser muy pequea,
cintica. podemos escribir para la distancia radial r del cuerpo
al centro de la Tierra en cualquier instante t,
2
1 r 2 1 1 2
W = K = M 1 v1 Mv12 r = R+h gt (2)
2 r2 2 2

2

1 2 r Y al sustituir esto en (1) obtenemos:
= Mv1 1 1
2 r2

25
Sistema de partculas Hugo Medina Guzmn

( R + h) 2 ( Ro + h ) (3) CONSERVACION DE LA CANTIDAD DE


v = = MOVIMIENTO ANGULAR DE UN SISTEMA
1 1 2
R + h gt 2 1 2 gt /( R + h) DE PARTICULAS.
2
Vamos a considerar un sistema de dos partculas,
Para calcular la magnitud de la desviacin hacia el como se muestra en la figura.
este, sea vy en el instante t, la velocidad del cuerpo
que cae en la direccin hacia el este, tal y como lo ve
un observador fijo con respecto a la superficie de la
Tierra. Entonces
v y = v o r
( Ro + h ) 1 2
= (R + h gt )
1 2 2
1 2 gt /( R + h)
1 2
2 gt 1 2
= ( R + h) 1 + ( R + h) + gt
R + h 2

d L1
Para la partcula 1: 1 = , donde
= gt 2 dt
1 = r 1 F 1 = r 1 F 12 + F 1ext

Donde la segunda igualdad se obtiene de la
utilizacin de (2) y (3) y la tercera, a continuacin,
mediante el hecho de que gt2 <<(R + h) y el empleo
del teorema binomial. Al integrar esta frmula para vy
se obtiene, para la desviacin total hacia el este y en F1 Es la fuerza total sobre la partcula 1.
el instante t,
1 3
y= gt .
3 F12 Es la fuerza ejercida por la partcula 2 y

Finalmente, puesto que el tiempo que necesita la


1 F1 ext Es la suma de las fuerzas externas sobre la
partcula para caer la distancia h es de ( 2hg ) 2 , la
partcula 1.
deflexin total hacia el este d, se puede expresar
como sigue

d L1
2 2h 1 = = r1 F12 + r1 F1 ext = 12 + 1 ext
d = h (4) dt
3 g
Por ejemplo, si se deja caer una partcula desde una Similarmente para partcula 2.
altura de 100 metros, su desviacin hacia el este,
segn esta frmula, se descubre que es (al sustituir los
valores h = 100 metros y = 7,2 x 10-5rad/s) de 2,2 d L2
cm. Esta desviacin es muy pequea y slo se puede 2 = = r2 F21 + r2 F2 ext = 21 + 2 ext
observar en condiciones controladas cuidadosamente.
dt

Es importante recordar la base fsica para la deflexin


d L1 d L2 d Ltotal
pronosticada en (4). Conforme la partcula desciende Sumando 1 + 2 = + =
hacia la superficie de la Tierra su velocidad dt dt dt
tangencial v debe aumentar para que el producto rv
sea constante. Por consiguiente, de esto se desprende Para los Torques internos tenemos:
que su velocidad tangencial debe sobrepasar a la del
punto de la superficie que se encontraba inicial e

inmediatamente por debajo, y, en esta forma, se 12 + 21 = r1 F12 + r2 F21
desva hacia el este.


Como F21 = F12

26
Sistema de partculas Hugo Medina Guzmn

12 + 21 = r1 F12 r2 F12 = r1 r2 F12


Al estar sujetos los dos hombres a la cuerda su


movimiento es circular y si consideramos que el piso
est en el plano xy,

De la figura: r1 + r12 = r2 r1 r2 = r12 Tenemos:

Reemplazando l
L inicial = 2 pk = lpk ,

2
12 + 21 = r12 F12 l lp '
L final = 2 p' k = k
4 2

Como r12 y F12 son paralelos: r12 F12 = 0 , y
Como L inicial = L final p ' = 2 p

12 + 21 = 0 La cantidad de movimiento lineal final de cada
hombre es el doble de la cantidad de movimiento
lineal inicial.
d Ltotal d Ltotal
De aqu: 1 ext + 2 ext = y ext =
dt dt CONSTANTE, Independiente, del tiempo

Ejemplo 33. Una partcula de masa m1 se desplaza


Vemos si ext = 0 Ltotal = CONSTANTE ,
sobre un plano horizontal con velocidad v 1 . Dos
independiente del tiempo. partculas de masas m2 y m3 unidas por una varilla de

Ejemplo 32. Dos hombres se encuentran en una pista masa despreciable se mueven con velocidad v 2 ,
de patinaje, ambos sostienen una cuerda de longitud como se indica en la figura.
l . Qu pasa con la cantidad de movimiento lineal

p de cada uno de ellos, si ambos jalan la cuerda y


acortan la distancia entre ellos a l / 2 ? Asumir que
se mueven en crculo y que la magnitud de sus
cantidades de movimiento son iguales.

Solucin.

Las nicas fuerzas externas al sistema son la fuerza


de la gravedad y la reaccin normal del piso, estas Suponiendo un choque totalmente plstico entre m1 y
fuerzas se cancelan. Las nicas fuerzas que, m2
intervienen en el sistema son las internas, por lo tanto
la cantidad de movimiento angular del sistema se m m
conserva. m1 = m2 = m3 = 1 kg, v 1 = 20 i , v 2 = 10 i ,
s s
l = 1m

27
Sistema de partculas Hugo Medina Guzmn

Calcular: b) Como parten del reposo, la cantidad de


movimiento total del sistema es cero.
a) La posicin del centro de masa respecto a la masa
m2 en el momento del choque.
P total = M v cm = 0
b) La ley del movimiento del centro de masa.

d

c.) La velocidad angular de rotacin alrededor del


Como v cm = r cm r cm = constante.
dt
centro de masa despus del choque.
El centro de masa permanece en la misma posicin.
Solucin.
c) Consideremos la cantidad de movimiento angular
a) En el momento del choque, tomando como con respecto al centro de masa antes y despus del
referencia la posicin de m2, el centro de masa est en: choque.

Posicin en el instante partida (t = 0). Antes del choque.

Posicin en el instante de encuentro.



L = r 1cm p 1 + r 2cm p 2 + r 3cm p 3


2d 1
r 1cm = i j , p1 = m1 v 1 = 20i
3 3

1
r 2 cm = (d v2t )i j , p 2 = m2 v 2 = 10i
3

2
r 3cm = (d v2t )i + j , p 3 = m3 v 2 = 20i
3
2d 2d 2d
m1 + m2 + m3
3 3 3
xCM = Nota: En los vectores posicin ( r 1cm , r 2 cm , r 3cm )
m1 + m2 + m3 solo ponemos la posicin en el eje y, porque la
posicin en x se va a anular con el producto vectorial.

m(2d ) 2d Reemplazando:
= =
3m 3
1 1 2
L = j (20i ) + j ( 10i ) + j ( 10i )
3 3 3
m1 (0) + m2 (0) + m3 (l )
yCM = 20 10 20
m1 + m2 + m3 L= k k+ k = 10k
3 3 3
l 1
= = m Despus del choque:
3 3

28
Sistema de partculas Hugo Medina Guzmn

Como despus del choque el sistema gira alrededor



del centro de masa con velocidad angular ,
podemos expresar la cantidad de movimiento angular
en funcin del momento de inercia.


L' = I , I = I 1 + I 2 + I 3
Solucin.
2 2 2
l l 2l 2
I = m1 + m2 + m3 = a) La cantidad de movimiento con que se acerca el
3 3 3 3 muchacho es:

2
luego L' = L = r p = rmv senk
3

Por conservacin de la cantidad de movimiento

Como r sen = l 0 L = ml 0 vk
angular L' = L
b) Cuando el muchacho se coge del poste, las fuerzas
2 rad
de reaccin centrpeta e impulsiva deben pasar por el
10k = = 15k poste por lo tanto no ejercen ningn torque sobre el
3 s muchacho y la cantidad movimiento angular se
conserva.


L = ml 0 vk = constante

La energa cintica despus de cogerse del poste es


4
K '= K.
5

K es la energa cintica antes de cogerse,


1 2
Ejemplo 34. Un muchacho va corriendo por la acera K= mv
con una velocidad constante v con sus brazos 2
estirados perpendicularmente a su recorrido. La
distancia entre los extremos de los dedos de sus Luego:
manos es 2l 0 . Cuando al correr pasa junto a un
poste, se coge al mismo con la mano izquierda, 4 41 2
levanta los pies del suelo, y gira por aire alrededor del K'= K = mv 2 = mv 2 (1)
5 52 5
poste.

a) Si su masa es, M. Cul es el valor de su cantidad 1 2


Tambin K ' = I , como L = Ik
de movimiento angular respecto al poste cuando corre 2
por la acera?
L
=
b) Si la fuerza de reaccin del poste no s1o lo hace I
girar, sino que adems proporciona una fuerza
impulsiva que hace frenar ligeramente su movimiento Reemplazando ste valor de en K :
hacia adelante, de modo que su energa cintica se
reduce a los cuatro quintos de su valor original. Cul
L2 (ml 0 v )
2
es su momento de inercia respecto al poste? K'= = (2)
2I 2I

Igualando (1) y (2)

29
Sistema de partculas Hugo Medina Guzmn

2 2 (ml 0 v ) 5 2
2

mv = Luego su momento de inercia es I = ml 0


5 2I 4

PREGUNTAS Y PROBLEMAS

1. Una masa m1 se sita en (x1, y1, z1) y otra masa m2 5. Suponga que la fuerza que acta sobre una pelota
en (x2, y2, z2). de tenis (m = 0,060 kg) en funcin del tiempo est
a) Hallar 1a distancia r0 entre m1 y m2. dada por la grfica de la figura. Usando mtodos
b) Hallar la distancia r1 entre m1 y el centro de masa. grficos estime:
c) Hallar la distancia r2 entre m2 y et centro de masa. a) El impulso total dado a la bola.
Respuesta. a) b) La velocidad de sta despus de haber sido
golpeada, suponiendo que estaba en reposo en el
r0 = (x 2 x1 )2 + ( y 2 y1 )2 + (z 2 z1 )2 momento de ser golpeada.
m2 r0 m1 r0
b) r1 = , c) r2 =
(m1 + m2 ) (m1 + m2 )
2. Dos partculas de masas m1 = 1 kg y m2 = 3 kg se
mueven por el espacio, sus vectores posicin Son:

r1 = 3i + tj t k , r2 = sen t 2 i + k
a) Hallar el centro de masa.
b) Cul es su aceleracin?
c) Hallar su aceleracin vista por un observador que
Respuesta. a) 4,5 Ns b) 75 m/s
se mueve con velocidad constante v = j + 3k .
Respuesta. a)
6. Un flujo de partculas idnticas de masa m y

1
[( ) (
r CM = 3 + 3sen t 2 i + tj + 3t t k
4
)]
velocidad uniforme v , inciden sobre un plano fijo de
1 1 rea A, la direccin forma un ngulo con la normal.
( )

b) a CM = cos t + 2t sen t i +
2 2 2
tk Despus del choque las partculas tienen una
2 16
c) igual que b) velocidad v ' , la direccin es simtrica a la de v .

3. Encontrar el centro de masa de una lmina delgada Tambin v = v ' .
mostrada en la figura a) Calcular el Impulso que se ejerce sobre cada
partcula en el momento del choque.
b) Calcular el valor de la fuerza comunicada a la
superficie por unidad de tiempo. Siendo n el numero
de partculas por unidad de volumen de chorro
incidente.

Respuesta. a) J = (2v cos )n ,
b) F = 2nAmv cos
2 2

Respuesta. y = 0,983m encima del centro del 7. Un nadador de 70 kg se lanza al agua desde el
orificio podio de una piscina con una velocidad de 3m/s en la
direccin de la figura. Calcular la fuerza ejercida
sobre el podio durante los 0,8s que el nadador ejerce
4. Una fuerza F = ti + t j + t k acta sobre un el esfuerzo sobre el mismo para impulsarse en el salto.
2 3

cuerpo en el intervalo de 0 t 6 s . Hallar el


impulso sobre el cuerpo.
Respuesta. 181 + 72j + 324l 18i + 72 j + 324k

30
Sistema de partculas Hugo Medina Guzmn

( )

Respuesta. F = 227i 818 j N Encontrar la velocidad v del obs.
d) Deducir del clculo anterior el alcance R del obs.
8. Un recipiente de 0,25 kg con capacidad para 5 kg
de agua se llena de un cao en 5 s. En el instante en
que el recipiente est medio lleno, la balanza lee 3,0
kg. Si no se salpica el agua, Cul es la velocidad del
agua que cae en dicho instante.

Respuesta. 2,45 m/s


M V
Respuesta. a) v = ,
9. Una bala de fusil de masa m y de velocidad m cos
constante v 0 , penetra en un bloque de madera fijo; la M V
b) v ' = 1 + ,
bala se detiene despus de recorrer una distancia d m cos
con un movimiento uniformemente retardado. m
e) Calcular la desaceleracin a de la bala, deducir la c) u v = v' cos ,
fuerza de desaceleracin. m+M
b) Calcular el tiempo de desaceleracin.
c) Cul es el impulso comunicado a la bala por el v = (u + Mv' cos )i + v' senj
bloque? Comparar con la cantidad de movimiento de
2v 2 Mv 2
la bala antes del choque. d) R = sen u + cos
Realizar la aplicacin numrica para v 0 = 600 m/s, d g M +m
=30 cm, m = 40 g
11. Desde la plataforma de un tren que se mueve con
v2 5 m una velocidad de 10 m/s se arroja un paquete de 25
Respuesta. a) a = 0 = 6 10 2 ,
2d s kg, Este es cogido en el aire por una persona que est
7 junto a la va. Desde el tren se observa que esta
F = ma = 24 10 N ,
persona retrocede con una velocidad de 7,5 m/s.
v 3 Cul es la masa de la persona?
b) t = 0 = 10 s
a Respuesta. 75 kg.
c) J = F t = 24 N, J = mv0
12. Un muchacho est en medio de un lago
10. Un can fijo sobre un vagn que se puede congelado sin friccin de tal manera que no puede
desplazar si friccin sobre una va rectilnea moverse. Para poder salir l lanza su sombrero de
horizontal con una masa total M. El can forma un masa 0,5 kg hacia el norte con una velocidad de 12
ngulo con la horizontal. m/s a 53 con la horizontal. Si la masa del muchacho
a) Si el vagn est en reposo el can dispara un obs es 60 kg y el radio del lago es 400 metros. Qu
de masa m, determinar la relacin entre las pasa7
velocidades v y V del obs y del can. Respuesta. El muchacho resbala hacia el sur y llega a
b) Si la velocidad del obs relativa al can es v la orilla 1 h 51 min despus.
(forma un ngulo con V ), determinar la relacin
13. Un paquete de 10 kg se descarga de una cinta
entre v ' y V . transportadora con una velocidad de 3 m/s y cae en
c) El vagn se desplaza con una velocidad rectilnea una vagoneta de 25 kg. Si la vagoneta est
constante u sobre la va antes del disparo. El obs inicialmente en reposo y puede rodar libremente,
tiene una velocidad v relativa al can en movimiento Cul es su velocidad final?
a la velocidad V despus del disparo:
Calcular la variacin de la velocidad (u - v) del
vagn

31
Sistema de partculas Hugo Medina Guzmn

Respuesta. a) xCM = - 3 1/3 m,



b) p1 = 25 i kg.m/s, p 2 = 0 , pCM = 25 i kg.m/s

c) v = 1 2/3 i m/s

Respuesta. v = 0,732 i m/s 17. Dos bolas P1 y P2 de masas m1 y m2 estn


suspendidas del cielorraso por dos hilos inextensibles
14. Un hombre de 75 kg se lanza al agua desde la de la misma longitud l ; P1 y P2 estn en contacto sin
proa de su bote de 50 kg. La componente horizontal presin con los hilos verticales.
de su movimiento es 1 m/s respecto al bote. Hallar las Se saca P1 de la posicin de equilibrio a un ngulo
velocidades del hombre y del bote respecto a un
observador en el muelle.
0 manteniendo el hilo tenso, luego se suelta sobre
a) Si el bote est inicialmente en reposo P2.
b) Si el bote se mova inicialmente hacia adelante con Calcular:
una velocidad de 2 m/s. No considerar prdidas de a) La velocidad de P1 justo antes del choque.
energa debido al agua b) Las velocidades v1 y v2 de P1 y P2
Respuesta. a) v1 = 0,4 m/s , v2 = 0,6 m/s inmediatamente despus del choque perfectamente
b) v1 = 2,4 m/s , v2 = 1,4 m/s elstico. Discutir este resultado para valores relativos
de las masas m1 y m2.
c) Las alturas de las posiciones limites de P1 y P2
15. Un can dispara un obs de 2,4 kg hacia arriba. despus del choque.
A1canza su mxima altura, 313,6 m y se parte en dos, Aplicacin numrica; l = l m . 0 = 60, m2 = m1/2
0,8 kg y 1,6kg. Las dos partes llegan a tierra
simultneamente. La pieza de 1,6 kg toca tierra a 480 Respuesta. a) v1 = 2 gl cos 0 , v1 = 3,13 m / s ,
m de la explosin (medida a lo largo del eje x).
m1 m2
a) Cunto tiempo tomara al obs volver a tierra si b) v '1 = v1 , v'1 = 1,05 m / s
no se hubiera partido? m1 + m2
b) Cul es la velocidad de cada una de las piezas
justamente despus de la explosin?
Para m1 > m 2 v1 y v'1 tienen el mismo sentido.
c) Encontrar la cantidad de movimiento de cada pieza
justamente antes de tocar tierra. Para m 2 > m1 v'1 tiene sentido contrario de v1 .
Respuesta. a) 8 segundos
2m1v1
v' 2 = , v '2 = 4,22 m / s
b) v (1, 6 ) = 60i m/s (16), v (0,8 ) = 120i m/s m1 + m2

c) p (1, 6 ) = 96i 125,44 j kg.m/s ,
v' 2 en todo caso tiene el mismo sentido que v1
v'12 v '2
c) h1 = = 0,056 m , h2 = 2 = 0,91 m
p (0,8 ) = 96i 62,72 j kg.m/s 2g 2g
(El movimiento es en el plano xy; g = 9,8 m/s2)
18. En un Juego de billar, la bola A est movindose
16. Un bloque de masa 10 kg est en reposo en el
origen segundo con masa 5 kg se mueve a lo largo del con 1a velocidad v 0 = 31 m/s cuando choca con las
eje x con velocidad de magnitud v 0 = 5 m/s. Los bolas B y C que estn juntas en reposo. Tras el
bloques choca quedan unidos. y se mueven en el eje x. choque, se observan la tres bolas movindose en las
La superficie tiene friccin despreciable. direcciones que muestra 1a figura, con = 30.
a) Cuando el bloque de 5 kg est en x = -10 donde Suponiendo Superficies lisas y choques
est centro de masa? perfectamente elsticos, hallar los mdulos de la
b) Encontrar la cantidad de movimiento de la masa de
5 kg, de la masa de 10kg y del centro de masa antes velocidades, v A , v B y vC .
del choque.
c) Cul es la velocidad del sistema combinado?

32
Sistema de partculas Hugo Medina Guzmn


Respuesta. v A = 1,5 m/s , v B = 1,29 m/s , vC 2,25 Respuesta. a) 0,943 m, b) 0,711 m, 0,37 m
m/s
23. Un objeto de 5 kg que se mueve con una
velocidad de 1,2 m/s choca directamente con un
19. Se dispara una bala de 39 g con una velocidad de
objeto de 20 kg que est en reposo. Se observa que el
500 m/ contra un bloque A de 5 kg de El coeficiente
objeto menor rebota con una velocidad de 0,6 m/s
de rozamiento entre el bloque A y la plataforma es
a) Cul es la prdida de energa cintica por el
0,5. Si la masa de la plataforma es 4 kg y puede rodar
impacto?
libremente, hallar:
b) Cul es el coeficiente de restitucin?
a) La velocidad final de la plataforma y e1 bloque.
Respuesta: a) K = - 0,675 J, b) = 0,875
b) La posicin final del bloque sobre la plataforma.
24. Una bola choca contra un plano liso formando un
ngulo 1 con la normal del mismo y rebota con un
ngulo 2 . Encontrar La expresin correspondiente
al coeficiente de restitucin

Respuesta. a) 2,16 m/s b) El bloque se detiene a


0,33 m de B.

20. La figura muestra dos masas sobre una


superficie con rozamiento despreciable. El coeficiente
de restitucin entre Las dos masas es 0,73; determinar:
a) Sus velocidades despus del choque.
b) La prdida de energa durante el choque.
tan 1
Respuesta. =
tan 2

25. Ira partcula de masa m1 tiene un choque frontal


perfectamente elstico con una partcula de masa m 2

Respuesta: a) v A = - 0,563 i m/s, v B = 6,94 i inicialmente en reposo. Cul es la prdida relativa de
energa cintica correspondiente a la partcula . m1
m/s
b) K = 41 J
K m1
Respuesta. =4 2
K m
21. Se deja caer una pelota al suelo desde una altura m2 1 + 1
y. Si el coeficiente de restitucin es , escribir m2
expresiones para el tiempo total que tardar la pelota
en dejar de dar bote y la distancia total que recorrer 26. Una masa m1 se mueve a lo largo del eje x con
en este tiempo. una velocidad v 0 a lo largo de una mesa sin friccin.

Respuesta. t =
2 y (1 + )
, s= y
(
1+ 2 ) Choca con otra nasa, la cual est inicialmente en
g (1 ) (
1 2 ) reposo. La masa m2 sale a lo largo del eje y. Si se
pierde la mitad de la energa cintica original en el
choque.
22. Una bola cae desde una altura h = 0,900 m sobre Cual es el mdulo de la velocidad y con que ngulo
una superficie lisa. Si la altura del primer rebote es h1 sale despus de la colisin?
= 0,800 m y la distancia d1 = 0,400 m, calcular:
a) El coeficiente de restitucin.
b) La altura y longitud del segundo rebote.

33
Sistema de partculas Hugo Medina Guzmn

3 v ' B v ' A v ' v '


Respuesta. v 2 = m1v 0 , = 0,9 B A = 0,9
(2
2 m2 + m1 m2 ) vA 3,43
1 v' B v' A = 0,9(3,43) = 3,09 (2)
mv 2 (m1 + m2 ) 2 Sumando (1) y (2):
= cos 1 1 0 = cos 1
m2 v2 3 m2 v' B +1,6v' B = 3,09 + 3,43
La velocidad de B inmediatamente despus del
27. Se deja en libertad el bloque A cuando = 90 y choque es
desliza sin rozamiento, hasta chocar con la bola B. Si v' B = 2,51 m/s
el coeficiente de restitucin es 0,90, calcular b) El diagrama del cuerpo libre de B, inmediatamente
a) La velocidad de B inmediatamente despus del despus del choque
choque.
b) La mxima traccin que soporta el hilo que
sostiene a B
c) La altura mxima a la que se eleva B.

v' 2B
Fr = mB ac T mB g = mB 0,9

v' 2
T = m B B + g
0,9
2,51 2

= 2 + 9,8 = 33,6 N
0,9
Solucin. c) Por conservacin de energa encontramos la altura
a) Por conservacin de energa encontraremos v A . mxima a la que se eleva B.

.
1
m A gr = m A v A2
2 1
m B v' 2B = m B gh
v A = 2 gr = 2(9,8)(0,6 ) 2
= 3,43 m/s v' 2 2,512
Por conservacin de la cantidad de movimiento h= B = = 0,321 m
2 g 2(9,8)
encontraremos v ' B
28. Un bloque de masa M est en reposo sobre una
masa sin friccin. Lo podemos golpear con un bloque
que se quede adherido o con un bloque muy duro con
el que se producir un choque perfectamente elstico.
Ambos bloques tienen masa m y pueden ser lanzados
can velocidad V0 En cul de los casos el bloque M se
v A = 3,43 m/s v B = 0 mover ms rpidamente? (considerar el movimiento
m A v A + m B v B = m A v' A + m B v' B en una sola dimensin).
(1,25)(3,43) = 1,25v' A +2v B Respuesta. a) v ' 2 =
m
V0 ,
v' A +1,6v' B = 3,43 (1)
m+M
2m
El coeficiente de restitucin es 0,90 b) v ' 2 = V0
(v' 2 v'1 ) v' B v' A m+M
= = = 0,9 En el segundo caso es el doble que en el primero.
(v 2 v1 ) vA

34
Sistema de partculas Hugo Medina Guzmn

29. Un cilindro A cae sobre otro B apoyado sobre un potencial del resorte comprimido es de 60 J y el
resorte de constante k = 3000 N/m desde una altura de
2m. Si el choque es perfectamente plstico, calcular: conjunto posee la velocidad inicial v 0 Si se rompe la
a) El desplazamiento mximo de B. cuerda cuando = 30 , hallar la velocidad
b) La prdida de energa en el choque. resultante de cada partcula

Respuesta. a) 3,47 cm , b) 8,18 J



30. Los parachoques se disean de tal manera que un Respuesta. v A = 9i + 5,2 j m/s y
automvil de 1600 g que golpea una pared rgida a la
velocidad de 12 km/h no sufra dao. Suponiendo que v B = 4i 3,5 j m/s
ese choque es perfectamente plstico. Calcular:
a) La energa absorbida por el parachoques durante el 34. Un depsito suelta arena a una banda
impacto. transportadora razn de 75 kg/s. Si la banda se mueve
b) La velocidad a la que el automvil puede golpear a con una rapidez constate v = 2,2 m/s. Qu fuerza se
otro de iguales caractersticas, que est en reposo sin necesita para mantenerla en movimiento? No tomar
daarse. en cuenta la friccin
Respuesta. a) 8890 J b) 24 km/h

31. Se dispara una bala de 25g en direccin


horizontal. La bala atraviesa el bloque A y queda
alojada dentro de bloque B. Por dicha causa los
bloques A y B comienzan a moverse con velocidades
iniciales de 2,4 y 1.8 m/s. respectivamente.
Hallar: Respuesta. 165 N
a) La velocidad inicial v 0 de la bala. 35. Un trineo lleno de arena se desliza sin friccin
b) La velocidad de la bala en el trayecto entre el por una pendiente de 30. La arena se escapa por un
bloque A y el B. agujero en el trineo a un ritmo de 2 kg/s. Si el trineo
parte del reposo con una masa inicial de 40 kg.
Cunto tard en recorrer 120m a lo largo de la
pendiente?
Respuesta. 7 segundos

36. Un cohete que consume combustible a un ritmo


constante k se encuentra sometido a la accin de una
fuerza externa constante de valor F adems de la
fuerza de reaccin de los gases. La masa inicial del
cohete ms combustible es m0 . La configuracin de
Respuesta. a) 470i m s b) 3261 m/s la tobera de escape es de tal manera que la velocidad
relativa de los gases es igual al negativo de la
32. Una explosin rompe un objeto en dos piezas una velocidad v del cohete.
de las cuales tiene 1,5 veces la masa de la otra. Si se a) Escribir la ecuacin del movimiento.
liberan 4500 J en la explosin. Cunta energa b) Obtener v (t ) .
cintica adquiere cada pedazo?
Respuesta. a) F = (m0 kt )
Respuesta. 1800 J, 2700 J dv
kv ,
dt
33. Cuando se rompe la cuerda que une las partculas
A y B, el resorte comprimido las obliga a separarse
(el resorte no est unido a las partculas). La energa

35
Sistema de partculas Hugo Medina Guzmn

Ft b) Las velocidades de A y B cuando la varilla ha


b) v (t ) = girado 90.
m0 c) Las velocidades de A y B cuando la varilla ha
girado 180.
37. Un cohete experimental se proyecta de forma que
pueda mantenerse inmvil sobre el suelo. El cuerpo
del cohete tiene una masa de 1200 kg y la carga de
combustible inicial es de 3600 kg,. e1 combustible se
quema y se expulsa con una velocidad de 2500 m/s.
Hallar la velocidad de consumo de combustible
necesario.
a) en el momento de encender el cohete.
b) cuando se consume la ltima partcula de
combustible.

3
Respuesta: a) p = mv 0 i , L = mlv 0 k
4
1 3 1 1
b) v A = v 0 i + v 0 j , v B = v 0 i v 0 j
4 4 4 4
1 1
c) v A = v 0 i , v B = v 0 i
2 2
Respuesta. a) 18,84 kg/s . b) 4.71 kg/s 4l. Se tiene una varilla rgida de masa despreciable
sujeta a un eje sin rozamiento de tal manera que la
38. Una bala de masa m se dispara con una velocidad varilla pueda rotar libremente. Al otro extremo de la
varilla hay un bloque de masa M. Si se dispara una
v B = v B i , si para x = x 0 , y = a (permanece bala de masa m con una velocidad v 0 tal como se
constante) Cul es su cantidad de movimiento muestra en la figura. Si la bala se incrusta en el
angular con respecto al origen en funcin de x? bloque, cul ser la velocidad angular del bloque

Respuesta. L = mv0 ak alrededor del eje7

39. Un obs de masa m se dispara de un can en el


origen, El obs se mueve en el plano y con una
velocidad inicial de magnitud v 0 y un ngulo con
el eje x.
a) Cul es el torque sobre el obs, con respecto al
origen en funcin del tiempo?
b) Cul es la cantidad de movimiento angular del,
obs con respecto al origen en funcin del tiempo?
mv0
Respuesta. =
c) Comprobar que =
dL
(m + M )a
dt

Respuesta. a) = v0 mgt cos i 42. Una barra de longitud b est pivotada en su
1 centro de tal manera que puede rotar en el plano
b) L = v 0 mgt cos i
2
horizontal. Dos nios estn sobre la barra en las
2
posiciones mostradas en la figura 7.59. a cual est
rotando con una velocidad angular en el sentido
40. Dos esferas pequeas A y B estn unidas por una
antihorario visto desde arriba. Si el nio de masa m1
varilla rgida de longitud l y masa despreciable. Las empieza a moverse hacia el centro tal que su distancia
dos masas reposan sobre una superficie lisa 2
horizontal cuando se comunica repentinamente a A la a el es b 4 at , Cul debe ser el movimiento del
nio de masa m2 para que la velocidad angular de la
velocidad v 0 = v 0 i . Hallar: a) La cantidad de barra permanezca constante? (La masa de la barra es
movimiento lineal y la cantidad de movimiento despreciable),
angular del sistema respecto al centro de masa.

36
Sistema de partculas Hugo Medina Guzmn

49. Un cuerpo de masa m1 = 2 kg se desliza sobre


una mesa horizontal sin friccin con una velocidad
inicial v1i = 10 m/s, frente a l movindose en la
misma direccin y sentido se encuentre el cuerpo de
masa m2 = 5 kg cuya velocidad inicial es v2i = 3 m/s,
ste tiene adosado un resorte en su parte posterior,
cuya constante elstica es k = 1120 N/m, cul ser la
mxima compresin del resorte cuando los cuerpos
Respuesta. Debe cambiar su distancia al centro de choquen?.
Respuesta. x = 0,28 m
b 2 m1 2 b
acuerdo a la ecuacin + at at 2
16 m2 2 50. Un bloque de 3,0 kilogramos, movindose sobre
una superficie sin friccin con una velocidad de 1,2
m/s, tiene una colisin perfectamente elstica con un
43. Un taco golpea a una bola de billar ejerciendo bloque de la masa M en el reposo. Despus de la
una fuerza promedio de 50 N durante un tiempo de colisin el bloque de 3,0 kilogramos retrocede con
0,01 s, si la bola tiene una masa de 0,2 kg, qu una velocidad de 0,4 m/s.
velocidad adquiri la bola luego del impacto?.
Respuesta. vf = 2,5 m/s

44. Una fuerza acta sobre un objeto de 10 kg


aumentando uniformemente desde 0 hasta 50 N en 4
s. Cul es la velocidad final del objeto si parti del a) La masa M es:
reposo?. b) La velocidad del bloque de masa M despus de la
colisin es:
Respuesta. vf = 10 m/s
c) Los bloques estn en el contacto para 0,20 s. La
fuerza media en el bloque de 3,0 kilogramos,
45. Se roca una pared con agua empleando una mientras los dos bloques estn en contacto, es:
manguera, la velocidad del chorro de agua es de 5 Respuesta. a) 6,0 kg, b) 0,8 m/ s, c) 24 N
m/s, su caudal es de 300 cm3/s, si la densidad del
agua es de 1 g/cm3 y se supone que el agua no rebota 51. El bloque de 8 kilogramos tiene una velocidad v y
hacia atrs, cul es la fuerza promedio que el chorro es detrs del bloque de 12 kilogramos que tiene una
de agua ejerce sobre la pared?. velocidad de 0,5 m/s. la superficie es de friccin
Respuesta. F = 1,5N despreciable. Los bloques chocan y se juntan.
Despus de la colisin, los bloques tienen una
46. Se dispara horizontalmente una bala de 0,0045
velocidad comn de 0,9 m/s.
kg de masa sobre un bloque de 1,8 kg de masa que
est en reposo sobre una superficie horizontal, luego
del impacto el bloque se desplaza 1,8 m y la bala se
detiene en l. Si el coeficiente de rozamiento cintico
entre el bloque y la superficie es de 0,2, cul era la
velocidad inicial de la bala?.
a) La prdida de energa cintica de los bloques
Respuesta. v1i = 1073 m/s debido a la colisin est la ms cercana a:

47. Se dispara una bala de 0,01 kg de masa contra un b) El impulso sobre el bloque de12 kg debido a la
pndulo balstico de 2 kg de masa, la bala se incrusta colisin es
en el pndulo y ste se eleva 0,12 m medidos Respuesta. a) 2,4 J, b) 4,8 N s
verticalmente, cul era la velocidad inicial de la
bala?. 52. Una bola de acero de 72 g se lanza desde el
reposo y cae verticalmente sobre una placa de acero.
Respuesta. v1i = 309,8 m/s
La bola golpes la placa y est en contacto con ella por
0,5 ms, la bola elsticamente, y vuelve a su altura
48. Una partcula A de masa mA se encuentra sujeta original. El intervalo de tiempo para el viaje es 0,30
por medio de un resorte comprimido a la partcula B s.
de masa 2mA, si la energa almacenada en el resorte a) La fuerza promedio ejercida sobre la bola durante
es de 60 J qu energa cintica adquirir cada el contacto es
partcula luego de liberarlas?. b) Asumiendo que la placa no se deforma durante el
Respuesta. Ec Bf = 20 J contacto. La energa elstica mxima almacenada por
la bola es:
Respuesta. a) 420 N, b) 0,08 J

37
Sistema de partculas Hugo Medina Guzmn

53. Una bala de la masa 0,01 kilogramos que se 56. En una demostracin una bola de acero pequea
mueve horizontalmente golpea un bloque de madera de la masa m se sostiene sobre una superbola de masa
de masa 1,5 kilogramos suspendida como pndulo. M (superbola es una bola de goma del coeficiente de
La bala se aloja en la madera, y juntos giran hacia restitucin muy alto). La combinacin junta se suelta
arriba una distancia de 0,40 m. cul era la velocidad del reposo. Cuando el superbola golpea el piso rebota
de la bala momentos antes del impacto con el bloque casi elsticamente, golpeando a bola de acero que
de madera? La longitud de la cuerda es 2 metros. todava est movindose hacia abajo. Esta colisin es
tambin bastante elstica, y consecuentemente bola
de acero se golpea y es lanzada derecho hasta una
altura H. Si h es la altura de la cual los objetos fueron
soltados, y M > > m, entonces bola de acero pequea
se levantar a una altura:
Respuesta. 9 h

57. Una muchacha de masa 50 kilogramos lanza una


bola de la masa 0,1 kilogramos contra una pared. La
bola golpea la pared horizontalmente con una
velocidad de 20 m/s, y rebota con esta misma
velocidad. La bola est en contacto con la pared
Respuesta. 66,7m/s 0,05 s, cul es la fuerza media ejercida sobre la bola
por la pared?
54. Una bala de 10 g se dispara verticalmente en un Respuesta. 80N
bloque de 8 kilogramos. El bloque se levanta 3 mm.
La bala penetra en el bloque en un intervalo de 58. La bola A, de la masa 3,0 kilogramos, se une a
tiempo de 0,001 s. asume que la fuerza en la bala es una barra ligera de 0,4 m, que gira libremente en P.
constante durante la penetracin. La bola B est suspendida de Q por una cuerda de 0,6
m y est en reposo. La bola A se levanta a cierto
nivel y se suelta. La bola A desciende, y tiene una
velocidad v1 = 3,6 m/s en el fondo, antes de chocar a
la bola B. Las velocidades de las bolas A y B despus
del choque son: v2 = - 1,2m/s y v3 =2,2 m/s...

a) La energa cintica inicial de la bala es:


b) El impulse en el bloque debido a la captura de la
bala es:
c) La penetracin de la bala en el bloque, es:
Respuesta. a) 190 J, b) 2,0 Ns, c)) 10 cm.

55. Una bala de 8 g se tira en un bloque de 4,0


a) La masa de la bola B es:
kilogramos, en reposo sobre una superficie horizontal
b) La magnitud del impulso sobre la bola A es:
sin friccin. La bala se aloja en el bloque. El bloque
c) La bola A rebota y gira un ngulo , donde la
se mueve hacia el resorte y lo comprime 3,0
centmetros. La constante de la fuerza del resorte es velocidad v4 es cero. El valor de es:
1500 N/m. d) La bola B se eleva hasta la altura h, donde la
velocidad v5 es cero. El valor de h es:
Respuesta. a) 6,6 kg, b) 14.4 N. s, c) 35 d) 0,25 m

59. Una pieza en forma de L se corta de una hoja


uniforme de metal. Cul de los puntos indicados es
a) La velocidad de la bala es: el ms cercano al centro de la masa del objeto?
b) ) El impulso del bloque (con la bala), debido al
resorte, durante el tiempo en el cual el bloque y el
resorte estn en contacto est es:
Respuesta. a) 290 m/s, b) 4,7 N.s

38
Sistema de partculas Hugo Medina Guzmn

Respuesta. Completamente inelstico

65. Las masas de los bloques, y las velocidades antes


y despus del choque estn dadas. Qu clase de
choque es?

Respuesta. C
Respuesta. Parcialmente inelstico
60. Las masas de los bloques, y las velocidades antes
y despus del choque estn dadas. Qu clase de
66. Las masas de los bloques, y las velocidades antes
choque es?
y despus del choque estn dadas. Qu clase de
choque es?

Respuesta. Parcialmente inelstico.

Respuesta. Parcialmente inelstico


61. Las masas de los bloques, y las velocidades antes
y despus del choque estn dadas. Qu clase de 67. Un resorte activa una bomba de juguete de 1,2 kg
choque es? sobre una superficie lisa a lo largo del eje x con una
velocidad de 0,50 m/s. en el origen O, la bomba
estalla en dos fragmentos. El fragmento 1 tiene una
masa de 0,4 kilogramos y una velocidad de 0,9 m/s a
lo largo del eje y negativo.

Respuesta. Perfectamente elstico.

62. Las masas de los bloques, y las velocidades antes a) La componente en x de la cantidad de movimiento
y despus del choque estn dadas. Qu clase de del fragmento 2 debido a la explosin es:
choque es? b) El ngulo , formado por el vector velocidad del
Respuesta. no posible porque la cantidad de fragmento 2 y el eje x es:
movimiento no se conserva. c) La energa liberada por la explosin es:
Respuesta. a) 0., N. s, b) 31, c) 0,32 J
63. Las masas de los bloques, y las velocidades antes
y despus del choque estn dadas. Qu clase de 68. Un cono trunco homogneo de metal tiene una
choque es? base circular mayor de radio 4 cm y la menor de radio
2 cm. Su altura es 6 cm. A qu distancia de su
dimetro mayor est situado el centro de masa?

Respuesta. Caracterizado por un incremento en


energa cintica.

64. Las masas de los bloques, y las velocidades antes


y despus del choque estn dadas. Qu clase de
choque es?
Respuesta. 2,36 cm

69. Cuatro masas puntuales se colocan como se


muestra en la figura: Cules son las coordenadas del
centro de masa?

39
Sistema de partculas Hugo Medina Guzmn

72. Un carro de 19 kg est conectado por medio de un


resorte comprimido con un carro 38 kg. Los dos
carros se estn moviendo a la derecha a una velocidad
de 25 m/s cuando el resorte se desenrolla y propulsa
repentinamente el carro de 19 kg hacia adelante con
una velocidad de 27 m/s. encontrar la velocidad del
segundo carro con respecto al centro de la masa del
sistema.
Respuesta. (23, 2,8)

70. Un alambre uniforme de longitud 60 cm y masa


60 g , est doblado en un tringulo rectngulo. Respuesta. 1 m/s
Cules son las coordenadas del centro de masa? 7
3. Una fuerza de 5,3 N es necesaria para sujetar a un
paraguas en un viento fuerte. Si las molculas del
aire tienen una masa de 4,7 x 10-26 kilogramos, y cada
una golpea al paraguas (sin rebotar) con una
velocidad de 2,0 m/s en la misma direccin, cuntos
tomos golpean al paraguas cada segundo? Asuma
que el viento sopla horizontalmente para no tomar en
cuenta la gravedad.
Respuesta. 5,6 x 1025 por Segundo
Respuesta. (10, 3)
74. Un cohete debe ser lanzado al espacio donde no
71. Una partcula de la masa 5,01 x 10-27 kilogramos, hay campo gravitacional. el 81% de la masa inicial
movindose a 1,88 x 105 m/s, choca con una del cohete es combustible y este combustible se
partcula idntica que inicialmente est en el reposo. expulsa con una velocidad relativa de 2300 m/s. si se
Despus de la interaccin, las partculas (que no asume que todo el combustible ser utilizado,
pueden ser distinguidas) se mueven con los ngulos encuentra la velocidad final de la ltima porcin de
55,4 y 34,6, ambos son medidos con respecto a la combustible expulsado relativo a un observador
direccin original del movimiento. Qu velocidades estacionario.
finales tienen las partculas? Respuesta. 1500 m/ s
Respuesta. 1,55 x 105 m/s a 346,
1,07 x 105 m/s a 55,4

40
Cuerpo rgido Hugo Medina Guzmn

CAPTULO 7. Cuerpo rgido


INTRODUCCION La suma de las fuerzas que actan sobre las n
En el capitulo anterior estudiamos el movimiento de partculas determinan la aceleracin del centro de
un sistema de partculas. Un caso especial masa.
importante de estos sistemas es aquel en que la
distancia entre dos partculas cualesquiera
aCM =
Fi
permanece constante en el tiempo, esto es un
M
CUERPO RIGIDO.
Tal como se mostr para un sistema de partculas,
A pesar que no existen cuerpos que sean
las fuerzas internas se anulan de pares, de forma
estrictamente rgidos, todos los cuerpos pueden ser
que solamente importarn las fuerzas externas tal
deformados, sin embargo el modelo del cuerpo
que
rgido es til en muchos casos en que la
deformacin es despreciable. M aCM = Fext
La descripcin cinemtica y dinmica de un cuerpo
extenso aunque este sea rgido en un movimiento en
tres dimensiones matemticamente es muy El movimiento de traslacin del cuerpo rgido es
complejo y es tratado en libros avanzados de como si toda su masa estuviera concentrada en el
dinmica. Es complejo porque un cuerpo tiene seis centro de masa y las fuerzas externas actuaran sobre
grados de libertad; su movimiento involucra l.
traslacin a lo largo de tres ejes perpendiculares y Todo el estudio que hemos lecho anteriormente para
rotacin alrededor de cada uno de estos ejes. No la partcula corresponde a la traslacin de un cuerpo
llegaremos a hacer un tratamiento general directo, rgido. No importa ni la forma, ni el tamao.
pero si desarrollaremos el movimiento del cuerpo
rgido en dos dimensiones. ROTACIN.
Es el movimiento en que uno de los puntos se
MOVIMIENTO DE UN CUERPO RGIDO considera fijo.
En esta parte expondremos algunos tipos de S se considera fijo un punto, el nico movimiento
movimiento de los cuerpos rgidos. posible es aquel en el que cada uno de los otros
puntos se mueve en la superficie de una esfera cuyo
TRASLACION. radio es la distancia del punto mvil al punto fijo.
Por traslacin entendemos al movimiento en el que Si se consideran dos puntos fijos, el nico
lodos los puntos del cuerpo se mueven en la misma movimiento posible es aquel en que todos los
direccin, con la misma velocidad y la misma puntos con excepcin de aquellos que se encuentran
aceleracin en cada instante. sobre la lnea que une los dos puntos fijos, conocida
como EJE, se mueven en circunferencias alrededor
de ste.

Por la definicin de centro de masa, tenemos:




rCM =
mi ri =
mi ri
m i M
Donde M es la masa total del cuerpo rgido y Cualquier desplazamiento de un cuerpo rgido
puede ser considerado como una combinacin de
M rCM = mi ri traslacin y rotacin.

Diferenciando dos veces


d2 d2
M 2 rCM = mi 2 ri
dt dt

M aCM = mi ai = Fi En los captulos anteriores ya hemos profundizado
bastante sobre movimiento de traslacin

1
Cuerpo rgido Hugo Medina Guzmn

estudiaremos aqu el movimiento de rotacin I) El teorema de Steiner o de los ejes paralelos.


alrededor de un eje y el movimiento de rotacin El momento de inercia del cuerpo respecto a un eje
traslacin. es igual al momento de inercia del cuerpo respecto a
un eje paralelo al anterior y que pasa por el centro
CANT1DAD DE MOVIMIENTO ANGULAR de masa es el producto de la masa del cuerpo por el
DE UN CUERPO RGIDO cuadrado de la distancia entre los ejes.
La cantidad de movimiento angular de una partcula I 0 = I CM + Md 2
respecto a un punto es
Demostracin. La figura siguiente representa la
L = r p = r m v seccin de un cuerpo en el plano del papel, CM es
En coordenadas polares: el eje normal al plano del papel a travs del centro
de masa y O es un eje paralelo. Escogiendo un
dr
r = rr , v = r + r&t& elemento diferencial de masa dm , escribamos la
dt expresin para los momentos de inercia con

dr respecto a los dos ejes.


L = rr m r + rt
dt

L = mr 2 r t

r t tiene la direccin y sentido de

L = mr 2 I CM = rCM
2
dm I 0 = r 2 dm
M M
Si consideramos al cuerpo rgido como n partculas
que giran alrededor de un eje, la cantidad de usando la ley de los cosenos, obtenemos:
movimiento angular de ste ser la suma de la r 2 = rCM
2
+ d 2 2rCM d cos
cantidad de movimiento angular de cada una de las reemplazando
partculas.
I 0 = rCM
2
(
+ d 2 2rCM d cos dm )
L total = m r + m r + ........ + m r
1 1
2
2 2
2
n n
2 M

I 0 = rCM
2
dm + d 2 dm 2d rCM cosdm
( )

2 2 2 M M M
= m r + m r + ........ + m r
1 1 2 2 n n El primer trmino

n 2
rCM dm = I CM
=

m r i i
2


M
i =1 El segundo trmino
La cantidad entre parntesis es el MOMENTO DE d 2 dm = Md 2
INERCIA DEL CUERPO RGIDO alrededor de un M
eje. El tercer trmino es cero porque es la suma en todo
n el cuerpo d los productos del elemento de masa y
I = mi ri 2 sus distancias al eje a travs del centro de masa, de
i =1 aqu:
Es importante darse cuenta que el momento de I 0 = I CM + Md 2
inercia depende de la distribucin de la masa del
cuerpo.
En el caso de un cuerpo rgido continuo, II. El teorema de la figura plana.
El momento de inercia de una figura plana con
los mi tienden a dm y respecto a un eje perpendicular a la misma es igual
se transforma en
M
, de aqu: a la suma de los momentos de inercia de la figura
plana con respecto a dos ejes rectangulares en el
I = r 2 dm plano de la figura los cuales se intersecan con el eje
M dado
Como m = V , donde es la densidad y V el Demostracin:
volumen del cuerpo: En la figura siguiente el eje z pasa por O
perpendicular al piano y. Elegimos un elemento
dm = dV diferencial de masa dm y escribimos los momentos
Tenemos: I = V
r 2 dV de inercia de la figura para cada uno de los tres ejes.
Para muchos cuerpos de forma geomtrica simple
sta integral puede evaluarse fcilmente.

Dos teoremas que simplifican los clculos del


momento de inercia son:

2
Cuerpo rgido Hugo Medina Guzmn

2 2
= 7 mb + 5ma
Aqu comprobamos
Iz = Ix + Iy

b) Momento de inercia de una varilla delgada rgida


de longitud l y masa m, con respecto a un extremo
y con respecto al centro de masa.
Solucin.

I x = y 2 dm , I y = x 2 dm , I z = r 2 dm
M M M
2 2 2
con r =x +y
r dm = (x + y )dm
2 2 2
M M
Tomemos un elemento diferencial dx, cuya masa es:
= x dm + y dm
2 2
M M M
dm = dx
Iz = Ix + Iy l
El momento de Inercia de la varilla es:
Ejemplo 1. A continuacin evaluaremos los l M
momentos de inercia algunos cuerpos simples. I O = x 2 dm = x 2 dx
M 0 l
a) Hallar el momento de inercia del sistema
mostrado en la figura, las masas son puntuales
unidas por varillas rgidas de masa despreciable.
=
M l 2

l 0
x dx =
M 3
3l
x [ ] l
0

1 3
= Ml
3
El momento de inercia de la varilla con respecto al
centro de masa

Solucin.
Momento de inercia respecto al eje x.

[ ]
l l
M M 3
I x = y mi I CM = 2
2 2
x dx = x 2
3l
i l l

2
l
2
1
= m(0 ) + 2m(0 ) + 3m(b ) + 4m(b )
2 2 2 2
= Ml 3
12
2 Aqu comprobamos:
= 7 mb 2
Momento de inercia respecto al eje y. l
I O = I CM + M
2
I y = xi2 mi c) Momento de inercia un anillo de masa M y radio
R, en el plano xy, Con respecto a los ejes x, y, z.
Solucin.
= m(0 ) + 2m(a ) + 3m(a ) + 4m(0 )
2 2 2 2

2
= 5ma
Momento de inercia respecto al eje z.

I z = ri 2 mi

= m(0 ) + 2m(a ) + 3m a + b
2 2
( 2 2
) + 4m(b) 2 La masa del elemento diferencial ds = Rd es:

3
Cuerpo rgido Hugo Medina Guzmn

M M
dm = ds = d
2R 2
El momento de inercia del anillo con respecto al eje
z es:
2 M
I z = R 2 dm = R 2 d
M 0 2
MR 2 2
= [ ]0 = mR2
2
Por el teorema de la figura plana
Iz = Ix + Iy Consideremos la esfera como una serie de discos.
Por simetra Tomemos un disco diferencial como se muestra en
Ix = Iy la figura, su radio es r = R 2 z 2 , su espesor
Luego dz.
La masa del disco es:
Iz 1
Ix = Iy = = MR 2
2 2 dm =
M 2
V
M
(
r dz = R 2 z 2 dz
V
)
d) El momento de inercia de un disco de radio R y
masa M con respecto al eje perpendicular que pasa 4 3
por su centro. M es la masa de la esfera y V = R el
3
Solucin. volumen de la esfera.
El momento de inercia del disco con respecto al eje
z es:
1 1M
dI z =
2
dmr 2 =
2V
( 2
R 2 z 2 dz )
El momento de inercia de la esfera lo encontramos
integrando esta expresin desde z = - R a z = R.
1M
I z = dI z =
R

2V
R
( 2
)
R 2 z 2 dz

8 MR 5
Consideremos como elemento diferencial al anillo =
M
V

0
R
( )
2
R 2 z 2 dz =
15 V
de radio r y ancho dr, su masa es:
M 2M 2 2
= MR
dm = 2rdr = 2 rdr 5
R 2
R
El momento de inercia de este anillo con respecto al
Para encontrar el momento de inercia con respecto a
eje perpendicular que pasa por O es
un eje arbitrario como se muestra en la figura
2M siguiente aplicamos el teorema de Steiner.
dI O = r 2 dm = r 2 rdr
R2
2M 3
= r dr
R2
El momento de inercia del disco es:
R
R 2M 3 2M r 4
I O = dI O = r dr =
0 R2 R 2 4 0
1
= MR 2
2
e) El momento de inercia de una esfera con respecto 2
a un eje que pasa por su centro. I P = I O + Md 2 = MR 2 + Md 2
Solucin. 5
2
2 R 2
I P = Md 1 +
5 d
En el caso en que R << d podemos considera como

4
Cuerpo rgido Hugo Medina Guzmn

si fuera una masa puntual y el momento de inercia los torques producidos por las fuerzas externas que
se reduce a: actan sobre el sistema es igual al cambio de la
I O = Md 2 cantidad de movimiento angular.

dL
Ejemplo 2. Hallar el momento de inercia de un =
disco de masa M y radio R que gira alrededor de un dt
eje paralelo a un dimetro y que pasa por el borde Esto es vlido tambin para el cuerpo rgido, donde
del disco. L es la cantidad de movimiento angular can
respecto al eje x de la figura anterior.

Solucin. d L d

Como L = I = I
dt dt
Siendo I el momento de inercia del cuerpo en torno
al eje dado, es constante en el tiempo y

d
=I
dt

d
Como = , aceleracin angular del cuerpo
Por el teorema de las figuras planas dt

Iz = Ix + Iy ;
= I
Adems por simetra Esta expresin tiene similitud a la ley de Newton

I x = I y,
F = ma
Por tanto
Ejemplo 3. Una barra uniforme de longitud L y
Ix = Iz/2 = MR2
masa M, que gira libremente alrededor de una
bisagra sin friccin, se suelta desde el reposo en su
Aplicando el teorema de Steiner
posicin horizontal, como se muestra en la figura.
Calcular la aceleracin angular de la barra y su
I = MR2 + MR2
aceleracin lineal inicial de su extremo.
= 5/4 MR2

SEGUNDA LEY DE NEWTON PARA


ROTACION
En esta seccin vamos a analizar el movimiento de
un cuerpo rgido que gira en torno a un eje fijo en el
espacio.

Solucin.
Como el torque de la fuerza en la bisagra es cero,
se puede calcular el torque en torno a la bisagra
producido por la otra fuerza externa que acta sobre
la barra, que es su peso, suponiendo que la barra es
homognea y que el peso acta en su centro
geomtrico. Entonces:
1
El cuerpo gira en torno al eje x. Si = (t ) es el = rMg = LMg
desplazamiento angular del punto del cuerpo desde
2
la lnea referencial, la velocidad angular del cuerpo Como = I , y el momento de inercia de la barra
es: 1 2
es I = ML .
d 3
=
dt 1
Como cada punto del cuerpo gira a la misma Se tiene: I = LMg
2
velocidad angular , el desplazamiento (t ) de
cualquier punto describe el desplazamiento del
cuerpo como un todo.
Para el sistema de partculas vimos que la suma de

5
Cuerpo rgido Hugo Medina Guzmn

1
LMg =
( )
5 r 2 b 2 gh 2 sen
= 2 =
3g (
7r 2 5b 2 )
1 2L
ML2
3 Ejemplo 5. Se tiene un disco de masa M y radio R,
Para calcular la aceleracin lineal del extremo de la que pueda girar libremente alrededor de un eje que
barra, usamos la ecuacin at = L . pasa por su centro. Se enrolla una cuerda alrededor
del disco, se tira la cuerda con una fuerza F. Si el
Reemplazando : disco est inicialmente en reposos Cul es su
3 velocidad al tiempo t?
a t = L = g
2
Ejemplo 4. Una esfera rueda sobre una barra, con
seccin en forma de U, inclinada. Determinar la
aceleracin.

Solucin.
El momento de inercia del disco con respecto al eje
es:
1
I= MR 2
2
La direccin de la cuerda siempre es tangente al
disco por lo que el torque aplicado es:
Solucin.
= FR
Las fuerzas que actan sobre la esfera son el peso, Como = I
P, la reaccin normal del plano, R, y la fuerza de
rozamiento Ff.
Tenemos =
I
Reemplazando
Como la reaccin R y el rozamiento Ff estn FR 2F
aplicados en el eje instantneo de rotacin no = =
realizan ningn torque, slo el peso: 1 MR
MR 2
2
= hmg sen , siendo h = (r 2 b 2 ) Siendo constante = 0 + t
12

2F
Como 0 = 0 = t = t
El momento de inercia de la esfera con relacin al MR
eje instantneo de rotacin es
Ejemplo 6. Se sujeta una masa M a una cuerda
2 2 ligera enrollada alrededor de una rueda de momento
I= mr + mh 2 de inercia I y radio R.
5 Hallar La tensin de la cuerda, la aceleracin y su
velocidad despus de haber descendido una
Aplicando la ecuacin fundamental de la dinmica distancia h desde el reposo.
de rotacin:

hmgsen hgsen
= = =
I (2mr / 5 + mh ) (2r 2 / 5 + h 2 )
2 2

la aceleracin lineal ser: a = h

Solucin.
h 2 gsen gsen La figura siguiente muestra los diagramas de cuerpo
a= =
(2r / 5 + h ) (2r / 5h 2 + 1)
2 2 2 libre.

6
Cuerpo rgido Hugo Medina Guzmn

a) Por el teorema de las figuras planas, tenemos


que:
Iz = Ix + Iy ;
Adems por simetra
Aplicando la segunda ley de Newton a la masa M I x = I y,
Mg T = Ma (1) Por tanto
Iz 1 1
Aplicando la segunda ley de Newton para rotacin = LR = (2R )R = R
2 2 3
Ix =
al disco 2 2 2
TR = I ,
a
( )
= 1,6.10 (0,05) = 6,28x10-5 kg m2
1 3

como a = R = b) Al comunicarle un momento angular


R L = 7,9 x10-4 kg m2/s,
a L 7,9 10 4
TR = I o TR 2 = Ia (2) 0 = =
R I 6,28 10 5
Resolviendo (1) y (2) obtenemos = 12,58 rad/s
M c) = 50 dina cm = 50x10-5 Nx10-2 m
a= g,
M + I R2 = 5x10-6 N m
Por lo tanto la ecuacin del movimiento en trminos
I R2 angulares ser:
T= Mg
M + I R2 1
= 0 + 0t + t 2 = 12,6t 0,0398t 2 , y
2
Siendo un movimiento con aceleracin constante = 12,6 0,079t
v 2 = v02 + 2as Siendo = 0 para t = 158 s.
Conocemos: a , v0 = 0 , s = h :
Ejemplo 8. Maquina de atwood tomando en
2Mg cuenta la polea.
v2 = h
M + I R2
2 Mg
v= h
M + I R2

Ejemplo 7. Un anillo de 5 cm de radio, grosor


despreciable y densidad 1,6 g/cm, se pone en
rotacin alrededor de un dimetro cuando se le
comunica un momento angular de 7900 g cm2/s.
a) Hallar la expresin analtica y el valor numrico
del momento de inercia respecto del eje de giro.
b) Con qu velocidad angular empieza a girar?
c) Si el rozamiento con el aire y los pivotes origina
un par de fuerzas cuyo torque es de 50 dina cm, La polea es un disco de masa M y radio R. La figura
cul ser la ecuacin del movimiento que efecta muestra los diagramas de cuerpo libre de cada una
el anillo?, cunto tiempo tarda en pararse? de las partes de la mquina de atwood.
(Nota 1 N = 105 dinas)
Solucin.

7
Cuerpo rgido Hugo Medina Guzmn

Planteando la segunda ley de Newton para cada


Aplicando la segunda ley de Newton a cada una de masa:
las partes.
Masa M1: m1 g T1 = m1 a ,
T1 M 1 g = M 1 a (1) T2 m 2 g = m2 a
Masa M2: Para la polea:
M 2 g T2 = M 2 a (2) a
Polea:
= T R T R = I = I R
1 2

T2 R T1 R = I Como el hilo no desliza,


1 a 1 a = R
= MR 2 = MRa (3) Por lo tanto tenemos tres ecuaciones:
2 R 2 m1 g T1 = m1 a ,
Resolviendo (1), (2) y (3), obtenemos:
T1 = M 1 ( g + a ) , T2 m 2 g = m2 a ,
T2 = M 2 ( g a ) y T1 T2 = I 2
a
(m2 m1 ) R
a= g Que sumadas dan lugar a:
(m2 + m1 + M 2) (m1 m2) g = a(m1 + m2 + I/R2).
Por lo tanto a vale:
Ejemplo 9. Una polea homognea de radio R, masa m1 m2 5
M y momento de inercia I, gira alrededor de su eje, a= g = 9,8
I 18
debido a la accin de dos masas m1 y m2. m1 + m2 + 2 25 +
R = 0,3 m, m1 =15 kg, m2 = 10 kg, M = 20 kg, I =18 R 0,3 2
kg m2. = 0,22 m / s2
Calcular: a 0,22
a) La aceleracin angular de la polea. y = = = 0,73 rad / s2
b) Las tensiones de las cuerdas. R 0,3
c) La tensin del soporte que fija el sistema al techo b) De las ecuaciones anteriores obtenemos:
T1 = m1 g m1 a = 15( g a ) = 143,7 N.
T2 = m 2 ( g + a ) = 100,2 N.
c) Considerando todas las fuerzas que actan sobre
la polea, que debe estar en equilibrio:

Solucin.
a) Vamos a suponer que el sistema acelera hacia el
lado de la masa mayor M.
F = 0
S = P + T1 + T2 = 20 x 9,8 + 146,67 + 102,22
= 445 N

8
Cuerpo rgido Hugo Medina Guzmn

Ejemplo 10. La figura representa un cilindro kg y de la garganta de la polea pequea pende otra
macizo y homogneo de radio R = 20 cm y masa masa de 100 kg que tiende a hacer girar a las poleas
M = 20 kg. A su periferia va arrollado un hilo ideal en sentido contrario al anterior. El momento de
de cuyo extremo libre cuelga una masa m = 8 kg. inercia del sistema formado por las dos poleas es de
Por una hendidura muy fina se le arrolla otro hilo 10 kg m2. Al dejar el sistema en libertad, se pone en
ideal a una distancia del eje horizontal r = 10 cm, a movimiento espontneamente. Se pide:
cuyo extremo libre se le aplica una fuerza constante a) En qu sentido se mueven las poleas?
F = 200 N. Calcular: b) Valor de la aceleracin con que se mueve cada
a) Momento de inercia del cilindro respecto a un eje una.
que coincida con una generatriz. c) Aceleracin angular de las poleas.
b) Aceleracin con que sube la masa m. d) Tensin de la cuerda que sostiene la masa de 100
c) Aceleracin angular del cilindro. kg cuando el sistema est en movimiento.
d) Tensin del hilo que sostiene la masa. Solucin.

Solucin.
a) Aplicando el teorema de Steiner,
I = MR2 +MR2 = 3/2 MR2
a) Cuando las poleas estn inicialmente en reposo,
los pesos coinciden con las tensiones.
Por tanto T1 = 200 N, y T2 = 1000 N.
El momento que ejerce T1 valdr
1 = T1 R1 = 200 Nm
El que ejerce T2 valdr
2 = T2 R2 = 300 N m.
Por tanto, al ser el momento de la fuerza T2 mayor,
la polea girar de modo que la masa M1suba.
b) Podemos plantear dos ecuaciones: b) y c) Planteando la ecuacin fundamental de la
T mg = ma y dinmica a cada masa y a la polea, tendremos:
1 a 1 T1 M 1 g = M 1 a1
Fr TR = I = MR 2 = MRa
2 R 2 T1 M 1 g = M 1R1 (1)
Que conducen a: M 2 g T2 = M 2 a 2
1 M 2 g T2 = M 2R2 (2)
Fr mgR = a mR + MR .
2 2 1 = I
Por lo tanto la aceleracin a vale:
Fr mgR 20 15,68 T2 R2 T1 R1 = I (3)
a= = De las tres ecuaciones obtenemos :
1 1,6 + 2
mR + mR M 2 gR2 M 1 gR1
2 =
= 1,2 m / s2 M 2 R22 + M 1 R12 + I
a 1,2 30 20
c) = = = g = 2,51 rad / s2.
R 0,2 20 + 9 + 10
= 6 rad/s2. La aceleracin de cada masa ser:
d) T = mg + ma = 8 (9,8 +1,2) a1 = R1 = 2,51 m/s2,
= 88 N.
a 2 = R2 = 0,75 m/s2
Ejemplo 11. Dos poleas cuyos radios son 1 m y d) T2 = M 2 g M 2R2 = 904,7 N
0,3 m, estn acopladas pegada una a la otra en un
plano vertical, formando un bloque que gira Ejemplo 12. Un rollo de 16,0 kg de papel con
alrededor de su eje de rotacin comn. De la radio R = 18,0 cm descansa contra la pared
garganta de la polea grande pende una masa de 20

9
Cuerpo rgido Hugo Medina Guzmn

sostenido por un soporte unido a una varilla que (40,0 31,54)(18,0 102 )
pasa por el centro del rollo. La varilla gira sin = = .
friccin en el soporte, y el momento de inercia del I (0,260)
papel y la varilla alrededor del eje es de 0,260 kg. = 4,71 rad/s 2
m2. El otro extremo del soporte est unido mediante
una bisagra sin friccin a la pared de modo que el
soporte forma un ngulo de 30,0 con la pared. El Ejemplo 13 Se debe aplicar una sola fuerza
peso del soporte es despreciable. El coeficiente de adicional a la barra de la figura para mantenerla en
friccin cintica entre el papel y la pared es equilibrio en la posicin mostrada. Puede
despreciarse el peso de la barra.
k = 0,25 . Se aplica una fuerza vertical constante a) Calcule las componentes vertical y horizontal de
F = 40,0 N al papel, que se desenrolla. la fuerza requerida.
a) Qu magnitud tiene la fuerza que la varilla b) Qu ngulo debe formar sta fuerza con la
ejerce sobre el rollo de papel al desenrollarse? barra?
b) Que aceleracin angular tiene el rollo? c) Qu magnitud debe tener?
d) Dnde debe aplicarse?

Solucin.
Solucin. a) La tensin en el resorte es W2 = 50 N, y la
En el punto de contacto, la pared ejerce una fuerza fuerza horizontal sobre la barra debe equilibrar la
F f de la friccin dirigida hacia abajo y una fuerza componente horizontal de la fuerza que el resorte
ejerce sobre la barra, y es igual a
normal N dirigida a la derecha. Esto es una
(50 N) sen 37 = 30 N, a la izquierda en la
situacin donde es cero la fuerza neta en el rodillo,
pero el torque neto no es cero. figura.
La suma de fuerzas verticales La fuerza vertical debe ser
Fvar cos = F f + W + F , F f = k N , 50 cos 37 + 10 = 50 N, arriba
b)
Las fuerzas horizontales
Fvarsen = N . 50 N
arctan = 59
De aqu tenemos: 30 N
Fvar cos = k N + F + W c)
Fvarsen = N . (30 N) 2 + (50 N) 2 = 58 N.
a) Eliminando N y resolviendo para Fvar da d) Tomando torques alrededor (y midiendo la
distancia de) del extremo izquierdo
W +F 50 x = (40)(5,0 )
Fvar =
cos k sin x = 4,0 m
40,0 + (16,0) (9,80) Donde solamente las componentes verticales de las
= = 266 N fuerzas ejercen torques.
cos 30 (0,25)sen30
b) Con respecto al centro del rodillo, la barra y la Ejemplo 14. Imagine que est tratando de subir
fuerza normal ejercen el torque cero. una rueda de bicicleta de masa m y radio R a una
La magnitud del torque neto es acera de altura h; para ello, aplica una fuerza
( F F f ) R , y F f = k N horizontal F. Qu magnitud mnima de F logra
subir la rueda si la fuerza se aplica
Puede calcularse reemplazando el valor
a) al centro de la rueda?
encontrado para Fvar en cualquiera de las b) En la parte superior de la rueda?
relaciones anteriores; c) En cul caso se requiere menos fuerza?
F f = k Fvarsen = 33,2 N .
Luego,

10
Cuerpo rgido Hugo Medina Guzmn

Aplicando la segunda ley de Newton en la masa C:


m C g T = mC a
8 g T = 8 R A
Aplicando la segunda ley de Newton de la rotacin
en el conjunto giratorio:
TR B = I
Solucin. Resolviendo el sistema formado:
a) Tome los torques respecto a la esquina superior 8 gR B TR B = 8R B2
8 gRB = 8aRB + I
2
de la acera.
TR B = I
La fuerza F acta a una distancia perpendicular 8 gR B 35,28
Rh y = 2
= = 0,66 rad/s2
el peso acta en una distancia perpendicular 8 R B + I 53,18
b) a 0 = R0 = 0,6 m/s2
R 2 (R h ) =
2
2 Rh h 2 .
Igualando los torques para encontrar la fuerza
necesaria mnima, c) (4 s ) = t = 2,65 rad/s

2 Rh h 2 aN = 2 RD = 6,34 m/s2
F = mg .
Rh
b) El torque debido a la gravedad es el mismo, pero EQUILIBRIO ESTTICO
En el captulo 5 vimos que para que una partcula
la fuerza F acta a una distancia perpendicular estuviera en equilibrio esttico era suficiente que La
2 R h, tal que la fuerza mnima es fuerza resultante fuese cero.

F = mg
2 Rh h 2
.
F = 0
2R h Esta condicin tambin, es necesaria para que un
c) Se requiere menos fuerza que cuando la fuerza se cuerpo rgido este en equilibrio, pero no es
aplica en parte alta de la rueda. suficiente que solamente el centro de masa este en
reposo, el cuerpo puede girar Es necesario que el
Ejemplo 15. Un disco homogneo A gira alrededor momento de: fuerzas o torque con respecto al centro
del eje y bajo la accin de la masa C unida a una de masa sea nulo.

cuerda que pasa por una polea sin peso ni
rozamiento enrollada alrededor del tambor =0
cilndrico macizo B, solidaria del disco A. A ste A continuacin desarrollaremos algunos ejemplos
est unida una masa puntual D, como indica la de aplicacin. En muchos de ellos la fuerza de la
figura. Las masas A, B, C y D son respectivamente gravedad ejercida sobre las diversas partes de un
65, 15, 8 y 4 kg. Se supone que la cuerda cuerpo puede sustituirse por una sola fuerza, el peso
permanece siempre horizontal. Calcular: total actuando en el centro de gravedad.
a) Aceleracin angular del disco. Si la aceleracin de la gravedad no vara a lo largo
b) Aceleracin tangencial de D. del cuerpo, el centro de gravedad coincide con el
c) Aceleracin normal de D, 4 s despus de partir centro de masa.
del reposo.
Ejemplo 16. Demostrar que cuando un cuerpo est
en equilibrio y el torque con respecto al centro de
masa es cero, el torque con respecto a cualquier
punto tambin es cero.
Solucin.

Solucin.
a) Calculemos en primer lugar el momento de
inercia del sistema A-B-D. En la figura
1 1 1
I = m A R A2 + m B RB2 + m D RD2 rO es el vector del centro de masa a O
2 2 2
= 51,56 kg m2

11
Cuerpo rgido Hugo Medina Guzmn

ri es el vector del centro de masa al punto donde



acta Fi .

rOi es el vector del punto O al punto donde acta

Fi .
De la figura vemos:

Solucin.
ri = rO + rOi a) Sustituir la fuerza vertical dada por otra igual
El torque total alrededor de O es paralela cuya lnea de accin pase por el centro de
masa.


O = r Oix F i = i ri rO F i =
i

r F r F
i
i i
i
O i = CM rO F i
i

Como rO es constante

O = CM rO F i b) Hacer girar el plano del par, hasta desplazarlo
i hasta la lnea A B.

Para un cuerpo en equilibrio Fi = 0

tal que O = CM

Si CM = 0 , el torque alrededor de cualquier
punto debe ser cero y viceversa.

Ejemplo 17. Par de fuerzas. Dos fuerzas iguales y


opuestas que actan en la figura siguiente se c) Se cambian los mdulos de las fuerzas a F de tal
denominan par de fuerzas, Segn se indica modo que:
a
F ' b = Fa F ' = F
b

F es el valor de cualquiera de las fuerzas y


d = ( x 2 x1 ) es la distancia entre ellas.
El momento o torque producido por estas fuerzas
con respecto a O es:
O = Fx 2 Fx1 = F ( x 2 x1 ) = Fd
Este resultado no depende de la seleccin del punto Ejemplo 19. Sobre una placa slida actan cuatro
O, el momento producido por un par es el mismo fuerzas de mdulos
respecto a cualquier punto del espacio. F1 = 28,3 N, F2 = 60 N, F3 = 20 N y F4 = 50 N.
Como se indican en la figura. Hallar la tuerza
Ejemplo 18. Una fuerza vertical F que acta en A. resultante sobre la placa y determinar su lnea de
en el slido rectangular mostrado en la figura, accin.
queremos sustituirla por otra cuya lnea de accin
pasa por el centro de masa ms un par de fuerzas
que acten horizontalmente aplicados en A y B.

12
Cuerpo rgido Hugo Medina Guzmn


r = xi + yj
Tal que

( ) (30i + 40 j )

= 6k = r F = xi + yj
= (40 x 30 y )k
De aqu:
(40 x 30 y ) = 6
20 x 15 y = 3
Solucin. Esta es la ecuacin de la lnea de accin de la
Utilizando el cuadriculado obtenemos: fuerza; si esta tuerza a de situarse en algn punto

del borde inferior de la placa, y = - 0,2 m..


r1 = 0,2i 0,2 j , Obtenemos
2 2 3 + 15 y 3 + 15( 0,2)
F1 = 28,3 i + 28,3 x= =0
2 j = 20i + 20 j
=
2 20 20


La figura siguiente muestra la fuerza resultante:
r2 = 0,1i 0,2 j , F2 = 60 j

r3 = 0,2i + 0,1 j , F3 = 20i

3 3
r4 = 0,1i + 0,2 j , F4 = 50 i 50 j
4 4
= 30i 40 j
La fuerza resultante es

F = F1 + F2 + F3 + F4
Ejemplo 20. Se tiene una escalera d masa M y
= (20 20 + 30 )&i& + (20 + 50 40 ) j
largo L apoyada contra la pared .No hay friccin en
= (30i + 40 j )N la pared y el coeficiente de friccin del piso es .
Cul es el mnimo ngulo de inclinacin para que
El torque resultante respecto al centro de masa es la no comience a resbalar?
suma de los torques individuales.

= 1+ 2 + 3 + 4
Siendo:

1 = r1 F1
( ) (
= 0,2i 0,2 j 20i + 30 j = 0 )

2 = r2 F2
= (0,1i 0,2 j ) (60 j ) = 6k Solucin.
La figura siguiente muestra el diagrama del cuerpo

3 = r3 F3 libre de la escalera.

= (0,2i + 0,1 j ) ( 20 j ) = 2k

4 = r4 F4
(
= 0,1i + 0,2 j ) (20i 40 j ) = 2k
Reemplazando:

= 6k Nm

Para determinar la lnea de accin de la tuerza,


consideremos que el punto de aplicacin de la
fuerza resultante es: Condicin para que el centro de masa no acelere:

13
Cuerpo rgido Hugo Medina Guzmn

F x = 0 = N x N y , R es a reaccin de la pared.
Como el sistema est en equilibrio
F = 0 = Mg N y
y
De aqu obtenemos: F = 0
N y = Mg , N x = N y = Mg Fx = R cos T cos = 0

Condicin de no rotacin: Fy = Rsen Tsen Mg mg = 0
La suma de momentos de fuerza con respecto al
centro de masa es cero.
L L L
Con = 0 alrededor de cualquier punto.
N y cos N y sen N x sen = 0 Tomamos momentos con respecto a O.
2 2 2 L
Reemplazando las fuerzas: TLsen mg Mgx = 0
L L L 2
Mg cos Mg sen Mg sen = 0 De esta ltima ecuacin obtenemos
2 2 2
2sen = cos L
TLsen mg
1 x= 2
= tan 1 Mg
2 Si T = Tm obtenemos el valor mximo de x.
Otra forma: Si estuviramos interesados en conocer R, sera
En lugar de tomar el centro de masa como origen mejor tomar momentos con respecto al otro
tomemos extremo inferior de la escalera. extremo.
Tomando momentos con respecto a este punto.
L Ejemplo 22. Un albail de 75 kg camina sobre un
Mg cos N x Lsen = 0 tabln de 3 m de largo y 80 kg apoyado sobre dos
2 vigas distantes 2 m, tal como indica la figura. Cul
Reemplazando el valor de Nx:
es la mxima distancia x que puede recorrer, sin
L que caiga?
Mg cos MgLsen = 0
2
1
= tan 1
2
Obtenemos la misma respuesta porque no importa
con respecto a que eje tomemos el torque.

Ejemplo 21. Una viga de masa m se empotra a la Solucin.


pared como se muestra en la figura y se sujeta por Para que el tabln gire, el torque del peso del
medio de un alambre. Si la tensin en el alambre albail respecto del punto O, ms el torque del peso
excede Tm el alambre se rompe. Para qu valor de de la parte de tabln que sobresale, debe ser mayor
o igual que el torque del peso de la parte de tabln
x, el alambre se romper por una masa M colocada apoyada entre las vigas:
sobre la viga? Llamando a la densidad lineal del tabln:
M
= , haciendo d = 2 m, L = longitud del
L
tabln, M = masa tabln, m = masa albail
tendremos:

mgx + (L d )g
(L d ) = dg d
2 2
Solucin.
La figura muestra el diagrama del cuerpo libre del
[
mx + d 2 (L d )2 = ] M
2L
(
2 Ld L2 , )
sistema viga-masa. M
x= (2d L ) = 0,53 m
2m
Ejemplo 23. Un bal de masa M se empuja sobre
un suelo con coeficiente de rozamiento
a) Qu fuerza F se ejerce si el bal se mueve con
aceleracin constante a?
b) Si el bal se mueve con velocidad constante?

14
Cuerpo rgido Hugo Medina Guzmn

c) Qu fuerza se necesita para inclinar el bal? La componente horizontal de la fuerza ejercida en


la barra por la bisagra debe equilibrar la fuerza

F aplicada, y as tiene magnitud 120,0 N y es
hacia la izquierda.
Tomando torques alrededor del punto A
(120,0 N)(4,00 m) + FV (3,00 m)
Solucin. La componente vertical es 160 N, el signo menos
La figura siguiente muestra el diagrama del cuerpo indica una componente hacia abajo, ejerciendo un
libre del bal. torque en una direccin opuesta a la de la
componente horizontal.
La fuerza ejercida por la barra en la bisagra es igual
en magnitud y contrario en la direccin a la fuerza
ejercida por la bisagra en la barra

Ejemplo 25. La caja es arrastrada sobre una


superficie horizontal con rapidez constante por una
fuerza. El coeficiente de friccin cintica es de 0,35.
a) Aplicando la segunda ley de Newton. a) Calcule la magnitud de F.
b) Determine el valor de h con el cual la caja
F x = F k ( N 1 + N 2 )Ma , comience a volcarse.
F y = N 1 + N 2 Mg = 0
Resolviendo las ecuaciones:
F = M (a + k g )
b) En el caso que el bal va con velocidad constante
a = 0 y F = M k g
c) Para analizar la inclinacin del bal tenemos que
Solucin.
escribir la ecuacin de momentos con respecto al
borde delantero, sin rotacin = 0, luego a) F = F f = k = k mg
b = (0,35)(30,0 kg)(9,80 m s )
2
= bN 1 hF +
2
Mg = 0
= 103 N
Cuando el bal empiece a inclinarse, empezar a b) Con respecto al borde delantero de la caja.
rotar en el sentido horario y N1 = 0, de aqu: El brazo de palanca del peso es
bMg 0,250
F= = 0,125 m
2h 2
y la aceleracin: El brazo de palanca h de la fuerza aplicada es
F b entonces
a= k g = k g mg 1
M 2h h = (0,125) = (0,125)
F k
Ejemplo 24. El extremo A de la barra AB de la 0,125
figura descansa en una superficie horizontal sin = = 0,36 m.
friccin, y el extremo B tiene una articulacin. Se
0,35
ejerce en A una fuerza horizontal F de magnitud
120 N. Desprecie el peso de la barra. Calcule las TRABAJO Y ENERGIA EN ROTACIN.
componentes horizontal y vertical de la fuerza Consideremos un cuerpo que gira alrededor de un
ejercida por la barra sobre la articulacin en B. eje tal como se muestra en la figura

Solucin. La energa cintica de un elemento de masa dm que


gira a una distancia r del eje de rotacin es:

15
Cuerpo rgido Hugo Medina Guzmn

1 1 2 1 2
dK = dmv 2 , v = r = I2 I1
2 2 2
1 = K 2 K 1 = K
dK = dm 2 r 2
2 E1 trabajo neto realizado por las fuerzas externas
Integrando. al hacer girar un cuerpo rgido alrededor de un eje
1 2 2 fijo es igual al cambio en la energa cintica de
K = dK = r dm rotacin.
M 2
como es constante. Por la analoga que existe entre las expresiones para
1 el movimiento lineal y el movimiento angular,
K = dK = 2 r 2 dm podemos decir que un torque ser conservativo a
2 M
condicin que exista una funcin potencial
El trmino integral es el momento de inercia del
cuerpo con respecto al eje de rotacin U = U ( ) de tal modo que el trabajo efectuado por
1 2
K= I , cuando el cuerpo sufre un desplazamiento
2
Para relacionar la energa cintica, al trabajo
(
angular ( 2 1 ) es la diferencia U (1 ) U ( 2 ) .)
efectuado sobre el cuerpo por un torque . As pues se deduce que:
Supongamos que se aplica una fuerza externa nica U (1 ) U ( 2 ) = K 2 K 1
F, que acta en el punto P del cuerpo.
K 1 + U (1 ) = K 2 + U ( 2 ) = constante
Cuando el sistema no es conservativo
( ) (
W NO CONSERVATI VO = K 1 + U (1 ) K 2 + U ( 2 ) )
POTENCIA
La rapidez con que se realiza este trabajo es:
dW d
= =
dt dt

Expresin que corresponde a la potencia
El trabajo realizado por F a medida que el cuerpo instantnea.
gira recorriendo una distancia infinitesimal P =
ds = rd en un tiempo dt es:
Ejemplo 26. Para la barra giratoria, calcular su
dW = F d s = Fsen rd rapidez angular y la rapidez lineal de su centro de
Como Fsen r es el torque de la fuerza F masa y del punto mas bajo de la barra cuando est
vertical.
alrededor del origen se puede escribir el trabajo
realizado para la rotacin infinitesimal como:
dW = d
Cuando el cuerpo gira en torno a un eje fijo bajo la
accin de un torque. El cambio de su energa
cintica durante el intervalo dt se puede expresar
como:
dK d 1 Solucin.
dK = dt = I 2 dt
dt dt 2
d
= I dt = I dt = I dt
dt
Como
= I y d = dt
Obtenemos:
dK = d = dW
Si se ntegra esta expresin se obtiene el trabajo
total
2 2
W12 = d = Id Usando el principio de conservacin de la energa,
1 1
considerando que la energa potencial se calcula

16
Cuerpo rgido Hugo Medina Guzmn

respecto al centro de masa y la energa cintica es 1 1 1


de rotacin: m1v 2 + m2 v 2 + I 2 + m1 gH
=
2 2 2
Ei = E f K i + U gi = K f + U gf
1 I
Cuando la barra esta inicialmente horizontal no m1 + m2 + 2 v 2 = (m2 m1 )gH
tiene Ki y cuando esta vertical tiene solo Kf, 2 R
entonces: Donde se ha usado la relacin v = R , despejando
L 1 11 v se obtiene:
Mg = I 2 = ML2 2
2 2 23 2(m2 m1 )gH
v=
I
3g m1 + m2 + 2
= R
L
Para calcular la rapidez del centro de masa, se usa:
Ejemplo 28. Sobre un cilindro homogneo de
L 1 radio R y masa M. tiene El cual tiene libertad de
vcm = r = = 3gL
2 2 girar sin friccin sobre un eje, como se muestra en
En el punto mas bajo la rapidez es la figura. Si se le aplica en su borde una fuerza
tangencial de magnitud F.
v = 2v cm = 3 gL a) Cul es la aceleracin angular del cilindro?
b) Cual es la velocidad angular y la energa
Ejemplo 27. Para el sistema de la figura, las masas cintica del cilindro al tiempo t?
tiene momento de inercia I en torno a su eje de c) qu cantidad de trabajo aplica la fuerza durante
rotacin, la cuerda no resbala en la polea y el este intervalo t?.
sistema se suelta desde el reposo.
Calcular la rapidez lineal de las masas despus que
una ha descendido H y la rapidez angular de la
polea.

Solucin.
El momento de inercia del cilindro en torno a su eje
es:
1
I= MR 2
2

a) Con = I = , = F0 R
I
Solucin. F0 R 2F
tenemos = = 0
Como no hay roce en la polea, se conserva la 1
energa, que aplicada a cada masa m1 y m2, MR 2 MR
suponiendo que m2 se encuentra inicialmente en la 2
parte superior del sistema, es: b) Siendo constante
Ei = E f = 0 + t
2 F0
Si 0 = 0 , = t , = t
MR
La energa cintica:
2
1 11 2 F
K = I 2 = MR 2 0 t
2 22 MR
F2 2
= 0 t
M
c) El trabajo realizado
F02 2
K 1i + K 2i + U 1i + U 2i W = K = K 2 K 1 = t 0
M
= K1 f + K 2 f + K p + U 1 f + U 2 f F02 2
= t
0 + m2 gH M

17
Cuerpo rgido Hugo Medina Guzmn

2R 1
Otra forma de calcular es: Luego W = MgR = M 2 R 2
2 4g 4
W = d , = F0 R (constante) Otra forma de evaluar el trabajo es por la
1
conservacin de la energa.
W = F0 R( 2 1 ) = F0 R
1 2
2 F0 I 0
W = K = K 2 K 1 =
Con = , 0 = 0 2
MR 11 1
2 2
= MR = M R
2 2
1 1 2F F 22 4
= t 2 = 0 t 2 = 0 t 2
2 2 MR MR
Finalmente Ejemplo 30. Una plataforma cilndrica uniforme
F F 2
2 de 180 kg de masa y 4,5 m de radio se frena de 3,2
W = F0 R 0 t 2 = t 0
rev/s al reposo en 18 s cuando se desconecta el.
MR M motor. Calcular la potencia de salida del motor (hp)
para mantener una velocidad constante de 3,2 rev/s.
Ejemplo 29. Un carrete de hilo delgado tiene radio Solucin.
R y masa M. Si se jala el hilo de tal modo que el Como primer paso debemos conocer cul es el
centro de masa del carrete permanezca suspendido torque de frenado que tenemos que vencer para
en el mismo lugar. mantener la velocidad constante, ese torque lo
a) Qu fuerza se ejerce sobre el carrete? calcularemos de la siguiente manera:
b) Cunto trabajo se habr realizado cuando el frenado = I frenado
carrete gira con velocidad angular ?
Solucin. 1
I= MR 2 .
La figura muestra al carrete suspendido. 2
2 1
frenado = = =
t t 2 t1 t
1 MR 2
frenado = MR 2 =
2 t 2t
La potencia es:
MR 2 2
P = =
El carrete solo tiene movimiento circular ya que 2t
est en equilibrio vertical Siendo
Aplicando las leyes de Newton: M =180 kg, R = 4,5 m,
rev 2 rad rad
F = 0 y T Mg = 0 = 3,2
s rev
= 6,4
s
,

= I TR = I t = 18 s.
1 2
Como I = MR , obtenemos: P=
(180 )(4,5) (6,4 )
2 2
= 40889,73 W
2 2(18)
1 Como 1 hp = 735,5 W
MgR = MR 2
2
2g P = 55,6 hp
y =
R Ejemplo 31. Se sujeta una masa M a una cuerda
a) La fuerza que se ejerce sobre el carrete es ligera enrollada alrededor de una rueda de momento
T = Mg de inercia I y radio R. Hallar La tensin de la
b) Como el trabajo realizado es: cuerda, la aceleracin y su velocidad despus de
haber descendido una distancia h desde el reposo.
W = , donde = TR = MgR 2 Resolver desde el punto de vista de energa.
constante = 0 + 2
2 2
Siendo
Si 0 = 0 2 = 2 y
2 2R
= =
2 4g

18
Cuerpo rgido Hugo Medina Guzmn

Solucin.
Por el principio de conservacin de la energa Etotal
= constante
Al inicio del movimiento toda la energa es
potencial, si consideramos como nivel cero el Las masas M1 y M2 inicialmente estn en reposo en
indicado en la figura (a).
la posicin y = 0 , despus de soltarlas una sube y
Ei = Mgh la otra baja como muestra la figura.
Las masas estarn movindose con velocidad v la
Polea tendr una velocidad angular .
Como no hay rozamiento por la conservacin de la
energa
E1 = E 2
1 1
0 = + M 1v 2 + M 2 v 2 +
2 2
1 2
I + M 1 gy M 2 gy
2
La energa final es pura energa cintica, de la nasa v 1
Siendo =
2
, I = MR , tenemos:
M con velocidad v antes de chocar y el disco con R 2
momento de Inercia I con velocidad angular 1 M 2
= v R , figura (b). M 1 + M 2 + v = (M 1 M 2 )gy
2
2 2
Ef =
1 1 1 1 v
Mv 2 + I 2 = Mv 2 + I 2(M 1 M 2 )
2 2 2 2 R v2 = gy
M
2
v 1 M1 + M 2 +
= M + 2 2
2 R Para un movimiento uniformemente acelerado
Como E i = E f v 2 = 2ay
Comparando:
v2 1
Mgh = M + 2 (M 2 M 1 )
2 R a= g
(M 2 + M 1 + M 2 )
2 2M
y v = gh
M +1 R2 Ejemplo 33. Una canica slida uniforme de radio r
parte del reposo con su centro de masa a una altura
2M h sobre el punto ms bajo de una pista con un rizo
v= gh
M +1 R2 de radio R. La canica rueda sin resbalar. La friccin
de rodamiento y la resistencia del aire son
despreciables.
Ejemplo 32. Resolver la mquina le Atwood
a) Qu valor mnimo debe tener h para que la
utilizando Conceptos de trabajo y energa,
canica no se salga de la pista en la parte superior del
Solucin.
rizo? (Nota: r no es despreciable en comparacin
con R.)
b) Qu valor debe tener h si la pista est bien
lubricada, haciendo despreciable la friccin?

19
Cuerpo rgido Hugo Medina Guzmn

1 2 1 2 7 2
g (h 2 R + r ) = v + v = v
2 5 10
7
g (h 2 R + r ) = v 2
10
2
Reemplazando el valor de v :
7
g (h 2 R + r ) = g (R r )
Solucin. 10
a) De a a B, la distancia que la canica ha cado es 7
y = h (2 R r ) = h + r 2 R. h 2 R + r = (R r )
10
7
h = 2 R r + (R r )
10
7
= 2R r + (R r )
10

27 17
= R r
10 10
El radio de la trayectoria del centro de masa de la
= 2,7 R 1,7 r
canica es R r , .
b) En ausencia de friccin no habr rotacin.
Luego:
1 2
mgy = mv
2
2
Sustituyendo las expresiones para y y v en
trminos de los otros parmetros da
1
h 2R + r = (R r )
2
La condicin para que la canica permanezca en la Resolviendo obtenemos
pista es
v2 5 3
Fr = mac mg = m (R r )
h=
2
R r.
2
v 2 = g ( R r ). Ejemplo 34. La figura muestra tres yoyos idnticos
La velocidad se determina del teorema del trabajo - que inicialmente estn en reposo en una superficie
energa, horizontal. Se tira del cordel de cada uno en la
1 2 1 2 direccin indicada. Siempre hay suficiente friccin
mgy = mv + I para que el yoyo ruede sin resbalar. Dibuje un
2 2 diagrama de cuerpo libre para cada yoyo. En qu
Se tiene: direccin girar cada uno? Explica tus respuestas
y = h (2 R r )
v
=
r
Se sabe que para una esfera
2 2
I= mr
5
Reemplazando estos valores en la ecucin de la Solucin. En el primer caso, F y la fuerza de la
energa: friccin actan en direcciones opuestas, y la fuerza
2
1 2 1 2 2 v de friccin tiene el torque mayor que hace rotar el
mg (h 2 R + r ) = mv + mr yo-yo a la derecha. La fuerza neta a la derecha es la
2 25 r diferencia F F f , tal que la fuerza neta es a la

derecha mientras que el torque neto causa una


rotacin a la derecha.

20
Cuerpo rgido Hugo Medina Guzmn

Para el segundo caso, el torque y la fuerza de 1 2 1 2


friccin tienden a dar vuelta al yoyo a la derecha, y mgh = mv + I
el yo-yo se mueve a la derecha. 2 2
En el tercer caso, la friccin tiende a mover al yo- Rodar sin resbalar significa
yo a la derecha, y puesto que la fuerza aplicada es v 2 2
vertical, el yoyo se mueve a la derecha.
= , I = mr
r 5
1 2 1 2
Tal que I = mv
2 5
7
mgh = mv 2
10
7v 2 7(17,82 m/s)
h= =
10 g 10(9,80 m/s 2 )
Ejemplo 35.
= 23 m
Una canica uniforme baja rodando sin resbalar por
el trayecto de la figura, partiendo del reposo. 1 2 1 2
b) I = mv , Independiente de r.
a) Calcule la altura mnima h que evita que la 2 5
canica caiga en el foso. c) Todo es igual, excepto que no hay el trmino de
b) El momento de inercia de la canica depende de energa rotacional cintica en K:
su radio. Explique por qu la respuesta a la parte (a) 1 2
no depende del radio de la canica. K= mv
c) Resuelva la parte (a) para un bloque que se 2
desliza sin friccin en vez de una canica que rueda. 1
Compare la h mnima en este caso con la respuesta mgh = mv 2
a la parte (a).
2
2
v
h= = 16 m .
2g
Comparado con la altura de la parte (a), 16 /23 =
0,7, es el 70 %.

Ejemplo 36. Una esfera slida uniforme rueda sin


resbalar subiendo una colina, como se muestra en la
figura. En la cima, se est moviendo
horizontalmente y despus se cae por un acantilado
vertical.
a) A qu distancia del pie del acantilado cae la
Solucin.
a) Encuentre la velocidad v que necesita la canica esfera y con qu rapidez se est moviendo justo
en el borde del hoyo para hacerlo llegar a la tierra antes de tocar el suelo?
plana en el otro lado. b) Observe que, al tocar tierra la esfera, tiene mayor
La canica debe viajar 36 m horizontalmente rapidez de traslacin que cuando estaba en la base
mientras cae verticalmente 20 m. de la colina. Implica esto que la esfera obtuvo
Use el movimiento vertical para encontrar el energa de algn lado? Explique.
tiempo.
Tome + y hacia abajo.
v 0 y = 0, a y = 9,80 m/s 2 , y y0 = 20 m, t = ?
1
y y 0 = v0 y t + a y t 2 t = 2,02 s
2
Solucin.
Luego x x 0 = v 0 x t v 0 x = 17,82 m/s.
a) Use la conservacin de la energa para encontrar
Utilice la conservacin de la energa, donde el la velocidad v 2 de la bola momentos antes que
punto 1 est en el punto de partida y el punto 2 est
salga de la parte alta del acantilado. Sea el punto 1
en el borde del hoyo, donde v = 17,82 m/s.
en la base de la colina y el punto 2 en la cima de la
Haga y = 0 en el punto 2, tal que colina.
y 2 = 0 e y1 = h Tome y = 0 en la base de la colina, tal que
K1 + U 1 = K 2 + U 2 y1 = 0 e y2 = 28,0 m.
K1 + U 1 = K 2 + U 2

21
Cuerpo rgido Hugo Medina Guzmn

1 2 1 2 1 1 en un eje sin friccin que pasa por su centro. Un


mv1 + I1 = mgy2 + mv22 + I22 bloque de masa M se suspende del extremo libre del
2 2 2 2 hilo. El hilo no resbala en la polea, y el cilindro rue-
Rodar sin resbalar significa = v r y da sin resbalar sobre la mesa. Si el sistema se libera
del reposo, qu aceleracin hacia abajo tendr el
1 2 11 2 1
I = mr (v / r ) 2 = mv 2 bloque?
2 25 5
7 7
mv12 = mgy 2 + mv 22
10 10
10
v 2 = v12 gy 2 = 15,26 m s
7
Considere el movimiento de proyectil de la bola,
despus de salir de la cima del acantilado hasta Solucin.
justo antes de tocar tierra. Tome + y hacia abajo. Hacer este problema usando la cinemtica implica
Utilice el movimiento vertical para encontrar el cuatro incgnitas (seis, contando las dos
tiempo en el aire: aceleraciones angulares), mientras que usando
v0 y = 0 , a y = 9,80 m/s 2 consideraciones de la energa se simplifican los
clculos.
y y 0 = 28,0m , t = ? Si el bloque y el cilindro ambos tienen velocidad v,
la polea tiene velocidad angular v/R y el cilindro
1 tiene velocidad angular v/2R, la energa cintica
y y 0 = v 0 y t + a y t 2 t = 2,39 s
2 total es
Durante este tiempo la bola viaja horizontalmente 1 M ( 2 R) 2 MR2
K = Mv2 + (v 2 R ) 2 + (v R) 2 + Mv2
x x 0 = v0 x t = (15,26 m s )(2,39 s ) = 36,5 m. 2 2 2
Justo antes de tocar tierra,
v y = v0 y + gt = 23,4 m/s y 3
= Mv 2 . (1)
vx = v0 x = 15,26 m/s 2
Esta energa cintica debe ser el trabajo hecho por
v = v x2 + v y2 = 28,0 m s la gravedad; si la masa que cuelga desciende una
distancia y,
K = Mgy. (2)
De (1) y (2):
2
v2 = gy
3
Para aceleracin constante
b) En la base de la colina, v 2 = 2ay,
v 25,0 m s Por comparacin de las dos expresiones obtenemos:
= = g
r r a=
La razn de la rotacin no cambia mientras la bola 3
est en el aire, despus de dejar la parte alta del
acantilado, tal que momentos antes de tocar tierra Ejemplo 38. Una barra de largo 2L y masa M est
15,3 m/s articulada en un extremo a un punto fijo O,
= inicialmente en reposo y horizontal. Si ella se
r suelta, comienza a rotar respecto a la articulacin
La energa cintica total es igual en la base de la
bajo el efecto del peso de la barra. Determine la
colina y momentos antes de tocar tierra, pero
reaccin en la articulacin y la velocidad angular de
momentos antes de tocar tierra poco de esta energa
la barra en funcin del ngulo que ella ha girado.
es energa cintica rotatoria, as que la energa
cintica de traslacin es mayor.

Ejemplo 37. Un cilindro slido uniforme de masa


M y radio 2R descansa en una mesa horizontal. Se
ata un hilo mediante un yugo a un eje sin friccin
que pasa por el centro del cilindro de modo que ste
puede girar sobre el eje. El hilo pasa por una polea Solucin.
con forma de disco de masa M y radio R montada

22
Cuerpo rgido Hugo Medina Guzmn

a) Momento de inercia de la barra con respecto a un


Por conservacin de energa tenemos que extremo
11 2 1
M (2 L ) Mgsen = 0
2
I A = ML2
23 3
Luego la velocidad angular de la barra es: Por conservacin de energa.
2 3g 3g L 11 2 2
= sen = sen Mg = ML
2L 2L 2 23
d2 3g
Adems R H = M L cos , =
dt 2 L
d2 L 1
RV Mg = M 2 ( Lsen ) vCM = = 3gL
dt 2 2
Entonces b) La aceleracin angular en dicho instante.
L
Mg
1 d 2 A 2 = 3g
R H = ML sen = =
2 sen d I A 1 ML2 2 L
3
1 d 2 3 g
= ML sen sen Ejemplo 40. Una barra de longitud 2L y masa M se
2 sen d 2L coloca sobre un plano horizontal liso. Si la barra es
tirada por una fuerza constante F, inicialmente
9 perpendicular a la barra y aplicada en un extremo,
= MLsen cos la barra comienza a moverse sobre el plano. La
4 fuerza se mantiene aplicada a ese mismo extremo
d2 manteniendo su direccin original. Determine una
RV = Mg M 2 ( Lsen ) ecuacin para el ngulo que gira la barra en funcin
dt del tiempo.
1 d 3g
= Mg ML cos sen
2

2 cos d 2L
5 9
= Mg Mg cos 2
2 4
Solucin.
Ejemplo 39. Una barra de longitud L y masa M se
coloca verticalmente sobre un plano horizontal liso,
en reposo. Si ella es perturbada levemente comienza
a caer. Determine:
a) La velocidad del centro de masa de la barra justo
cuando ella se coloca horizontal.
b) La aceleracin angular en dicho instante.

El torque respecto al centro de masa conduce a


1
FLsen = ML2
3
3F
= sen
L
Ejemplo 41. Una barra de longitud L y masa M
Solucin. puede oscilar libremente en torno a uno de sus

23
Cuerpo rgido Hugo Medina Guzmn

extremos que se mantiene fijo, bajo la accin de su puramente energa cintica.


peso. Escriba la ecuacin diferencial para el ngulo 1 2 11 1
que ella gira. K= I = MR 2 2 = MR 2 2
Solucin. 2 22 4
Por conservacin de energa.
1 2 1 2k 2
k = MR =
2 2 2

2 4 MR 2
2k 2
Finalmente =
MR 2

TRASLACIONES Y ROTACIONES
COMBINADAS
Hasta ahora solo hemos tomado en consideracin la
Por conservacin de energa
rotacin del cuerpo en torno a un eje fijo en el
11 2 L espacio.
E= ML2 Mg cos La finalidad de esta seccin es estudiar el caso en
23 2
que el eje de rotacin si acelera tambin vamos a
Derivando respecto al tiempo presentar tres mtodos analticos de resolver este
caso.
1 L
ML2 + Mg sen = 0 Primer mtodo
3 2 Aplicamos la segunda ley de Newton para
Finalmente traslacin relativa ejes no rotantes a travs del

3g centro de masa. Para ilustrar este mtodo y los otros


+ sen = 0 tambin, consideremos un cuerpo de radio R, masa
2L M y momento de inercia respecto a su entro masa I,
al que se le obliga a rodar sin deslizamiento a lo
Ejemplo 42. Un pndulo de torsin consiste en un largo de una superficie horizontal por medio de una
disco uniforme de masa M y radio R suspendido de fuerza F que acta en su centro de masa, La tuerza
una barra delgada y vertical de masa despreciable y
de friccin F f y la reaccin N actan tal como se
que puede torcerse al dar vuelta al disco alrededor
de su eje, como se indica en la figura. La barra tiene muestra en la figura siguiente.
una Constante de elasticidad torsional k.
inicialmente se hace girar el disco un ngulo
respecto del equilibrio y luego se le suelta desde el
reposo. Determinar su velocidad de rotacin cuando
llega nuevamente a la posicin de equilibrio.

EL cuerpo se mueve con una aceleracin horizontal


a que es la que corresponde a su centro de masa, y
a su vez rota con aceleracin angular .
Como rueda sin deslizamiento la relacin entre el
desplazamiento lineal y el desplazamiento angular
es x = R .
Solucin.
Con la ley de Hooke para rotacin,
La velocidad es
= k
dx d
El trabajo para torcer un ngulo es: =R v = R
1 2 dt dt
W = d = ( k )d = k
0 0 2 La aceleracin es
Este trabajo queda como energa potencial. dv d
1 =R a = R
U ( ) = k 2 dt dt
2
Al liberarse esta se convierte en energa cintica. Aplicando la segunda ley de Newton para traslacin
Al pasar por el punto de equilibrio la energa es

24
Cuerpo rgido Hugo Medina Guzmn

F F f = Ma En este caso como la aceleracin del centro masa es


a, la aceleracin angular del cuerpo alrededor de O
Aplicando la segunda ley de Newton para rotacin es = a R .
alrededor del centro de masa
RF f = I CM Aplicando la segunda ley de Newton para
traslacin:
F F f = Ma
Eliminando F f y , obtenemos:
I Aplicando la segunda ley de Newton para rotacin a
M + CM2 a = F alrededor de O:
R
FR = I O
La aceleracin
F Como =a R 2
y I O + I CM + MR :
a= con la segunda ecuacin,
I
M + CM2
R (I CM + MR 2 ) Ra = FR a=
F
I
M + CM2
Si para t = 0: R
x0 = 0 , v0 = 0 , Tercer mtodo
Siendo a = constante Este mtodo Consiste en usar las ecuaciones de la
energa directamente.
La velocidad es:
Es un Sistema Conservativo
v = v 0 + at
K + U = Constante
F Resolveremos por este mtodo el ejemplo anterior.
v= t
(
M + I CM R 2 ) Puesto que no hay deslizamiento la tuerza de
friccin sobre el cuerpo no trabaja sobre el mientras
rueda. Siendo un sistema conservativo la fuerza F
El desplazamiento es: se puede deducir de una funcin Potencial U = - Fx
1 2 donde x es la coordenada horizontal del centro de
x = x0 + v0 t + at nasa.
2
La energa E del cuerpo es:
1 F E = K +U
x= t2
(
2 M + I CM R 2 ) 1 1
K = I CM 2 + Mv 2 , U = Fx
2 2
Segundo mtodo
En este mtodo escribimos la ecuacin para 1 1
Luego: E = I CM + Mv Fx
2 2
traslacin igual que en el anterior mtodo, pero para 2 2
la rotacin se aplica la segunda ley de Newton con v
respecto al eje de rotacin que pasa a travs del Siendo =
punto de reposo instantneo (punto de apoyo en el R
movimiento) si tal punto no existe no puede usarse 1 I
este mtodo E = v 2 CM2 + M Fx
Como ilustracin veamos el ejemplo anterior. El 2 R
punto contacto es el punto fijo instantneo O, De aqu podemos evaluar la velocidad considerando
consideremos que este no desliza y todos los otros que para el instante inicial x = 0, y v = 0, por
puntos de eje momentneamente rotan alrededor de consiguiente E = 0.
el. 1 2 I CM
v 2 + M Fx = 0 y
2 R
1 2 I CM
v 2 + M = Fx
2 R
2 Fx
v=
I CM
+M
R2

25
Cuerpo rgido Hugo Medina Guzmn

Siendo un movimiento con aceleracin constante y F f obtenemos:


Eliminando
v = 2ax Mgsen
De esto a=
F M + I CM R 2
a= Considerando que para t = 0: s = 0, y v = 0.
I CM
+M Mgsen
R2 v = t ,
2
M + I CM R
Otra forma de calcular la aceleracin.
1 Mgsen 2
s = t
Considerando que 2 M + I CM R 2
dE
E = Constante =0
dt Para un anillo:
dE 1 2 I CM 1
= v 2 + M Fx = 0 I CM = MR 2 , s = gsen t 2
dt 2 R 4
Para un disco:
dv I dx 1 1
v CM2 + M F =0 I CM = MR 2 , s = gsen t 2
dt R dt 2 3
dv dx Para una esfera:
Como =a y =v 2 5
dt dt I CM = MR 2 , s = gsen t 2
I 5 14
va CM2 + M Fv = 0 Para un plano sin friccin (sin rodadura)
R 1
F s= gsen t 2
a= 2
I
M + CM2 Por la ecuacin de energa
R
Si para t = 0: K 0 = 0 y U 0 = 0
Ejemplo 43. Analizar el movimiento de un cuerpo E = K0 + U0 = 0
de radio R, momento de inercia respecto a su centro
de masa I que rueda sin deslizar hacia abajo en Llamando h a la cada del centro de masa desde la
plano inclinado de ngulo . posicin de reposo, tenemos:
1 1
K= Mv 2 + I CM 2 ,
2 2
U = Mgh = Mgs sen = 0 ,
=v R
1 2 I
v M + CM2 Mgs sen
2 R

Solucin.
2 Mgsen
v= s
Como se muestra en la figura hay dos fuerzas que M + I CM R 2
actan sobre el cuerpo, Mg acta en el centro de
gravedad y la fuerza de contacto que se Ejemplo 44. Usar la conservacin de la energa
descompone en la reaccin normal N y la fuerza de para describir el movimiento de rodadura de un
friccin Ff. cuerpo rgido de masa M que rueda por un plano
Vamos a resolver por el primer mtodo.
inclinado y rugoso.
Traslacin:
Mgsen F f = Ma
Rotacin:
RF f = I CM
Por la condicin de no deslizamiento:
=a R

26
Cuerpo rgido Hugo Medina Guzmn

Estudiar el movimiento.

Solucin.
Se supone que el cuerpo rgido parte del reposo
desde una altura h y que rueda por el plano sin
resbalar la conservacin de energa da: Solucin.
E = cte K + U g = cte Vamos a resolver primero por las ecuaciones del
movimiento de Newton.
K i + U gi = K f + U gf Traslacin.:
Pero Mg T = Ma
Ki = 0 y Ugf = 0, entonces Rotacin.:
1 1 RT = I CM
Mgh = I cm 2 + Mvcm
2

2 2 Como:
Como 1 a
vcm= R = vcm/R, se reemplaza en la ecuacin I CM = MR 2 , = :
2 R
anterior
1 v2 1 2 1 a 1
I cm cm2 + Mvcm = Mgh RT = MR 2 = MRa
2 R 2 2 R 2
Despejando cm se obtiene: De aqu se obtenemos:
2 gh 1 2
vcm = T= Ma y a = g
I + I cm MR 2 2 3
Por ejemplo, para una esfera slida uniforme de El yo-yo funciona segn este principio, est
proyectado para que a sea mucho menor que g.
2
momento de inercia I cm = MR 2 , se puede
5 Resolviendo por conservacin de la energa
calcular su vcm en el punto ms bajo del plano y su E = K +U =
aceleracin lineal. 2
1 11 v
2 2 gh 2 gh 10 Mv 2 + MR 2 Mgy
vcm = = = gh 2 22 R
1+
(2 5)MR 2
1+
2 7
dE
MR 2 5 Como E = constante =0
dt
10 dy dv
vcm = gh Tambin v = y a=
7 dt dt
La aceleracin lineal se puede calcular con la Con esto encontramos que
ecuacin
2
v2 a= g
v 2
cm =v 2
cmi + 2a cm x a cm = cm 3
2x
De la geometra de la figura, se tiene: h = x sen , Ejemplo 46. Estudiar el movimiento de un disco
donde x es la longitud del plano, reemplazando en homogneo de radio R y masa M, sobre el que acta
acm: una fuerza horizontal F aplicada un punto variable a
5 lo largo de una lnea vertical que pasa por el centro,
gxsen segn se indica en la figura. Supngase el
7 5
a cm = = gsen movimiento sobre un plano horizontal.
2x 7
Ejemplo 45. Un disco homogneo de radio R y
masa M tiene una cuerda enrollada alrededor, segn
vemos en a figura. Sujetando el extremo libre de la
cuerda a un soporte fijo, se deja caer el disco.

27
Cuerpo rgido Hugo Medina Guzmn

(0)
3F f = F 1 2 = F F f =
F
R 3
En la ecuacin (3)
F 1
F (0 ) + R = I CM = MR 2
3 2
F 1
R = MR 2
3 2
2F
=
Solucin.
3MR
En la figura vemos que la fuerza F se aplica a una El cilindro rueda hacia la derecha.
distancia h sobre el centro.
d) Si F se hace muy grande tal que Ff tiende a
Suponiendo que F f acta hacia la izquierda. aumentar, tan pronto como sobrepase el valor
Aplicando las leyes de Newton del movimiento: mximo posible de la tuerza de rozamiento (N), el
Traslacin disco deslizar.
F F f = Ma (1) Se debe hacer una nueva hiptesis, esta vez se
tienen tambin las ecuaciones (1), (2) y (3) pero
N Mg = 0 (2) a R.
Rotacin alrededor del centro de masa
1 Ejemplo 47. Un carrete de radio interior R1 y radio
Fh + F f R = I CM = MR 2 (3) exterior R2 se halla sobre un suelo spero. Se tira de
2
l con una tuerza F mediante un hilo arrollado en
a
Considerando = torno a su cilindro interior. Se mantiene un ngulo
R con la horizontal. Se observa que hay un ngulo
h Crtico 0 , tal que < o , el carrete rueda sin
2F + 2 F f = Ma (3a)
R deslizar en el sentido del cual se tira de l, y para
Igualando (1) y (3a) > o el carrete rueda sin deslizar en sentido
h contrario, Cul es el valor del ngulo critico.
F F f = 2F + 2F f
R
h
3 F f = F 1 2
R
Discusin:
h R
a) F f = 0 , cuando 1 2 =0 h=
R 2
Esto quiere decir si F se aplica a R/2 del centro, la
fuerza de rozamiento es cero.
Solucin.
b) Si h = R Aplicando las leyes de Newton del movimiento;
R F Traslacin:
3F f = F 1 2 = F F f = F cos F f = Ma = MR2 (1)
R 3
el rozamiento es en sentido contrario al indicado y Fsen Mg + N = 0
la ecuacin (3) se convierte en: Rotacin:
F 1 F f R2 + FR1 = I CM
F (R ) R = MR 2
3 2 F f R2 = FR1 I CM
2 1
F = MR R1 I
3 2 Ff = F R1 CM (2)
R2 R2
4F
= Eliminando la fuerza F f ., reemplazando (2) en
3MR
Esto indica que el cilindro rueda hacia la derecha. (1):

c) Si disminuye h hasta que h = 0.

28
Cuerpo rgido Hugo Medina Guzmn

R I 1
F cos F 1 R1 CM = Ma Ff = Ma
2
R2 R2
Que sustituida en la primera da:
R I 2F
F cos F 1 R1 + CM = MR2 a) a = ,
R2 R2 3M
R2 cos R1 MR22 I CM a 2F
F = b) = = ,
R R2 R 3MR
2
(R2 cos R1 ) d 1 F
c) F f = Ma =
=F =
(MR 2
2 I CM ) dt 2 3
Ejemplo 49. Un disco de masa M y radio 2R se
La rotacin har que el movimiento del carrete ser apoya sobre un plano horizontal spero de modo
d que puede rodar sin resbalar con su plano vertical.
hacia adelante cuando >0 El disco tiene un resalto de radio R como se indica
dt
en la figura, en el cual se enrolla una cuerda que se
R2 cos R1 > 0 tira con una fuerza horizontal constante F,
R determine:
cos > 1 a) La aceleracin del centro de masa del disco.
R2 b) La aceleracin angular del disco.
d c) La fuerza de roce.
El movimiento ser hacia atrs cuando <0
dt
R2 cos R1 < 0
R
cos < 1
R2
d Solucin.
El ngulo crtico es cuando =0
dt
R2 cos R1 = 0
R
cos = 1
R2

Ejemplo 48. Un disco de masa M y radio R se


apoya sobre un plano horizontal spero de modo
que puede rodar sin resbalar con su plano vertical.
Si se tira del centro del disco con una fuerza Ahora F F f = Ma , N Mg = 0
horizontal constante F, determine: 1
M (2 R )
2
a) La aceleracin del centro de masa del disco. F f 2 R + FR =
b) La aceleracin angular del disco. 2
c) La fuerza de roce. 2 a
Solucin. = 2 MR = MRa
2R
Simplificando:
2 F f + F = Ma = F F f
Ff = 0
De donde resulta:
F
a) a =
m
Aqu F
b) =
F F f = Ma , N Mg = 0 , 2MR
c) F f = 0
1 1
Ff R = MR 2 = MRa
2 2
Entonces

29
Cuerpo rgido Hugo Medina Guzmn

Ejemplo 50. Un disco de masa M y radio R tiene A partir del punto A en que el piso es spero
enrollada una cuerda en su periferia y cae partiendo deslizar primeramente sobre el plano spero, pero
del reposo mientras la cuerda que se sostiene de su acabar rodando sin deslizar.
extremo se desenrolla. Determine: En la parte intermedia habr una aceleracin a que
a) La aceleracin de bajada del disco. disminuye a la velocidad de v1 a v 2 y una
b) La tensin de la cuerda.
aceleracin angular que disminuye a 1 , la
hace igual a cero y cambia su rotacin hasta que
llega la velocidad angular a un valor tal que
2 = v2 R .
Aplicando las leyes de Newton en la figura
siguiente.

Solucin.

Traslacin: N = Ma , N Mg = 0
1
Rotacin: RN = I CM = MR 2
2
2 g
Aqu Mg T = Ma , De esto obtenemos: a = g , =
R
1 1 La velocidad es: v = v1 + at = v1 gt
TR = MR 2 = MRa
2 2 La velocidad angular es:
1 v1 2 g
De donde Mg Ma = Ma = 1 t = t
2 R R
2 Parta encontrar el tiempo en que el disco deja de
a) a = g
3 resbalar, debe cumplirse: v = R
1 1
b) T = Ma = Mg vi = k Rj = Ri
2 3
Ejemplo 51. Se da a un cilindro homogneo de
(v1 gt ) = v1 2g t R
R R
radio R y masa M con una velocidad horizontal v1
2 v1
y una velocidad angular 1 en sentido opuesto a 2v1 = 3gt t =
3 g
las agujas del reloj 1 = v1 R en la parte sin con este valor de t
rozamiento de la superficie horizontal. Ms all del 2 v1 v1
punto A, cambia la superficie de manera que a la v 2 = v1 g =
derecha de A el coeficiente de rozamiento es . 3 g 3
La velocidad final es un tercio de la inicial

Solucin.
En la parte lisa el cuerpo se mueve con velocidad
horizontal constante v1 hacia la derecha, rotando Ejemplo 52. Se lanza una bola de billar con una
con velocidad angular 1 en el sentido antihorario. velocidad inicial v 0 sobre una mesa horizontal,

30
Cuerpo rgido Hugo Medina Guzmn

existiendo entre la bola y la mesa un coeficiente de 5gt 2v


rozamiento . Calcular la distancia que recorrer v0 gt = t= 0
hasta que empiece a rodar sin deslizamiento. 2 7 g
Qu velocidad tendr en ese instante? la velocidad en ese instante es
Aplicar para el caso v 0 = 7 m/s, = 0,2. 5
v= v0 = 5 m/s, t = 1,02 s
7
La distancia recorrida
1
x = v0 t gt 2
2
Solucin. 2
La fuerza de rozamiento N = mg se opone al 2v02 1 2v0 12v02
movimiento, siendo adems la fuerza resultante, por
= g =
7 g 2 7 g 49g
lo que:
mg = ma , a = g = 6,12 m.
La velocidad de la bola comenzar a disminuir de Ejemplo 53. Un tambor tiene un radio de 0,40 m y
tal modo que: un momento de la inercia de 5,0 kg m2. El torque
v = v0 at = v0 gt . producido por la fuerza de friccin de los cojinetes
Al mismo tiempo, sobre la bola que inicialmente no de anillo del tambor es 3,0 Nm. Un anillo en un
rueda, (0 = 0) acta un momento de fuerza: extremo de una cuerda se desliza en una clavija
corta en el borde del tambor, y una cuerda de 15 m
de longitud se enrolla sobre el tambor. El tambor
est inicialmente en reposo. Una fuerza constante
se aplica al extremo libre de la cuerda hasta que la
cuerda se desenrolla y se desliza totalmente de la
clavija. En ese instante, la velocidad angular del
= F f R = mgR tambor es de 12 rad/s. El tambor despus decelera y
se detiene.
que producir una aceleracin angular =
I
mgR 5g
= = =
I 2 2R
mR 2
5
Por lo que la velocidad angular ir aumentando:
5gt
= t = a) Cul es la fuerza constante aplicada a la cuerda?
2R b) Cul es la cantidad de movimiento angular del
tambor en el instante en que la cuerda deja el
La velocidad de un punto de la periferia de la esfera tambor?
vale v P = R , que ir aumentando con el tiempo, c) Cul es el trabajo negativo realizado por la
porque aumenta con el tiempo. friccin?
d) Qu tiempo el tambor estuvo en movimiento?
Movimiento con la cuerda?
Solucin.
a)
Trabajo de la fuerza F + trabajo de la friccin
= Energa cintica ganada al terminarse la cuerda
1
Fs + f = I O 2
2
15 1
F (15) 3,0 = (5,0 )(12)
2

0,4 2
Por tanto, observamos que la velocidad de la bola F = 31,5 N
disminuye, y la velocidad de la periferia de la bola b)
aumenta. En el momento en que la velocidad de la L = I O = (5)(12)
periferia se iguale a la velocidad de traslacin, se = 60 kg.m2/s
conseguir la rodadura, es decir el no deslizamiento. c)
v = v P v = R Movimiento con la cuerda

31
Cuerpo rgido Hugo Medina Guzmn

15 m
W f 1 = f = 3 = - 112,5 J = 0,75
0,4 s2
Movimiento sin la cuerda mg T = ma
1 1 T 20
W f 2 = I O O2 = (5,0)(12) = 360
2
m= =
2 2 g a 9,8 0,75
Trabajo total = 2,21 kg
W f = W f 1 + W f 2 = 482,5 J b)
d) a 0,75
= =
O = I O R 0,4
FR f I O 31,5(0,4 ) 3,0 = 5,0 rad
= 1,875 2
s
31,5(0,4) 3,0 rad
1 =
5,0
= 1,92
s O = I O TR = I O
Por otra parte TR 20(0,4 )
IO = =
0 12 1,875
o = 1t1 t1 = = = 6,25 s
1 1,92 = 4,27 kg m
2

Movimiento sin la cuerda


O = I O 3 = 5 2 Ejemplo 55. El radio de una rueda de 3,0
kilogramos es 6,0 centmetros. La rueda se suelta
3 rad del reposo en el punto A en un plano inclinado 30.
2 == = 0,6 La rueda gira sin deslizar y se mueve 2,4 m al
5 s
punto B en 1,20s.
0 = 0 + 2t 2 a) Cul es el momento de inercia de la rueda?
0 12 b) Cul es la aceleracin angular de la rueda?
t2 = = = 20s
2 0,6
El tiempo total es 26,25 s

Ejemplo 54. Una rueda tiene un radio de 0,40 m y


se monta en cojinetes sin friccin. Un bloque se
suspende de una cuerda que se enrolla en la rueda.
La rueda se libera de reposo y el bloque desciende Solucin.
1 1
1,5 m en 2,00 segundos. La tensin en la cuerda
mR 2 = (3kg )(0,06m )
2
a) I O =
durante el descenso del bloque es 20 N. 2 2
a) Cul es la masa del bloque? = 0,0054 kg m2
b) Cul es el momento de inercia de la rueda? b)
mgsen30 Ff = ma F f R = I O
I
F f = O
R
I
mgsen30 O = mR
R
mgsen30 3(9,8)(0,5)
= =
IO 0,0054
Solucin. + mR + 3(0,06)
R 0,06
a) 14,7 rad
= = 54,4 2
1 2 0,27 s
h= at
2
2h 2(1,5) Ejemplo 56. Una masa de 20 kg se halla sobre un
a= 2 = plano inclinado 30, con el que tiene un rozamiento
t (2)2 cuyo coeficiente vale 0,3, unida a una cuerda sin
masa e inextensible que pasa por una polea de MP =
160 kg, cuyo radio geomtrico es de 20 cm y radio

32
Cuerpo rgido Hugo Medina Guzmn

de giro rg = 15 cm. De dicha cuerda pende una = 10 + 5,2 = 15,2 kg.


masa de 40 kg que es abandonada libremente.
Calcular:
a) Aceleracin con que se mueve el sistema.
b) Tensiones en la cuerda.
c) En qu rango de valores de la masa que pende,
el sistema estar en equilibrio?
2
Momento de inercia de la polea I P = Mrg .
Solucin.
a) Partiendo de la suposicin de que la masa
colgante acelera hacia abajo, plantearemos las tres
ecuaciones correspondientes al movimiento de las Si la masa m2 se hace an menor, llegar un
tres masas: momento en que ser arrastrada por m1. Esto
m2g - T2 = m2a producira una inversin en el sentido de la fuerza
de rozamiento. El valor mximo de m2 deber
cumplir ahora:
m2 = m1sen + m1 cos
= 10 5,2 = 4,8 kg.
Por tanto, entre 0 y 4,8 kg el sistema acelerar de
modo que m2 suba; entre 4,8 y 15,2 kg,
permanecer en equilibrio; y para ms de 15,2 kg
m2 acelerar hacia abajo.
T1 m1gsen + m1 g cos = m1 a ,
a Ejemplo 57. Porqu una esfera que rueda se
T2 R T1 R = I = M P rg2 detiene? En esta parte vamos a tratar de explicar la
R resistencia al rodamiento.
Sumando las tres ecuaciones siguientes La figura siguiente muestra una esfera de masa M y
m0 g T2 = m2 a , radio R la cual est rodando con una velocidad
angular y avanza con una velocidad v = R .
T1 m1gsen + m1 g cos = m1 a
2
rg
T2 T1 M P a
R
Obtenemos:
m2 g m1gsen + m1 g cos
rg
2

= a m1 + m2 + M P
R
m2 m1sen + m1 cos
a= 2
g Solucin.
rg La fuerzas que actan sobre la esfera son el peso
m1 + m2 + M P
Mg 1a reaccin del piso N y la fuerza de friccin
R
40 10 5,2 F f . Si aplicamos la segunda ley de Newton a la
= 2
g traslacin.
15
60 + 160 Ff = M g
20
= 1,62 m/s2

b) debe haber una aceleracin a y v decrecera. Si


T2 = m2 ( g a ) = 327 N, aplicamos segunda ley de Newton a la rotacin.
2
RF f = I CM
rg la aceleracin angular depende de Ff. por
T1 = T2 a = 181 N.
R consiguiente Ff acta incrementando .
c) El valor mnimo que hace que la masa m2 acelere En resumen: en traslacin Ff. acelera, en rotacin
hacia abajo se produce cuando a = 0, es Ff. desacelera, esto aparentemente es una
decir: contradiccin.
Por otra parte Mg y N estn en la lnea vertical que
m2 = m1sen + m1 cos por el centro de masa y no causan efecto en el

33
Cuerpo rgido Hugo Medina Guzmn

movimiento horizontal.
Si la esfera y el plano son rgidos, de modo que la
esfera est en contacto solo en un punto, tampoco
originan alrededor del centro de masa. .porque
actan a travs de l
Para resolver la Contradiccin suprimamos la
idealizacin de que todos los cuerpos son rgidos, la
esfera se aplana un poco y el nivel de La superficie
se hunde Ligeramente (ver la figura a continuacin)

Sobre la varilla acta el peso Mg y la reaccin R.


La velocidad angular en este instante se puede
encontrar aplicando la ecuacin de la energa.
L L 1
Mg = Mg cos + I O 2
2 2 2
1 2
Como I O = ML
La reaccin N acta delante del centro de masa, 3
produciendo un torque N = dN de resistencia al L L 11
Mg = Mg cos + ML2 2
rodamiento. 2 2 23
N RF f = I CM 3g
a
2 = (1 cos )
Como N = Mg , F f = Ma , = :
L
R 6g 3g 2
a = sen = 2sen
N RMa = I CM L 2 L 2
R 6g
I = sen 2
N = a CM + RM L 2
R Aplicando la segunda Ley de Newton para
traslacin a lo largo de la varilla.
2 2
Para una esfera: I CM = MR L
5 F = ma c R Mg cos = M 2
2
7
Luego: N = MRa , como N = Mg Cuando La varilla deja de presionar R = 0, y:
5 L
N 7R Mg cos = M 2
d= = a 2
Mg 5 g reemplazando el valor de 2 encontrado

Ejemplo 58. La figura muestra una varilla


6g L
homognea de masa M y longitud L en posicin Mg cos = M sen 2
vertical. La cual se deja caer desde el reposo. L 22
a) A que ngulo entre la varilla y la vertical, la Simplificando
varilla ya no presionar al piso?
b) Con qu coeficiente de friccin el extremo de cos = 6 sen 2 cos 2 = 5 sen 2
La varilla no resbalar hasta este momento? 2 2 2
5
tan =
5
De aqu: = 48,2
b) Para que la varilla no resbale tenemos en la
figura siguiente.
Las componentes de R son:

R = Rsen i + R cos j
Solucin.
a) La figura siguiente muestra .la varilla cuando
forma un ngulo con la vertical.

34
Cuerpo rgido Hugo Medina Guzmn

La condicin para que la varilla no resbale es:


F f Rsen Por la conservacin de la cantidad de movimiento
angular.
Con F f = N y N = R cos I1
R cos Rsen I 2 2 = I 11 2 = 1
I2
tan Siendo I 2 < I 1 , resulta 2 > 1
El coeficiente de rozamiento del piso debe ser
Su velocidad aumenta.
cuando menos igual a tan para que llegue sin
.
deslizar hasta el ngulo Ejemplo 60. Esta vez el mismo estudiante sentado
Para = 48,2 1,12 sobre el mismo banco, sostiene en sus manos en
posicin vertical al eje de rotacin de una rueda de
bicicleta, la rueda gira alrededor de ese eje vertical
CONSERVACION DE LA CANTIDAD DE
MOVIMIENTO ANGULAR. con velocidad angular 0 , el estudiante y el banco
Anteriormente hemos visto que: estn en reposo (a).
El estudiante gira el eje de la rueda en ngulo
dp dL con la vertical (b), como no hay torque respecto al
F= y tambin =
dt dt eje vertical, la cantidad de movimiento angular con
y mostramos que para un cuerpo rgido. respecto al eje vertical debe conservarse.

d L total
ext =
dt
Si no hay torque externo con respecto a algn eje la
cantidad de movimiento angular ser constante con
respecto a ese eje.

L total = Constante
o expresado en funcin del momento de inercia
apropiado.

I = Constante
Esta relacin nos va a ser muy til como veremos a
continuacin. Inicialmente se tiene

Ejemplo 59. Un estudiante est sentado sobre un L = I 0 0 k
banco giratorio montado sobre cojinetes sin friccin
que puede girar libremente alrededor de un eje Cuando se inclina la rueda (respecto al eje vertical)

vertical como se muestra en la figura (a). El
L' = L estudiante +banco + L rueda
estudiante sostiene en las manos extendidas dos
pesas. Su momento de inercia en esta posicin es I1 = I e e + I 0 0 cos k
y su velocidad angular 1 . No actan sobre l
Siendo I e el momento de inercia del estudiante y
torques no equilibrados y en consecuencia su
cantidad de movimiento angular tiene que banco respecto al eje vertical, e su velocidad
conservarse. angular con respecto a ese eje.
Cuando el estudiante acerca las manos al cuerpo, su
momento de inercia vara, figura ( b) ahora es I2 y Como L = L'
su velocidad angular ser 2
I e e + I 0 0 cos k = I 0 0 k

35
Cuerpo rgido Hugo Medina Guzmn

I de 2 rad/s2 durante 6 segundos, momento en el cual


e = 0 0 (1 cos ) k encoge los brazos y acerca sus piernas al eje hasta
Ie tener un momento de inercia de 4 kg.m2 .
Es la velocidad angular del estudiante con el sentido Determinar su velocidad de giro final.
de giro inicial de la rueda.
Cuando la rueda se invierte se invierte totalmente
= 2 , y:
2I 0
e = 0 k
Ie

Ejemplo 61. Una persona est sentada en una silla


giratoria manteniendo los brazos extendidos con
una pesa en cada mano. Gira con una frecuencia de
2 Hz. El momento de inercia de la persona con los Solucin.
pesos es de 5 kg m2. Hallar:
a) la nueva frecuencia cuando encoja los brazos y Despus de un tiempo t de iniciar el giro, su
disminuya el momento de inercia a 2 kg m2. velocidad angular ser:
b) La variacin de energa cintica del sistema.
c) De dnde procede este incremento de energa
1 2 1
cintica? at = (2 )(6 ) = 36 rad/s
2
(t ) =
2 2

al acercar brazos y piernas al eje, el torque de las


fuerzas sigue siendo nulo, por lo que se conserva la
cantidad de movimiento angular, I

(I )Antes = (I )Despus
Solucin. I Antes
a) Al encoger los brazos, estn actuando fuerzas y Despus = Antes
torques de fuerzas internas, por lo que podemos I Despus
admitir que se conserva la cantidad de movimiento
angular.
7
L1 = L2 I 11 = I 2 2 = 36 = 63 rad/s
4
I I
2 = 1 2 , 2 f 2 = 1 2 f1 ,
I2 I2 Ejemplo 63. Un muchacho de 25 kg corre con
velocidad de 2,5 m/s hacia un tiovivo en reposo de
I 5 radio 2 m cuyo, momento de inercia vale 500 kg
f 2 = 1 f1 = 2 = 5 Hz
I2 2 m2. Hallar la velocidad angular y frecuencia del
conjunto despus de que el muchacho suba al
1 1 L2 L2 tiovivo justo en el borde.
b) K = I 2 22 I 112 =
2 2 2I 2 2I1
L = I 11 = 5(2 2 ) = 20 kg m 2 s ;
1 1
K = 200 2 = 60 2 J .
2 5
El signo positivo nos indica que hay un aumento de
energa cintica.
c) Este incremento de energa cintica procede de la
energa qumica almacenada en los msculos del
brazo. Solucin.
La cantidad de movimiento angular del muchacho
Ejemplo 62. Un patinador, con los brazos respecto al centro del tiovivo es:
extendidos y las piernas abiertas y con un momento
L1 = mvR = (25)(2,5)(2 ) = 125 kg m2/s
de inercia respecto a su eje vertical de 7 kg.m2 ,
inicia un giro sobre si mismo con una aceleracin

36
Cuerpo rgido Hugo Medina Guzmn

El momento de inercia del conjunto tiovivo- IO


2,0
muchacho es ' = = 1,5
I = Im + IT = 25x22 + 500 I 'O
2,256
= 600 kg m2
rad
Planteando la igualdad entre la cantidad de = 1,33
movimiento angular inicial y final, tendremos: s
L1 = L2 , mvR = (I m + I T ) Reemplazando:
1 1
I 'O ' 2 = (2,256)(1,33)
2
mvR 125 Energa despus =
= = 2 2
(I m + I T ) 600 =2J
= 0,208 rad/s Se pierde 4,05 -2 = 2,05
2,05
f = = 0,033 Hz fraccin de energa = = 0,5
2 4,05
= 1,99 r.p.m.
Ejemplo 65. Una barra rgida de masa M y largo L
Ejemplo 64. Una tornamesa con radio de 8,0 m y gira en un plano vertical alrededor de un eje sin
momento de inercia de 2,0 kg.m2. La placa friccin que pasa por su centro. En los extremos de
tornamesa rota con una velocidad angular de 1,5 la barra se unen dos cuerpos de masas m1 y m2.
rad/s sobre un eje vertical que pasa a travs de su Calcular la magnitud del momento angular del
centro en cojinetes sin friccin. Una bola de 0,40 sistema cuando su rapidez angular es y la
kg se lanza horizontalmente hacia el eje de la aceleracin angular cuando la barra forma un
tornamesa con una velocidad de 3,0 m/s. La bola es
ngulo con la horizontal.
cogida por un mecanismo con forma de tazn en el
borde de la tornamesa.
a) Cul es cantidad de movimiento angular de la
bola alrededor del eje de la tornamesa?
b) Qu fraccin de energa cintica se pierde
durante la captura de la bola?

Solucin.
El momento de inercia por el eje de rotacin del
sistema es igual a la suma de los momentos de
inercia de los tres componentes, con los valores de
la tabla se obtiene:
2 2
1 L L
I = ML2 + m1 + m2
12 2 2
L2 M
Solucin. = m1 + m2 +
a) La cantidad de movimiento angular de la bola 4 3
alrededor del eje de la tornamesa es cero Como el sistema gira con rapidez angular , la
b) magnitud del momento angular es:
1 2 1 L2 M
Energa antes = mv + I O 2 L = I = m1 + m2 +
2 2 4 3
1 1 Para calcular la aceleracin angular usamos la
= (0,4 )(3,0 ) + (2,0 )(1,5)
2 2
relacin
2 2
= 4,05 J t
t = I = , al calcular el torque total
1 I
Energa despus = I ' O '
2

2 en torno el eje de rotacin, se obtiene:


Para calcular esta energa necesitamos conocer I0 y L L
t = m1 g cos m2 g cos
. 2 2
I ' O = I O + mR 2 = 2,0 + (0,4)(0,8)
2
1
= 2,256 kg/ m 2 = (m1 m2 )gL cos
2
Lantes = Ldespus I O = I 'O '

37
Cuerpo rgido Hugo Medina Guzmn

Reemplazando en los valores de I y de t , se a) El momento de inercia ser la suma del momento


de inercia de una varilla, ms el de la esfera. Como
obtiene la aceleracin angular: la esfera est a L+R del eje, aplicamos Steiner:
t 2(m1 m2 )g cos 2 1
= = Ie = me R 2 + me (L + R ) , I V = mV L2
2
I L(m1 + m2 + M 3) 5 3
I = I e + IV
Ejemplo 66. En la figura las masas m1 y m2 se
2 1
= me R + me (L + R ) + mV L
conectan por una cuerda ideal que pasa por una 2 2 2
polea de radio R y momento de inercia I alrededor 5 3
de su eje. La mesa no tiene roce, calcular la 2 1
I = (0,25)(0,1) + (0,25 )(0,85) + (0,5)(0,75)
2 2 2
aceleracin del sistema.
5 3
Solucin. Primero se calcula en momento angular = 0,27 kg.m2
del sistema de las dos masas ms la polea:
2
b) L = I = 0,27 = 0,27( 2f )
T
12
= 0,54 = 0,345 kgm2 / s
60
Ejemplo 68. Un cilindro de 50 kg y 20 cm de radio,
v gira respecto de un eje vertical que coincide con su
L = m1vR + m2 vR + I eje de simetra, debido a una fuerza constante,
R
Luego se calcula el torque externo sobre el sistema, aplicada a su periferia que, despus de 40 s de
la nica fuerza externa que contribuye al torque iniciado el movimiento, alcanza 200 r.p.m.
total es m1g, entonces el torque es Calcular:
El valor de la fuerza y el torque de la fuerza
= m1 gR . aplicada.
Entonces se tiene:
dL
=
dt
d v
m1 gR = (m1 + m2 )vR + I
dt R
dv I dv
m1 gR = (m1 + m2 )R +
dt R dt
I Solucin.
m1 gR = m1 + m2 + 2 Ra La frecuencia de rotacin adquirida vale:
R
200
m1 g f = Hz
a= 60
I La velocidad angular:
m1 + m2 + 2
R 200 20 rad
= 2f = 2 =
60 3 s
Ejemplo 67. Una varilla de 500 g y 75 cm de La aceleracin angular:
longitud, lleva soldada en un extremo una esfera de
10 cm de radio y 250 g de masa. Calcular: rad
= =
a) El momento de inercia cuando gira, alrededor de t 6 s2
un eje perpendicular a la varilla que pasa por el Por otra parte el momento de inercia del cilindro
extremo libre. vale:
b) La cantidad de movimiento angular del conjunto 1 1
mR 2 = (50)(0,2 ) = 1 kgm2.
2
si gira a 12 rpm. I=
2 2
Luego el torque de la fuerza aplicada

= FR = I = (1) = 0,52 Nm.
6
La fuerza tangencial:
0,52
Solucin. F= = = 2,6 N
R 0,2

38
Cuerpo rgido Hugo Medina Guzmn

Ejemplo 69. Un anillo de masa M y radio R (ICM = 1


MR2), cae en rodadura pura sobre un plano longitud l I CM = Ml 2 se sostiene de un
inclinado que forma un ngulo con la 12
horizontal. extremo mediante un pivote sin friccin. La barra se
a) Hacer el DCL. del anillo. encuentra inicialmente en reposo en forma vertical
b) Hallar la aceleracin del centro de masa del cuando un proyectil de masa m impacta sobre ella y
anillo. queda incrustado instantneamente. La velocidad
c) Encontrar el valor de la friccin entre el plano inicial del proyectil es v 0 . Hallar:
inclinado y el anillo. a) La cantidad de movimiento angular del sistema
d) Cul debe ser el mnimo valor del coeficiente de respecto del pivote justo antes de la colisin.
rozamiento esttico entre el plano y el anillo para
que este se encuentre en rodadura pura? b) La velocidad angular de giro del sistema despus
que el proyectil se incrusta en la barra.
c) La altura mxima que alcanzar el CM de la
barra.
d) El trabajo del proyectil cuando se incrusta contra
la barra.

Solucin.
a) El DCL. del anillo.

Solucin.
a) La cantidad de movimiento angular del sistema
b) Segunda ley de Newton para la traslacin respecto del pivote justo antes de la colisin.
Mgsen F f = Ma Lantes = mv0 d
b) La velocidad angular de giro del sistema despus
Segunda ley de Newton para la rotacin
que el proyectil se incrusta en la barra.
a
I = F f R MR 2 = Ff R
R
F f = Ma
Reemplazando el valor de Ff en la primera
ecuacin.
Mgsen Ma = Ma Mgsen = 2Ma
1
Finalmente a = gsen
2
c) El valor de la fuerza de friccin entre el plano
inclinado y el anillo.
1 Lantes = Ldespus
F f = Ma = Mgsen
2 1
d) El mnimo valor del coeficiente de rozamiento mv0 d = Ml 2 + (d )d
esttico entre el plano y el anillo para que este se 3
encuentre en rodadura pura debe de cumplir
1 mv0 d
Ff = k N = Mgsen =
2 1 2 2
Ml + md
Mgsen 1 3
k = = tan
2 Mg cos 2 c) La altura mxima que alcanzar el CM de la
barra.
Energa justo despus del choque
Ejemplo 70. Una barra uniforme AB de masa M y

39
Cuerpo rgido Hugo Medina Guzmn

1 l velocidad angular del bloque despus del choque.


= I O 2 Mg + mgd
2 2
1 2 2
= I O = Ml + md
3
Energa cuando alcanza el punto ms alto
l
= Mg + mgd (1 cos )
2
Por conservacin de energa: Solucin.
Energa justo despus del choque = energa cuando Cantidad de movimiento angular antes del choque
alcanza el punto ms alto. con respecto al eje O.

1 l L antes = r p = rmv0sen k = mbv0 k
I O 2 Mg + mgd
2 2 Para encontrar la cantidad de movimiento angular
l despus del choque, segn la figura siguiente.
= Mg + mgd (1 cos )
2
1 l
I O 2 = Mg + mgd cos
2 2
1
I O 2
cos = 2
l
Mg + mgd
2
[ ]

(mv0 d ) 2 L despus = I p + (m + M )b 2
1 1 2 2
Ml + md 2 Por conservacin de la cantidad de movimiento
23 1 2
2
Ml + md angular
= 3

l L antes = L despus
Mg + mgd
[ ]

2 rmv0 sen k = I p + (m + M )b 2
= m 2 v 02 d 2 rmv0 sen
l 1 = k
2 M + md Ml 2 + md 2 g
2 3 [
I p + (m + M )b 2 ]
hmx = l(1 cos )
Ejemplo 72. Se tiene una plataforma circular que
puede rotar sin friccin alrededor de un eje
d) El trabajo del proyectil cuando se incrusta contra perpendicular al centro. E1 momento de inercia de
la barra.
la plataforma con respecto al eje es I p . Un insecto
1 l
W = E = mv 02 Mg mgd de masa m se coloca sobre la plataforma a una
2 2 distancia b del eje. El sistema se hace girar con una
1 l velocidad angular 0 en el sentido horario. El
- I O 2 + Mg mgd
2 2 insecto empieza a correr en una circunferencia de
radio b alrededor del eje con una velocidad de
1 2 1
= mv0 I O
2
magnitud constante v 0 , medida relativa a tierra.
2 2
a) Cual es la cantidad de movimiento angular total
Ejemplo 71. Un bloque de masa M se pega a una si el insecto corre con la plataforma?
plataforma circular, a una distancia b de su centro. b) Cul ser si corre en oposicin a la rotacin de
La plataforma puede rotar, sin friccin, alrededor de la plataforma?
c) Es posible que el pequeo insecto pueda detener
un eje vertical alrededor de su centro. Siendo I p su la gran plataforma? Cmo?
momento de inercia con respecto a sta. Si un Solucin.
proyectil de masa m que se mueve con una La cantidad de movimiento angular del sistema
velocidad horizontal v 0 , como se muestra en la antes que el insecto comience a correr es:
figura, incide y queda en el bloque. Encontrar la

40
Cuerpo rgido Hugo Medina Guzmn

Resolver usando la conservacin de la cantidad de


( ) ( )

L = I p + mb 2 0 = I p + mb 2 0 k movimiento angular.

a) Cuando el insecto corre en el mismo sentido del Solucin.


giro con mdulo de velocidad v 0 su cantidad de En la parte lisa no hay fuerza de friccin, en la parte
spera aparece la tuerza de friccin, cuya lnea de
movimiento angular es: accin est en el plano. Por tanto, la cantidad de
L' = (I p + mb 2 ) ' mbvo k

movimiento angular del disco respecto a un punto
de referencia en el plano permanecer Constante
Pero como la cantidad de movimiento angular es durante todo el movimiento (por ejemplo A).
constante. La cantidad de movimiento angular total La cantidad de movimiento antes de llegar a A.
es:

L' = L = (I p + mb 2 ) 0 k


L = r M v1 = I 0 1

Como r v1 = rv1sen k = Rv1 k ,
b) En este caso, como en el caso anterior 1 v
I0 = MR 2 , 1 = 1 k = 1 k
L' = L 2 R
( ) 1 1

L' = I p + mb 2 0 k L = MRv1 k + MRv1 k = MRv1 k
2 2
c) Si es posible, tomando el caso a) La cantidad de movimiento angular despus de
( )

pasar A y haber 1legado a rodar sin deslizar. Se
L' = I p + mb 2 ' mbvo k
v2
(
= I + mb k
2
p ) 0
traslada con velocidad v 2 tal que 2 =
R
.

La plataforma se detiene cuando ' = 0 , es decir:


(
mbv0 k = I p + mb 2 0 k )
Esto sucede cuando

v0 =
(I p + mb 2 )
0
mb
En el sentido indicado en el caso a).

Ejemplo 73. Se da a un cilindro homogneo de L' = r M v 2 = I 0 2
radio R y masa M con una velocidad horizontal v1
y una velocidad angular 1 en sentido opuesto a Como r v 2 = rv 2 sen k = Rv 2 k ,
las agujas del reloj 1 = v1 R en la parte sin 1 v
I0 = MR 2 , 2 = 2 k = 2 k
rozamiento de la superficie horizontal. Ms all del 2 R
punto A, cambia la superficie de manera que a la 1 3
derecha de A el coeficiente de rozamiento es . L ' = MRv 2 k MRv 2 k = MRv 2 k
2 2

41
Cuerpo rgido Hugo Medina Guzmn


Igualando L' = L , tenemos: 1 R
2

= I f = MR 2 + m f
3 1 v 2 4
MRv 2 k = MRv1 k v 2 = 1
2 2 3 1
MR 2 + mR 2
Ejemplo 74. Un proyectil de masa m y velocidad f = 2 2
0
v0 se dispara contra un cilindro slido de masa M y 1 2 R
radio R. El cilindro est inicialmente en reposo MR + m
montado sobre un eje horizontal fijo que pasa por
2 16
su centro de masa. El proyectil se mueve M
+m
perpendicular al eje y se encuentra a una distancia 1 2 0
D < R sobre el eje. Calcular la rapidez angular del =
2 M m
sistema despus que el proyectil golpea al cilindro y +
queda adherido a su superficie. 2 16
b)
1 11
Ki = I i02 = MR 2 + mR 2 02
2 22
1
= (M + 2m )R 202
4
Solucin.
1 1 R
2
El momento angular del sistema se conserva, 1
Kf = I f 2f = MR 2 + m 2f
entonces 2 2 2 4
Li = L f = 1 1 R
2
M 2 + m 2
2
2
MR + m 0
1 2 2 4 M 2 + m 16
mv0 D = I = MR 2 + mR 2 2
2 = 1 M + m M + 2m R 2 2
0
4 8 M + m / 8
mv0 D
= = 1 M + 2m (M + 2m )R 2 2
1 0
MR 2 + mR 2 4 M + m/8
2 Como M + 2m > 1
M + m/8
Ejemplo 75. Un disco de masa M y radio R gira en La energa rotacional aumenta.
un plano horizontal en torno a un eje vertical sin
roce. Un gato de masa m camina desde el borde del Ejemplo 76. La barra horizontal de la figura tiene
disco hacia el centro. Si la rapidez angular del un momento de inercia respecto al eje de rotacin
sistema es 0 cuando el gato est en el borde del de 5x10-3 kg m2, y cada una de las bolas que pueden
disco, calcular: deslizar sobre ella pesan 50 g y se consideran de
a) la rapidez angular cuando el gato ha llegado a un dimensiones despreciables. El conjunto est girando
punto a R/4 del centro, libremente alrededor del eje O-O con las bolas
b) la energa rotacional inicial y final del sistema. dispuestas simtricamente respecto al eje y sujetas
Solucin. por un hilo AB de 20 cm. Si se rompe el hilo
Llamando Id al momento de inercia del disco e Ig al cuando el conjunto gira a 20 rad/s, determinar la
momento de inercia del gato, el momento de inercia nueva velocidad angular cuando las bolas lleguen a
total inicial y final del sistema es: los topes del extremo de la barra.
1
Ii = Id + I g = MR 2 + MR 2
2
2
1 R
I f = MR 2 + m
2 4
a) Como no hay torques externos sobre el sistema
en torno al eje de rotacin, se puede aplicar la
conservacin de la cantidad de movimiento angular
I ii = I f f
1 2
I f = 2 MR + mR 0
2

Solucin.

42
Cuerpo rgido Hugo Medina Guzmn

Empecemos calculando el momento de inercia del 2


conjunto, cuando las bolas estn separadas 20 cm. f = 180 = 120 rpm.
(2 + 1)
I1 = Ibarra + Ibolas = Ibarra + 2 m r12 = 5x10-3 kg m2 +
0,1x0,12 = 6x10-3 kg m2 GIROSCOPOS Y TROMPOS - MOVIMIENTO
Cuando se alejen hasta los topes: DE PRECESION
I2 = Ibarra + Ibolas = Ibarra + 2 m r22 El girscopo es una rueda montada en rodamientos
= 5x10-3 kg m2 + 0,1x0,252 sin friccin, en tal forma que la rueda tiene libertad
de rotar en cualquier direccin con respecto al
= 11,25x10-3 kg m2 marco que lo sujeta.
La rotura del hilo libera fuerzas exclusivamente Para lograr esto se necesitan tres gmbalos
internas, por lo que se conservar la cantidad de (correspondientes a los tres espacios
movimiento angular del sistema: dimensionales). Como los rodamientos no tienen
L1 = L2 I 11 = L2 2 friccin no se ejercen torques sobre la rueda. Esto
significa que una vez iniciado el giro, el eje de
I 6 rotacin permanecer fijo no importando que
2 = 1 1 = 20 = 10,67 rad / s
movimiento se de al mareo exterior. La direccin en
I2 11,25
el espacio del eje no variar.
Hasta ahora vimos el movimiento rotacional en que
Ejemplo 77. Un disco de 2 kg de masa y 10 cm de el eje de rotacin est fijo, o tiene movimiento de
radio gira alrededor de su eje a 180 r.p.m.. Encima, traslacin sin cambio en su direccin. La mayora
pero sin que exista contacto, se encuentra otro disco de los movimientos rotacionales quedan en estas
de 1 kg de masa, del mismo radio y en reposo. categoras, pero en el caso de un trompo o
Cuando el disco superior se deja caer, ambos se girscopo en rotacin no se cumple lo anterior. Si
mueven solidariamente. Calcular la velocidad se hace girar rpidamente el rotor de este aparato y
angular final. luego se coloca un extremo libre del eje de rotacin
sobre un soporte fijo, como se muestra en la figura.
El girscopo no caer del soporte sino que se
mantiene en posicin casi horizontal mientras que
el eje de su rotor gira lentamente en un plano
horizontal, esta rotacin lenta del eje se conoce
como PRECESION.

Solucin.

Cuando el disco superior se posa sobre el inferior,


el torque de las fuerzas sigue siendo nulo por lo que
se conserva la cantidad de movimiento angular,
I .
Veamos como se origina la precesin.
(I )Antes = (I )Despus Consideremos un girscopo simplificado mostrado
en la figura siguiente, un disco cilndrico muy
macizo de masa M y radio a que tiene libertad para
I1 girar sin friccin en torno a una varilla muy ligera y
I1i = (I1 + I 2 ) f f = i delgada, a lo largo de su eje.
I1 + I 2

Como el Momento de inercia de un disco es


.m.R2 se obtiene:

1
m1 R 2
2 m1
f = i =
1 2 1 2 (m1 + m2 ) i
m1 R + m2 R
2 2
Un extremo de la varilla se apoya en A. que est a
En este caso particular: una distancia l del disco. Si se mantiene la varilla

43
Cuerpo rgido Hugo Medina Guzmn

horizontal, y se hace girar al disco con una es de 2,7 kg m2. La velocidad angular inicial de la
velocidad angular en torno a su eje y luego, se rueda es de 55 rad/s en sentido antihorario. En un
suelta. momento dado la profesora gira 180 el eje de la
Como actan dos nicas fuerzas el peso Mg y la rueda pasando a girar con -55 rad/s en sentido
reaccin del apoyo R, podra pensarse que el disco contrario al anterior. Calcular:
a) La velocidad angular adquirida por la silla y el
caera. Si L0 fuera cero sucedera esto, pero el sentido de giro.
torque que produce Mg es: b) El trabajo realizado por la profesora.

( )
= (li ) Mgk = Mglj

este torque produce un cambio en la cantidad de


movimiento angular

( )

d L = dt = Mglj dt
la magnitud. de este cambio es:
dL = Mgldt
Por otra parte: dL = L0 d
d Mgl
De aqu Mgldt = L0 d y =
dt L0 Solucin.
a) Dado que no hay momentos externos sobre la
1 silla giratoria podemos considerar que el momento
Como L0 = L0 = Ma 2 ;
2 angular no vara.
d Mgl 2 gl L1 = I RUEDA 1 ,
= = 2
dt 1
Ma 2
a L2 = I RUEDA ( 1 ) + I SILLA 2
2 I RUEDA 1 = I RUEDA ( 1 ) + I SILLA 2
Por consiguiente el disco no caer, en lugar de ello
girar en el plano horizontal xy (ver la figura 2I
siguiente) en torno al eje vertical a travs del punto
2 = RUEDA 1
I SILLA
de apoyo A.
2(0,02)
2 = 55 = 8,15 rad /s
2,7
(Positivo, por tanto en el sentido de rotacin inicial
de la rueda)
b)
W = E = E 2 E1
1 1 1
I SILLA 22 + I RUEDA ( 1 ) I RUEDA 1
2 2
=
La velocidad angular de esta precesin es:
2 2 2
d 2 gl 1
= = = = I SILLA 22 = 89,6 J
dt I a 2 2
El trabajo es por tanto la energa adquirida por la
silla, ya que la energa de la rueda no vara.
Ejemplo 78. Una profesora de fsica se encuentra
sentada en una silla giratoria manteniendo en sus
Dicho trabajo, positivo, es producido por la fuerza
manos una rueda de bicicleta como se indica en la
muscular (interna) de la profesora.
figura. El momento de inercia de la rueda respecto a
su eje es de 0,2 kg m2, y el momento de inercia de
la profesora ms la rueda respecto del eje de la silla

PREGUNTAS Y PROBLEMAS

1. El centro de masa de una pelota de radio R, se rotacional y la energa cintica de traslacin.


mueve a una rapidez v. La pelota gira en torno a un Considere la pelota una esfera uniforme.
eje que pasa por su centro de masa con una rapidez
angular . Calcule la razn entre la energa 2. Un volante en la forma de un cilindro slido de
radio R = 0,6 m y masa M = 15 kg puede llevarse

44
Cuerpo rgido Hugo Medina Guzmn

hasta una velocidad angular de 12 rad/s en 0,6 s por


medio de un motor que ejerce un torque constante.
Despus de que el motor se apaga, el volante
efecta 20 rev antes de detenerse por causa de la
friccin (supuesta constante). Qu porcentaje de la
potencia generada por el motor se emplea para
vencer la friccin?
Respuesta. 2.8%. .
Respuesta. a) 2(Rg/3), b) 4(Rg/3), c) (Rg).
3. Un bloque de masa m1 y uno de masa m2 se
conectan por medio de una cuerda sin masa que 6. Un peso de 50 N se une al extremo libre de una
pasa por una polea en forma de disco de radio R, cuerda ligera enrollada alrededor de una pelota de
momento de inercia I y masa M. As mismo, se deja 0,25 m de radio y 3 kg de masa. La polea puede
que los bloques se muevan sobre una superficie en girar libremente en un plano vertical en torno al eje
forma de cua con un ngulo como muestra la horizontal que pasa por su centro. El peso se libera
6 m sobre el piso.
figura. El coeficiente de friccin cintico es para
a) calcular la tensin de la cuerda, la aceleracin de
ambos bloques. Determine
la masa y la velocidad con la cual el peso golpea el
a) la aceleracin de los dos bloques y
piso.
b) la tensin en cada cuerda.
b) Calcular la rapidez con el principio de la
conservacin de la energa.
Respuesta. a) 11,4N, 7,6 m/s2, 9,5 m/s, b) 9,5 m/s.

7. Una ligera cuerda de nylon de 4 m est enrollada


en un carrete cilndrico uniforme de 0,5 m de radio
y 1 kg de masa. El carrete est montado sobre un
eje sin friccin y se encuentra inicialmente en
Respuesta. reposo. La cuerda se tira del carrete con una
a) (m2sen - )(m1 + m2cos )g/(m1 + m2 + M), aceleracin constante de 2,5 m/s2. a) Cunto
b) T1 = m2g + m1a, T2 = T1 + Ma. trabajo se ha efectuado sobre el carrete cuando ste
alcanza una velocidad angular de 8 rad/s?
4. Una masa m1 y una masa m2 estn suspendidas b) Suponiendo que no hay la suficiente cuerda sobre
por una polea que tiene un radio R y una masa m3. el carrete, Cunto tarda ste en alcanzar esta
La cuerda tiene un masa despreciable y hace que la velocidad angular?
polea gire sin deslizar y sin friccin. Las masas c) Hay suficiente cuerda sobre el carrete?
empiezan a moverse desde el reposo cuando estn Respuesta. a) 4 J, 1,6 s, c) s.
separadas por una distancia D. Trate a la polea
como un disco uniforme, y determine las 8. Una barra uniforme de longitud L y masa M gira
velocidades de las dos masas cuando pasan una alrededor de un eje horizontal sin friccin que pasa
frente a la otra. por uno de sus extremos. La barra se suelta desde el
reposo en una posicin vertical. En el instante en
que est horizontal, encuentre
a) su rapidez angular,
b) la magnitud de su aceleracin angular,
c) las componentes x e y de la aceleracin de su
centro de masa, y
d) las componentes de la fuerza de reaccin en el
eje.
Respuesta. a) (3g/L), b) 3g/2L,
c) (3/2 + )g, d) (-3/2 + )Mg.
5. Un disco slido uniforme de radio R y masa M
puede girar libremente sobre un pivote sin friccin
que pasa por un punto sobre su borde.
Si el disco se libera desde el reposo en la posicin
mostrada por el crculo.
a) Cul es la rapidez de su centro de masa cuando
el disco alcanza la posicin indicada en el crculo
punteado?
b) Cul es la rapidez del punto ms bajo sobre el
9. Los bloques mostrados en la figura estn unidos
disco en la posicin de la circunferencia punteada?
entre si por una polea de radio R y momento de
c) Repetir para un aro uniforme
inercia I. El bloque sobre la pendiente sin friccin

45
Cuerpo rgido Hugo Medina Guzmn

se mueve hacia arriba con una aceleracin constante


de magnitud a.
a) Determine las tensiones en las dos partes de la
cuerda,
b) encuentre el momento de inercia de polea.

L
Respuesta. x = (1 s )
2
Respuesta. a) T1 = m1 (a + gsen ) , 12. Determinar la tensin en el cable AB que
Impide que el poste BC deslice. En la figura se ven
T2 = m2 ( g a ) los datos esenciales. La masa del poste es de 18 kg.
2 g g Suponer que todas las superficies son lisas.
b) m2 R m1R 2 m2 R 2 m1R 2 sen
a a
10. Un cuerpo plano est sometido a cuatro fuerzas
como se indica en la figura.
a) Hallar el mdulo y direccin del torque actuante
respecto a un eje perpendicular al plano y que pasa
por el punto A.
b) Respecto a un eje que pasa por el punto B.
e) Respecto a un eje que pasa por el punto C.
d) Determinar la fuerza equivalente y su lnea de
accin.
e) Sustituir esta fuerza por otra que est aplicada en
A y un par de fuerzas o cupla aplicadas en los
puntos B y C y hallar el valor mnimo de estas Respuesta. T = 46,2 N
fuerzas.
13. Un hombre de 70 kg, sostiene un objeto de 31,9
kg. Como se indica en la figura. La polea carece de
rozamiento. La plataforma sobre la que est situado
el hombre est colgada mediante dos cuerdas en A y
otras dos en B. Cul e tensin de una de las
cuerdas en A?

Respuesta.
a) = 23 Nm, b) = 23 Nm, c) = 24 Nm,
Respuesta. 124,5 N
d) F = i + j , y = x 23 ,
23 14. Reemplace la fuerza de 1000 N de la figura por
e) FB = ( 3i + 4 j ) = FC una fuerza que pasa por A y una cupla cuyas
25 fuerzas actan verticalmente a travs de B C.
11. Un marco cuadrado de lado L. Se cuelga de un
clavo rugoso de coeficiente de rozamiento esttico
s . A qu distancia del vrtice est clavado si el
marco est a punto de deslizar?

46
Cuerpo rgido Hugo Medina Guzmn

Respuesta. 3,82 s

18. Si se aplica La fuerza F a una cuerda ligera



atada a un bloque con el sistema de poleas mostrado
Respuesta. FA = 800i + 600 j , FB = 467 j , en la figura. Cul es el mximo peso que puede
levantar?

FC = 467 j

15. Un hombre de 60 kg que camina a 2 m/s


atraviesa un tabla de 30 kg y 10 m de larg
a) Cul es la fuerza sobre el soporte B en funcin d
tiempo?
b) Si la mxima fuerza que puede resistir B es 490
Cundo y dnde caer al ro el hombre?
Considerar que el peso del hombre siempre acta en
direccin de la vertical que pasa por su centro de
masa.
Respuesta. 3F

19. El rodillo que se ve en la figura tiene una masa


de 339 kg Que fuerza F es necesaria para subir el
rodillo sobre el bloque?
Respuesta: a) FB = (12t + 15)9,8 N, b) t = 2,92
s, x = 5,83 m de A.

16. Un hombre de masa m quiere subir por una


escalera. La escalera tiene masa M, largo L y forma
un ngulo con e piso. El coeficiente de friccin
entre la escalera y e peso es , mientras que la
pared no tiene friccin.
a)A qu altura de la escalera puede llegar antes que Respuesta. F =3949,4 N
comience a resbalar?
b) Si el ngulo es el mayor sin que la escalera 20. La lnea de accin de una fuerza de 1N est en
el plano xz y corta el eje z en un punto que dista 0,6
sola puede estar sin resbalar, cul es la altura a la
m del origen.
que puede llegar el hombre?
a) Cul es el torque respecto al eje y si el ngulo
Respuesta. a)
comprendido entre la direccin de la fuerza y el eje
1
(m + M )Lsen 2 ML cos M cos
z es 60?
b) Si el ngulo e l80?
c) Si el ngu1o es 330?
b) L 2 Respuesta. a) = 0,52 N m , b) = 0
c) = - 0,3 N m
17. El disco A tiene una masa de 2 kg y un radio de
7,5 cm, se coloca en contacto con una correa que se 21. Dos discos de masa 10 kg y radio R = 0,3 m
mueve con una velocidad v = 15 m/s. Sabiendo que cada uno estn conectados mediante una cuerda. En
el coeficiente de rozamiento entre el disco y la el instante mostrado en la figura, la velocidad
correa es 0,2, calcular tiempo necesario para que el angular del disco B es de 20 rad/s en sentido
disco alcance una velocidad angular constante. horario. Calcular cunto sube el disco A cuando la
velocidad angular del disco B sea de 4 rad/s.

47
Cuerpo rgido Hugo Medina Guzmn

2
Respuesta.
3mgR

Respuesta. 1,54 m 25. Una esfera de l00 kg de masa y 0,6 m de


dimetro baja rodando, partiendo del reposo, por un
22. Un cilindro de masa ni y radio r rueda sin plano inclinado 25. recorriendo 30 m..
deslizar sobre la cara interior de una superficie a) Cul es su energa cintica al cabo de los 30 m?
cilndrica de radio R. Sabiendo que la esfera parte b) Cul es la velocidad de su centro de masa?
del raposo en la posicin indicada en la figura, Respuesta. a) 1268 kg m, b) 13,3 m/s
obtener:
a) La velocidad de la esfera al paso por B. 26. Un pasajero viaja de pie en un mnibus. El
b) El mdulo de la reaccin normal en cada mnibus se mueve con una velocidad de 50 km/h
instante. cuando el conductor aplica los frenos. El mnibus
desacelera de modo uniforme durante una distancia
de 15 ni hasta detenerse. Qu ngulo respecto a la
vertical deber inclinarse el pasajero para evitar su
cada?
Respuesta. 33,27 hacia atrs.

27. a) Cmo podra distinguirse una esfera de oro


de otra de plata si ambas tuviesen el mismo peso, el
mismo radio y las dos estuvieron pintadas del
mismo color?
4
Respuesta. a) g (R r )(1 cos ) , b) Cmo podra distinguir un huevo duro de uno
fresco si estuvieran juntos?
3
mg
b) (7 4 cos ) 28. Un carrete cilndrico hueco y uniforme tiene
3 radio interior R/2, radio exterior R y masa M . Est
montado de manera que gira sobre un eje horizontal
23. A que altura sobre la mesa debe golpearse una fijo. Una masa m se conecta al extremo de una
bola de billar con un taco mantenido cuerda enrollada alrededor del carrete. La masa m
horizontalmente para que la bola comience su desciende a partir del reposo una distancia y durante
movimiento sin rozamiento entre ella y la mesa? un tiempo t. Demuestre que el torque debido a la
fuerza de roce entre el carrete y el eje es:
y 5 y
= R m g 2 2 M 2
t 4 t

Respuesta. 7/5R

24. Un cilindro homogneo de masa m y radio R


descansa sobre un plano horizontal. Se aplica un
torque, segn se indica en la figura. Hallar el valor
del coeficiente de rozamiento entre la rueda y el
plano para que aparezca rodadura pura. 29. Un cilindro de 10 kg de masa rueda sin deslizar
sobre una superficie horizontal.
En el instante en que se su centro de masa tiene una
rapidez de 10 m/s, determine:
a) la energa cintica traslacional de su centro de
masa,

48
Cuerpo rgido Hugo Medina Guzmn

b) la energa rotacional de su centro de masa, y c) a) el origen,


su energa total. b) el punto ms alto de su trayectoria,
Respuesta. a) 500 J, b) 250 J, c) 750 J. c) justo antes de chocar con el suelo.
mv03
30. Una esfera slida tiene un radio de 0,2 m y una Respuesta. a) 0, b) sen 2 cos ,
masa de 150 kg. Cunto trabajo se necesita para 2g
lograr que la esfera ruede con una rapidez angular 2mv03
de 50 rad/s sobre una superficie horizontal? c) sen 2 cos
(Suponga que la esfera parte del reposo y rueda sin g
deslizar).
37. Un disco slido uniforme de masa M y radio R
31. Un disco slido uniforme y un aro uniforme se gira alrededor de un eje fijo perpendicular su cara.
colocan uno frente al otro en la parte superior de Si la rapidez angular es , calcular la cantidad de
una pendiente de altura h. Si se sueltan ambos desde movimiento angular del disco cuando el eje de
el reposo y ruedan sin deslizar, determine sus rotacin
rapideces cuando alcanzan el pie de la pendiente a) pasa por su centro de masa, y
Qu objeto llega primero a la parte inferior? b) pasa por un punto a la mitad entre el centro y el
borde.
32. Una bola de boliche tiene una masa M, radio R
y un momento de inercia de (2/5)MR2. Si rueda por 38. Una partcula de 0,4 kg de masa se une a la
la pista sin deslizar a una rapidez lineal v, Cul es marca de 100 cm de una regla de 0,1 kg de masa.
su energa total de funcin de M y v? La regla gira sobre una mesa horizontal sin friccin
Respuesta. 0,7Mv2. con una velocidad angular de 4 rad/s. Calcular la
cantidad de movimiento angular del sistema cuando
33. Un anillo de 2,4 kg de masa de radio interior de la regla se articula en torno de un eje,
6 cm y radio exterior de 8 cm sube rodando (sin a) perpendicular a la mesa y que pasa por la marca
deslizar) por un plano inclinado que forma un de 50 cm,
ngulo de = 37 con la horizontal. En el momento b) perpendicular a la mesa y que pasa por la marca
en que el anillo ha recorrido una distancia de 2 m al de 0 cm.
ascender por el plano su rapidez es de 2,8 m/s. Respuesta. a) 0,43 kgm2/s, b) 1,7 kgm2/s.
El anillo continua ascendiendo por el plano cierta
distancia adicional y despus rueda hacia abajo. 39. Una mujer de 60 kg que est parada en el borde
Suponiendo que el plano es lo suficientemente largo de una mesa giratoria horizontal que tiene un
de manera que el anillo no ruede fuera en la parte momento de inercia de 500 kgm2 y un radio de 2
superior, qu tan arriba puede llegar?
m. La mesa giratoria al principio est en reposo y
34. Una barra rgida ligera de longitud D gira en el tiene libertad de girar alrededor de un eje vertical
plano xy alrededor de un pivote que pasa por el sin friccin que pasa por su centro. La mujer
centro de la barra. Dos partculas de masas m1 y m2 empieza a caminar alrededor de la orilla en sentido
se conectan a sus extremos. Determine la cantidad horario (cuando se observa desde arriba del sistema)
de movimiento angular del sistema alrededor del a una rapidez constante de 1,5 m/s en relacin con
centro de la barra en el instante en que la rapidez de la Tierra.
cada partcula es v. a) En qu direccin y con qu rapidez angular gira
Respuesta. ( m1 + m2)vD. la mesa giratoria
b) Cunto trabajo realiza la mujer para poner en
35. Un pndulo cnico consta de masa M que se movimiento la mesa giratoria?
mueve en una trayectoria circular en un plano Respuesta. a) 0,36 rad/s, antihorario.
horizontal. Durante el movimiento la cuerda de
longitud L mantiene un ngulo constante con la 40. Una barra uniforme de masa M y longitud d gira
vertical. Muestre que la magnitud de la cantidad de en un plano horizontal en torno de un eje vertical
movimiento angular de la masa respecto del punto fijo sin friccin que pasa por su centro. Dos
de soporte es: pequeas cuentas, cada una de masa m, se montan
sobre la barra de manera tal que pueden deslizar sin
gM 2 L3 sen 4 friccin a lo largo de su longitud. Al principio las
L= cuentas se fijan por medio de retenes ubicados en
cos
las posiciones x (donde x < d/2) a cada lado del
centro, tiempo durante el cual el sistema gira una
36. Una partcula de masa m se dispara con una
rapidez angular . Repentinamente, los retenes se
rapidez vo formando un ngulo con la horizontal.
quitan y las pequeas cuentas se deslizan saliendo
Determine la cantidad de movimiento angular de la
de la barra. Encuentre,
partcula respecto del origen cuando sta se
a) la rapidez angular del sistema en el instante en
encuentra en:
que las cuentas alcanzan los extremos de la barra, y

49
Cuerpo rgido Hugo Medina Guzmn

b) la rapidez angular de la barra despus de que las


cuentan han salido de ella.

41. Un bloque de madera de masa M que descansa


sobre una superficie horizontal sin friccin est
unido a una barra rgida de longitud l y masa
despreciable. La barra gira alrededor de un pivote
en el otro extremo. Una bala de masa m que se
desplaza paralela a la superficie horizontal y normal
a la barra con rapidez v golpea el bloque y queda
incrustada en l.
45. A una bola de boliche se le da una rapidez
a) Cul es la cantidad de movimiento angular del
inicial vo en una canal de manera tal que
sistema bala-bloque?
inicialmente se desliza sin rodar. El coeficiente de
b) Qu fraccin de la energa cintica original se
pierde en la colisin? friccin entre la bola y la canal es . Demuestre que
durante el tiempo en que ocurre el movimiento de
Respuesta. a) mvl , b) M/(M+m). rodamiento puro,
a) la rapidez del centro de masa de la bola es 5vo/7,
42. Una cuerda se enrolla alrededor de un disco y
uniforme de radio R y masa M. El disco se suelta b) la distancia que recorre es 12 vo2/49 g.
desde el reposo con la cuerda vertical y su extremo (Sugerencia: Cuando ocurre el movimiento de
superior amarrado a un soporte fijo. A medida que
rodamiento puro, vcm = R. Puesto que la fuerza de
el disco desciende, demuestre que
friccin proporciona la desaceleracin, a partir de la
a) la tensin en la cuerda es un tercio del peso del
segunda ley de Newton se concluye que acm = g.)
disco.
b) La magnitud de la aceleracin del centro de masa
46. El alambre de un carrete de masa M y radio R se
es 2g/3, y
desenrolla con una fuerza constante F. Suponiendo
c) la rapidez del centro de masa es (4gh/3).
que el carrete es un cilindro slido uniforme que no
Verifique su respuesta a la pregunta c) utilizando
desliza, muestre que, a) la aceleracin del centro de
mtodos de energa.
masa es 4F/3M, y
b) la fuerza de friccin es hacia la derecha y su
43. Una pequea esfera slida de masa m y de radio
magnitud es igual a F/3.
r rueda sin deslizar a lo largo de la pista mostrada
c) Si el cilindro parte del reposo y rueda sin
en la figura. Si parte del reposo en la parte superior
deslizar, Cul es la rapidez de su centro de masa
de la pista a una altura h, donde h es grande
despus que ha rodado una distancia D?
comparada con r
Respuesta. c) (8FD/3M).
a) Cul es el valor mnimo de h (en funcin de R)
de modo que la esfera complete la trayectoria?
b) Cules son las componentes de fuerza de la
esfera en el punto P si h = 3R?

47. Suponga un disco slido de radio R al cual se le


da una rapidez angular o alrededor de un eje que
pasa por su centro y despus se baja hasta una
superficie horizontal y se suelta, como en la.
44. Un proyectil de masa m se mueve a la derecha Suponga tambin que el coeficiente de friccin
con rapidez v0. El proyectil golpea y queda fijo en entre el disco y la superficie es .
extremo de una barra estacionaria de masa M y a) Calcular la rapidez angular del disco una vez que
longitud D que est articulada alrededor de un eje ocurre el rodamiento puro.
sin friccin que pasa por su centro. b) Calcular la prdida fraccionaria de energa
a) Encuentre la rapidez angular del sistema justo cintica desde el momento en que el disco se suelta
despus de la colisin. hasta que ocurre el rodamiento puro
b) Determine la prdida fraccionaria de energa c) Muestre que el tiempo que tarda en ocurrir el
mecnica debida a la colisin. movimiento de rodamiento puro es R o/3 g.
d) Muestre que el tiempo que recorre el disco antes
de que ocurra el rodamiento puro es R2 o 2/18 g.

50
Cuerpo rgido Hugo Medina Guzmn

1
Respuesta. = sen -1 . Estar en
M r2
+ 2
m R
M r2
reposo solo si + 2 1
m R
48. La figura muestra un carrete de alambre que
descansa sobre una superficie horizontal. Cuando se 51. Los discos A y B son del mismo material y
tira, no se desliza en el punto de contacto P. El tienen el mismo espesor, pudiendo girar 1ibemente
carrete se tira en las direcciones indicadas por alrededor de un eje vertical. El disco B se encuentra
medio de los vectores F1, F2, F3 y F4. Para cada en reposo cuando se deja caer sobre el disco A. el
fuerza determine la direccin en que rueda el est girando con una velocidad angular de 400 rpm.
carrete. Advierta que la lnea de accin de F2 pasa Sabiendo que la masa del disco A es de 4 kg,
por P. calcular:
a) La velocidad angular final de los discos.
b) La variacin de la energa cintica experimentada
por el sistema.
R A = 0,1 m, RB = 0,15 m,

49. El carrete mostrado en la figura tiene un radio


interior r y un radio externo R. El ngulo entre la
fuerza aplicada y la horizontal puede variar.
Demuestre que el ngulo crtico para el cual el Respuesta. a) 334 rpm, .b).- 6,5l J
carrete no rueda y permanece estacionario est dado
por cos = r/R. 52. Una bala de 3g se dispara, con una velocidad
(Sugerencia: En el ngulo crtico la lnea de accin horizontal de 550 m/s, contra. Una varilla de
de la fuerza aplicada pasa por el punto de contacto.) madera AB de longitud L = 0,750 m. La varilla que
inicialmente est en reposo, se encuentra
suspendida de una cuerda de longitud L = 0,750 m.
Sabiendo que h = 0,150 m, calcular las velocidades
de cada uno de los extremos de la varilla
inmediatamente despus de que la bala se haya
incrustado.

50. Se tiene un carrete sobre un plano inclinado, el


cual tiene enrollado un hilo delgado y su extremo
libre sujeta una masa m por medio de una polea sin
friccin y masa despreciables. Se asume que la
masa del carrete M est distribuida uniformemente
en un crculo de radio R. Determinar el ngulo de
inclinacin al cul el centro de gravedad del
carrete estar en reposo.


Respuesta. v A = 0,566i , v B = 6,22i

53. Un tabln masa M se apoya sobre un pequeo


pivote D. Un gimnasta A de masa m est de pie
sobre el extremo C del tabln, un segundo gimnasta
B de la misma masa m salta desde la altura h y cae

51
Cuerpo rgido Hugo Medina Guzmn

sobre el tabln en E. Suponiendo que este choque es


perfectamente inelstico, determinar la altura que
alcanzar el gimnasta A. (El gimnasta A permanece
de pie completamente rgido).

Respuesta. = 2,1 rad/s

55. Una rueda de bicicleta de 82 cm de dimetro


tiene una platina de acero enrollada en su parte
exterior de modo que la masa resultante del sistema
puede suponerse que est situada toda ella en la
m2h periferia de la rueda, siendo M = 7,3 kg
Respuesta.
(2m + M 3)2 sosteniendo los dos extremos del eje con las manos
en la posicin horizontal. El eje sobresale 15,2 cm a
cada lado de la rueda. Mientras la rueda est
54. Un disco macizo de 1,2 kg de masa y 10 cm de girando con una velocidad angular de 25,12 rad/s se
dimetro est montado en un extremo de un eje de hace girar el eje con las manos en un plano
masa despreciable que est pivotado alrededor de horizontal alrededor de su centro. Calcular el valor
un punto a 6 cm del, centro del disco en el otro y direccin de la fuerza que deber ejercer en cada
extremo del eje, a una distancia de 10 cm del mano para producir una velocidad angular de
pivote, se cuelga un objeto de 0,96 kg de masa. Si la precesin de 0,628 rad/s alrededor del centro.
velocidad angular de giro del disco es 37,37 rad/s. Respuesta. un par de fuerzas de 64,6 N aplicadas
Cul es la velocidad de precesin? en cada extremo del eje.

52

Potrebbero piacerti anche